You are on page 1of 473

igh-Yield study guide that correlates with the Doctors In Training Step 1 Review Course video series Featuring:

Brian Jenkins, MD Mike Mclnnis, MD Chris Lewis, MD Jennifer Shuford, MD Hampton Richards, MD

i
r r r r r
DOCTORS IN TRAINING

Daily quizzes, mnemonics, notes and diagrams to promote active learning Bridges the gap between basic and clinical sciences

BETTER DOCTORS. BETTER WORLD.

AUSE?

ND 7 ADDITIONAL DAYS 1-WEEK EXTENSION

UDED:
7 additional days of online video lecture access The ability to pause your online video lectures access once for any length of time within your designated )urse year. ansider watching all of the deo lectures, pausing the >urse, and reviewing them II again.

withcodeSPAC15
AMty to purchase a I -week cone mansion vrll expire 3 months after your aart date ol Part 2 video lectufe access.

NE A E ED N

I PHARMACOLOGY
Comprehensive pharmacology review with an emphasis on high-yield information for USMLE and COMLEX exams Clinical applications throughout Prepare early for third year rotations Our Solid Pharmacology series features lectures from our experienced physicians, entertaining learning components, mnemonic and animations to encourage active learning. This lecture series includes valuable information that will help you in classes, exams and your Step 1/Level 1 studies.

NH L . EP

ALSO, CHECK OUT OUR OTHER SOLID SERIES COURSES:

urn 3-*ii'

LECTURES CAN BE PURCHASED INDIVIDUALLY OR AS AN ENTIRE COURSE WITH DIT'S FLEXIBLE LEARNING CREDIT SYSTEM AND CAN BE WATCHED ON YOUR SCHEDULE.

USMLE Step I Review Study Guide 2012 edition

DOCTORS IN TRAINING

BETTER DOCTORS. BETTER WORLD.

Brian Jenkins, MD

Doctors In Training.com: USMLE Step I Review, 2012 edition

Author: Brian Jenkins. MD

Copyright 2012 Doctors In Training.com, LLC www.doctorsintraining.com Doctors In Training.com 1701 River Run, Suite #703 Fort Worth, TX 76107
All Rights Reserved. This text is protected by copyright. No part of this book may be reproduced in any form or by any means, including photocopying, or utilized by any storage and retrieval system, without written permission from the copyright owner.

Selected images from Lippincott Williams & Wilkins Atlas of Anatomy "*^~ rr 2008 Wolters Kluwer Health, Inc. All rights reserved. WHlicHTIS

&

Wl

HCIPIS

a Wolters Kluwer business


DISCLAIMER: THE AUTHOR DISCLAIMS ANY UABILITY. LOSS. INJURY. OR DAMAGE INCURRED AS A CONSEQUENCE. DIRECTLY OR INDI RECTLY. OF THE USE AND APPLICATION OF ANY OF THE CONTENT AND MATERIAL CONTAINED IN THIS TEXT. ALTHOUGH THE INFORMA TION IN THIS TEXT HAS BEEN CAREFULLY REVIEWED FOR CORRECTNESS. THE AUTHOR CANNOT ACCEPT ANY RESPONSIBILITY FOR ANY ERRORS OR OMISSIONS THAT MAY BE MADE. THE AUTHOR MAKES NO WARRANTY. EXPRESS OR IMPLIED. AS TO THE COMPLETENESS. CUR RENCY OR ACCURACY OF THE CONTENTS OF THIS TEXT. THE INFORMATION CONTAINED IN THIS TEXT SHOULD NOT BE CONSTRUED AS SPECIFIC INSTRUCTIONS FOR INDIVIDUAL PATIENTS. MANUFACTURER'S PRODUCT INFORMATION AND PACKAGE INSERTS SHOULD BE REVIEWED FOR CURRENT INFORMATION. INCLUDING CONTRAINDICATIONS. DOSAGES. AND PRECAUTIONS. For problems, questions, or concerns, you may contact the author at support@doctorsintraining.com.

Copyright 2012, Doctors In Trainmg.com. LLC All Rights Reserved.

This Study Guide contains page references to several medical educational resources. Below is a key to the abbreviations that accompany the various page number references appearing in this text.
FAII Le, T, Bhushan, V, et al. First Aid for the USMLE Step 1 2011. New York, NY: McGraw-Hill; 2011. Le, T, Bhushan, V, et al. First Aid for the USMLE Step 1 2011. New York, NY: McGraw-Hill; 2012. Hall, JE. Guyton and Hall Textbook of Medical Physiology. 12th ed. Philadelphia, PA: Saunders Elsevier; 2011. Kumar, V, Abbas, AK, et al. Robbins and Cotran Pathologic Basis of Disease. 8th ed. Philadelphia, PA: Saunders Elsevier; 2010. Moore, KL, Dalley, AF, & Agur, AMR. Clinically Oriented Anatomy. 6th ed. Philadelphia, PA: Lippincott Williams & Wilkins; 2010. Longo, DL, Fauci, AS, Kasper, DL, Hauser, SL, Jameson, JL, Loscalzo, J, eds. Harrison's Principles of Internal Medicine. Vol. 2. 18th ed. New York, NY: McGraw-Hill; 2012. Brunton, LL, Chabner, BA, & Knollman, BC, eds. Goodman & Gilman's The Pharmacological Basis ofThempeutics. 12th ed. New York, NY: McGraw-Hill; 2011.

Key to Abbreviations

FAQ

Phys R

COA

GG

Neither the Doctors in Training USMLE Step I Review course, nor this Study Guide, is endorsed by or afliated with any of the medical education resources referenced in this text.

Course Viewing Options Historically, the Doctors In Training live Step I review course was structured as 5 hours of lecture per day (Monday through Friday) for 3 weeks. We recognize, however, that this schedule may not meet the individual needs of every student. To make the course as exible and helpful as possible, the online course is broken into 148 lectures, which can be viewed in more or fewer than 15 days, as desired.
30-day course 21-day course 18-day course 15-day course 12-day course 10-day course 5 lectures/day 7 lectures/day 8 lectures/day 10 lectures/day 12 lectures/day 15 lectures/day

Although you have the exibility to view the lectures in any order, we strongly recommend that you watch the videos in the order in which they appear in the Table of Contents, completing each row before proceeding to the next one, regardless of how many lectures you view in a day.
Copyright 2012. Doctors In Training.com. LLC. All Rights Reserved.

2012 Doctors In Training Step 1 Review Course Table of Contents


Lecture 1 Foundations 1 - Principles and Embrvolo The 21 Fundamental Principles of Human Life Embryology Part 1 Embryology Part 2 Foundations 5 - Cellular Cellular Response 1Acute and Chronic Inammation Cellular Response 2 Control of the Extracellular Environment Neuro 3 - Brainstem and Cranial Nerves lefts, Arches, Pouches ranial Nerves pecialty Centers in the Brainstem Endocrine 2 - Adrenals Adrenal Steroids D Cushing Syndrome Other Adrenal Pathology Immunology 1-Basics and Antieen Presentation Immunology Basics Antigen Presentation Immunology 3 - B cells. Antibodies, and Cytokines B Cells and Antibodies Cytokines and Immunosuppressants Lecture 2 Foundations 2 - Cellular Order Cell Cycle and Protein Organelles Protein Processing Lecture 3 Foundations 3 - Cellular Structure and Function Cytoskeleton and Other Cellular Components Plasma Membrane Lecture 4 Foundations 4-Cel Suffering and Death Apoptosis and Necrosis Cellular Injury

Foundations 6 - Cellular Adaptations Cellular Response 3Internal Adaptations Cellular Response 4 Atrophy and Aging

Neuro 1-Basics Organization of the Brain Neurotransmitters

Neuro 2 - Hypothalamus and Slee. Hypothalamus Sleep

Pharm Basics 1ParasvmDathetics Parasympathetic Activation Parasympathetic Inhibition Pharm Basics 3Sympathetics Sympathetic Agents Sympathetic Blockers Immunology 2 -T Cells and Monocytes T Cells Monocytes and Macrophag Immunology 4Complement, Immunodeciency Complement, Hypersensitivity, and Granulocytes Immunodeciencies Pharm Basics 4Pharmacokinetics Enzyme Kinetics and Pharmacokinetics Pharmacodynamics Gl 6-Small Intestine Enterocytes and Absorption Diseases of the Small Intestine Biochem 7-Fuel Use Ketones Feeding, Fasting, and Exercise Ethanol and Caloric eciencies

Pharm Basics 2 - Cellular Communication Autonomic Nerve Terminals G-protein-Linked 2 Messengers Gl 1-Oropharynx Mouth, Tongue, Salivary Glands Nose and Sinuses Biochem 2-RNA RNA Basics Transcription

Endocrine 1 - Pituita Basic Overview of Endocrine Pituitary and Prolactin Other Pituitary Hormon Endocrine 3-Thyroid Thyroid Basics and Hyperthyroidism Hypothyroidism and Thyroid Cancer Biochem 3 - Protein tRNA and Protein Synthesis Protein Modication

Micro 1 - HIV Gl 2-Esophagus HIV Basics Basics of the Esophagus Highly Active Antiretroviral Esophageal Pathology Therapy (HAART)

ir@M!"-=iiJta:'i'i3iiMi
Gastritis, PUD, and Antaci Other Gl Pathology Duodenum Pancreas Biochem 5- Ener Potential Fates of Pyruvate Pyruvate Dehydrogenase, TCA and ETC Gl 7 - Liver and Cirrhosis Embryology of the Liver Hepatocytes Cirrhosis and Portal Hypertension Liver Diseases

of the Stomach Gastric Physiology

Endocrine 4- Pancreas Endocrine Pancreas and DM DKA and DM Medications Biochem 6-HMPShunt nd Other Sugars MP Shunt uctose, Galactose, and Lactose

Biochem 4-Glucose Glycolysis and Gluconeogenesis Glycogen Endocrine 5 - Obesif Physiology of Obesity Metabolic Syndrome and Obesity

2012 Doctors In Training Step 1 Review Course Table of Contents


Lecture 1 Gl 8 - Liver Pathology Hepatitis J Additional Liver Diseases Lecture 2 Gl 9-Gall Bladder Biliary Anatomy, Physiology, and Jaundice Biliary Tract Disease Pharm Basics 5-Dm Metabolism Drug Metabolism Alcohol Metabolism and Drug Sufxes Biochem 11-Minerals Calcium, Iron, and Zinc Trace Minerals and Toxic Metals Lecture 3 Lecture 4 Biochem 8 - Lipids Biochem 9 - Amino Acids Lipid Transport and Nitrogen Fatty Acids and Cholesterol Amino Acid Basics Nitrogen Metabolism Heme 1 - Clotting Factors Coagulation Basics Agents Affecting Clotting Factors Heme 3 - Nonhemolytic Anemias Microcytic Anemia Macrocytic and Nonhemolytic, Normocytic Anemias Erythrocyte Basics RBC Pathology

Biochem 10 - Amino Acid Disorders K Amino Acid Disorders

Biochem 1 - DNA DNA Basics DNA Interactions

Heme 5-Platelets Platelet Basics Platelet Disorders M Other Bleeding Disorders Antiplatelet Medications

IIIMiilIi

of Blood

Biochem 12 - Fat-Soluble
Vitamins and Antioxidan Vitamins D, K, and A Vitamins C, E, and Other Antioxidants

Lymph Leukemias Chronic Myeloproliferative Disorders Multiple Myeloma

Heme 4-Hemolytic Anemias Intrinsic Hemolytic Anemias Extrinsic Hemolytic Anemias Biochem 13-Water uble Vitamins Folic Acid, Vitamin B12, and Vitamin B6 Vitamins B5, Niacin, Riboavin, Thiamin! and Biotin Micro 3 - Protozoa & Helminths Protozoa Helminths

Gl 10 - Large Intestine 1 Large Intestine Basics, Cecum, and Appendix N Ascending, Transverse, and Descending Colon

Gl 11 - Large Intestine 2 Distal Large Intestine Anus

Micro 2Enterobacteriaceae & Diarrhea Enterobacteriaceae, UTI, and Antibiotics Diarrhea and Food

Oncology 1-Basics Genetics of Cancer Cancer Risk Factors

Oncology 2 - Neoplastic Progression Neoplastic Progression Downstream Effects of Neoplasia Medicine in Society 2Bias & Statistics Bias and Error Condence Interval

Medicine in Society 1Evidence Based Medicine Studies and Diagnostic Tests Application of Test Data

Poisoning Oncology 3 - Cancer Prevention and Host Defense Cancer Prevention Host Defense Against Cancer Medicine in Society 3 Public Health Disease Prevention: Pediatrics and Obstetrics Prevention of Adult Diseases

Oncology 4 - Cancer Drugs Cancer drugs

Medicine in Society 4Geriatrics Physiological and Psychological Changes Diseases and Resources for the Elderly

IIMira.-fSyi.-g IIBHilfHTff
Healthcare System Ethics Healthcare System Part 1 Ethical Principles Healthcare System Part 2 Ethical Scenarios

llTPTTFPJi'rTTf
and Phvsiolo Diaphragm and Respiration Respiratory Tree and Alveoli Circulation Lung Volumes and Hemoglobin Pulmonary Circulation and Pulmonary Hypertension

2012 Doctors In Training Step 1 Review Course Table of Contents


Lecture 1 Oxygenation and Ventilation Oxygenation Ventilation PulmonmfMzm Cancer and Infections Lung Cancer Lung Infections Micro 7-Cell Wall Inhibitors Cephalosporins Other Cell Wall Inhibitors Lecture 2 Pulmonary 4 - Extreme Environments Altitude, Aviation, and Space Deep Sea and Hyperbarics Micro4-Bas Bacterial Structures Exotoxins Micro 8 - TB and TB Drugs Mycobacteria and Tuberculosis Anti-Tuberculosis Agents Lecture 3 Pulmonary 5 - PE, COPD, and Asthma PE, DVT, and COPD Asthma Lecture 4 Pulmonary 6 - Restrictive Lung Disease Restrictive Lung Diseases and ARDS Pneumoconiosis and Sarcoidosis icro 6 - Gram Positives and Penicillins Other Gram Positives Penicillins Cardiovascular 1Overview and Ventricle Development Atrial Development and Fetal Circulation Cardiovascular 5 - Edema and Shock Capillaries and Edema Capillaries and Shock Cardiovascular 9 - ECG and Arrhythmias Atria to AV Conduction Ventricular Arrhythmias Cardiovascular 13 - M Ml Pathophysiology Diagnosis and Treatm of Ml Neuro 5-Brainstem in Cross Section Anatomy of the Brainstem Cranial Nerve Lesions Neuro 9-Basal Gan Basal Ganglia and Hypokinesis Hyperkinesis

Gram Positives and Staph. Streptococcal Species Micro 9-Other Nonstaining Bacteria Spirochetes and Zoonotics Other Nonstaining Bacteria

Cardiovascular 2DevelODmental Pathola U Right-to-Left Shunts Left-to-Right Shunts a ' [^H f J i M i i H I f a l Cardiovascular 6 - Cardiac Cycle V Atria and Ventricles Heart Sounds and Jugular Waves Cardiovascular 10 - BP and HTN W Physiology of BP Regulation Hypertensio Cardiovascular 14 - Other Cardiac Pathology X Cardiomyopathies and Endocarditis Other Cardiac Pathology I Neuro 6 - Occlusion Syndromes Brainstem Occlusion Syndromes 1 Brainstem Occlusion Syndromes 2 Neuro 10-Spinal Cord and Lesions Spinal Cord Basics ! Spinal Cord Lesions

Cardiovascular 3 - Cardiac Output Cardiac Output Variables Charting Cardiac Output Cardiovascular 7 - Heart Murmurs Systolic Murmurs Diastolic Murmurs Cardiovascular 11Antihvoertensives Antihypertei Choice of i H l i l ' J ' M l H l ff p H Cardiovascular 15Vascular Disorders Vascular Disorders 1 Vascular Disorders 2 Neuro 7 - Vascular Events Arterial Events Venous and Ventricular Events

Cardiovascular 4 - Heart Failure Heart Failure Pathophysiology Heart Failure Medicatio Cardiovascular 8Electrophysiology Action Potentials Antiarrhythmics Cardiovascular 12 Atherosclerosis therosclerosis an_ anginal Therapy and ipid-Lowering Agents Neuro 4 - Regions of the Brain Cerebral Cortex Cerebral Circulation Neuro 8 - Movement Movement Execution Modiers of Movement

Neuro 11 - Brachial Plexus and Upper Extremity Nerves Brachial Plexus Anatomy Nerves of the Upper Extremity

Neuro 12 - Lower Extremity and Skeletal Muscle Lower Extremity Skeletal Muscle

Rheum and Derm 1- Bone and Bone Disorders Bone Histology and Pathology Paget Disease and Bone Tumors

Endocrine 6 - Calcium Metabolism Basics of Calcium Metabolism Hyperparathyroidism and Hypoparathyroidism

Rheum and Derm 2-Joint Rheum and Derm 3-Joint Rheum and Derm 4 Basics Diseases Systemic Disorders Basics s Lower B a s i canda n d L o w e r O s t e o a r t h r i t i s a n d S y s t e m i c R h e u m a t i c E x t r e mJoints Joints Treatment Diseases Extremity i t y iiiT?jmiiii]i.-{^-.Mj.Upper Extremity Joints wi'^BiuffMlSB'

2012 Doctors In Training Step 1 Review Course Table of Contents


Lecture 1 Neuro 13-Sensation Skin Sensation BB Other Demyelinating Diseases Otic Sensation Neuro 14 - Eve Eye Structures C Neural Aspects of the Eye Lecture 2 Rheum and Derm 5 Dermatolo Derm Basics and Derm Path Part 1 Derm Path Part 2 Psychology Basics Learning and Therapy Lecture 3 Micro 10 - Mvcolo Mycology Arthropods Lecture 4 Micro 11 - Antifungals and Neuro Review Antifungals Neuro Review

Psvch2-ChildPsvch Infants and Early Childhood Adolescence Neuro 17 - Seizures Seizures Anti-Seizure Medications

Uffiyn'niril'liiUM
Delirium, Dementia, Alcohol Substance Abuse Psvch 4 - Psvchosis and Mania Psychosis and Schizophrenia Mania and Bipolar Disorder Nephron and Diuretics
^^yni^yj ~d 11; -i W r>T:

Neuro 15-Dementia and Neuro 16-Tumorsand Headache Anesthetics Dementia Brain Tumors Headache Anesthetics

Psvch 5 - Depression Major Depressive Disorder (MDD) Antidepressant Medication Renal 3-Metabolic Disorders Renal Metabolic Basics pHandADH Micro 13-Protein Synthesis Inhibitors Protein Synthesis Inhibitors Other Antibiotics Review 1-Micro by Systems Micro by Systems 1 Micro by Systems 2 Reproduction 3 - Male

Psvch 6 Anxiety Ego Defe.. and Personali

Renal 1-Basics Anatomy and Embryology Glomerulus

:|p1--iii-Hi

E K S a
Renal 4-Glomerular Pathology Nephritic Syndrome Nephrotic Syndrome Micro 14 - DNA Viruses Herpes Viruses Other DNA Viruses Renal 5-Other Renal Patholo Other Renal Pathology 1 Other Renal Pathology 2 Micro 15 - RNA Viruses RNA Viruses Part 1 RNA Viruses Part 2 Micro 12 - Other Gram Negative Bacteria Gram Negative Basics Specic Gram Negative Organisms Micro 16-Viral Bas Viral Basics Part 1 Viral Basics Part 2

Review 2 - Side Effects and Reproduction l-Anatom\ Antidotes Drug Side Effects Antidotes Reproductive Anatomy Reproductive Embryology 4-Estrogen Reproduction 5-Vagina, and Progesterone I Cervix, and Uterus Female Reproductive Cycle Vaginal and Cervical Hormonal Birth Control Pathology and Menopause Uterine Pathology Reproduction 8Chromosomal Disorders Trisomies and Mental Retardation Other Chromosomal Disorders

Reproduction 2 Androgens and Testes Androgens Testes Pathology Reproduction 6 - Ovaries Ovarian Pathology Ovarian Tumors

Reproduction 7Pregnancy Pregnancy Complications Labor and Delivery

Review 3 - Genetic

Review 4-Genetic
Diseases: AR and X-linked Autosomal Recessive Diseases and Lysosomal Storage Diseases X-Linked Recessive Disorders Review 6 - Final Review The Wisdom and. Medicine

Biochem 14-Basic Genetics Genetic Laboratory Techniques Inheritance

Disorders: AD and Trinucleotide Repeats Autosomal Dominant Diseases Part 1 Autosomal Dominant Diseases Part 2 and Trinucleotide Repeats Reproduction 9 - Breast Review 5 - Pediatric Review Benign Breast Conditions Breast Tumors and Cancer Pediatric Review Part 1 Pediatric Rev

The 21 Fundamental Principles of Human Life


I. What are the 21 fundamental principles of human life? 1) Human sufcient life is a system of , Human life requires

2) Human life is organized into 3) Every level of human life has 4) As a system of energy and matter exchange, human life. A. The law of B. Since establishing order requires energy,. C. Energy-consuming reactions (AKA ) must be coupled with energy releasing reactions (AKA ). 5) In order for human life to grow, must take place. 6) Each stakeholder in human life is 7) As specialization increases, the various stakeholders of human life can be categorized as being more or more relative to other stakeholders. 8) Each stakeholder in human life has

9) Human life is organized into multiple 10) The specialized stakeholders of human life that are designed to establish order. 11) Human life is in a constant state of_

o
p
stakeholders, ^

12)

In

order

to

maintain

the

harmony

of

13) When a stakeholder of human life goes through a cycle of. 14) Harmony among the stakeholders in human life requires a method of allowing. 15) The more stakeholders that

o
z CO
i

-rj 25.

16) Every stakeholder of human life has which ultimately were n 17) 1) 8 19) Each stakeholder in human life makes efforts to $

cr
The only way to alleviate S2_ <$

20) 2) 1

Copyright

2012.

Doctors

lnTraining.com.

LLC

All

Rights

Reserved.

Embryology Part I
Fetal landmarks (FAII pi 18) (FAI2 pi24) Morula Inner Cell Mass Eplblast
A gastrulation epidermis hair, mill mmary gland ant pituitary lent Inner tar teeth enamel

Trophoblast

placenta embryonic membrane Endoderm Pircntnymeof;


liver thyroid pancreas respiratory i tonsils thymus bladder, urethra tympanic cavity 'auditory tube tract

Hypoblast

nturulation

Intermediate: 'urogenital (Including kidney and gonads)

via notoi"1"

(covers amnion) * lateral and ventral ' "" of 9"t (with endode body wall (with "cardiovascular and lymp ectoderm) Myotomes"

N-ur.,IC..-,t(>lls: * Schwann cells "- tUs B anocytes Ha of adrenal I septum I root ganglia -facial structures .js In A Inhibits ml

uetamai;
vertebral bodies, mine lamina, cannulus post pituitary fibrosus of Inter* pineal body vertebral discs us " pulposls from the

Dermomyotomes

OermoSsmej.; dermis

Eeimtrej intrinsic back muscles (erector spinae)

Hyporrnrci intercostal!
obliques tansverse ah rectus abdo dorsal appendages - extensor intral appendages - exor m

'.. What is the formula for Gibbs free energy?

What developmental structure matches the following description? Fetal placental structure that secretes hCG Maternal component of the placenta

Order the following molecules by how much energy they contain that can be made available to fuel endergonic reactions: pyruvate, adenosine monophosphate, glucose, adenosine, adenosine triphosphate.

lat are the stages of an embryo between conception and an inner cell mass?

Copyright 2012. Doctors In Training.com, LLC. All Rights Reserved.

Embryology part 2
GG: Chapter 23, 66 Fetal landmarks (FAII pi 18) (FAI2 p!24) Embryologic derivatives (FAII pi 19) (FAI2 pl26) Neural development (FAII pi 18) (FAI2 pl2S) Teratogens (FAII pl20) (FAI2 pl27) (GGpl84S) Important genes ofembryogenesis (FAII pll8) (FAI2 pl24) Fetal alcohol syndrome (FAII p!20) (FAI2 pl28) (GG p64l)

6. What are the neural crest derivatives of the following adult structures? Peripheral Nervous System Ear Eye Adrenal Gland Mouth Heart Digestive System Thyroid Skin

o c
z
%

Homeobox (HOX) Genes


Blueprint for skeletal morphology Code for transcription regulators

o
z
I

Mutation in Homeobox HOXD-13 synpolydactyiy (extra fused digit between 3rd and 4th fingers)
Retinoic acid alters HOX gene expression

-rj D

n
- & >
to C D

D Q_

9 5
Copyright 2012. Doctors In Training.com. LLC. All Rights Reserved.

1 o

3 cr -.

[3]

other ndings would you expect to see?

9. List as many drugs as you can that are contraindicated in pregnancy.

During what weeks of fetal development does organogenesis take place? (FAI2pl24)

What is the relationship between the notochord, the neural plate, the neural tube, and the neural crest cells? (FA 12 p 125)

What is the embryologic origin of the following adult structures? (FAI2 pl26) Anterior pituitary
Cornea Lens Retina Olfactory epithelium Mammary glands Salivary glands Sweat glands

Copyright 2012. Doctors In Training.com. LLC. All Rights Reserved.

uestion Warm-Up
origin of the tissue just below the anal canal? (FAI2 pl26)

What effect might the following teratogens have on a developing fetus? (FAI2 pl27) ACE inhibitors Aminoglycosides Diethylstilbestrol Tetracyclines Valproic acid

Which vitamin should not be supplemented in large amounts during pregnancy? (FA I PI27)

Cell Cycle and Protein Organelles

Nuclear Localization Signals Amino acids sequences: 4-8 amino acids; rich in lysine, arginine, and protein Essential component of proteins bound for or residing in the nucleus (e.g., histones) Nuclear pores recognize these signals and transport proteins into the nucleus via ATPase A mutation in a single amino acid may prevent nuclear transport

5. Cell Cycle Basics (FA 11 p76) (R p86)


Cyclins + cyclin-dependent kinases (CDK) phosphorylate target proteins to drive the cell cycle All cyclins are degraded by ubiquitin protein ligase when their cell-cycle specific job is complete p2l, p27, and p57 bind to and inactivate cyclin-CDK complexes (p53 controls the activation of p2l)

Gl - S
Cyclin D binds/activates CDK4 * phosphorylation of Rb protein -> Rb protein is released from transcription factor E2F * with E2F unbound, the cell is free to transcribe/synthesize components needed for progression through the S phase (cyclin E, DNA polymerase, thymidine kinase, dihydrofolate reductase) Cyclin E binds/activates CDK2 * the cell is allowed to progress into S phase

G2-*M
Cyclin A - CDK2 complex - mitotic prophase Cyclin B - CDKI complex activated by cdc25 -* breakdown of nuclear envelope (nuclear lamin breakdown) and initiation of mitosis

Copyright 2012. Doctors In Training.com. LLC. All Rights Reserved.

What molecules provide the structural framework for DNA and the nuclear envelope?

What must be present on a protein in order for that protein to gain entry into the nucleu

Which types of proteins are responsible for fostering the progression through the cell cycle

Which cydin-CDK complexes assist in the progression from Gl phase to S phase?

Which tumor suppressor proteins prevent the progression of the cell into S phase?

Which cyclin-CDK complexes assist in the progression from G2 phase to M phase?

Protein Processing
Phys: Chapter 2 Endoplasmic Reticulum and Golgi S- Rough ER (FAII p76) _j Smooth ER (FAII p76) C Golgi apparatus (FAII p77) Q Cell Trafcking (FA/2 p80) Enzyme Terminology (FAII p95) _j Protein Degradation -= - Proteasome (FAI2 p80) - Lysosome

< L >

U 12.1-cell disease (FA 12 p80)


' Deficiency in mannose phosphorylation ( N no mannose-6-phosphate to target lysosomal proteins * secretion out of cell instead of into lysosomes 7 death by age 8 C (+) corneal clouding, coarse facies, HSM, skeletal abnormalities, restricted joint movement, +/- MR

13. Chaperones and Heat Shock Proteins:


Chaperones assist in the proper folding and transport of polypeptides across the ER, Golgi, and beyond. Some chaperones are synthesized constantly and are involved in normal intracellular protein trafcking. Others chaperones are induced by stress such as heat (heat shock proteins hsp70, and hsp90). These chaperones "rescue" shock-stressed proteins from misfolding. If the ) folding process is not successful, the chaperones facilitate degradation of the damaged protein. L- This degradative process often involves ubiquitin (also a heat shock protein), which is added to the abnormal protein and marks it for degradation by the ubiquitin-proteasome complex.
Copyright 2012. Doctors In Training.com. LLC All Rights Reserved.

End of Session Quiz


14. What molecule targets proteins in the endoplasmic reticulum for lysosomes?

15. What is decient in l-cell disease? (FAQ p80)

16. What are the different methods that a cell uses to break down proteins (proteolysis)?

17. Which cell types are rich in smooth ER?

o c
z

o
z in
to

n C D
o a . ( D
~ 5
Copyright 2012, Doctors In Training.com. LLC. All Rights Reserved.

[7]

Cytoskeleton and Other Cellular Components


Phys: Chapter 2 Cytoskeleton - Cytoskeletal elements (FAII p78) - Immunohistochemicalstains (FAII p78) - Microtubule (FAII p78) - Cilia structure (FAII p78) - Kartagener syndrome Free soluble cytoplasmic elements - Free ribosomes - Enzymes - Water, elements (potassium), organic molecules (amino acids, glucose) - Cytoskeleton Insoluble cytoplasmic inclusions

- Glycogen

- Lipid droplets Sarcoplasmic reticulum Mitochondria (Phys pl6) Peroxisome

4. Intermediate Filament Structures


i f t Intermediate Filaments Vimentin Support cellular membranes Keep certain organelles fixed in cytoplasm Desmin Cytokeratin \ Glial brillary acid proteins (GFAP) Peripherin Neurolaments (L, M, H (molectular wt) Nuclear lamins (A, B,C) Muscle cells (smooth, skeletal, heart) Epithelial cells (keratin in desmosomes and hemidesmosomes) Astrocytes, Schwann cells, other neuroglia Neurons Axons within neurons Structural Component of: Connective tissue (broblasts, leukocytes, endothelium)

1 3 U
I m m

e f e

z
.

D 0
_i_

Nuclear envelope and DNA within

Copyright @ 2012. Doctors In Training.com, LLC All Rights Reserved.

lick Quiz
What drugs act on microtubules WMMKrii

What two fundamental substances are required to make most things work inside the

Which organelle is responsible for the breakdown of very long-chain fatty acids?

What are the intermediate laments for the following tissue types and cellular structui Connective tissue Muscle tissue Epithelial tissue Axons What are the defects seen in Kartagener syndrome? (FAII p78)

Plasma Membrane
Phys: Chapters 2, 4 R: Chapter 2 Plasma membrane - Composition (FAII p78) - Sodium pump (FAII p79) - Sodium-mediated diffusion (Phys pS4) Arachidonic acid (FAII p39l) (R p58)

Copyright 3 2012. Doctors In Training.com. LLC. All Rights Reserved.

10. Label the steps outlining the derivatives of arachidonic acid.


cell membrane phospholipids

140

16 18

1 7

Blocks receptors to the agents on the left

o t5 c
LL_ "O C

11. Tyrosine kinase receptor


Transmembrane receptors that bind an extracellular ligand then intracellularly transfer a phosphate group (phosphorylate) from ATP to selected tyrosine side chains on specic cellular proteins including itself (autophosphorylation). The rst step in the signaling cascade that is initiated by tyrosine kinase receptors is autophosphorylation. PDGF and other growth factor receptors: single-pass transmembrane protein Insulin and IGF-1 receptors: - 2a subunits (bound by disulde bonds) - bind extracellular ligand - 23 subunits-tyrosine kinase activity

rd

tJ
3
i_

lo
i_

U
I m
CO

I z
D

o o

[10]

Copyright 2012. Doctors In Training.com. LLC. All Rights Reserved.

12. Clathrin

BBfa
|PElSf|
rtctPtor clathrin coat

Vesicle formation I

-.:

I*.
naked transport vesicle

'cargo molecule

dynamln

A) Normal LDL Receptor


LDL binding site ' LDL receptor protein

clathrin coated pit binding site

B) Mutant LDL Receptor

LDL receptor protein with abnormal coated pit binding site

A mutant LDL receptor lacks the coated-pit binding site but retains a functioning LDLbinding site. As a result, cells with mutant receptors are able to bind LDL normally but are unable to ingest it. Individuals with this mutation have a higher risk of dying prematurely from a myocardial infarction.

Copyright 2012. Doctors In Training.com. LLC. All Rights Reserved.

" . Which arachidonic acid product causes the following effect? (FAII p39l) Increased bronchial tone Decreased bronchial tone Increased platelet aggregation Decreased platelet aggregation Increased uterine tone Decreased uterine tone Increased vascular tone Decreased vascular tone 14. What are the two most abundant substances in plasma membranes? (FAI I p78)

. What drugs act on the arachidonic acid product pathway? What enzymes do they affect! (FAII p39l)

. What protein is involved in transporting an endocytosed vesicle from the plasma membrane to the endosome?

Copyright 2012. Doctors In Training com. LLC. All Rights Reserved.

Question Warm-Up
, /hat drugs interfere with microtubule functioning? (FAI2 p8l) Which cell types are constantly regenerating themselves due to an absence of the GO phase and a short Gl phase? (FAI2 p79)

Apoptosis and Necrosis


R: Chapter I
Apoptosis (FAII p220)

Intrinsic apoptosis vs. extrinsic apoptosis (FAII p220) (FAI2 p244) Necrosis (FAI I p220)

4. Mechanisms of Apoptosis (FAI I p220) (R p27)


Apoptosis is initiated when: (I) cells are deprived of important cell signals such as growth factors, (2) cell stress is present, (3) DNA damage is present and the DNA repair process fails p53 triggers apoptosis, (4) cytokines such as TNF trigger apoptosis, or (5) cytotoxic T cells insert granzyme B into cells * activation of caspases - If p53 is mutated or absent, it cannot induce apoptosis in the presence of severe DNA damage Caspase proteases execute apoptosis and have Cysteine protease that cleaves after aspartic acid residues Extrinsic Pathway (Death Receptor-Mediated) via activation of cell surface death receptor - Type I TNF receptor I (TNFRI) - Fas (CD95): Fas ligand binds Fas * grouping of 3 or more Fas molecules to form a binding site for FADD * Fas-associated death domain (FADD) binds inactive caspase-8 (caspase-10 in humans) * cleavage and activation of caspase-8 * cleaving and activation of other pro-caspases * apoptotic proteolytic cascade (pathway used in selection of T cells) - FLIP protein may bind to and inhibit cleavage of pro-caspase-8 thereby inhibiting apoptosis Intrinsic Pathway (Mitochondrial) - Increased mitochondrial permeability * release of pro-apoptotic molecules into the cytoplasm - Bcl-2 proteins regulate apoptosis. - Bcl-2 and Bcl-x prevent apoptosis (Bcl-2 is homologous to Ced-9) - Bak Bax, and Bim are pro-apoptotic - Cells undergo stress or are deprived of important cell signals -* Bcl-2 and Bcl-x are lost from mitochondrial membranes and are replaced with Bak, Bax, and Bim > mitochondrial membrane permeability increases * caspase activating proteins (i.e., cytochrome c) and AIF (apoptosis inducing factor) leak out - Cytochrome c binds cytosolic Apaf-I (apoptosis activating factor-1 which is homologous to Ced-4) * this complex activates caspase-9 - Apaf-1 may be directly inhibited by Bcl-2 and Bcl-x - AIF (apoptosis inducing factor) binds to and neutralize various inhibitors of apoptosis * apoptosis is no longer inhibited During apoptosis, substances that recruit phagocytes are secreted and marker molecules are placed on the cell surface so that phagocytosis can take place prior to necrosis and inammation. CD31 is expressed by healthy cells to prevent phagocytosis.
Copyright @ 2012. Doctors In Training.com, LLC. All Rights Reserved.

Cellular Injury
R: Chapter I Causes of cell injury (Rpll) Reversible vs. irreversible injury (FA 12 p245) Mechanisms of cell injury (R pi 7) - ATP depletion - Mitochondrial damage - Inux of calcium and loss of calcium homeostasis - Accumulation of oxygen-derived free radicals (oxidative stress) - Free radical injury (FAII p222) - Defects in membrane permeability - Damage to DNA and proteins Ischemia-hypoxia vs. ischemia-reperfusion - Infarcts: red vs. pale (and reperfusion) (FAII p22l)

c 9. What cellular byproducts might you detect in the serum when the following cell types are injured?)
Cardiac myocytes Skeletal myocytes Hepatocytes Salivary gland cells Pancreatic exocrine cells RBCs

. Ti P L
Which metals are known to facilitate the generation of oxygen free radicals? What cellular particles are responsible for handling oxygen free radicals? (R p2l) In order to prevent cellular death from cellular injury, what specic things would you nee to optimize to prevent the cellular injury in the rst place? (R pi 8)

Copyright 2012. Doctors In Training.com. LLC. All Rights Reserved.

3 Question Warm-Up
1. What are the characteristic features of a cell undergoing apoptosis? (FAQ p244)

2. What are some of the substances that can trigger apoptosis?

3. What are some of the cell types that are derived from the neural crest? (FA 12 pi 26) Peripheral Nervous System Ear Eye Adrenal Gland Mouth Heart Digestive System Thyroid Skin

Cellular Response I - Acute and Chronic Inammation


R: Chapter 2 Inammation (FAII p22l) - Acute inammation - Leukocyte extravasation (FAII p222) - Mediators of inammation (R p56) Patterns of inammation (R p66) - Serous - Fibrinous - Suppurative Transudate vs. exudate (FAII p223) Chronic inammation - Role of macrophages (R p7l) - Granulomatous diseases (FAII p223) (R p73) Markers of inammation - Erythrocyte sedimentation rate (FAII p223) - C-reactive protein (R p74) - Systemic effects (R p74)

o c
z
%

o
z c> / Ln
C D

o
I
C D

C. O

5'

Copyright @ 2012. Doctors In Training.com. LLC. All Rights Reserved.

[15]

4. Elevated ESR (FAI I p223, FAI2 p248)


Polymyalgia rheumatica Temporal arteritis Disease activity in RA and SLE Infection, Inflammation (e.g., osteomyelitis) Malignancy

5. C-Reactive Protein (R p498)


Acute-phase reactant synthesized by the liver Part of the innate immune response: opsonizes bacteria and activates complement Can be secreted from cells within atherosclerotic plaques to activate local endothelial cells to induce a prothrombotic state and increase the adhesiveness of the endothelium to leukocytes Elevations are a strong predictor of Ml. stroke, PAD, and sudden cardiac death CRP can be lowered by smoking cessation, exercise, weight loss, and statins

Cellular Response 2 - Control of the Extracellular Environment


R: Chapter 3 Collagen (R p95) Collagen synthesis and structure (FAI2 p83) Fibroblasts Osteogenesis imperfecta (FAI2 p83) Ehlers-Danlos syndrome (FAI2 p83) Alport syndrome (FAI2 p84) Elastin (FAI2 p84) Mechanisms of angiogenesis (R p99) Cutaneous wound healing (FAI2 p248) (R pl02)

10. Epithelial Wound Healing


0-3 hours - Hemorrhage and clotting 12-24 hours - Acute inflammation (PMN) 2-4 days - Macrophage infiltration and epithelial cell migration 3-5 days - Granulation tissue (especially at wound edges) Months - Collagen production (type III then type I)

Copyright 2012, Doctors In Traimng.com. LLC All Rights Reserved.

What is the underlying dysfunction in Chediak-

rome?(FAI2p8l)

How does having a high cholesterol content in the plasma membrane affect the function of the plasma membrane? (FA 12 p8l)

What area of the colon is most susceptible to ischemic damage? (FAI2 p245)

Cellular Response 3 - Internal Adaptations


R: Chapter /, 3
Strengthening - Hypertrophy vs. hyperplasia (R p6) Replacement - Stem cells (R p82) - Liver regeneration (R p93) Irreparable damage - Metaplasia

4. Where can tissue stem cells be found in the following locations?


Bone Liver Hematopoietic stem cells in the marrow Oval cells in the canals of Hering Neural precursor cells in the subventricular zone and dentate gyrus of the hippocampus Hair follicle bulge Interfollicular areas of the surface epidermis Sebaceous glands Deep within the intestinal crypts Satellite cells beneath the myocyte basal lamina Limbal stem cells at the cornea-epithelium junction

Brain
Skin

Intestine Skeletal muscle Cornea

Copyright 2012, Doctors In Training.com. LLC. All Rights Reserved.

ick Quiz
What female hormone is known for Does the compensatory growth of muscle bers occur primarily as a result of hyperpla or hypertrophy? (R p6) Does myometrial growth in pregnancy occur primarily as a result of hyperplasia or hypertrophy? (R p7) What organelle becomes hypertrophied in hepatocytes with chronic phenobarbftal use?(R p7) Which four transcription factors appear to be essential in pluripotential stem cells? (R p84 What can happen to the cells of the lower esophagus in response to chronic acid reux? (Rp 10)

Cellular Response 4 - Atrophy and Aging


R: Chapter I Atrophy (FAII p22l)(Rp9) Cellular aging and telomerase (R p39-40)

Quiz
What is actually occurring at the cellular level during atrophy? (R plO)

What is a lipofuscin granule? (R plO)

What enzyme mitigates the aging effects of cellular division by maintaining chromoson length? (R p40)

.. What is currently the known as the most effective way of prolonging life span? (R p4l]

15. What family of proteins is induced by caloric restriction? (R p4l)

Copyright 2012. Doctors In Traming.com. LLC. AH Rights Reserved.

uestion Warm-Up
What adult cell types arise from neural crest cells? (FAI I pi 19) (FAI2 pl26)

Which amino acids are modied in the Golgi apparatus? (FAI I p77) (FA12 p80)

Failure of what process results in l-cell disease? (FAI I p77) (FAI2 p80)

Organization of the Brain


R: Chapter 28 Embryology - Regional specication of developing brain (FAII pl26) (FA/2 pl33) - Neural tube defects (FAII pl26) (FAI2 pl33) (R pl284) - Forebrain anomalies (FAII pl27) (FAI2 pl34) - Posterior fossa malformations (FAII pl27) (FAI2 pl34) - Syringomyelia (FAII pl27) (FAI2 pl34) (R pl286) (H p3373)

What abnormalities are often found with an Amold-Chiari malformation? (FAI I pl2, (FAI2pl34)

What are the classic presenting symptoms of a syringomyelia? (FAI I pi27) (FA12 pi3'

What amniotic uid lab abnormality might point you to a diagnosis of anencephaly? pl27)(FAI2p!34)

Copyright 2012. Doctors In Training.com. LLC. All Rights Reserved.

Neurotransmitters
R: Chapter 28 GG: Chapter 14 Phys: Chapter 45 Cells of the nervous system (FAII p396) (FAI2 p434) Blood-brain barrier (FAII p398) (FAI2 p436) Neurotransmitters (FAII p397) (FAI2 p435) (GG p376) (Phys pSSO)

7. What are the 4 major dopaminergic pathways, and what is the result of blocking these pathways?

Major pathway Mesocortical pathway

Result of blocking

Mesolimbic pathway

Nigrostriatal pathway

Tuberoinfundibular pathway

8. What disorder is thought to arise from reduced norepinephrine activity? Increased norepinephrine activity?

9. What disease is associated with the degeneration of the basal nucleus of Meynert and less CNS acetylcholine?
to
"to CO

u
I

[2]

Copyright 2012. Doctors In Training.com. LLC All Rights Reserved.

d of Session Quiz
TA * J l i r f l i l l T J V i 9 l l U v A a C 1 l 4 I I H a * J I l l r i l f 4 i l <

(FAQ p434,436) Look like fried eggs under histologic staining Form multinucleated giant cells in the CNS when infected with HIV Myelinates multiple CNS axons Myelinates one PNS axon Damaged in Guillain-Barre syndrome Damaged in multiple sclerosis Macrophages of the CNS Cells of the blood brain barrier

In which neurological diseases is acetylcholine altered? (FAI I p397) (FAQ p435)

12. What is the main inhibitory neurotransmitter of the CNS? In which diseases are level altered? (FAI I p397) (FAI2 p435)

What are the components of the blood-brain barrier? (FAI I p398) (FAI2 p436)

Copyright & 2012. Doctors In Training.com. LLC. All Rights Reserved

L.fi'w-i u'Jiii/'/rfiin'i
Which neurotransmitters have altered levels ln*TiwSrH! isorder? (FAI I p397) (FAI2 p43!

What arachidonic acid product has actions that oppose that of prostacyclin? (FAI I p39l) (FAI2p429)

What organelle and cytochrome is particularly important in intrinsic apoptosis? (FAIIp220)(FAI2p244)

Hypothalamus
Phys: Chapter 58 Hypothalamus (FAII p398) (FAI2 p436) (Phys p7l5) Leptin (FAI2 p436) (Phys p846) Posterior pituitary (FAII p398) (FAI2 p436) Oxytocin (Phys p90S) Melatonin

Which nucleus of the hypothalamus ts the following description? (FAI I p398) (FAI2 p43 Considered the "master clock" for most of our circadian rhythms Regulates the parasympathetic NS Destruction results in hyperthermia Regulates the sympathetic NS Produces antidiuretic hormone (ADH) to regulate water balance Receives input from the retina Savage behavior and obesity result from stimulation Savage behavior and obesity result from destruction Stimulation leads to eating and destruction leads to starvation Regulates the release of gonadotropic hormones (i.e., LH and FSH) Responsible for sweating and cutaneous vasodilation in hot temperatures Responsible for shivering and decreased cutaneous blood flow in the cold Destruction results in neurogenic diabetes insipidus Destruction results in inability to stay warm Releases hormones affecting the anterior

Copyright 2012. Doctors In Training.com, LLC. All Rights Reserved.

Hypothalamus
Phys: Chapter 59 H: Chapter 27 Sleep hygiene (Hp216) Sleep stages (FAII p62) (FAI2 p64) (Phys p72l) (H p2l3) REM sleep (FAIf p62) (FAI2 p64) (Phys p72l) Sleep patterns of depressed patients (FAII p62) (FAI 2 p64) Insomnia (H p2l6) - Nonbenzodiazepine hypnotics (FAII p433) (FA12 p47S) Restless legs syndrome (H p2l8) Narcolepsy (FAII p63) (FAI2 p65) (H p220) Circadian rhythm (FAII p63) (FAI2 p65) (H p2l4,221) Sleep terror disorder (FAII p63) (FA/2 p65) (H p22l) Nocturnal enuresis (H p22l)

5. What are the components of good sleep hygiene in the treatment of insomnia? (H p2l6)
Keep a consistent sleep schedule and avoid daytime naps Dark, quite room - The darker the room, the better. Wear sleep masks and earplugs if necessary Limit caffeine and alcohol at least 4-6 hrs prior to bedtime Avoid TV, computer, and video games for at least 1-2 hrs prior to bedtime Exercising early in the day Cool temperature Reserve bed for sleep and sex only Avoid pets in bed

Avoid eating before bedtime

What medications are common in the treatment of insomnia? What makes each one unique?
Melatonin Valerian Antihistamines (Benadryl, Tylenol PM, doxylamine) Trazodone TCAs such as amitriptyline, doxepin Long acting benzodiazepines such as temazepam, lorazepam, clonazepam, diazepam, chlordiazepoxide Zolpidem (Ambien), zaleplon (Sonata) Eszopiclone (Lunesta) Ramelteon (Rozerem) Non-addictive, OTC, vivid dreams, safe for < 3 months OTC herbal remedy, studies show no benet Commonly used by patients rst-line, associated with poor sleep quality, not for long-term use, anticholinergic side effects (avoid in the elderly) Antidepressant, increases REM sleep, small risk of priapism Antidepressant, small risk of arrhythmias (obtain EKG prior to use), anticholinergic side-effects (avoid in the elderly) Addictive, short-term only (< 35 days) ^<

m C

"D

a
3 c
Q_ (/} fD fD

Act at the benzo receptor, short-term only (< 35 days), rebound insomnia when discontinued May be used long-term (FDA 2004) Non-addictive because it works at melatonin receptors instead of GABA/benzo receptors; avoid if hepatic insufciency; long-term studies are lacking

Copyright 2012, Doctors In Training.com. LLC. All Rights Reserved.

[5]

7. Restless legs syndrome (H p2l8)


The sensation of unpleasant paresthesias that compels the patient to have voluntary, spontaneous, continuous leg movements Usually a primary, idiopathic disorder Secondary RLS can result from iron defitiency, end-stage renal disease, diabetic neuropathy. Parkinson disease, pregnancy, rheumatic diseases (RA), varicose veins, caffeine intake, etc. Treatment pramipexole or ropinirole qHS (or levodopa/carbidopa), iron replacement, avoid caffeine, donazepam qHS, gabapentin, opioids

8. What is the treatment for narcolepsy? (H p220)


Avoidance of drugs that cause sleepiness Scheduled naps (once or twice a day for 10-20 min) Stimulants - modafinil is first-line Support group attendance If cataplexy then use venlafaxine, fluoxetine, or atomoxetine Sodium oxybate (GHB) can assist in sleep and reduce cataplexy

9. At what age should nocturnal enuresis be treated? What are the treatment options?
Enuresis cannot be diagnosed until 5 years of age (chronological and developmental) Treatment is usually delayed until the child is at least 7 years of age First-line: behavioral interventions: - Start toilet training if not yet attempted - Motivational therapy (e.g., star charts) - Restrict uids before bed (with a compensatory increase in daytime uids) - Night time chaperone to the toilet or scheduled wakening to avoid using alarm clock - Enuresis alarm (pad with alarm device) in bed for dassic conditioning. This is most effective n l o n g - t e r m t h e r a p y. ^ Second-line: pharmacologic interventions: 7J} - High likelihood of recurrence upon discontinuation -q - Imipramine (Tofranil) for short-term (up to 6 weeks) C - Desmopressin (DDAVP) orally (FDA 2007 * intranasal desmopressin is no longer indicated for enuresis due to risk of hyponatremic seizures) D - Indomethacin suppository rtJ

>v

CN

D LU

Z
L J

Copyright

2012.

Doctors

In

Training.com,

LLC. All

Rights

Reserved.

of Session Q\
10. During what sleep stage would a man have variable BP, penile tumescence, and van^-

at drugs are used to shorten Stage 4 sleep?

at is the "key" correspond to the different prindple neurotransmitter involved in tat EEG waveforms to initiating sleep? What is the stages of sleep?

What is the pathway by which retinal information induces the release of melatonin? (FAII p63) (FAI2p65)

Copyright CO 2012. Doctors lnTraining.com, LLC. All Rights Reserved.

i#JTTJi ? wTm /i77i7n*


What brain structure is resp (FAII p62) (FAI2p64)

Which areas of the hypothalamus regulate the autonomic nervous system? (FAII p398) (FAI2p436)

Clefts, Arches, Pouches


Branchial apparatus (FAII pl27) (FAI2 pl35) Branchial cleft derivatives (FAII pl27) (FAI2 pl36) Branchial pouch derivatives (FAII pl27) (FAI2 pl36) Cranial nerve labeling

Cranial Nerves
COA: Chapter 7 Brain stem (FAII p4l5) (FAI2 p454-45S)

4. Label the cranial nerves as they come of the brainstem:

Superior cerebellar peduncle

%?i
Middle cerebellar peduncle

Optic tract Cerebral peduncle

f \ 2 4
Pyramids

Dorsal

Ventral
Copyright <." 2012. Doctors In Tratntng.com. LLC AH Rights Reserved.

uick Quiz
5. What nerves innervate the branchial arches? Later what structures are derived from these arches?

6. From which branchial pouch are the following structures derived? Middle ear and eustachian tubes Superior parathyroids Inferior parathyroids Epithelial lining of the palatine tonsil Thymus 7. Which cranial nerve is responsible forthe following actions? (FAI I p4l6) (FAI2 p456) Eyelid opening Taste from anterior 2/3 of tongue Head turning Tongue movement Muscles of mastication Balance Monitoring carotid body and sinus chemo- and baroreceptors

Cranial Nerves
COA: Chapter 7 Extraocular muscles and nerves (FAII p42l) (FAI2 p46l) (COA p898) Testing extraocular muscles (FAII p422) (FA/2 p462) Cavernous sinus (FAII p4l8) (FAI2 p458) Trigeminal nerve - Mastication muscles (FAII p4l9) (FAI2 p459) (COA p92l) Testing cranial nerves (H p3236) - KLM sounds (FAI I p4l9) - Cranial nerve reexes (FAII p4l6) (FAI2 p456) Cranial nerve review (FAII p4l6) (FAI2 p456)

Copyright 2012. Doctors In Traimng.com, LLC. All Rights Reserved.

8. Cavernous sinus

Lippmcott Williams & Wilkins Atlas of Anatomy 2C08 Wolters Kluwer Health. Inc. All rights reserved.

Specialty Centers in the Brainstem


"^ Reticular activating system ' C Sympathetic chain vs. parasympathetic ofCN III, VII, IX, X (COA p57-64) rrj Vagal nuclei (FAII p4l7) (FAI2 p457) L- Horner syndrome (FAII p4l3) (FAI2 p453) (COA p9l3)

Copyright 2012. Doctors In Training.com. LLC. All Rights Reserved.

on Quiz
^^^M^^^^^^S^^^SmSuBmvemoS^SSMuSe^^^&^^^^S^^^^^^^^^^^^^n

ich cranial nerves have their nuclei in the pons? (FAI I p4l6) (FAQ p456)

Which cranial nerves have their nuclei in the midbrain? (FAI I p4l6) (FAQ p456)

What information is communicated at the nucleus solitarius? Nucleus ambiguus? Dors motor nucleus?

A woman involved in an accident cannot turn head to the left and has a right shoulde droop. What structure is damaged?

14. A 19-year-old patient presents with a furuncle on his philtrum, and the cavernous sim becomes infected. What neurological decits might you see in this patient?

What are the muscles of mastication? (FAI I p4l9) (FAI2 p459)

Copyright 2012. Doctors In Training.com. LLC. All Rights Reserved.

UiriMBIi?Jwil TOI<aWin?i^ii^^w*^Bii^rSTiTfiTOilM leukemia In a population of 1,000 hotel beach umbrella monitors on a dean beach, 50, develop leukemia What is the attributable risk? (FAI I p54) (FAI2 p54) Which antihypertensive drug ts the following side effects? (FAI I p280) (FAI2 p306) First dose orthostatic hypotension Hypertrichosis Cyanide toxicity Dry mouth, sedation, severe rebound HTN Bradycardia, impotence, asthma exacerbation Reflex tachycardia Metabolic alkalosis Elevated anti-histone antibodies Hypercalcemia An abdominal aortic aneurysm is most likely a consequence of what process? (FAQ p295)

Cerebral Cortex
Phys: Chapter 57 Cerebral cortex functions (FAII p402) (FAI2 p440) (Phys p698) Homunculus (FA11 p403) (FAI2 p440) Cerebral arteries - cortical distribution (FAII p404) (FAI2 p442) Overview of cerebral areas - Common brain lesions (FAII p403) (FAI2 p44l) Aphasia (FAII p404) (FAI2 p442) (Phys p699) (H p202)

4. Non-Dominant Aphasia Non-dominant Broca (Expressive Dysprosody) - inability to express emotion or inflection in speech Non-dominant Wernicke (Receptive Dysprosody) - inability to comprehend emotion or inflection in speech

Copyright IS 2012. Doctors In Training.com. LLC. All Rights Reserved.

Hemispatial neglect syndrome Poor repetition Poor comprehension Poor vocal expression Personality changes Dysarthria Agraphia and acalculia Hyperorality, hypersexuality, disinhibrted behavior A patient with a cortical lesion is unaware of his neurologic deciency. Where is the lesion?

What typically is the cause of a lesion to the mamillary bodies?

Cerebral Circulation
COA: Chapter 7 Carotid arteries Carotid stenosis Vertebral arteries Circle of Willis (FAII p405) (FAI2 p443) (COA p883-885)

8. Label the following diagram of the Circle of Willis:

Copyright K) 2012. Doctors In Training.com. LLC. All Rights Reserved

[14]

Copyright 2012. Doctors In Training.com, LLC. All Rights Reserved.

Question Warm-Up
What heart defect is associated with the following disorder? (FAI I p269) (FAQ p294) Chromosome 22q 11 deletions Down syndrome Congenital rubella Turner syndrome Marfan syndrome

in which section of the brainstem is each of the cranial nerve nuclei located? (FAI I p4 (FAQ p456)

What information is communicated at the nucleus solitarius? Nucleus ambiguus? Doi motor nucleus?

Anatomy of the Brainstem


COA: Chapter 7 Pons Midbrain Medulla

Copyright 2012. Doctors In Training.com. LLC. All Rights Reserved

4. Midbrain

5. Pons

c o 'tj
m
C D

2 u
E tJ

C O C O

6. Medulla

e " CD
I

2 D
[16]
Copyright 2012. Doctors In Training.com, LLC. All Rights Reserved.

ick Quiz

Where does the corticospinal tract decussate in the brainstem? Where does the dors column-medial lemniscal pathway decussate? (FAI I p4IO) (FAQ p450) What artery supplies the following medullary structures: spinal thalamic tract, inferior cerebellar peduncle, nucleus ambiguus, and lateral spinothalamic tract?

Cranial Nerve Lesions


H: Chapter 376 Circle of Willis lesions (FAII p405) (FAI2 p444) Cranial nerve lesions (FAII p4l8) (FAI2 p458) Facial lesions (FAII p4l9) (FAI2 p459) (H p3360)

10. If the right vagus nerve or nuclei is damaged, then to which side will the uvula deviate?
Since the muscles of the left soft palate are working to raise the palate and the muscles on the right are not, the uvula will deviate to the left (opposite the side of the lesion).

11. If the portion of the right motor cortex (or right corticobulbar tract) that innervates the soft palate is damaged, to which side will the uvula deviate? Since the soft palate bers from the right motor cortex (or right corticobulbar tract) travel to the left nucleus ambiguus, the uvula will deviate to the right (toward the side of the lesion). 12. A patient comes to your ofce and before you notice any other symptoms, you see that the patient's uvula deviates to the left when she says "Ah." What neurological areas might be damaged in order for this abnormality to be seen?
(Work backward to gure these problems out) left deviation of uvula means that the muscles of the left palate are raising the palate and the muscles of the right palate are not. What innervates the right soft palate? - Right vagal nerve (CN X) - Right nucleus ambiguus - Left corticobulbar tract - Soft palate portion of left motor cortex

13. If the right hypoglossal nerve or nuclei is damaged, then to which side will the tongue deviate when sticking-out?
Since the tongue is "pushed" out, the muscles on the functional side (in this case the left side) will "push" the tongue to the non-functional side (in this case the right side) (toward the side of the lesion).

14. If the portion of the right motor cortex (or right corticobulbar tract) that innervates the tongue is damaged, to which side will the tongue deviate?
Since tongue bers from the right motor cortex travel to the left hypoglossal nucleus, the tongue will deviate to the left (away from the side of the lesion).
Copyright 2012. Doctors In Training.com. LLC. All Rights Reserved.

15. A patient comes to your ofce and before you notice any other symptoms, you see that the patient's tongue deviates to the left when he sticks out his tongue. What neurological areas might be damaged in order for this abnormality to be seen?
(Work backward to gure these problems out) left deviation of the tongue means that the muscles of the right tongue are pushing the tongue out unopposed by the muscles of the left tongue. What innervates the left tongue? - Left hypoglossal nerve (CN XII) - Left hypoglossal nudeus (anterior spinal artery vertebral artery) - Right corticobulbar tract - Tongue portion of the right motor cortex

16. How do the symptoms of a lesion to the cortical motor region of the face differ from a lesion of the facial nerve or nucleus? (FAI I p4l9) (FAQ p459)
Lesion in cortical motor face region - Paralysis of contralateral side of lower face Lesion of facial nerve or nudeus - Paralysis of ipsilateral side of entire face 17. Explain the above answer. The facial motor nudeus receives motor bers for the lower face from the opposite motor cortex and motor bers for the upper face from both motor cortices. Therefore, if a lesion occurs in the facial region of the left motor cortex, there is still sufcient innervation for the right upper face from the right motor cortex. However, since the left motor cortex is the only cortex to innervate the right lower face, there will be paralysis in the right lower face.

18. What name is given to a collection of symptoms indicative of a lesion of the facial nerve or nucleus?

o
$ 19. What diseases might have Bell's palsy as a complication? (FAI I p4l9) (FAQ p459)
to Lyme disease O * Herpes zoster (j AIDS _ Sarcoidosis ' Tumors C * Diabetes 3 (hint My Lovely Bella Had An STD) C
i_

D LU
[18]

Copyright 2012. Doctors In Traimng.com. LLC. All Rights Reserved.

0. How can a stroke of the facial motor cortex be distinguished from Bell's palsy?

iat should you immediately think about in a patient with "bilateral Bell's palsy"?

How do the symptoms of a lesion to the cortical motor region of the face differ from lesion of the facial nerve or nucleus? Lesion in cortical motor face region- Lesion of facial nerve or nucleus-

3. A patient comes to your ofce and before you notice any other symptoms, you see th the patient's uvula deviates to the right when she says "Ah." What neurological areas might be damaged in order for this abnormality to be seen?

4. A patient has leftward deviation of the tongue on protrusion and has a right sided spastic paralysis. Where is the lesion?

j. A patient cannot blink his right eye or seal his lips. What is the diagnosis and which ne is affected?

Copyright'<) 2012. Doctors In Training.com. LLC. All Rights Reserved.

Which type of vasculitis ts the following description? (FAI I p277-279) (FAQ p304-305) Necrotizing granulomas of lung and necrotizing glomerulonephritis Necrotizing immune complex inflammation of visceral/renal vessels Young Asian women Young asthmatics Infants and young children; involved coronary arteries Most common vasculitis Associated with hepatitis B infection

What portion of the brain is supplied by the anterior cerebral artery? Middle cerebral artery? (FAI I p404) (FAQ p442)

How does the presentation of a right parietal lobe lesion differ from the presentation of a left parietal lobe lesion? (FAI I p403) (FAQ p44l)

Brainstem Occlusion Syndromes I


H: Chapter 370 Effects of strokes (FAII p405) (FAI2 p444) (H p3284) 4. What is the hallmark sign of a general brainstem lesion? Alternating syndromes: with long tract symptoms on one side (i.e., hemiparalysis) and cranial nerve Symptoms on the other 5. What causes and what are the symptoms of Weber syndrome? Midbrain infarction resulting from occlusion of the paramedian branches of the posterior cerebral artery: - Cerebral peduncle lesion contralateral spastic paralysis (AKA contralateral hemiparesis) - Oculomotor nerve (CN III) palsy ipsilateral ptosis, pupillary dilation, and lateral strabismus (eye looks down and out) 6. What causes and what are the symptoms of medial medullary syndrome? Damage to which areas cause these symptoms? Caused by occlusion of a paramedian branch of anterior spinal artery (from vertebral artery) unilateral infarct of medial portion of rostral medulla (AKA anterior spinal artery syndrome) - Contralateral spastic hemiparesis (pyramid/corticospinal tract damage) - Contralateral tactile and kinesthetic defects (medial lemniscus damage) - Tongue deviates toward side of the lesion (hypoglossal nucleus/nerve damage) - (Note that pain and temperature sensation are preserved)

Copyright fc> 2012. Doctors In Training.com, LLC. All Rights Reserved.

7. What causes and what are the symptoms of lateral medullary syndrome (AKA Wallenberg syndrome)? Damage to which areas cause these symptoms? Caused by occlusion of one of the posterior inferior cerebellar arteries (PICA) unilateral infarct of lateral portion of rostral medulla (AKA posterior inferior cerebellar artery syndrome) - Loss of pain and temp, over contralateral body (spinothalamic tract damage) - Loss of pain and temp, over ipsilateral face (trigeminothalamic tract damage) - Hoarseness, difculty swallowing, loss of gag reex (nucleus ambiguus: glossopharyngeal and vagus damage) - Ipsilateral Horner syndrome (descending sympathetic tract) - Vertigo, nystagmus, nausea/vomiting (vestibular nuclei damage) - Ipsilateral cerebellar decits (i.e., ataxia, past pointing) (inferior cerebellar peduncle damage)

Brainstem Occlusion Syndromes 2


H: Chapter 370 MLFsyndrome (FAII p424) (FAI2 p464) (H p239) Locked-in syndrome (H p2247) 10. What is the cause of medial inferior pontine syndrome? What are the symptoms? ", Damage to which areas cause these symptoms? ( Caused by occlusion of a paramedian branch of the basilar artery unilateral infarct of medial aspect of Q\ inferior pons | - Contralateral spastic hemiparesis (corticospinal tract damage) / - Contralateral loss of light touch/vibratory/kinesthetic sensation (medial lemniscus damage) ( - Paralysis of gaze to side of lesion (damage to pontine gaze center: PPRF and abducens nucleus) - Ipsilateral paralysis of lateral rectus muscle (damage to abducens nerve bers) | (Note that pain and temperature sensation are preserved) !

Copyright 2012. Doctors In Training.com. LLC. All Rights Reserved

What is the cause of lateral inferior pontine syndrome? What are the symptoms? Damage to which areas cause these symptoms?
Caused by occlusion of one of the anterior inferior cerebellar arteries (AICA) unilateral infarct of lateral aspect of inferior pons (AKA anterior inferior cerebellar artery syndrome) - Ipsilateral facial nerve paralysis (facial nucleus and nerve ber damage) - Ipsilateral loss of taste from anterior 2/3 of tongue (solitary nucleus and nerve bers) - Ipsilateral deafness and tinnitus (cochlear nucleus and nerve ber damage) - Nystagmus, vertigo, nausea/vomiting (vestibular nucleus and nerve ber damage) - Ipsilateral limb and gait ataxia (damage to middle and inferior cerebellar peduncles) - Ipsilateral loss of pain and temperature sensation from the face (spinal trigeminal nucleus and nerve ber damage) - Contralateral loss of pain and temperature sensation (damage to spinothalamic tract) - Ipsilateral Horner syndrome (damage to descending sympathetic tract) - (No contralateral body paralysis or loss of light touch/vibratory/kinesthetic sensation)

superior pontine syndrome. What are the symptoms of this syndrome? Damage to which areas cause these symptoms?
- Ipsilateral loss of taste from anterior 273 of tongue (solitary nucleus and nerve bers) - Ipsilateral limb and gait ataxia (damage to middle and inferior cerebellar peduncles) - Ipsilateral loss of pain and temperature sensation from the face (spinal trigeminal nucleus and nerve ber damage) - Ipsilateral loss of light touch and vibration sensation from face (main sensory trigeminal nucleus and nerve ber damage) - Ipsilateral jaw weakness and deviation of jaw toward side of lesion (trigeminal motor nucleus and nerve ber damage) - Contralateral loss of pain and temperature sensation from body (damage to spinothalamic tract) - Ipsilateral Horner syndrome (damage to descending sympathetic tract)

A patient cannot abduct her left eye on lateral gaze and convergence is normal. She is also having difculty smiling. In what part of the CNS is there a lesion?

A lesion of what artery can cause a locked-in syndrome?

.-.. is the clinical presentation of a patient with internudear ophthalmoplegia? (FAI I p424) (FAQ p464)

Copyright 2012. Doctors In Training.corn. LLC. All Rights Reserved.

On auscultation of a patient, you hear a pansystolic murmur at the apex with radiation the axilla. What is the most likely cause of this murmur? (FAI I p259) (FAQ p285)

Your study shows that high LDL does not increase one's risk of CAD. What type of error is this? (FAI I p56) (FAQ p57)

What medication inhibits alcohol dehydrogenase? Which medications inhibit acetaldehyde dehydrogenase? (FAI I p246)

Arterial Events
R: Chapter 28 H: Chapters 370, 378 Aneurysms (FAII p405) (FAI2 p445) Intracranial hemorrhage (FAII p406) (FAI2 p446) (R pl288, 1295) (H p3293) - Subarachnoid hemorrhage - Epidural hematoma (H p3379) - Subdural hematoma (H p3379) - Parenchymal hemorrhage (H p3294) - Interventricular hemorrhage Ischemic brain disease (FAII p407) (FAI2 p447) (H p327l)

4. Intraventricular hemorrhage in the newborn Hemorrhage into the ventricular system Most common in premature/very low birth weight infants (< 32 wks, <l500g) within the first 72 hours of life Originates from the germinal matrix in the subependymal, subventricular zone that gives rise to neurons and glia during development All infants born at younger than 30-32 wks gestational age should receive a screening US to detect

Copyright 2012, Doctors In Training.com. LLC. All Rights Reserved.

with berry aneurysms? A 28-year-old woman is involved in a motor vehicle accident (MVA). She initially feels ne, but minutes later she loses consciousness. CT scan reveals an intracranial hemorrhage that does not cross suture lines. Which bone and vessel were injured in the crash? A 40-year-old man with a history of Marfan syndrome and hypertension presents with a severe headache. A head CT is normal at presentation and examination of the CSF reveals numerous red blood cells. What is the cause of the man's headache? An 85-year-old man with Alzheimer disease falls at home and presents 3 days later with severe headache and vomiting. What is the most likely diagnosis and structures were damaged?

Venous and Ventricular Events


R: Chapter 28 Dural venous sinuses (FAII p407) (FAI2 p447) Ventricular System (FAII p408) (FAI2 p448) (COA p878) Hydrocephalus (FAII p408) (FAI2 p448) (R pi283) Pseudotumor cerebri (H pi26)

9. Circulation of CSF (COA p88l)

Lippincott Williams 6 Wilkms Atlas of Anatomy 200S Wolters Kluwer Health. Inc. All rights reserved.

Copyright 2012, Doctors In Training.com. LLC. All Rights Reserved.

PSEUDOTUMOR CEREBRI (AKA Benign Intracranial Hypertension) 10. What are the characteristic features of pseudotumor cerebri?
Young, obese female Headaches - daily (worse in the morning) pulsatile, possible nausea/vomiting, possible retroocular pain worsened by eye movement Papilledema Most worrisome sequelae is vision loss CT scan: Absence of ventricular dilation, no tumor or mass

What treatment options are available for managing pseudotumor cerebri? Confirm absence of other pathology with CT and MRI of the head (r/o central venous thrombosis) Discontinue any inciting agents (e.g. vitamin A. tetracyclines, corticosteroid withdrawal) Weight loss in obese patients Acetazolamide - first line (start 250mg qid or 500mg bid * increase to 500mg qid to lOOOmg qid) Invasive treatment options - Serial lumbar punctures - Optic nerve sheath decompression - Lumboperitoneal shunting (CSF shunt)

Where is CSF generated? Where is CSF reabsorbed? (FAI I p408) (FAQ p448)

14. What is the difference between communicating and noncommunicating hydrocephalus? (FAII p408) (FAQp448)

clinical features characterize normal pressure hydrocephalus? (FAI I p408) (FAI2p448)

Copyright 2012. Doctors In Training.com, LLC. All Rights Reserved.

What are ve hereditary thrombosis syndromes? (FAI I p356) (FAQ p388)

Which cancer drugs are known for being cardiotoxic? (FAI I p36l, 363) (FAQ p400)

Movement Execution
Phys: Chapters 54, 55 Lower motor neurons -Reex arc (P p658) - Clinical reexes (FAII p4l4) (FAI2 p454) - Primitive reexes (FAII p4l4) (FAI2 p454) Upper motor neurons - Motor cortex (Phys p667) - Thalamus (FAII p399) (FA12 p437)

Copyright 2012. Doctors In Training.com. LLC. All Rights Reserved.

4. Muscle Spindle Control (Phys P657, 661) Muscle spindle: monitors muscle length Extrafusal muscle fibers: functional unit of muscle Intrafusal muscle fibers: regulate length - Muscle stretch results in intrafusal stretch which stimulates la afferent * DH alpha motor neuron, causing reex muscle contraction - Hint: help you pick up a heavy suitcase Golgi tendons: monitor tension rather than length (perpendicular to intrafusal muscle fibers) - Provide inhibitory lb afferent feedback - Hint: cue you to drop a heavy suitcase Gamma loop: regulates sensitivity of reex arc CNS the Y motor neuron contracts intrafusal fiber (central part of spindle), increasing the sensitivity of the reex arc

Muscle Spindle, Gamma Loop, & Golgi Tendon Organ


Golgi tendon Descending pathway Intrafusal muscle bers Muscle spindle Extrafusal muscle

bers

5. What portion of the thalamus relays the following information? (FAI I p399) (FAI2 p437) Somatosensory from body (via medial lemniscus and spinothalamic) Communications with prefrontal cortex; memory loss results if destroyed Cerebellum (dentate nucleus) and basal ganglia motor cortex Trigeminothalamic and taste pathways to somatosensory cortex Retina occipital lobe Basal ganglia prefrontal, premotor, and orbital cortices Mamillothalamic tract cingulate gyrus (part of Papez circuit) Integration of visual, auditory, and somesthetic input Dentate nucleus and basal ganglia * supplementary motor cortex (Auditory info) brachium of inferior colliculus primary auditory cortex

Copyright .' 2012. Doctors In Training.com. LLC. All Rights Reserved.

6. What are the differences b motor pathway? 7. What portion of the thalamus relays the following information? (FAI I p399) (FAQ p437) Somatosensory from body (via medial lemniscus and spinothalamic) Cerebellum (dentate nucleus) and basal ganglia motor cortex Trigeminothalamic and taste pathways to somatosensory cortex Retina occipital lobe Basal ganglia * prefrontal, premotor, and orbital cortices (Auditory info) Brachium of inferior colliculus primary auditory cortex What reexes might you check for in a newborn exam to assess an infant for healthy neurological function? (FAI I p4l4) (FAQ p454)

Modiers of Movement
Phys: Chapter 56 Lateral corticospinal tract (FAII p4IO) (FAI2 p450) (Phys p669) Motor neuron signs (FAII p4IO) (FAI2 p450) Cerebellum (FAII p399) (FAI2 p437) (Phys p68l) Tremor (FAII p402) (FAI2 p439) (H p3327)

CEREBELLUM (FAII p399) (FAQ p437) (Phys p682) 9. What are the longitudinal zones of the cerebellum starting with the most medial? Vermis Intermediate (paravermal) zones (right and left) Lateral hemispheres (right and left)
Lateral Intermediate Eart of part of emisphere He

Anterior Lobe,

Anterior Lobe

.fco.

y Primary Fissure Midbrain


'Culmen' I I
i i

Posterior Lobe

I Decllve i Posterior Lobe

I Folium I

Medulla

Flocculonodular Lobe Posterolateral Fissure " Flocculonodular Lobe

Flocculus

Copyright 2012. Doctors In Training.com. LLC. All Rights Reserved.

10. Simplied Transverse Section from the Cerebellar Cortex

Molecular Layer

Stellate Basket

v-r*i

eTT^ v

~^M-

Purkinje Layer

Purkinje w

Granule Layer

Golgi #- <# Granular

IPSP to deep Climbing bers cerebellar nuclei and vestibular nuclei

Mossy bers

11. Describe the general ow of information through the cerebellum. Inputs (mossy and climbing fibers) cerebellar cortex * Purkinje fiber * deep nuclei of cerebellum output targets

c
oo

Copyright 2012. Doctors In Training.com, LLC. All Rights Reserved.

[29]

12. What structure provides the major output pathway from the cerebellum?
Brachium conjunctivum (AKA superior cerebellar peduncle) contralateral VL of thalamus

13. Based on the primary source of information brought into the cerebellar cortex, which cerebellar regions are referred to as the vestibulocerebellum, spinocerebellum, and cerebrocerebellum? (To which deep nuclei do these regions project?)
Vestibulocerebellum - Flocculonodular lobe and vermis ( fastigial) Spinocerebellum - Vermis and paravermal regions ( fastigial and interposed) Cerebrocerebellum - Lateral hemispheres ( dentate) Spinocerebellum

Intermediate hemisphere .^brocerebellum (lateral hemisphere)/' ,

Denate nucleus Interposed

H
Vestibulo cerebellum Balance and eye movement

To m o t o r To l a t e r a l To m e d i a l and premotor descending descending To vestibular cortices systems systems nuclei Motor planning Motor execution

14. Motor control on which side of the body would be affected with a lesion on one side of the cerebellar hemisphere?
Motor control ipsilateral to the side of the lesion would be affected (because the cerebellum contralateral thalamus cortex corticospinal tract body contralateral to cortex).

15. What neurological abnormalities can be attributed to damage of the spinocerebellum (vermis and paravermis)?
Postural instability Slurred/slowing of speech

Hypotonia Pendular knee jerk reflexes

Copyright S3 2012, Doctors In Training.com. LLC. All Rights Reserved.

16. What symptoms are seen in anterior lobe (anterior vermis) syndrome? What is the most common cause of anterior lobe syndrome? The most anterior portion of the vermis belongs to the legs. Ataxia/dystaxia of legs (even when the trunk is supported) broad-based, staggering gait Chronic alcohol abuse * thiamine deficiency degeneration of cerebellar cortex (starting at the anterior lobe) 17. What neurological decits can be attributed to damage of the cerebrocerebellum (lateral hemisphere)? Lack of coordination of voluntary movements (with respect to both the timing and rate of the movement) Delays in initiating movements and trouble stopping movements Dysmetria (impaired ability to control the distance, speed, and power of a movement) Intention tremor

TREMOR (FAI I p402) (FAI2 p439) 18. What is the difference between essential tremor, resting tremor, and intention tremor? (FAI I p402) Essential tremor- family history of tremor, occurs with movement and at rest Resting tremor- associated with Parkinson disease, disappears with voluntary movement Intention tremor- assodated with cerebellar damage, appears only with voluntary movements 19. What are the features of essential (AKA familial) tremor? Rapid fine tremor of head, hands, arms, and/or voice (think Katharine Hepburn) 50% of patients have a family history of tremor Occurs with movement and at rest Treated with beta-blocker (propranolol), primidone (an anticonvulsant), clonazepam, or alcohol (pts often self medicate) 20. What neurological decits can be attributed to damage of the vestibulocerebellum (vermis and occulonodular)? Disequilibrium: difficulty in maintaining balance Abnormal eye movements (such as cerebellar nystagmus that is more pronounced when patient looks Z. to the side of the lesion) Q] 21. What is the most common cause of damage to the occulonodular lobe? Q Medulloblastoma in childhood CO

z o
$
1 3

3 C D

Copyright 2012. Doctors In Training.com. LLC. All Rights Reserved.

[31]

raHlRraRBwWTP!
to affect movement? (FAI I p4IO) (FAI2 p450)

What diseases specically damage the area above? (FAI I p4l I) (FAQ p45l)

What are classic signs of an upper motor neuron lesion? Of a lower motor neuron lesion? (FAII p4IO)(FAI2p450)

What is the difference between essential tremor, resting tremor, and intention tremor? (FAI I p402) (FAI2 p439)

Copyright 2012, Doctors In Trainmg.com. LLC. All Rights Reserved.

estion WarmWhat is the rate-limiting step in heme synthesis? (FAI I p354) (FAQ p386)

What is the structure of HbH? What disease results in HbH production? What is the structure of Hb Bart's? What disease results in Hb Bart's production? (FAI I p349) (FAQ p380)

What excitatory neurotransmitter is involved in pain?

Basal Ganglia and Hypokinesis


Phys: Chapter 56 H: Chapter 372 Basal ganglia (FAII p400) (FAI2 p438) (Phys p690) Parkinson disease (FAII p40l) (FAI2 p438) (R pl3l9) (Phys p693) (H p33l7) Surgical treatment for Parkinson disease (H p3324) - Hemiballismus (FAII p40l) (FAI2 p439) (H p3332) Parkinson medications - Parkinson disease drugs (FAII p435) (FAI2 p478) (H p332l) (GG p6l4) - L-dopa (levodopa)-carbidopa (FAII p435) (FAI2 p478) (Hp332l) (GGp6l4) - Selegiline (FAII p436) (FAI2 p478) (GGp6l8) Differential diagnosis of hyporeexia

4. Basal Ganglia

Neostriatum ct direct ay pathway

-iri.vif.iiH mi
pars reticula. Substantia Nigra pars compai

MMitirrpTrT IHjnaiEluEOaSa

Subthalamic Nucleus

Copyright 2012. Doctors In Training.com. LLC. All Rights Reserved.

What are the cardinal features of Parkinson disease? (FAI I p40l) (FAQ p438)

28-year-old chemist presents with MPTP exposure. What neurotransmitter is depleted?

How do the following structures normally impact movement globus pallidus internal segment, subthalamic nucleus, and substantia nigra pars compacta?

Hyperkinesis
R: Chapter 28 H: Chapter 372 Types of hyperkinesis and dyskinesis - Chorea (FAII p40l) (FAI2 p439) - Athetosis (FAII p40l) (FAI2 p439) - Myoclonus (FAII p40l) (FA12 p439) - Dystonia (FAII p40l) (FAI2 p439) (H p3328) Huntington disease (FAII p40l) (FAI2 p439) (R pi322) (H p3330) Huntington drugs (FAII p436) (FAI2 p479) (GGp622)

Differential diagnosis of hyperreexia

8. A man in his 40s begins to develop eariy dementia and uncontrollable movements of his upper extremities. In what portion of the brain do you expect to see atrophy

A male patient presents with involuntary ailing of one arm. Where is the lesion?

What are the Cs of Huntington disease? (FAI I p40l) (FAQ p439)

What neurotransmitters are altered in Huntington disease? (FAI I p40l) (FAQ p439)

Copyright 2012, Doctors In Training.com. LLC. All Rights Reserved.

What defect is associated with the following type of murmur? (FAI I p259) (FAQ p28 , Crescendo-decrescendo systolic murmur best heard in the 2nd-3rd right interspace close to sternum Rumbling late diastolic murmur with an opening snap Pansystolic (AKA holosystolic or uniform) murmur best heard at the 4th-6th left intercostal spaces Continuous machine-like murmur (in systole and diastole) What organisms are known for causing endocarditis? (FAI I p275) (FAQ p30l)

What four drug regimen is used to treat TB? (FAI I pl9l) (FAQ p2Q)

Spinal Cord Basics


COA: Chapter 4 Spinal cord and associated tracts (FAII p409) (FAI2 p449) Spinal nerves (FAII p408) (FAI2 p448) Spinal cord - lower extent (FAII p409) (FAI2 p448) Spinal tract anatomy and functions (FAII p4IO) (FAI2 p450)

4. Label the cross-sectional view of the spinal cord.

Copyright 2012. Doctors In Training.com. LLC. All Rights Reserved.

Which spinal tract conveys the following information? (FAI I p409) (FAQ p449) Touch, vibration, and pressure sensation Voluntary motor command from motor cortex to body Voluntary motor command from motor cortex to head/neck Alternate routes for the mediation of voluntary movement Pain and temperature sensation Important for postural adjustments and head movements Proprioceptive information for the cerebellum

Where does each of the following spinal tracts decussate/cross over? (FAI I p4IO)(FAQ p450) Dorsal columns Lateral corticospinal Spinothalamic tract

Spinal Cord Lesions


R: Chapter 28 H: Chapters 373, 374, 377, 380 Spinal cord lesions (FAII p4ll) (FAI2 p45l) (H p377) Poliomyelitis (FAII p4ll) (FAI2 p45l) Oligodendroglia (FAII p396) (FAI2 p434) Multiple sclerosis (FAII p425) (FAI2 p465) (R pl3IO) (H p3395) Internuclear ophthalmoplegia (FAII p424) (FAI2 p464) (H p239) Werdnig-Hoffman disease (FAII p4ll) (FAI2 p45l) Amyotrophic lateral sclerosis (FAII p4ll) (FAI2 p452) (R pi324) (H p3345) Tabes dorsalis (FAII p4l2) (FAI2 p452) Friedreich ataxia (FAII p4l2) (FAI2 p452) Brown-Sequardsyndrome (FAII p4l2) (FAI2 p452)

7. Selected hereditary ataxias: Friedreich ataxia (FAI I p4l2) (FAI2 p452) (H p3343) Hereditary vitamin E deficiency (mutations in the alpha-tocopherol transfer gene protein) Ataxia-telangiectasia (FAI I p2l4) (FAI2 p238) (R pl323) (H p3343) Metachromatic leukodystrophy (FAI I pill, 426) (FA 12 pi 16) Wilson disease (FAI I p333) (FA12 p364,466) (H p3334)

Copyright 2012. Doctors In Training.com, LLC. All Rights Reserved.

What are the ndings of Brown-Sequard syndrome? (FAI I p4Q) (FAQ p452)

vnat is the classic presentation of a syringomyelia? What malformation is associated syringomyelia? (FAI I pi27,411) (FAQ pl3,451)

... What clinical presentation would lead you to suspect amyotrophic lateral sclerosis as a diagnosis? (FAI I p4l I) (FAQ p452)

What are some of the more classic presenting scenarios for multiple sclerosis? (FAI I p425) (FAQ p466)

Copyright 2012. Doctors In Training.com. LLC. All Rights Reserved

What is pulsus paradoxus, and what,

With what hematologic disease would you expect to see the following? (+) Ham's test Heinz bodies Basophilic stippling (+) Osmotic fragility test (+) DEB test D-dimer Coomb's (+) Coomb's (-) (+) Ristocetin test Outline the ow of fetal circulation (FAI I pQ5) (FAQ pi 32)

Brachial Plexus Anatomy


COA: Chapter 6 Brachial plexus (COA p72l) (H p347l) Brachial plexus lesions (FAII p373) (FAI2 p408) Erb-Duchenne palsy (FAII p375) (FAI2 p4IO) Klumpke palsy and thoracic outlet syndrome (FAII p375) (FAI2 p4IO)

Copyright 2012. Doctors In Trainmg.com. LLC. All Rights Reserved.

4. Brachial plexus in neck dissection

Lippmcott Williams 8 Wilkins Atlas of Anatomy 2008 Wolters Kluwer Health, Inc. All rights reserved.

5. Label the following diagram of the brachial plexus:


Trunks Cords

^^^^^^^^^^^J
3

"

r%

{ ] W i l l

^^^^^^l

II

Copyright 3 2012. Doctors lnTraining.com. LLC. All Rights Reserved.

Label the following diagram

cniai plexus:

lat are the symptoms of a lesion to the C5 and C6 nerve roots? (FAI I p375) (FAQ p4IO)

What are the symptoms of a lesion to the inferior trunk of the brachial plexus? (FAI I p375) (FAI2p4IO)

JL> Nerves of the Upper Extremity


COA: Chapter 6 rd p Upper extremity innervation (FAII p372) (FAI2 p407) Cj Upper extremity nerves (FAII p374) (FAI2 p409)
Ct3 Distortions of the hand (FAII p375) (FAI2 p4IO)

Jq Long thoracic nerve (FAII p376) (FAI2 p4ll) Hand muscles (FAII p376) (FAI2 p376)

Copyright 2012. Doctors In Trammg.com. LLC. All Rights Reserved.

9. What does the RADIAL nerve innervate, and what is seen if this nerve is damaged?

Triceps - Cannot extend forearm, loss of triceps reflex Skin of posterior arm - Loss of sensation of posterior arm Deep branch - Supinator & Brachioradialis - Loss of brachioradialis reex - Extensor muscles of forearm - WRIST DROP (cannot extend wrist) - Abductor pollicis longus - WRIST DROP - Skin of posterior forearm - Loss of sensation of posterior forearm Superficial branch - Skin of lateral, posterior hand - Loss of sensation of lateral, posterior hand Hint: radial nerve innervates the BEST: Brachioradialis, Extensors of wrist and ngers, Supinator, and Triceps Saturday Night Palsy: Radial nerve compression against the spiral groove of the humerus > weak wrist and finger extension, weak brachioradialis reex, but normal triceps.

Lippincott Williams 8 Wilkms Atlas of Anatomy 2008 Wolters Kluwer Health. Inc. All rights reserved.

Copyright 2012. Doctors In Training.com. LLC. All Rights Reserved.

10. What does the ULNAR nerve innervate, and what is seen if this nerve is damaged? Flexor carpi ulnaris - CLAW HAND*
Medial !4 of the flexor digitorum profundus - CLAW HAND* Deep branch - Hypo thenar muscles - cannot adduct the thumb - Adductor pollicis - cannot adduct the thumb - Ulnar two lumbricals - cannot abduct tor adduct ngers - All interosseous muscles - cannot abduct tor adduct ngers Superficial branch - Skin over medial hand - loss of sensation of medial hand *CLAW HAND - Cannot ex DIP joint 4th and 5th digits, atrophy of interosseus muscles and inability to extend interphalangeal joints when trying to straighten ngers.

Lippincott Williams 8 Wilkins Atlas of Anatomy 2008 Wolters Kluwer Health, Inc. All rights reserved.

Copyright 2012. Doctors In Training.com. LLC. All Rights Reserved.

What does the MEDIAN nerve innervate, and what is seen if the nerve is damaged?
Pronator teres - Cannot pronate Flexor carpi radialis - Weak wrist flexion Palmaris longus -Weak wrist flexion Flexor digitorum superficialis- Weak wrist flexion Flexor pollicis longus - Cannot flex thumb Pronator quadratus - Cannot pronate Lateral 1/2 of flexor digitorum profundus - BISHOP'S HAND / HAND OF BENEDICTIONf Radial two lumbricals - BISHOP'S HAND / HAND OF BENEDICTIONf Skin over lateral palm - Loss of sensation of lateral palm Skin of distal first 3 Yi digits - Loss of sensation of distal first 3 */i digits y^T Recurrent branch / - Thenar muscles -APE HAND:): (cannot oppose thumb) / (

fBISHOP'S HAND/HAND OF BENEDICTION - loss of PIP exion in digits 1-3 and loss of DIP flexion in digits 2-3 * when patient attempts to make a fist, digits 2 and 3 remain partially extended (COA p768) +APE HAND - thumb movements are limited to exion/extension in the plane of the palm CARPAL TUNNEL SYNDROME - median nerve entrapment at the carpal tunnel (enclosed by the inelastic exor retinaculum ventrally, and the carpal bones dorsally)

Lippincott Williams 8 Wilkins Atlas of Anatomy 2008 Wolters Kluwer Health, Inc. All rights reserved.

12. An elderly woman with chronic osteoarthritis and diffuse pain now presents with numbness and tingling over the lateral digits of her right hand that sometimes radiates up to the elbow. Exam reveals wasting of the thenar eminence. What is the diagnosis?

Copyright 2012. Doctors In Training.com. LLC. All Rights Reserved.

13. What does the MUSCULOCUTANEOUS nerve innervate, and what is seen if this nerve is damaged?
Biceps - Weak arm and forearm flexion, and forearm supination Coracobrachial - Weak arm flexion Brachialis - Weak forearm flexion Skin of lateral forearm - Loss of sensation of the lateral forearm

Lippincott Williams 8 Wilkins Atlas of Anatomy 2008 Wolters Kluwer Health. Inc. All rights reserved.

14. What nerves run with the following arteries?


- Dorsal scapular artery Lateral thoracic artery Posterior circumflex artery Suprascapular artery Thoracodorsal artery Deep brachial artery Ulnar artery Brachial artery Anterior interosseous artery Posterior interosseous artery
Copyright 2012. Doctors In Training com. LLC. All Rights Reserved.

Session Quiz
15. What nerve is damaged when a patient presents with the following symptom (upper extremity)? Claw hand Ape hand Wristdrop Scapular winging Unable to wipe bottom Loss of forearm pronation Cannot ab- or adduct fingers Weak lateral rotation of arm Loss of arm and forearm flexion Loss of forearm extension Trouble initiating arm abduction Unable to abduct arm beyond 10 degrees Unable to raise arm above horizontal What nerve is most at risk of injury with the following types of fractures/injury? Shaft of the humerus Surgical neck of the humerus Supracondyle of the humerus Medial epicondyle I* Loss of arm abduction Anterior shoulder dislocation Injury to the carpal tunnel

17. A patient falls off a motorcycle and lands on his right shoulder On physical exam you notice his shoulder has an abnormal conguration. X-rays indicate an anterior dislocatii of his shoulder. What artery and nerve are most at risk of being damaged?

v nigh-school athlete falls on his arm during practice. In the ER, a radiograph shows a midshaft break of the humerus. Which nerve and which artery have the highest risk of being damaged? What muscular actions are affected?

A patient presents with decreased pain and temperature sensation over the lateral aspects "< of both arms. Where is the lesion?

Copyright 2012. Doctors In Tr.nning.com. LLC. All Rights Reserved.

What drug categories do the following medications fall under? (FAI I p247) (FAQ p274) Losartan Vecuronium Ticarcillin Desipramine Enalapril Lorazepam Rosiglitazone

Which branchial arches develop into the following structures? (FAI I pi26) (FAQ pl36) Common carotid artery Aortic arch Right subclavian Pulmonary arteries

What is the classic clinical presentation of athyrogiossal duct cyst? (FAI I pl3l) (FAQ pi31

_j Lower Extremity
-^ COA: Chapter 5 < D -^ Lower extremity nerves and ^' Sensory of the lower limbs muscles (FAII p376) (FAI2 p4ll) "O Arteries of the leg

Copyright 2012. Doctors In Training.com. LLC. All Rights Reserved.

4. LE Nerves (FAI I p376) Femoral nerve Psoas and iliacus. Pectineus Sartorius Rectus femoris Vastus - Lateralis - Intermedius Superior gluteal nerve Tensor fascia latae Gluteus medius Gluteus minimus Inferior gluteal nerve Gluteus maximus

- Medialis Obturator nerve Adductor


- Magnus - Longus - Brevis Gracilis (Pectineus)

Sciatic nerve (tibial branch)


Biceps femoris Semitendinosus Semimembranosus Adductor magnus Sacral plexus Gemellus superior and inferior Piriformis Quadratus femoris

Deep peroneal nerve Peroneus tertius Tibialis anterior Extensor hallicus longus Extensor digitorum longus Supercial peroneal nerve Peroneus longus Peroneus brevis Supercial peroneal nerve Peroneus longus Peroneus brevis

Tibial nerve Gastrocnemius Soleus Plantaris Poplrteus Flexor hallucis longus Flexor digitorum longus Tibialis posterior

z m

S
O
1 i i i

fD

&

a
c
>

Copyright 2012. Doctors In Training.com. LLC. All Rights Reserved.

[47]

5. Femoral Nerve

6. Obturator Nerve

It ,,
(<*

~J.l

I4 1

Lippincott Williams 8 Wilkins Atlas of Anatomy 2008 Wolters Kluwer Health. Inc. All rights reserved.

Copyright i<3 20I2. Doctors In Training.com, LLC. All Rights Reserved.

7. Sciatic and Common Fibular Nerve

8. Sciatic and Tibial Nerve

Uppincott Williams 8 Wilkins Atlas of Anatomy 2008 Wolters Kluwer Health. Inc. All rights reserved.

Copyright 2012. Doctors In Training.com. LLC. All Rights Reserved.

9. Lower Extremity Dermatomes

Lippincott Williams & Wilkins Atlas of Anatomy 2008 Wolters Kluwer Health. Inc. All rights reserved.

Copyright G? 2012. Doctors In Training.com. LLC. All Rights Reserved,

jt/h:c
Exam of a patient revea leg. What muscular defect would you also expect to be present?

A 20-year-old dancer reports decreased plantar exion and decreased sensation overth back of her thigh, calf, and lateral half of her foot. What nerve is involved?

, patient fractures her bula neck. What nerve is most at risk of being damaged?

Skeletal Muscle
Phys: Chapters 6, 7 -^ Types of muscle bers (FAII p378) (FAI2 p4l2) ^ Muscle conduction to contraction (FAII p377) (FAI2 p4l2) (Phys p83) D Skeletal and cardiac muscle contraction (FAII p378) (FAI2 p4l3) (Phys p73) >_ Dantrolene (FAII p435) (FAI2 p477)

0 14. Label the following sarcomere:

Copyright 2012. Doctors In Training com. LLC. All Rights Reserved.

iiiij

nPI/X'J' LL

Which two muscle receptors are responsible for opening the sarcoplasmic reticulum in response to depolarization? (FAI I p377)

What drug prevents the release of calcium from the sarcoplasmic reticulum of skeletal muscle? (FAI I p435)

17. For what conditions is this drug useful? (FAI I p435)

Which type of muscle ber would usually be dominant in the gastrocnemius muscle? (FAII p378)

Copyright 2012. Doctors In Trainmg.com. LLC. All Rights Reserved.

WK91' L=~i "f J i li/< liuZ A young woman is found to nave snore stature ana snortenea tin ani -Mhroir?; ^;ilSI8BBB^8i8BBEi^fSB8miCTrMHE What endocrine disorder is most likely responsible for these manifestations? What is the rate-limiting enzyme of beta-oxidation of fatty acids? (FAI I p95) What is the treatment for nephrogenic diabetes insipidus? (FAI I p295)

Skin Sensation
Phys: Chapters 46-48 Sensory corpuscles (FAII p397) (FAI2 p435) Additional sensory nerves Landmark dermatomes (FAII p4l3) (FA12 p435) Clinical important landmarks (FAII p37l) (FAI2 p405) Peripheral nerve layers (FAII p397) (FAI2 p435) Spinothalamic tract and dorsal columns (FAII p4IO) (FAI2 p450) Schwann cells (FAII p396) (FAI2 p435) Guillain-Barre syndrome (FAII p426) (FAI2 p466) (H p3473)

SENSORY RECEPTORS (FAI I p397) (FAQ p435) 4. Compare Merkel receptors, Meissner's corpuscles, Rufni endings, and Pacinian corpuscles based on location in the skin tissue and on whether they are slowly or rapidly adapting.

Slowly adapting
Supercial layers Deep layers (dermis)

Rapidly adapting

5. What is the difference between a slowly adapting receptor and a rapidly adapting receptor?
Slowly adapting: sends a continuous electrical signal throughout a continuous stimulus Rapidly adapting: sends an electrical signal only at the beginning and end of a continuous stimulus

<U 6. What sensory receptor communicates with the following information? (FAI I p397) (FAI2p435) Pricking pain (fast, myelinated)
_ Burning or dull pain and itch (slow, unmyelinated) -\ Receptor for cold sensation s Receptor for warm sensation D Vibration and pressure Hz Dynamic/changing light, discriminatory touch Static/unchanging light touch Proprioception information - muscle length monitoring 54 1 r * r Proprioception U informationTmuscleJensionmonitoring,.g h......R eDe r v e d . J Copyright -WO . LJoctois In Traimng.com.'eLC. All Ri ts s

7. What sensory receptor matches the following description?


Resembles an onion in cross section Robust spindle-shaped structures found particularly on the soles of the feet Found only in areas of skin without hair (fingertips, lips, eyelids, etc.) Simplest sensory receptor thought to be pain receptor or thermoreceptors Touch receptor that is tough to distinguish from melanocytes

8. What sensory receptor communicates with the following information? (FAI I p397)(FAQ Pricking pain (fast, myelinated) Burning or dull pain and itch (slow, unmyelinated) Vibration and pressure Dynamic/changing light, discriminatory touch Proprioception information - muscle length monitoring Proprioception information - muscle tension monitoring E" Static/unchanging light touch 9. What is the landmark for a pudendal nerve block? (FAI I p37l) (FAQ p405)

What CSF changes are present in Guillain-Barre syndrome? (FAI I p426) (FAQ p466)

.at are the classic manifestations of Guillain-Barre syndrome? (FAI I p426) (FAQ p4^

Other Demyelinating Diseases


R: Chapter 28

Other demyelinating and dysmyelinating diseases (FAI I p426) (FAQ p466) (R pi309)

Copyright & 2012. Doctors In Training.com. LLC. All Rights Reserved.

Otic Sensation
Phys: Chapter 55 H: Chapter 21 COA: Chapter 7 Tympanic membrane (COA p967) Inammatory ear diseases (R p754) - Acute otitis media (COA p978) - Otitis externa (COA p978) - Cholesteatoma (H p249) (R p754) Ossicles (COA p970) Cochlea (COA p973) Vestibular apparatus (Phys p674) Vertigo (FAII p427) (FAI2 p467) (H pl78)

12. Weber Test


Normal ^Midline Conductive hearing loss lateralizes to the side of the affected ear Sensorineural hearing loss lateralizes to the side opposite the affected ear 13. Rinne Test Normal Air conduction > Bone conduction (AC>BC) Conductive hearing loss BC > AC

14. Given the following exam ndings, what is the diagnosis?


Weber Patient A Patient B Patient C Patient D Midline Right Left Midline Rinne (left) Rinne (right) Diagnosis

AOBC AOBC AOBC BC>AC

AOBC BO AC AOBC BO AC

c o
to < L > CO

15. Cholesteatoma Overgrowth of desquamated keratin debris within the middle ear space that may eventually erode the ossicular chain and external auditory canal Causes: negative middle ear pressure (chronic retraction pocket) from eustachian tube dysfunction or direct growth of epithelium through a TM perforation Commonly associated with chronic middle ear infection PE: grayish-white "pearl/' lesion behind or involving the TM, conductive hearing loss, vertigo Treatment surgical removal usually involving tympanomastoidectomy and reconstruction of the ossicular chain

o
D
[56]

Copyright 2012, Doctors In Training.com. LLC. All Rights Reserved.

Session Qu
; s

Which organisms are most commonly responsible for acute otitis media7 (R p754)

What organism is most commonly responsible for otitis externa7

Chronic otitis media can sometimes result in a cystic lesion that is lined by keratinizing squamous epithelium which can be metaplastic that is lled with amorphous debris. Wie the name of this condition? (R p754)

Copyright O 2012. Doctors In Training.com. LLC. All Rights Reserved.

internal acoustic meatus. What is the diagnosis? 2. What are the different mechanisms by which heart contractility can be increased?

What medication is used to treat the following parasitic infection? (FAII pl60-l63)(FAI2pl75-l77) Trichomonas or Gardnerella Plasmodium vivax or ovale I ( FA I I p 2 5 5 ) ( FA I 2 por 8 l ) Pediculosis capitis 2 pubis

Eye Structures
COA: Chapter 7 H: Chapter 28 Eye and retina (FAII p420) (FAI2 p459) (COA p889) Eye pathology (FAII p420) (FAI2 p460) Aqueous humor pathway (FAII p420) (FAI2 p460) Glaucoma (FAII p42l) (FAI2 p46l) (COA p9l2) (H p234) Glaucoma drugs (FAII p430) (FAI2 p47l) (GGpl785) Cataract (FAII p42l) (FAI2 p46l) Papilledema (FAII p42l) (FAI2 p46l)

Glaucoma (fai i d42IWFai:


4. Outline the ow of aqueous humor.
Formed in a capillary bed in the ciliary body secreted into the posterior chamber flows between the angle formed by the lens and the iris diaphragm into anterior chamber reabsorbed by the canal of Schlemm

5. What is the pathogenesis of glaucoma?


Blocked canal of Schlemm aqueous humor not reabsorbed increased pressure atrophy of optic

6. Open Angle Glaucoma Common, insidious form; almost always bilateral Risk factors: older than 40, black, diabetes Early stage- asymptomatic Late stage- areas of reduced/absent vision, contraction of visual field (peripheral * central)

7. Acute Angle-Closure Glaucoma


Emergency Abrupt onset of pain, nausea, colored halos, rainbows around light Red, teary eye with hazy cornea and fixed, mid-dilated pupil (not reactive to light) that is firm to palpation

8. Diagnosis
Cupping of the optic disk (cup:disc ratio greater than 1:2) Tonometry
Copyright <Q 2012. Doctors In Tramtng.com. LLC. All Rights Reserved.

Cataracts (faq p42i) (FA12 p46i)


9. What is the classic presentation of a patient that has a cataract?
Painless, progressive decrease in vision manifested with difficulty driving at night, reading road signs, or reading ne-print Usually bilateral, but often unilateral Near-sightedness is often an early manifestation Possible disabling

60-year-old male has a hard time driving at night because of worsening vision and the appearance of halos around oncoming headlights. What is the diagnosis? 11. What 5 drug dasses are used in the treatment of glaucoma? (FAI I p430) (FAQ p47l) 12 How does the optic disc appearance differ in glaucoma when compared to hydrocephalus? (FAII p42l) (FAQp46l) (FAIIp42l)(FAI2p46l)

Neural Aspects of the Eye


H: Chapter 28 Retinal detachment (FAII p423) (FAI2 p463) (COA p9IO) Age-related macular degeneration (FAII p423) (FAI2 p463) (H p235) Pupillary control (FAII p422) (FAI2 p462) Pupillary light reex (FAII p422) (FAI2 p462) (COA p9ll) Cranial nerve III in cross section (FAII p423) (FAI2 p463) Visual eld defects (FAII p423) (FAI2 p463) (H p227)

Pupillary Light Reex (FAII p422) (FAI2 p462) 14. Outline the pupillary light reex pathway.
Cells of retina * optic tract * optic tract/nerve * pretectal nucleus * bilateral Edinger-Westphal nuclei * preganglionic parasympathetic fibers in oculomotor nerve ciliary ganglion postganglionic parasympathetic fibers * pupillary sphincter of iris * pupillary constriction (miosis)

15. Describe what light reexes will be seen in both eyes if the right optic nerve is damaged prior to the pretectal nucleus (AKA afferent defect).
No constriction of either the left or right eye when light is shined in the right eye Both pupils constrict if the light is shined in the left eye

16. Describe what light reexes will be seen in both eyes if the right oculomotor nerve is damaged (AKA efferent defect).
Right eye will not respond to light shone in either the right or left eye Left eye will constrict when a light is shined in either eye
Copyright 2012. Doctors In Training.com. LLC. All Rights Reserved.

What eye abnormality is seen in a p (FAI I p424) (FAQ p464)

A patient cannot adduct her left eye on lateral gaze but convergence is normal. What structure is damaged?

Light stimulus in patient's right eye produces bilateral pupillary constriction. When the light is shown in the left eye, there is a paradoxical bilateral pupillary dilatation. What is the defect?

A woman presents with headache, visual disturbance, and amenorrhea. What is the diagnosis?

hat is the treatment for dry age-related macular degeneration? (FAI I p423) (FAQ p463)

Copyright 2012, Doctors In Training.com. LLC. All Rights Reserved.

What neoplasm is most commonly responsible for the following paraneoplastic syndrom. (FAII p229) (FAQp255) ACTH * Cushing syndrome Erythropoietin > polycythemia ADH SIADH What are the endogenous agonists to the different opioid receptors? (FAI I p430) (FAQ p47l

Which medication ts the following description (FAI I p430) (FAQ p47l)? Opioid cough suppressant commonly used with the expectorant guaifenesin Opioid used in the treatment of diarrhea Opioid used in the treatment of acute heart failure Opioid receptor antagonist Non-addictive weak opioid agonist Partial opioid agonist that causes less respiratory depression

Dementia
R: Chapter 28 H: Chapter 371 Dementia (FAII p425) (FAI2 p465) (H p3300) Alzheimer disease (FAII p425) (FAI2 p465) (R pl3l3) (Phys p727) (H p3305) Alzheimer drugs (FAII p436) (FAI2 p479) (GG p6l9) Pick disease (FAII p425) (FAI2 p465) (R pl3l8) (H p33IO) Lewy body dementia (FAII p425) (FA/2 p465) (R pi321) (H p33l2) Creutzfeldt-Jakob disease (FAII p425) (FAI2 p465) (H p33l2, 3441) Prions (FAII pl75) (FAI2 pl95) (R pl308) (H p344l) Other causes of dementia 4. Identify the neuronal pigment/inclusion that matches the following statement: Intranuclear inclusions seen in herpes simplex encephalitis Cytoplasmic inclusions pathognomonic of rabies Neuronal inclusions characteristic of Parkinson disease Cytoplasmic inclusion bodies associated with aging Dark cytoplasmic pigment in neurons of the substantia nigra and locus coeruleus, not seen in patients with Parkinson Eosinophilic, rod-like inclusions in hippocampus of Alzheimer patients Diagnostic of Alzheimer disease Filamentous inclusions that stain with silver, do not survive neuronal death Filamentous inclusions that stain with PAS and ubiquitin 5. What diseases are associated with Lewy bodies?

Copyright 2012. Doctors In Training.com. LLC. All Rights Reserved.

What are the usual components of a "dementia work-up"?

Which neurodegenerative disease matches the following statement? Lou Gehrig disease (ALS) Senile plaques, neurofibrillary tangles Presents at birth as "floppy baby" Lewy bodies Dementia often associated with frequent falls and/or syncope Atrophy of caudate nucleus chorea Depigmentation of substantia nigra Both upper and lower motor neuron signs (spasticity and weakness) Dementia + visual hallucinations Dementia + personality changes and/or aphasia

What allele is associated with Alzheimer disease? Why is Alzheimer disease so common in patients with Down syndrome? (FAI I p425) (FAQ p465)

What is the mechanism of action of the drugs used in the treatment of Alzheimer disease? (FAI I p436)

^ Headache
~j2 H: Chapter 14 rd ^ Headache (FAII p427) (FAI2 p467) .p - Tension (H pi20) '<~ - Migraine (H pi 14) - Cluster (H pi22) - Other headaches (H pi24-128) Headache medications \ - Sumatriptan (FA11 p436) (FAI2 p479) (H pi 19) ^ HEADACHE (FAI I P427) (FAQ p467) 10. What is the typical presentation of a tension headache? O Constant, chronic pain (lasts hours/days/weeks/months) r~T- Occurring in the frontal or occipital regions (most often bilateral) or as a band around the head j-jl No associated symptoms such as light/loud noise sensitivity, visual changes, nausea/vomiting, or focal neurological changes

Copyright 2012. Doctors In Training.com, LLC. All Rights Reserved.

11. What are the main diagnostic criteria for migraine without aura? At least 5 attacks
Headache lasting 4-72hrs (2-48hrs in children) At least 2 of the following: - Unilateral location - Pulsating quality - Moderate to severe intensity (inhibits or prohibits daily activities) - Aggravated by dimbing stairs or similar activity At least I of the following: nausea and/or vomiting; photophobia and/or phonophobia

12. What are the typical symptoms of a cluster headache?


Strictly unilateral Severe aching/boring pain in the retro-orbital/periorbital region (Doesn't throb like a migraine. No aura.) Duration of 30 min-3 hours, occurs daily (often at the same time), and continues for an interval of 4-8 weeks May be assodated with partial Horner syndrome (ptosis and miosis), ipsilateral nasal congestion or eye redness, rhinorrhea, or tearing

13. What is the most likely cause of headache based on the following description?
Made worse by foods containing tyramine Obese female with papilledema Jaw muscle pain when chewing Periorbital pain with ptosis and myosis Photophobia and/or phonophobia Bilateral frontal/occipital pressure Lacrimation and/or rhinorrhea Elevated ESR "Worst headache of my life" Headache + extraocular muscle palsies Scintillating scotomata prior to headache Headache occurring either before or after orgasm Responsive to 100% oxygen supplementation Trauma to the head headache begins days after the event, persists for over a week and does not go away

m C
Ln I

S
u >
3
a >
{D

(D

D CL

I D f

P J Q_

J D O ZT

Copyright 2012. Doctors In Training.com. LLC. All Rights Reserved.

[63]

tension headache? (FAI I p427) (FAQ p467)

What is the mechanism of action of sumatriptan? (FAI I p436) (FAQ p479)

What are the contraindications to sumatriptan use? (FAI I p436) (FAQ p479)

What is the usual treatment for migraine headaches in pregnancy?

What would suspect as a cause of headache in a patient using topical retinoic acid for acne?

A 20-year-old woman that has migraine headaches each proceeded by an aura should never be prescribed which medication?

Copyright 2012. Doctors In Training.com. LLC. All Rights Reserved.

est ion Warm-Up


An ER trauma patient is found to have multiple masses scattered throughout the bo and brain as seen on a trauma protocol CT scan of the head, neck, chest, abdomen, and pelvis. Assuming that this represents multiple metastatic lesions, what do you suspect as the primary source of the cancer given that metastases are found in the b parenchyma? (FAI I p230) (FAQ p256)

abel the following diagram of lung volumes.

Copyright O 2012. Doctors In Trainmg.com, LLC. All Rights Reserved.

Brain Tumors
R: Chapter 28 H: Chapter 379 Primary brain tumors (FAII p428) (FAI2 p468) (R pl330) (H p3382) Herniation syndromes (FAII p429) (FAI2 p470) (H p2248) Uncal herniation (FAII p429) (FAI2 p470) (H p2248)

uick Quiz
What are the 3 most common primary brain tumors in adults? What are the 3 most common in children?

Which primary brain tumor ts the following description? Pseudopalisading necrosis Polycythemia Neurofibromatosis II Associated with von Hippel-Lindau syndrome Foamy cells, high vascularity Hyperprolactinemia * galactorrhea, amenorrhea, anovulation Psammoma bodies Fried-egg appearance Perivascular pseudorosettes Bitemporal hemianopia worst prognosis of any primary brain tumor child with hydrocephalus Homer-Wright pseudorosettes

Anesthetics
GG: Chapter II, 17, 19,20 Barbiturates (FAII p432) (FAI2 p474) (GG p469) Benzodiazepines (FAII p433) (FAI2 p475) (GG p458) Anesthetics - general principles (FAII p433) (FAI2 p475) (GG p527) Inhaled anesthetics (FAII p433) (FAI2 p476) (GG p539) Intravenous anesthetics (FAII p434) (FAI2 p476) (GGp532) Local anesthetics (FAII p434) (FAI2 p477) (GG p565) Neuromuscular blocking drugs (FAII p435) (FAI2 p477) (GG p258) Dantrolene (FAII p435) (FAI2 p477)

Copyright 2012, Doctors In Training.com. LLC All Rights Reserved.

6. Depolarizing vs. Non-Depolarizing Neuromuscular Blockers NOIBBUG


Tr a i n - o f - f o u r DEPOLARIZING BLOCK PHASE I i Constant but diminished , PHASED NONDEPOLARIZING BLOCK

Te t a n u s

', Constant but diminished

Posttetanic . potentiation l"SJ

Absent

Present "o
D

"a

lllll '
7. Succinylcholine vs. Tubocurarine
SUCCINYLCHOLINE PHASE I Succinylcholine AHminictratinn Administration Tubocurarine Administration Effect of neostigmine ] Posttetanic facilitation Rate of recovery ! Response to a
tetanic stimulus '

PHASE II Augmented Augmented Antagonistic

TUBOCURARINE

Additive Antagonistic Augmented

Antagonistic Additive Antagonistic

4-8 min Sustained3 Fasciculations

> 20 min3 Unsustained

30-60 min Unsustained

Initial excitatory effect ] on skeletal muscle '

PHASE II Succinylcholine Administration (Img/kg)

Copyright 3 2012. Doctors In Trainmg.com. LLC. All Rights Reserved.

Which anesthetic ts the following description? (FAI I p433-435) (FAQ p476-477) IV, associated with hallucinations and bad dreams Inhaled, SE nephrotoxic IV, most common drug used for endoscopy Inhaled, SE convulsions/seizures Inhaled, SE hepatoxic IV, used for rapid anesthesia induction & short procedures Inhaled, used for rapid anesthesia IV, decreases cerebral blood flow (important in brain surgery) IV, does not induce histamine release like morphine High triglyceride content increases risk of pancreatitis with long-term use

What is the mechanism of action of dantrolene? (FAI I p435) (FAQ p477)

). What is the mechanism of action of local anesthetics? Which nerve bers are blocked rst with local anesthesia? (FAI I p434) (FAQ p477)

. What drugs can be used to reverse neuromuscular blockade? (FAI I p435) (FAQ p477)

Copyright 2012, Doctors In Training.com. LLC. All Rights Reserved.

uestion Warm-Up HElIirSsfBfjfHff!!^


*]ff^<lll*"lllL*J

' Polycythemia Neurofibromatosis II Hyperprolactinemia galactorrhea, amenorrhea, anovulation Psammoma bodies Perivascular pseudorosettes Loss of peripheral vision Worst prognosis of any primary brain tumor

A 25-year-old female presents with sudden uniocular vision loss and slightly slurred speech. She has a history of weakness and paresthesia that have resolved. What is th most likely diagnosis?

What is the classic triad of symptoms in multiple sclerosis? (FAI I p425) (FAQ p465)

Seizures
H: Chapter 369 Phys: Chapter 59 Seizures (FAII p426) (FAI2 p466) (Phys p725) (H p325l) Sturge-Weber syndrome (FAII p427) (FAI2 p467) Tuberous sclerosis (FAII p427) (FAI2 p467) (R pi342)

4. Trigeminal Neuralgia AKA Tic Douloureux "Lightning-like" pain, "Electric shocks," along a division of the trigeminal (usually maxillary) triggered by light touch (wind, bed sheets.) Rx: carbamazepine or other anticonvulsant (phenytoin, gabapentin, topiramate) 5. Tuberous Sclerosis (FAI I p86, 424) (FAQ p90, 467) Autosomal dominant; 1/5,000- 1/10,000 live births Complete penetrance BUT variable expression Most common mutations are in the TSCI or TSC2 genes - TSCI gene hamartin protein - TSC2 gene > tuberin protein Classic triad: seizures, mental retardation, angiofibromas Additional findings: hypomelanotic macules (ash-leaf spots), retinal hamartomas, cortical tubers (glioneuronal hamartomas) Tumor associations (FAI I p222): renal angiomyolipoma, cardiac rhabdomyoma, astrocytoma (subependymal giant cell astrocytoma
Copyright 2012. Doctors In Traimng.com. LLC. All Rights Reserved.

In regards to seizures,' (FAI I p426) (FAQ p466)

IiS8BRI!8i1Sitt: srsrsfe

What are the most common causes of seizures in children? (FAI I p426) (FAQ p466)

8. A 10-year-old child "spaces-out" in class (stops talking midsentence and then continues as diagnosis? Iff nothing had happened). During the spells, there is slight quivering of lips. What is the

Anti-Seizure Medications
H: Chapter 369 GG: Chapter 21 Epilepsy drugs (FAII p43l) (FAI2 p473) (H p326l-3267) (GG p590-606) Epilepsy drug toxicities (FAII p432) (FAI2 p474) (GG p590-606) Phenytoin (FAII p432) (FAI2 p474) (GGp59l)

9. Drug of choice for partial (simple and complex) and tonic-clonic seizures: 10.4 other drugs useful in partial and tonic-clonic seizures:

Drug of choice for absence seizures: 12. Used to treat status epilepticus and eclampsia: <} 13. Common side-effects of epilepsy drugs:

00 14. Additional SE of phenytoin: 15. Which anti-epileptics are teratogens? 16. What drugs cause Stevens-Johnson syndrome? 17. What drugs are known to cause agranulocytosis?
Copyright 2012. Doctors In Training.com. LLC. All Rights Reserved.

18. Hepatotoxic anti-epileptics:

19. What drugs induce the P450 system?

20. Which anti-epileptics block Na+ channels?

21. Which anti-epileptics work by potentiating the effects inhibitory effects of GABA?

What drugs are known for causing Stevens-Johnson syndrome? (FAI I p245) (FAQ p2

13. How is barbiturate overdose managed? How is benzo overdose managed? (FAI I p432-433) (FAQ p474-475)

.4. What side effects are common to most all of the anti-epileptics? (FAI I p432) (FAQ p474)

. What are the toxic side effects of phenytoin? (FAI I p432) (FAQ p474)

Copyright (-3 2012. Doctors In Training.com. LLC. All Rights Reserved.

uestion Warm-Up
Which collagen is typically decient in Ehlers-Danlos syndrome? In osteogenesis imperfecta? (FAI I p80) (FAQ p83) What regulates the progression of GI phase of the cell cyde to S phase? (FAI I p76) (FAQ p'

3. What cranial nerves innervate the tongue in the following ways? (FAI I pQ9) (FAQ pl3 Taste in the anterior 2/3 Taste in posterior 1/3 (main innervater) Motor Sensation in the anterior 2/3 Sensation in the posterior 1/3

Parasympathetic Activation
GG: Chapters 8-10 Phys: Chapter 60 Central and peripheral nervous system (FAII p235) (FAI2 p262) Effects of parasympathetic activation (GG pl79-l8l, Table 8-1) (Phys p734-736. Table 60-2) Cholinergic agonists (FAII p238) (FAI2 p265) (GG p222) Alzheimer anticholinesterases (GG p252) Myasthenia gravis (FAII p386) (FAI2 p422) (GG p25l) Cholinesterase inhibitor poisoning and pralidoxime (FAII p238) (FAI2 p265) (GG p247-249)

4. How do the sympathetic and parasympathetic nervous systems affect the following body structures?
Sympathetic Heart
r~\/A

Parasympathetic

tye

Salivary glands Bronchiolar smooth muscle Bladder Male GU Gl tract

5. What are the symptoms of excess parasympathetic activity? (FAI I p238) (FAQ p265)

Copyright 2012. Doctors In Training.com. LLC. All Rights Reserved.

6. Alzheimer disease anticholinesterases Donepezil Galantamine Rivastigmine 7. Myasthenia Gravis (FAI I p386) (FAQ p422)
Antibodies to the acetylcholine receptor Most common board question presentation: Tensilon test Thymus pathology: - 50% associated with thymic - 20% associated with thymic. - 1596associated with thymic. Myasthenic crisis - rapidly progressing weakness esp. in Rx: that worsens throughout the day

muscles

8. What drug regenerates acetylcholinesterases after organophosphate poisoning? (FAII p238) (FAQ p265)

lat is the antidote for organophosphate poisoning? (FAI I p238) (FAQ p265)

). Which anticholinesterases are used in the treatment of Alzheimer disease?

Copyright 2012, Doctors In Traming.com, LLC. All Rights Reserved.

Parasympathetic Inhibition
GG: Chapters 9, II Anticholinergics (FAII p239) (FAI2 p266) (GGp225) - Atropine (FAII p239) (FAI2 p266) - Muscarinic antagonists (FAII p239) (FAI2 p266) - Urge incontinence treatment (GG p23l, p232 Table 9-3) Contraindications to anticholinergics (GG p234) Other drugs with anticholinergic side effects Nicotinic acetylcholine receptors (GG p255) - Central and peripheral nervous system (FAII p235) (FAI2 p262) - ACh receptors (FAII p23S) (FA/2 p262) Hexamethonium (FAII p239) (GGp272)

11. What are the symptoms of inhibiting parasympathetic activity? (FAII p239) (FAQ p266)

12. What drugs inhibit parasympathetic activity? What are their uses? (FAII p239) (FAQ p266)

13. What anticholinergics are used in the treatment of urge type urinary incontinence? }>
Torterodine Oxybutynin Darifenacin and Trospium solifenacin ^ Sr (/)

14. In what patient populations is atropine contraindicated? (FAI I p239) (FAQ p266) ^ I 1 5 . W h a t o t h e r m e d i c a t i o n s h a v e a n t i c h o l i n e r g i c s i d e e ff e c t s ? &
First generation HI blockers: diphenhydramine (Benadryl), doxylamine (Unisom), chlorpheniramine ft Traditional neuroleptics (FAI I p453) (FAI2 p498) Tricydic antidepressants (FAI I p455) (FAI2 p500)

Amantadine

(FAI

pl95)

(FAI2

P2I6)

}.

o tn
Copyright 2012. Doctors In Training.com. LLC. All Rights Reserved.

[3]

What are the symptoms of excess p

c activity? (FAI I p238) (FAQ p265)

What are the symptoms of inhibiting parasympathetic activity? (FAI I p239) (FAQ p266)

18. Identify the following drugs as a direct cholinergic agonist, anti-acetylcholinesterase, antimuscarinic, or cholinesterase regenerator Physostigmine Tropicamide Pilocarpine Benztropine Echothiophate Scopolamine Oxybutynin Edrophonium Atropine To t t e r o d i n e Donepezil Trospium Pralidoxime Rivastigmine Bethanechol Homatropine Neostigmine Pyridostigmine Darifenacin . Carbachol Ipratropium

t, A caused this? What is the shortness of breath, salivation, miosis, and diarrhea. What gardener presents with mechanism of action?

. Atropine is not effective in reversal of organophosphate poisoning. What drug would best help this patient?

21. Which of the muscarinic antagonists discussed could be used to improve FEVI in a patient with COPD?

A 30-year-old schizophrenic man now has urinary retention due to his neuroleptic. What do you treat it with?

In the dark, both pupils are dilated, (see image) In the light, the control pupil is miotic while the pupil given drug X remains mydriatic. What is drug X?

Copyright 2012. Doctors In Trainingcom. LLC. All Rights Reserved.

Q u e s t i o n Wa r m - U p
Which agents are often used in the treatment ot urge incontinence

What group of genes is responsible for skeletal development? (FAI I pi 18) (FAQ pQ4)

Which cell types are derived from the neural crest? (FAI I pi 19) (FAQ pQ6)

Autonomic Nerve Terminals


GG: Chapter 8 Phys: Chapter 60 Autonomic drugs (FAII p237) (FAI2 p264) (GG pl82) (Phys p73l)

4. Fill in the diagram of acetylcholine synthesis and neurotransmission:

Cholinergic

Causes

paralysis pj

4 Fates of ACh

Copyright 2012. Doctors In Traimng.com. LLC. All Rights Reserved.

5. Fill in the diagram of norepinephrine synthesis and neurotransmission:

Fates of NE

What substances inhibit the reuptake of norepinephrine? (FAI I p237) (FAQ p264)

Outline the pathway for the generation of norepinephrine from tyrosine. (FAI I p237) (FAQ p264) (FAI I pi07) (FAQ pi 12)

m What substances stimulate the release of norepinephrine from neurons? (FAI I p237)
(FAI2p264)

Copyright 2012, Doctors In Training.com. LLC. All Rights Reserved.

9. Fill in the diagram of acetylcholine synthesis and neurotransmission:

Cholinergic

*i
Causes 4 Fates of ACh

10. Fill in the diagram of norepinephrine synthesis and neurotransmission:

Noradrenergic

m
Fates of NE

Copyright 2012. Doctors In Training.com, LLC, All Rights Reserved.

G-protein-Linked 2nd Messengers


Phys: Chapter 45 R: Chapter 3 GG: Chapter 3 G-protein-linked 2nd messengers (FAII p236) (FAI2 p263) (Phys p548) (R p90, p9l Fig. 3-9) (GG pS2)

11. Fill the signal transmission pathways:

/
2 3

hormone binds 7-pass transmembrane receptor

growth factor binds tyrosine kinase receptor

7 '

13

1
i

c o "S Iu

1 1 10 1
9 11

i r

12

"c E o U
i_

affected target proteins and/or gene regulatory proteins ^

i 14 i 15 i 16 i 17 i 18

/$ *&

_D

13 U I

activated alpha subunit of G-Protein

hormone binds 7-pass transmembrane receptor

growth factor binds tyrosine kinase receptor

/
activated phospholipaseC

activated alpha subunit of 6-Protein

adaptor protein Ras-activating protein active Ras protein activation of protein kinase I activation of protein kinase II

active adenylate cyclase

inositol triphosphate (IP3)

C O

increased Intracellular calcium increased levels of cyclic AMP calcium bound calmodulin activation of protein kinase A activation of CaM-kinase

i i

diacytglycerol (DAG)

< C D

i I i

< Q_

affected target proteins and/or gene regulatory proteins

activation of protein kinase C

activation of protein kinase III

1 1

[8]

Copyright <? 2012. Doctors In Training.com. LLC All Rights Reserved.

ess/on Qui
What G protein class do the following receptors stimulate.

Outline the pathway by which stimulation of a Gq receptor activates protein kinase C. (FAII p236) (FAQp263)

14. Outline the pathway by which stimulation of a Gs receptor activates protein kinase A. (FAII P236) (FAQp263)

at enzymes are used in the catabolism of norepinephrine?

6. What enzyme catalyses the conversion of tyrosine to dopa? (FAI I pl07) (FAQ pi 12)

Copyright 2012. Doctors In Training.com. LLC. All Rights Reserved.

Sympathetic Agents
GG: Chapter 12 Sympathomimetics (FAII p240) (FAI2 p266-267) (GG p277) en Sympathoplegics (FA 11 p24l) (FA 12 p267) (GG p295)

'-P C2 4. Label the following spinal cord representation seen at levels Tl - l_3

Copyright 2012. Doctors In Training.com. LLC. All Rights Reserved.

(FAQ p266-267) Clonidine Dopamine Phenylephrine Albuterol Norepinephrine Isoproterenol Epinephrine Dobutamine Terbutaline

6. Which sympathomimetic matches the following statement? (FAI I p240) (FAQ p266-267) Given as a nebulizer for asthma Drug of choice for anaphylaxis Most common first line agent for patients in cardiogenic shock Most common first line agent for patients in septic shock Given subQ for asthma Used by ENT to vasoconstrict nasal vessels Used to treat ADHD What drug(s) would be most appropriate in a patient in shock because they maintain renal blood ow?

Sympathetic Blockers
GG: Chapter 12 Alpha-blockers (FAII p24l) (FAI2 p268) (GG p304) Beta-blockers (FAII p242) (FAI2 p269) (GGp3IO)

Copyright 2012. Doctors In Training.com. LLC. All Rights Reserved.

How does blood pressure response xo pnenyiepnnne administration cnange ir an AWWggBBJ:llii!MKffi'.IHi' a-blocker is administered beforehand? Why is this different than the change seen when epinephrine is used rather than phenylephrine? (FAI I p24l) (FAQ p268)

What are the common side effects of (^-blockers? Which patient populations should use caution when taking p-blockers? (FAI I p242) (FAQ p269)

What are the various clinical uses for the following sympathomimetics? (FAII p240) (FAQp266-267) Dopamine Clonidine Amphetamine Terbutaline Epinephrine What are the various clinical applications of beta-blockers? (FAI I p242) (FAQ p269)

Copyright 2012. Doctors In Training.com. LLC. All Rights Reserved.

PI38)

What conditions are associated with an elevated ESR (erythrocyte sedimentation rate (FAII p223) (FAQp248)

What is the clinical use fortiotropium? (FAI I p235)

Enzyme Kinetics and Pharmacokinetics


GG: Chapter 2 Enzyme kinetics (FAII p232) (FAI2 p258) Pharmacokinetics (FAII p232) (FAI2 p2S9) (GG p28) Dosage calculations (FAII p233) (FAI2 p2S9) (GG p28)

4. Label the following graphs of enzyme kinetics:


Graph of Michaelis-Menten Kinetics Lineweaver-Burk Plot

Substrate Concentration [SI

5. Label the following plots as either competitive or noncompetitive inhibition:


Graph of Michaelis-Menten Kinetics Lineweaver-Burk Plot

Substrate Concentration |S] Copyright 2012. Doctors In Training.com. LLC. All Rights Reserved.

6. What 4 pharmacokinetics equations are most important to know for Step I?

60 kg man with status asthmaticus is being given an IV infusion of drug X at 60 mg/hr. The larance of drug X is 2L/h, and the volume of distribution is approximately 0.5 L/kg. 48 iurs after administration has begun, the asthma attack is under control. At this time, the ncentration of drug X in his plasma is 20 mg/L What is the half-life of drug X in this patient?

What loading dose could have been used to reach the target concentration of 20 mg/L?

If the patient begins to show signs of toxicity, and the target dose is decreased to 10 mg/l what would you do to get to this level?

If the patient has a kidney disease, and the clearance is reduced to I L/h, but Vd is unchanged, what effect will this have on loading dose and maintenance dose?

Copyright 2012. Doctors In Traming.com. LLC. All Rights Reserved.

Pharmacodynamics
GG: Chapters 2-4 Efcacy vs. potency (FAII p233) (FAI2 p26l) (GG p45) Pharmacodynamics (FAII p234) (FAI2 p26l) Physiologic antagonism (FAII p234) (GG p4l, 46) Therapeutic index (FAII p234) (FAI2 p26l) (GG p73)

12. Partial vs. Full Agonists Decreased efcacy (lower Vmax)

Increased potency f Decreased potency

Dose (log scale) d of Session Quiz


13. What is the equation for half-l'rfe? (FAI I p232) iffSWTfSm

What effect will a noncompetitive antagonist have on Vmax and Km? (FAI I p232) (FAQ p258)

What effect will a competitive antagonist have on efcacy and potency? (FAI I p234)(FAI2

16. What are some medications that are known for having a low therapeutic index?

17. What is the competitive antagonist in cases of benzodiazepine overdose? (FAI I p234)(FAI2 p26l) ji>

How many half-lives does it take for a drug infused at a constant rate to reach 94% of steady state? What variables determine the half-life of a drug? (FAI I p232) (FAQ p25<

Copyright 2012. Doctors In Training.com. LLC. All Rights Reserved.

What effect does changing Km and Vmax have on potency and efcacy? (FAI I p233) (FAI2p26l)

2. What are the 4 main pharmacokinetic equations? (FAI I p232) (FAQ p259)

What enzyme is decient in PKU? What are the symptoms? (FAI I pi07) (FAQ pi 12)

Drug Metabolism
GG: Chapters 2, 6 Elimination of drugs (FAII p233) (FAI2 p260) Urine pH and drug elimination (FAII p233) (FAI2 p260) (GGpl8) Phase I vs. phase II metabolism (FAII p233) (FAI2 p260) (GG pl24) Cytochrome P450 (GG pl27) P450 interactions (FAII p245) (FAI2 p273) (GG pl29)

= 4. P450 Inhibitors: "PICK EGS"


\ Protease Inhibitors Isoniazid Cimetidine Ketoconazole Erythromycin ( Grapefruit juice Sulfonamides

ryj 5. P450 Inducers: "BCG PQRS"


' Barbiturates Griseofulvin Phenytoin Quinidine Rifampin Carbamazepine

Copyright 2012. Doctors In Training.com. LLC. All Rights Reserved.

6. Urine pH and Drug Elimination (FAI I p233) (FAQ p260) (GG pi8)

hf + A
acidic drug anion

H + B
basic drug uncharged base
pKa (acid dissociation constant) is the pH at which the amount of the non-protonated form (A' or B): the amount of protonated form (HA or BH+) If pH is low (acidic environment) and less than pKa, there will be more of the protonated form (HA or BH4) basic drugs (BH"1) get trapped If pH is high (basic environment) and higher than pKa, there will be more of the nonprotonated form (A" or B) acidic drugs (A-) get trapped Treat acidic drug OD (i.e., salicylates) with NaHC03 traps the acidic drug in the basic urine Treat basic drug OD (i.e., amphetamines) with NH4CI traps basic drug in the acidic urine

What is the general byproduct of Phase I metabolism? What is the general byprodu Phase II metabolism? What reactions take place in Phase I metabolism? What reactio, take place in Phase II metabolism? (FAI I p233) (FAQ p260) Phase I Phase II

8. Which hepatic phase of metabolism is lost rst by geriatric patients? Which phase is mediated by cytochrome p450? (FAI I p233) (FAQ p260)

ich medication overdose can be treated with sodium bicarbonate? (FAI I p233) (FAQ p260)

Which medication overdose can be treated with ammonium chloride? (FAI I p233) (FAQ p260) &

Copyright'-) 2012. Doctors In Training.com. LLC. All Rights Reserved.

Alcohol Metabolism and Drug Sufxes


R: Chapter 9 GG: Chapter 23 Ethanol metabolism (FAII p94) (FAI2 plOO) (R p4l2) (GG p630) Disulram (GG p643) Alcohol toxicity (FAII p246) (R p4!3-4l4, 1329) (GG p632) - Methanol (R pi329) (GG p63l) (H p367) - Ethylene glycol (H p367) Drug sufxes (FAII p247) (FAI2 p274)

What are some of the inducers of cytochrome P450? (FAI I p245) (FAQ p273)

What are some of the inhibitors of cytochrome P450? (FAI I p245) (FAQ p273)

What medication inhibits alcohol dehydrogenase? (FAI I p246)

Which medications inhibit acetaldehyde dehydrogenase? (FAI I p246)

lat drug category has the following ending? (FAI I p247) (FAQ p274) -azepam

-azole
-barbital -caine -cychne -navir -operidol -phylline

-tropin

-dronate -cnoi -mustine -statin -bendazole

-sartan -stigmine -curium/ -curonium -glitazone

Copyright 2012. Doctors In Training.com. LLC. All Rights Reserved.

Question Warm-Up
Which cytokine is particularly important in maintaining granulomas? (FAI I p223) (FAQ p What is the function of the lysosome? Outline the pathway by which stimulation of a Gq receptor activates protein kinase < i-FAII n?36) (FAQ p263)

Basic Overview of Endocrine


Phys: Chapter 74 H: Chapter 338 Review of hormone actions (Phys p883) Review of hormone origins (Phys p883) Signaling pathways of endocrine hormones (FAII p294) (FAI2 p32l) (Phys p886) (H p2869) Steroid/thyroid hormone mechanism (FA p295) (FAI2 p322) (Phys p89l)

4. What hormone has the following action(s)?


Stimulates bone and muscle growth Stimulates milk production and secretion Stimulates milk secretion during lactation Responsible for female secondary sex characteristics Stimulates metabolic activity Increases blood glucose level and decreases protein synthesis Responsible for male secondary sex characteristics Prepares endometrium for implantation / maintenance of pregnancy Stimulates adrenal cortex to synthesize and secrete Cortisol Stimulates follicle maturation in females and spermatogenesis in males Increases plasma calcium, increases bone resorption Decreases plasma calcium, increases bone formation Stimulates ovulation in females and testosterone synthesis in males Stimulates thyroid to produce TH and uptake iodine

5. From where are the following hormones secreted?


Growth hormone (GH) Thyroid hormone Glucocorticoids (Cortisol) Progesterone Prolactin Oxytocin Atrial natriuretic hormone (ANH) Glucagon Testosterone Follicle-stimulating hormone (FSH) Vasopressin (ADH) Calcitonin Thyroid-stimulating hormone (TSH) Epinephrine and norepinephrine Insulin Estradiol Estriol Estrone Estrogen in males Parathyroid hormone (PTH) Somatostatin Luteinizing hormone (LH) Mineralocorticoids (aldosterone) Adrenocorticotropic hormone (ACTH)

Copyright 2012. Doctors In Training.com. LLC. All Rights Reserved.

Pituitary and Prolactin


R: Chapter 24 Phys: Chapter 75 H: Chapter 339 Pituitary gland (FAII p288) (FAI2 p3l4) (Phys p89S) (H p2876) Hypothalamic-pituitary hormone regulation (FAII p290) (FAI2 p3l6) (R pl098-l099) (Phys p897) Prolactin - Prolactin regulation (FAII p290) (FAI2 p3l7) (Hp2886) - Hyperprolactinemia (R pi 103) (H p2887) 6. ACTH and MSH (H p2896) ACTH is synthesized as part of a large precursor called proopiomelanocortin (POMC), which also contains the sequences for other hormonal peptides, including the lipotropins, melanocyte-stimulating hormones (MSH) and beta-endorphin. 7. Hyperprolactinemia (FAI I p290, 484) (FAQ p3l7, 538) (H p2887) Causes - Pregnancy/nipple stimulation - Stress (physical or psychological) - Prolactinoma (assodated with bitemporal hemianopia) - Dopamine antagonists: antipsychotics (haloperidol, risperidone), domperidone, metoclopramide, methyidopa Premenopausal female symptoms - hypogonadism infertility, oligo/amenorrhea; rarely galactorrhea Postmenopausal female symptoms - none since already hypogonadal Male symptoms - hypogonadism (low testosterone) decreased libido, impotence, infertility (low sperm counts), gynecomastia, rarely galactorrhea 8. Somatostatin (FAI I p305, 317) (FAQ p334, 346) Produced throughout the Gl tract but notably by D cells in gut mucosa and pancreatic islet cells Also produced throughout the nervous system In the CNS, PNS, and peripheral organs somatostatin decreases endocrine and exocrine secretion, reduces splanchnic blood ow, reduces gastrointestinal motility and gallbladder contraction, and inhibits >.. secretion of most gastrointestinal hormones <3 Clinical Uses for somatostatin analogs (octreotide, somatostatin LAR, and lanreotide-P): Pituitary excesses: acromegaly, thyrotropinoma, ACTH-secreting tumors Gl endocrine excess: Zollinger-Ellison Syndrome, carcinoid syndrome, VIPoma (AKA pancreatic

b_

cholera), glucagonoma, insulinoma I - Certain diarrheal diseases - Need to reduce splanchnic circulation: portal hypertension (bleeding varices), bleeding peptic ulcers LU

Z c C U

Q Z LU

[2]

Copyright 2012. Doctors In Training.com. LLC. All Rights Reserved.

uick Quiz
'. A 50-year-old female complains of double vision, amenorrhea, and headaches. What is

I most likely diagnosis? replacement of tissue in the sella turcica with CSF. What is the mc A patient's MR! reveals likely clinical presentation? I. What hormones arise from the anterior pituitary?

Which hormones share a common alpha subunit?

Other Pituitary Hormones


H: Chapters 339, 340 Pituitary adenoma (FAII p300) (FAI2 p328) (R pi 100) (H p2880) Growth hormone (FAI2 p3l7) (Phys p898) (H p2890) - Acromegaly (FAII p300) (FAI2 p328) (Phys p893) (H p2895) - GH replacement (FAII p305) (FAI2 p334) Oxytocin (FAII p305) (FAI2 p334) (Phys p905) (H p2904)

What is the most common presentation of hyperprolactinemia in a female patient?

What are some of the possible clinical features of acromegaly? (FAI I p300) (FAQ p328)

15. What are some of the dinical uses for somatostatin?

What hormone has the following action(s)? Stimulates milk secretion during lactation Stimulates metabolic activity Increases blood glucose level and decreases protein synthesis Stimulates ovulation in females and testosterone synthesis in i

Copyright 2012, Doctors In Training.com, LLC. All Rights Reserved.

What is the precursor molecule of ACTH synthesis?

What is required for a molecule to enter into the nudeus through a nudear pore?

What are the various clinical applications of atropine? (FAI I p239) (FAQ p266)

Adrenal Steroids
R: Chapter 24 Phys: Chapter 77 H: Chapter 342 Adrenal cortex and medulla (FAII p288) (FAI2 p3l4) (R pi 148) (Phys p92l) Fetal adrenal gland (FAI2 p3l4) Steroid synthesis (FAII p29l) (FAI2 p3l8) (Phys p923) (H p2940) Congenital bilateral adrenal hyperplasia (FAII p29l) (FAI2 p3l8) (R pll52) (H p2959) Review of action of Cortisol, aldosterone - Cortisol (FAII p292) (FAI2 p3l9) (Phys p928) - Glucocorticoids (FA p305) (FAI2 p334) - Aldosterone (Phys p924) - Fludrocortisone

4. Complete the steroid synthesis pathway:

in

t 1 -,M

20 ( 21

a
^H

(^ M B

22C

Copyright @ 2012. Doctors In Training.com. LLC. All Rights Reserved.

Congenital Adrenal Hyperplasia (R pi 152) 5. What features characterize a deciency in 3 B-hydroxysteroid dehydrogenase?
Inability to produce

6. What features characterize a deciency in 17 a-hydroxylase?


Inability to produce sex hormones and Cortisol Increased production of mineralocorticoids (i.e. aldosterone) sodium and fluid retention

7. What features characterize a deciency in 21 a-hydroxylase?


Inability to produce Cortisol * Inability to produce mineralocorticoids Increased production of sex hormones

8. What features characterize a deciency in 11 B-hydroxylase? Inability to produce


Increased production of deoxycorticosterone (a weak mineralocorticoid) Increased production of sex hormones *

Cushing Syndrome
R: Chapter 24 Phys: Chapter 77 H: Chapter 342

Cushing syndrome (FAII p296) (FAI2 p323). (R pi 148) (H p2945)

Quick Quiz
9. An adult male with elevated serum Cortisol levels and signs of Cushing syndrome undergoes a dexamethasone suppression test I mg of dexamethasone does not decrease Cortisol levels, but 8 mg does. What is the diagnosis? 10. What are the symptoms of 21 a-hydroxylase deciency? Of II B-hydroxylase deciency? (FAII p29l)(FAI2p3l8)

o n
m
to
>

I. What food substance is an essential starting point in the synthesis of adrenal steroids? (FAII p29l)(FAI2p3l8)

12. What effect does Cortisol have on bone formation and immune system functioning? (FAII p292)(FAI2p3l9)
Copyright 2012. Doctors In Training.com. LLC. All Rights Reserved

3 = 5
e n

[5]

Other Adrenal Pathology


R: Chapter 24 Phys: Chapter 77 H: Chapters 342, 343, 351 Hyperaldosteronism (FAII p296) (FAI2 p323) (R pi 151) (H p2949) Adrenal insufciency (R pi 154) (H p2954) - Primary: Addison disease (FAII p296) (FAI2 p324) (R pll55) - Primary: Waterhouse-Friderichsen syndrome (FAII p296) (FAI2 p324) (R pll55) - Secondary (FAII p296) (FAI2 p324) (R pi 157) - Tertiary Stress (FAII p6l) (FAI2 p63) Pheochromocytoma (FAII p297) (FAI2 p324) (R pi 159) (Hp2962) Neuroblastoma (FAII p297) (FAI2 p324) MEN (FAII p303) (FAI2 p332) (R pi 161) (H p3072)

d of Session Quiz
...... . A very tan child with a pale mother presents to your clinic and is found to be hypotensive. What is the most likely diagnosis?

. What are all the potential side effects of glucocorticoid use? (FAI I p305) (FAQ p334)

. What are the clinical manifestations of Addison disease? What is the cause of Addison disease? (FAII p296)(FAI2p324)

. What tumor locations are associated with the 3 different types of multiple endocrine neoplasia? (FAI I p303) (FAQ p332)

'. What might a lab detect in the urine of a patient with pheochromocytoma7 (FAI I p297) (FAI2p324)

Copyright 2012. Doctors In Traming.com, LLC. All Rights Reserved.

3 Question Warm-Up
!HI!lrafe^:vj[ii .JH55S!
(FAQ p323-324) Disease Cushing syndrome Conn syndrome Addison dise Graves disease 2. What is the drug of choice for anaphylactic shock? Cardiogenic shock? Septic shock? (FAII p240) (FAQ p 266) Hormone that is increased or decreased

What cancers are associated with RET gene mutation?

Thyroid Basics and Hyperthyroidism


R: Chapter 24 Phys: Chapter 75 H: Chapter 341 Thyroid development (FAII pl30) (FAI2 pl38) (H p29ll) Thyroid histology (Phys p907) Thyroid hormones (FAII p295) (FAI2 p322) (R pi 107) (Phys p 907) (H p29l3) Hyperthyroidism vs. hyperthyroidism (FA p297) (FAI2 p325) Hyperthyroidism (FAII p298) (FAI2 p326) (R pi 108) - Graves disease (R pi 114) (H p2922) - Thyroid storm - Toxic multinodular goiter (R pi 116) (H p2932) -)od-Basedow reaction (H p2930) - Thyroid secreting teratoma (H p2927) Propylthiouracil, methimazole (FAII p305) (FAI2 p334) (GGpll47)

4. Classication of Eye Changes in Graves Disease

p
p
m *

j I

P
pi m

0 0 No signs/symptoms |i O n l y s i g n s , n o s y m p t o m s AI _ I Soft tissue involvement ) i ; Proptosis 4 4 I Extraocular muscle involvement | I ) > | Corneal involvement ' I Vi s i o n l o s s ( o p t i c n e r v e i n v o l v e m e n t ) 6

Copyright 6 2012. Doctors In Training.com. LLC. All Rights Reserved.

A 35-year-old female presents with diffuse goiter and hyperthyroidism. What are the most likely relative values of TSH and thyroid hormones?

What would you suspect to be the cause of hyperthyroidism in a patient presenting with the symptoms of hyperthyroidism in addition to the following ndings? (FAI I p297-298) (FAQ p325-326) Extremely tender thyroid gland Pretibial myxedema Pride in recent weight loss, medical professional Palpation of single thyroid nodule Palpation of multiple thyroid nodules Recent study using IV contrast (iodine) Eye changes: proptosis, edema, injection History of thyroidectomy or radioablation of thyroid What name is given to a thyroid-secreting teratoma7 (FAI I p49l) (FAQ p546)

8. What is the most common location for ectopic thyroid tissue? (FAI I pi30) (FAQ pl38)

Hypothyroidism and Thyroid Cancer


R: Chapter 24 Phys: Chapter 77 H: Chapter 341 Hypothyroidism (FAII p298) (FAI2 p325) (R pi 109) (H p29l9) - 5=1 Goiter (Rplll6)(H p2930) <3 Thyroid cancer (FAII p298) (FAI2 p326) (Rplll9)(H p2933) >v. Recurrent laryngeal nerve injury (FAII p404) (COA pi045)

Copyright 2012. Doctors In Traimng.com. LLC. All Rights Reserved.

'. A 48-year-old female has been suffenng with progressive lethargy and extreme sensrtivrty cold temperatures. What is the most likely diagnosis?

0. What type of thyroid cancer matches the following statement7 Most common type of thyroid cancer (70-75%) Second most common type of thyroid cancer (10%) Activation of receptor tyrosine kinases 1 Cancer arising from parafollicular C cells Commonly associated with either a RAS mutation or a PAX8-PPAR gamma I rearrangemen Commonly associated with rearrangements in RET oncogene or NTRKI 1 Most common mutation in the BRAF gene (serine/threonine kinase) I' Hashimoto thyroiditis is a risk factor 11. What is the mechanism of action of propylthiouraal? What other drug works like PTU? What are their side effects? (FAI I p305) (FAQ p334)

How would pregnancy affect serum thyroid hormone levels? (FAI I p295) (FAQ p322)

What is the medication of choice for the treatment of atrial brillation due to hyperthyroidism

Copyright Si 2012. Doctors In Traimng.com, LLC. All Rights Reserved.

What are the tumor markers for pancreatic cancer? (FAI I p336) (FAQ p367) Where would you expect to nd B cells and T cells in the spleen? In the lymph nodes? (FAI I p200)(FAI2p222)

3. What drug category has the following ending? (FAI I p247) (FAQ p274)
ane azine tidine tropin chol bendazole -azepam -conazole -triptyline -zosin -stigmine -dipine

Endocrine Pancreas and DM


Phys: Chapter 78 H: Chapter 344 Endocrine pancreas cell types (FAII p289) (FAI2 p3l5) (R pi 130) (Phys p939) Glucagon (FAI2p3l6)(R pi 132) (Phys p947) Insulin (FAII p289) (FAI2 p3l5) (Phys p939) Insulin-dependent organs (FAII p289) (FAI2 p3l6) Diabetes mellitus (FAII p30l) (FAI2 p329) (R pi 131) Sorbitol (FAIIpl04) (FAI2 pl09) Type I vs. type 2 DM (FAII p30l) (FAI2 p330) (R pi 134)

How is hemoglobin glycosylated in DM to form HgbAlc? Which type of diabetes mellitus ts the following descriptions? (FAI I p30l) (FAQ p330) Associated with obesity May cause ketoacidosis Strong genetic predisposition Associated with HLA DR3 & DR4 >. In which tissues will you nd GLUT-2 receptors? (FAI I p289) (FAQ p315) 7. Which tissues depend on insulin for glucose uptake? (FAI I p289) (FAQ p315) '. Which tissues are allowed to uptake glucose even in the absence of insulin because they use a GLUT-I receptor? (FAI I p289) (FAQ p315)

Copyright 2012. Doctors In Training.com, LLC, All Rights Reserved.

DKA and DM Medications


H: Chapter 344 GG: Chapter 43

Diabetic ketoacidosis (FAII p302) (FAI2 p330) (H p2976) Diabetes drugs (FAII p304) (FAI2 p333) (GGpl248, 1255) (H p2996)

9. Common causes of DKA


(Almost always related to a state of excess glucagon, catecholamines, or corticosteroids) Infection (pneumonia, gastroenteritis, UTI) Diabetic medication reduction or omission Severe medical illness (Ml, CVA, trauma) Undiagnosed DM Dehydration Alcohol or drug abuse (especially stimulants) Corticosteroids

10. Newer DM Agents


Sitagiiptin (Januvia) and Saxagliptin (Onglyza) - Inhibitors of dipeptidyl peptidase IV (DPP-IV) which affects glucagon-like peptide (GLP-I) among other hormones - Prolongs incretin actions, which decreases glucagon secretion and increases insulin secretion, delays gastric emptying Exenatide (Byetta) and Liraglutide (Victoza) - Exenatide is an analog of exendin, a hormone (derived from Gila monster saliva) with actions similar to GLP-I - Liraglutide is a synthetic analog of human GLP-1 - Mimic the actions of incretins, which decrease glucagon secretion and increase insulin secretion, delay gastric emptying. - Not approved for use while on insulin therapy - SE: possibly increased risk of acute pancreatitis. Pramlintide (Symlin) - Amylin analog, normally secreted with insulin, decreases glucagon secretion and gastric emptying m - Used only in patients taking insulin but in either type I or type 2 DM patients p-

o n
z m
I
"0 D

Copyright 2012. Doctors In Training.com. LLC. All Rights Reserved.

[II]

characteristics? (FAI 1 p304) (FAQ p333) Lactic acidosis is a rare but worrisome side effect Most common side effect is hypoglycemia Often used in combination with any of the other oral agents Also help lower triglycerides and LDL cholesterol levels Not safe in settings of hepatic dysfunction or CHF Should not be used in patients with elevated serum creatinine Should not be used in patients with liver cirrhosis, elevated serum creatinine, or inammatory bowel disease Hepatic serum transaminase levels should be carefully monitored when using these agents Not associated with weight gain, often used in overweight diabetics Metabolized by liver; excellent choice in patients with renal disease Primarily effects postprandial hyperglycemia MOA: closes K channel on B cells * depolarization Ca2+ influx insulin release MOA: inhibits a-glucosidase at intestinal brush border MOA: agonist at PPARv receptors improved target cell response to insulin

12. A 40-year-old male comes to the emergency room for the treatment of DKA. His type I diabetes is normally well controlled, and he doesn't know why his DKA developed this time. What is the differential diagnosis for the development of his DKA? (FAII p302) (FAQ p330)

Copyright @ 2012. Doctors In Trammg.com. LLC. All Rights Reserved.

est ion warmWhat co-factors are required for the function of pyruvate dehydrogenase? What other enzyme requires the same co-factors? (FAI I p99) (FAQ pi04)

What medications are used to shrink prolactinomas? (FAI I p300) (FAQ p328)

What are the functions of interleukins 1-5? (FAI I p208) (FAQ p230)

Physiology of Obesity
Phys: Chapter 71 H: Chapter 77 BMI (FAII p6l) (FAI2 p63) (Phys p850) (H p622) Adipocytes (Phys p82l) (H p624) Leptin (FA p398) (FAI2 p436) (Phys p846) (H p625) Hormonal regulation of fat utilization (Phys p825) Lipodystrophy (H pl993)

alamus? (FAI2p436)

What effect does stress have on adipocytes?

>. What are some of the causes of lipodystrophy?

Copyright 2012. Doctors In Training.com. LLC. All Rights Reserved

Metabolic Syndrome and Obesity


Phys: Chapter 71 R: Chapter 9 H: Chapters 77, 78, 242 Metabolic syndrome (H p\992) Obesity (Phys p850) (R p442) (H p629) Medical complications of obesity (R p442) (H p627) Recommended weight loss Methods of weight loss (H p634) (Phys p85l)

7. Metabolic Syndrome Diagnostic Criteria (ATPIII Clinical Criteria, Circulation 2005;l 1:1883) (H pi992) Diagnosis based on any 3 of the following:
Abdominal obesity: Waist circumference > 40in (102cm) in men, or > 35in (88cm) in women - (IDF criteria is a 94cm in men, and a80cm in women) - (Recognize that not all metabolic syndrome patients are overweight!) Triglycerides a 150 mg/dl HDL <40mg/dl in men, or <50mg/dl in women

BE a 130/85
Fasting serum glucose a lOOmg/dl (or 2hr post oral glucose a 140 mg/dl)

8. NASH: Nonalcoholic Steatohepatitis (H p2604) (Phys p860)


Most common causes: Obesity, DM II, Hyperlipemia, Insulin-resistance Due to insulin-resistance at the liver excess lipid accumulation in the liver Can progress to cirrhosis, lead to hepatocellular carcinoma, worsen Hep C progression Suspect if chronically elevated ALTs Diagnosis - Liver US, CT scan, or MRI can make the diagnosis. - Magnetic resonance spectroscopy (MRS) is the gold standard - Liver biopsy can also make the diagnosis. It is the only manner to identify those at risk for disease progression

Treatment
- Avoidance of all alcohol - Weight loss - most likely benecial, but no proven benet -H** - Control any diabetes mellitus aggressively to keep HbAI C < 7.0 5ft - TZDs (pioglitazone) - improves LFTs and possible histology improvement _Q - Metformin shows improvement in animal studies

I 9. Weight Loss Drugs (H p634) LT ) O r l i s t a t LU - Alters fat metabolism by inhibiting pancreatic lipases Z - Used for long-term obesity management (in conjunction with modied diet)
QC* - Side effects indude steatorrhea, Gl discomfort, and reduced absorption of fat-soluble vitamins

vJ Sibutramine O " Sympathomimetic serotonin and norepinephrine reuptake inhibitor Q - Used for short-term and long-term obesity management ~7 - Side effects indude hypertension and tachycardia, headache LU - Avoid SSRIs or MAOIs
- Contraindicated in CAD, cerebrovascular disease, CHF, arrhythmia

[14]

Copyright 2012. Doctors In Trainmg.com. LLC. All Rights Reserved.

uti

of Session Qui2

In which patient populations is sibutramine contraindicated?

What are the ve categories of criteria for the diagnosis of metabolic syndrome?

What liver disease is associated with obesity?

At what BMI is a patient considered obese? (FAI I p6l) (FAQ p63)

Copyright 2012. Doctors In Training.com, LLC. All Rights Reserved.

In osteomalacia and rickets, how will serum Ca2+, phos, alk phos, PTH, urine Ca2+, and urine phos compare to normal values? Which hormones work via tyrosine kinase second messengers?

Basics of Calcium Metabolism


Phys: Chapter 79 H: Chapters 46, 352, 353 Embryologic origin of parathyroids Parathyroid anatomy and histology (COA pi020) PTH (FAII p293) (FAI2 p320) (R pi 126) (Phys p962) (H p3096) Vitamin D (FAII p294) (FAI2 p32l) (Phys p960) (H p3092) Calcitonin (FAII p294) (FAI2 p32l) (Phys p966) Hypercalcemia (H p3099)

4. Hypercalcemia
90% Primary hyperparathyroidism - Solitary parathyroid adenoma (85%) - Parathyroid hyperplasia (15%) Malignancy - Squamous cell cancers (especially lung) (via PTH-related peptide) - Renal cell carcinoma - Breast cancer metastasis - Multiple myeloma (via local osteolytic factors) 10% Excess vitamin D ingestion Excess antacid ingestion = milk-alkali syndrome Granulomatous dz (excess vitamin D due to sarcoid or TB) Increased bone turnover (hyperthyroidism, vitamin A intoxication, immobilization) Thiazide diuretics decreased renal excretion of calcium 5. Signs/Symptoms of Primary Hyperparathyroidism Stones Bones Abdominal Groans

Psychic moans

Other

Copyright 2012. Doctors In Training.com. LLC. All Rights Reserved.

Quiz
. What are the 3 functions of vitamin D? ''. How does PTH affect calcium? How does PTH affect phosphate; 8. Which cancers are associated with hypercalcemia? 9. What cell type produces PTH? What cell type produces calcitonin? (FAII p293) (FAQ p 320)

Hyperparathyroidism and Hypoparathyroidism


Phys: Chapter 79 H: Chapters 46, 353 Hyperparathyroidism (FAII p299) (FAI2 p327) (R pll27) (H p3IOO) Causes of hypocalcemia (H p36l) Hypoparathyroidism (FAII p299) (FAI2 p327) (R pll29) Causes of hypoparathyroidism (H p3H4)

10. Hypocalcemia
Hypoparathyroidism - Parathyroidectomy (from damage during thyroidectomy) - Autoimmune destruction of parathyroids - Pseudohypoparathyroidism (kidneys unresponsive to PTH) - DiGeorge syndrome Poor calcium intake Vitamin D deficiency - Nutritional deciency and paucity of sunlight - Chronic renal failure Acute pancreatitis

11. What are two signs of hypocalcemia?

What are two the most common causes of primary hyperparathyroidism?

What are some possible causes of hypocalcemia?

Copyright.,' 2012. Doctors In Training.com. LLC. All Rights Reserved.

Question Warm-Up
Place the following agents in the appropriate categories: (FAI I p238 - FAI I p242) (FAQ p265-269) norepinephrine, metoprolol, timolol, scopolamine, phenoxybenzamine, bethanechol, isoproterenol, donepezil, prazosin, atropine, propranolol, labetalol, hexamethonium, atenolol, terbutaline, dopamin pilocarpine, carbachol, edrophonium, phenylephrine, glycopynvlate, phentolamine, neostigmine, benztropine, terazosin, pralidoxime, echothiophate, ipratropium, epinephrine, esmolol, oxybutynin Indirect cholinergic agonists
Ife!lMB;WHMMlMillT*nfcJrW5r1

02 Bi 32agonist02 Bi agonist= D2 > Bi > a I agonist= B2 agonist> Bi agonist> 02 agonist-

Nonselective (02 = Bi): Bi selective:

Which portions of the hypothalamus are inhibited by leptin? Which are stimulated? (FAII p398) (FAQp436)

Which cranial nerve relays the following information? (FAI I p4l6) (FAQ p456) Hypoxia measured by the carotid body Motor information for swallowing Blood pressure from the aortic arch Salivation from the sublingual glands Salivation from the parotid gland Blood pressure from the carotid

Copyright 2012. Doctors In Trainmg.com. LLC. All Rights Reserved.

Mouth, Tongue, Salivary Glands


R: Chapter 16 Phys: Chapter 64 Embryology - Tongue development (FAII pl29) (FAI2 pi37) - Cleft lip and cleft palate (FAII pi30) (FAI2 pi38) Tongue Salivary glands Salivary gland tumors (FAII p32l) (FAI2 p350) - Pleomorphic adenoma (R p757) - Warthin tumor (R p759) - Mucoepidermoid carcinoma (R p759)

What is the most common location of salivary gland tumors?

What is the most common salivary gland tumor? What is the histological appearance of this tumor?

nat is the second most common benign salivary gland tumor?

What is the most common malignant salivary gland tumor (the second most common tumor overall of the salivary gland)?

What nerves innervate the tongue? (FAI I pl29) (FAQ pi37)

Copyright Q 2012. Doctors In Training.com. LLC. All Rights Reserved.

Nose and Sinuses


R: Chapter 16 H: Chapter 317 Nose Eustachian tubes Paranasal sinuses Mucormycosis (H pi661) Tonsils and adenoids

9. Paranasal sinuses

Lippincott Williams S Wilkins Atlas of Anatomy 2003 Wollers Kl jwer Health, Inc. All rights reserved.

Copyright ) 2012. Doctors In Training.com. LLC. All Rights Reserved.

stion Warm-Up
Which HIV medications cause pancreatitis? Which cause rash? neuropathy and lactic acidosis? (FAI I pl97) (FAQ p2l8)

What is the mechanism of type IV hypersensitivity? (FAI I p2IO) (FAQ p233)

What birth defect can result from the following teratogenic agents? (FAI I pQO) (FAQ pl2 ACE inhibitors Folate antagonists Tetracyclines Lithium

Basics of the Esophagus


R: Chapter 17 H: Chapter 292 Overview of the cells, functional units, and function of the digestive tract - Digestive tract histology (FAII p3IO) (FAI2 p338) Embryology - Gl embryology (FAII pl30) (FAI2 pl38) - T fistula (FAII pi31) (FAI2 pi39) Anatomy - Digestive tract anatomy (FAII p3IO) (FAI2 p338) - Esophageal anatomy (FAII p3IO) Diverticula (H p2429) - Zenker diverticulum (FAII p327) (FA12 p356) - Traction - Epiphrenic GE junction Nitric oxide (FAII p3l7) (FAI2 p346) Achalasia (FAII p322) (FAI2 p3SI) (H p2430)

Copyright 2012. Doctors In Traimng.com, LLC. All Rights Reserved.

4. Abdominal Wall Defects Omphalocele


- Defect in abd wall; extruding viscera covered by sac (sac composed of peritoneum and amnion) - Liver often found protruding - Other anomalies common (50%) - (Gl, GU, CV, CNS, MS)

Gastroschisis - Defect in abd wall; extruding viscera not covered by sac - Liver never found protruding - Other anomalies less common (10-15%) - Defect lateral to umbilicus (R > L)

Rx: reduction and closure (silo if necessary)

5. Annular Pancreas
Two-thirds of patients remain asymptomatic throughout their life Symptom onset can occur at any age of life and depends on the severity of duodenal constriction Children present with gastric outlet obstruction (bilious vomiting, feeding intolerance, abdominal distention) In infants, it is associated with polyhydramnios, Down syndrome, esophageal and duodenal atresia, imperforate anus, and Meckel diverticulum Adults usually present at 20-50 years of age with abdominal pain, postprandial fullness and nausea, peptic ulceration, pancreatitis, and rarely biliary obstruction

6. Esophageal diverticula are named according to their location in the esophagus:


Zenker diverticulum (FAI I p327) (FAI2 p356) - Immediately above the upper esophageal sphincter Traction diverticulum - Near the midpoint of the esophagus Epiphrenic diverticulum - Immediately above the lower esophageal sphincter

A newborn is having trouble feeding. He vomits milk when fed and has a gastric air bubble on X-ray. What kind of stula is present7 What substance is important for relaxing the lower esophageal sphincter? (FAI I p3l7) (FAQ p346)(FAIIp322)(FAI2p35l) What are the two nervous tissue bundles that run through the Gl tract and where are they located? (FAII p3IO) (FAQ p338) What are the names of the diverticula found in the esophagus based on their anatomical locations?

Copyright 2012. Doctors In Training.com, LLC. All Rights Reserved.

Esophageal Pathology
H: Chapters 91, 292 R: Chapter 17 Esophageal pathologies (FAII p322) (FAI2 p35l) Barrett esophagus (FAII p322) (FAI2 p352) Esophageal cancer (FAII p323) (FAI2 p352) (H p764)

l?iTTTg>i?>7fjiT*jMM
A 60-year-old man with chronic reux presents with esophageal cancer What is the m< likely histologic subtype? What upper Gl problem is associated with the following ndings/symptoms? Specialized columnar epithelium seen in a biopsy from distal esophagus Biopsy of a patient with esophagitis reveals large pink intranuclear inclusions and host cell chromatin that is pushed to the edge of the nucleus Biopsy of a patient with esophagitis reveals enlarged cells, intranuclear and cytoplasmic inclusions, and a clear perinuclear halo An esophageal biopsy reveals a lack of ganglion cells between the inner and outer muscular layers Biopsy of mass in parotid gland reveals a double layer of I columnar epithelial cells resting on a dense lymphoid stroma Protrusion of the mucosa in the upper esophagus Outpouching of all layers of the esophagus found just above the LES Basal cell hyperplasia, eosinophilia, and elongation of lamina propria papilla seen in biopsy of esophagus I Goblet cells seen in the distal esophagus A PAS stain on a biopsy obtained from a patient with esophagitis reveals hyphate organisms Esophag

Copyright 2012. Doctors In Traming.com. LLC. All Rights Reserved.

To what immune cell does the following c (FAII p209) (FAQp23l) CD4 CDI4 CDI6 CDI9

What is the clinical use for the following antimuscarinic drug(s)? (FAI I p239) (FAQ p266) Ipratropium Atropine, homatropine, tropicamide Benztropine Scopolamine Distinguish the following types of organ transplant rejection. Mechanism Time Frame

Basics of the Stomach


H: Chapter 293 R: Chapter 17 Overview of blood supply to the Gl tract (COA p250) - Abdominal aorta and branches (FAII p3ll) (FAI2 p339) - Gl blood supply and innervations (FAII p3ll) (FAI2 p339) - Collateral circulation (FAII p3l2) (FAI2 p340) - Celiac trunk (FAII p3l2) (FAI2 p340) Gl secretory products (FAII p3l8) (FAI2 p347) Zollinger-Ellison syndrome (FAII p302) (FAI2 p33l)

Copyright 2012, Doctors In Training.com, LLC. All Rights Reserved.

ick Quiz
(FAN p3ll)(FAI2p339)

What are the arterial branches off of the common hepatic artery? (FAI I p3Q) (FAQ p3'

What disorder is characterized by a excess of gastrin? What is the treatment? (FAI I p302) (FAI2p33l)

Gastric Physiology
Phys: Chapter 64 H: Chapter 293 GG: Chapter 45 Physiology of stomach acid secretion Neurologic control of the stomach Gastric parietal cell (FAII P3I9) (FAI2 p348) Gl therapy (FAI I p337) (FAI2 p367)

What receptors found on gastric parietal cells regulate acid secretion? (FAI I p3l9) (FAQ p

What happens to serum pH at the time of gastric add secretion? (FAI I p3l9) (FAQ p348)

What cell type secretes the histamine that stimulates the histamine receptor on parietal thereby increasing gastric add production? (FAI I p337) (FAQ p367)

The vagus nerve stimulates G cells to release gastrin; however; the administration of a muscarinic antagonist such as atropine will inhibit the release of gastrin. Why?

Copyright 0 2012, Doctors In Training.com. LLC. All Rights Reserved.

How does the drug dose response curve change with the addition of a competitive antagoni: compared to a noncompetitive antagonist7 (FAI I p234) (FAQ p26l) Which autoimmune disorder matches the following description? (FAI I p2Q) (FAQ p235)
Anti-TSH receptor antibodies Antimitochondrial antibodies Anticentromere antibodies Antihistone antibodies Anti-smooth muscle antibodies

Which receptors use a Gs protein? (FAI I p236) (FAQ p263)

Gastritis, PUD, and Antacid


H: Chapter 293 Phys: Chapter 64 GG: Chapter 45 Gastritis (FAII p324) (FA12 p354) Peptic ulcer disease (FAII p325) (FAI2 p354) Treatment of peptic ulcers (FAI2 p354) Pharmacotherapy for diseases related to stomach acid - Antacid use (FAII p338) (FAI2 p369) - H2 blockers (FAII p337) (FAI2 p368) - PPIs (FAII p337) (FAI2 p368) - Bismuth (FAII p338) (FAI2 p368) - Sucralfate (FAII p338) (FA12 p368) - Misoprostol (FAII p338) (FAI2 p368) - Muscarinic antagonists (FAII p338) - Octreotide (FAII p338) (FAI2 p368)

uick Quiz
peptide ulcer? (FAI I p325) (FAQ p354) What enzyme is inhibited by PPIs? Name two different PPIs. (FAI I p337) (FAQ p368) What drugs and endogenous hormones regulate the secretion of gastric add? (FAI I p3l8) (FAI2p347) Which antadd matches the following statement? (FAI I p338) (FAQ p369) May cause diarrhea May cause constipation May cause rebound hypercalcemia May cause hvDokalemia
Copyright 2012. Doctors In Trainmg.com. LLC. All Rights Reserved.

Other Gl Pathology
H: Chapters 39, 91 GG: Chapter 46 Menetrier disease (FAII p324) (FAI2 p354) Stomach cancer (FAII p324) (FAI2 p354) Ondansetron (FAII p339) (FAI2 p369) Pyloric stenosis (FAII pi31) (FAI2 pi39)

' afTT* fT*WM M i w*TnV>


What upper Gl problem is assodated with the following ndings/symptoms? Stomach biopsy reveals neutrophils above the basement membrane, loss of surface epithelium, and brin-containing purulent exudate Small intestine biopsy reveals small lymphocytes with irregular nuclear contours and proliferation of these lymphocytes into the I mucosa and epithelial glands [ Stomach biopsy reveals lymphoid aggregates in the lamina propria, columnar absorptive cells, and atrophy of glandular structures Most common cause of duodenal ulcers | Diffuse thickening of gastric folds, elevated serum gastrin levels, biopsy reveals glandular hyperplasia without foveolar hyperplasia A patient taking NSAIDs forthe management of her gout develops anemia, has pain with eating and is positive on occult blood test What drug would most directly address the mechanism behind this patient's current problem?

Where is Virchow's node located? (FAI I p324) (FAQ p354)

An infant presents to the ER with vomiting. What presenting features would lead you to suspect congenital pyloric stenosis? (FAI I pl3l) (FAQ pl39) Age - 2-6 weeks old Male > female (classic presentation is 1st bom male) Nonbilious, projectile vomiting Dehydration Possibly hypokalemic, hypochloremic metabolic alkalosis (if vomiting for at least 5 days) Palpable olive-like bulge on abdominal exam

Copyright .j 2012. Doctors In Training.com. LLC. All Rights Reserved.

Which immunosuppressant ts the following description? (FAI I p2l5-2l6) (FAQ p240) Derivative of 6-mercaptopurine Causes phocomelia Nephrotoxic in 75% of patients

SE: acne, osteoporosis, HTN, hyperglycemia, immunosuppression * infection


Inhibits secretion of IL-2 and other cytokines Alkylating agent that requires bioactivation in liver Inhibits dihydrofolate reductase Which alpha adrenergic antagonists are used in the treatment of pheochromocytoma? (FAII p24l) (FAQp268) How is hnF\NA processed before it leaves the nucleus? (FAI I p72) (FAQ p75)

Duodenum
R: Chapter 17 Phys: Chapter 64 Embryology - Duodenal atresia (FAII p328) (FAI2 p358) - Pancreas and spleen embryology (FAII pl3l) (FAI2 pl39) Basic intra-abdominal anatomy - Retroperitoneal structures (FAII p308) (FA12 p336) - Important Gl ligaments (FAII p309) (FAI2 p337) - Portosystemic anastomoses (FAII p3l3) (FA12 p) Ampulla ofVater and sphincter ofOddi Gl hormones of the duodenum (FAII p3l7) (FAI2 p346) Enteric nervous system Prokinetici'motility agents (GG pl325) - Macrolides - Metoclopramide (FAII p339) (FAI2 p370) Brunner's glands (FAII p320) (FAI2 p349)

4. Prokinetic Agents (FAI I p338-339) (FAQ p369-370) TACh, T5-HT (carcinoid syndrome), ID2
Cholinergic agonists (bethanechol)
Acetylcholine-esterase inhibitors (neostigmine) Metoclopramide (Reglan) - (+5-HT4, -D2) Domperidone - (-D2) Cisapride - (+5-HT4) Macrolides - stimulate smooth muscle motilin receptors

Copyright 2012. Doctors In Training.com. LLC. All Rights Reserved.

Quick Quiz
r a 1 ' 5. \What1important secretory products are secreted from the following cells of the Gl t.

G cells I cells S cells

(FAII p3l7)(FAI2p346)

Parietal cells I - Chiefl cells Dcel s What Gl ligament matches the following description? (FAI I p309) (FAQ p337) Contains the portal triad and may be compressed to control bleeding Attaches the spleen to the posterior abdominal wall Attaches the spleen to the stomach

What is the mechanism of action of metoclopramide? (FAI I p339) (FAQ p370)

Pancreas
Phys: Chapter 64 R: Chapter 19 H: Chapters 93, 312, 313 Pancreatic enzymes (FAII p320) (FAI2 p349) Carbohydrate digestion (FAII p320) (FAI2 p349) Cystic brosis (FAII p87) (FAI2 p9l) Acute pancreatitis (FAII p335) (FAI2 p335) Chronic pancreatitis Pancreatic adenocarcinoma (FAII p336) (FAI2 p367)

Copyright'; 2012. Doctors In Traming.com. LLC. All Rights Reserved.

Which hormones stimulate pancre

Jon?(FAII p3l7)(FAI2p346)

What enzyme catalyzes the rate-limiting step in carbohydrate digestion? (FAI I p320) (FAI2p349)

i. After the loss of his job, a 35-year-old man has diarrhea and hematochezia. Intestinal biopsy shows transmural inammation. What is the diagnosis?

. A weight lifter undergoes emergency surgery for a life threatening condition. Examination of a section of his small bowel reveals focal hemorrhages. What is the process responsible for this?

What are the possible etiologies of acute pancreatitis? (FAI I p335) (FAQ p366)

at is the typical presentation of a patient with pancreatic insufciency? What is the treatment for pancreatic insufciency? (FAI I p323) (FAQ p353)

Copyright 2012, Doctors In Training.com. LLC. All Rights Reserved.

uestion Warm-Up
To which lymph nodes does cancer in the following areas most commonly metastasize? (FAII p200) (FAQp222) Stomach Duodenum, jejunum Sigmoid colon Rectum Testes Scrotum

What is the cause of achalasia7 How is achalasia diagnosed? (FAII p322) (FAI2p35l)

Name 5 drugs that inhibit acetylcholinesterase. What is the dinical application for each? (FAII p238) (FAQp265)

Enterocytes and Absorption


R: Chapter 17 Phys: Chapter 65 H: Chapter 294 Crypts ofLieberkuhn (Phys p787) Overview ofenterocyte functions Chylomicrons Carbohydrate absorption Selected vitamin and mineral absorption (FAII p320) (FAI2 p349) Malabsorption syndromes (FAII p323) (FAI2 p353) Celiac sprue (FAII p324) (FAI2 p353)

Copyright 2012. Doctors In Training.com. LLC All Rights Reserved.

B12 Absorption 4. What are the steps involved in the intake and absorption of B12?
Pepsin in the stomach releases B12 pt from its protein bound form Still in the stomach, B12 pt binds to salivary vitamin B12 binding proteins (called cobalophilins or R-binders) R-B12 complex is broken down in duodenum by pancreatic proteases Still in the duodenum, unbound B12 binds to (secreted by the cells in the stomach) IF-B12 complex binds to IF-specific receptors on cells of the B12 transverses the plasma membranes of the mucosal cell and is picked-up by a plasma protein called transcobalamin II

5. What specic organs are involved in the absorption of vitamin B12?


Salivary glands, stomach, pancreas, distal ileum

6. What is a Schilling test?


Radiolabeled cyanocobalamin given orally to determine if it was absorbed by the ileum Measurement of urinary excretion of radiolabeled B12 over a 24 hours period - Normal = >8% of the oral dose recovered in the urine

7. What are the three stages of the Schilling test?


1) Administration of radiolabeled B12 without intrinsic factor (to determine if there is a problem absorbing B12) 2) Administration of radiolabeled B12 with intrinsic factor (to determine if a lack of intrinsic factor is the cause of the problem) 3) Administration of radiolabeled B12 with pancreatic supplements (to determine if a lack of enzymatic degradation of R protein is the problem) 4) Administration of radiolabeled B12 after administration of antibiotics (tetracycline) or anti-inammatory drugs (prednisone) (to determine other causes of B12 malabsorption such as bacterial overgrowth)

Copyright 2012, Doctors In Training.com, LLC. All Rights Reserved.

/^

Diseases of the Small Intestine


H: Chapters 295, 296, 298, 299 R: Chapter 17 Crohn's disease (FAII p326) (FAI2 p355) - Sulfasalazine (FAII p339) (FAI2 p369) IBS(FAIIp326)(FAI2p3S6) Meckel diverticulum (FAII p327) (FAI2 p357) Small bowel obstruction - Adhesion (FAII p328) (FAI2 p358) Intussusception (FAII p328) (FAI2 p357) Ileus - Meconium ileus (FAII p328) (FAI2 p358) Necrotizing enterocolitis (FAII p328) (FAI2 p358) Ischemic colitis (FAII p328) (FAI2 p358) Angiodysptasia (FAII p328) (FAI2 p3S8) Carcinoid syndrome (FAII p302) (FAI2 p3S9)

12. What treatments are available for managing Crohn's disease? 5-ASA agents (e.g., mesalamine, sulfasalazine) - usual initial therapy for mild disease Azathioprine or mercaptopurine > methotrexate Anti-TNF agents (e.g., infliximab, adalimumab) Steroids +/- antibiotics for acute exacerbations 13. Irritable Bowel Syndrome (IBS): ROME II Diagnostic Guidelines At least I2wks of abdominal discomfort or pain in the preceding year assodated with one of the following: - Relief with defecation - Change in frequency of stool - Change in form of stool Characteristic of IBS: change in stool form or frequency (>3 daily or <3 wkly), straining, urgency, or feeling of incomplete passage, bloating/distention, mucus Not characteristic of IBS: anorexia, weight loss, malnutrition, progressively worsening pain, pain that prevents sleep Diarrhea Predominant Lab work-up: serum tissue transglutaminase antibody to r/o celiac sprue +/- CBC, Chem 7, TSH, ESR, stool leukocyte Tricydic antidepressants (desipramine) or SSRIs ('if TCAs not well tolerated) If woman with severe disease, alosetron Loperamide (Imodium) prn Constipation Predominant Fiber bulking agents

(Lotronex)

/j^
ON

I
C O

3 14. What are the common causes of small bowel obstruction (SBO)? A - Adhesions from previous surgeries (about 75% of cases) B - Bulge / Hernia (second most common cause) C - C a n c e r / Tu m o r s ( m o s t c o m m o n l y m e t a s t a t i c c o l o r e c t a l c a n c e r ) f l ) Other less common causes: volvulus, intussusception (FAI I p328) (FAI2 p357), Crohn's disease,
D

gallstone ileus, bezoar, bowel wall hematoma from trauma, inammatory stricture, congenital ^ malformation, radiation enteritis
Copyright 2012. Doctors In Training.com. LLC. All Rights Reserved.

[17]

31 L K'l 'T^M W1 M.#i' / f


What pathology of the small intestine is most closely associated with the following statements? A small intestinal mucosa laden with distended macrophages in the lamina propria (that are lled with PAS(+) granules and rod-shaped bacilli seen by electron microscopy) Gluten sensitivity Radiography reveals a "string-sign" in the terminal ileum Total or subtotal atrophy of the small bowel villi, plasma cells and lymphocyte inltration into the lamina propria and epithelium, and hyperplasia/elongation of crypts

What intestinal disorder is common in the NICU to premature babies that receive oral feds too soon? (FAI I p328) (FAI2p358)

In addition to colon cancer; what is one of the most common causes of Gl bleeding in the elderly? (FAI I p328) (FAQ p358)

What are the common causes of small bowel obstruction?

What are the classic symptoms of carcinoid syndrome? (FAI I p302) (FAQ p33l)

Copyright 2012, Doctors In Training.com. LLC. All Rights Reserved.

arm-Up
What immunodeciency matches the following statement? (FAI I p2l3-214) (FAQ p237-238) Neutrophils fail to respond to chemotactic stimuli Adenosine deaminase deficiency Failure of endodermal development Defective tyrosine kinase gene Associated with high levels of IgE

. How does hexokinase differ from glucokinase? (FAI I p97) (FAQ pi03)

i. What is the mechanism of action of the following drug categories? FAII pi97) (FAQp2l8) Protease inhibitors Nucleoside reverse transcriptase inhibitors Non-nudeoside reverse transcriptase inhibitors Fusion inhibitors

Embryology of the Liver


Hepatocytes
Phys: Chapter 70 R: Chapter 18 H: Chapter 303 Hepatocyte functions - Protein synthesis - Detoxication and metabolic conversions - Storage Hereditary hyperbilirubinemias (FAII p333) (FAI2 p363) Cirrhosis and Portal Hypertension R: Chapter 18 H: Chapters 307, 308 Alcoholic liver disease (FA11 p331) (FAI2 p36l) Cirrhosis and portal hypertension (FAII p330) (FAI2 p360) SA AG 4. Serum Albumin: Ascites Gradient (SAAG) SAAG = [albuminjserum - [albuminjascites SAAG > I.I in portal hypertension SAAG < I.I in cancer, nephrotic syndrome, TB, pancreatitis, biliary disease, connective tissue disease

Copyright 2012. Doctors In Training.com. LLC. All Rights Reserved.

What enzyme is responsible for the conjugation of bilirubin? (FAI I p32l) (FAQ p350) Which hereditary hyperbilirubinemia matches the following statement? (FAII p333) (FAQp363) Mildly decreased UDPGT Completely absent UDPGT Grossly black liver Responds to phenobarbetal Treatment includes plasmapheresis and phototherapy Asymptomatic unless under physical stress (alcohol, infection) What are the signs of portal hypertension? (FAI I p330) (FAQ p360)

Liver Diseases
R: Chapter 18 Phys: Chapter 70 H: Chapters 301, 302 Markers of liver/pancreas pathology (FAII p33l) (FAI2 p360) Nutmeg liver (FA II p33l) (FA 12 p362) Budd-Chiari syndrome (FAII p332) (FAI2 p362) Reye syndrome (FAII p33l) (FAI2 p36l)

What is seen in Budd-Chiari syndrome? What conditions are associated with BuddChiari syndrome? (FAI I p332) (FAQ p362)

What is the mechanism by which aspirin can cause Reye syndrome? (FAII p33l) (FAQp36l)

In a patient with an elevated alkaline phosphatase, an elevation in which serum lab markers would direct you to a diagnosis of liver disease? (FAI I p33l) (FAQ p360)

What might you see histologically in a patient with alcoholic hepatitis? (FAII P33l)(FAQp36l)

Copyright 2012. Doctors In Training.com, LLC. All Rights Reserved.

What is the result of a glycolytic enzyme deciency? (FAI I p98) (FAQ pi 04) What is result of a deciency in pyruvate dehydrogenase? (FAI I p99) (FAQ pi05) Which substances serve as chemotactic agents for leukocytes? (FAI I p222) (FAQ p24 3. Which cranial nerves innervate muscles derived from the branchial arches? (FAII pi28) (FAQ pl36)

Hepatitis
R: Chapter 18 H: Chapters 304-306 Viral Hepatitis (FAII pl7l) (FAI2 pl90) - Interferons (FAII pl98) (FAI2 p2l8) Hepatitis serologic markers (FAII pl72) (FAI2 pl9l) Autoimmune hepatitis Protozoal liver disease - Entamoeba histolytica (FAII pl60) (FAI2 pi75) - Leishmania donovani (FAII pl6l) (FAI2 pl77)

4. Autoimmune Hepatitis ANA (+), Anti-smooth muscle antibody (+), Anti-liver-kidney microsomal antibody (+) Anti-mitochondrial antibody (-)

Identify the hepatitis B status of the following patients based on their hepatitis B serolc markers. HepBsAg Negative Negative Positive Positive Negative HepBsAb HepBcAb Positive Positive Negative Positive IgM Negative Positive IgG Negative Positive Negative
Status

What medications are used in the treatment of chronic hepatitis B and C? (FAI I pl98 FAI I pi95) (FAQ p2l8, FAQ p2l6)

Which antibodies can be used to help make the diagnosis of autoimmune hepatitis?

Copyright 2012. Doctors In Training.com. LLC. All Rights Reserved.

Additional Liver Diseases


H: Chapters 92, 309 Wilson disease (FAII p333) (FAI2 p364) Hemochromatosis (FAII p334) (FAI2 p364) Ct\-Antitrypsin deciency (FAII p332) (FAI2 p362) Liver tumors - Hepatic adenoma - Hepatic angiosarcoma - Hepatocellular carcinoma (FAII p33l) (FAI2 p36l)

8. Hepatic Adenoma
Most often in females 20-44 (OCP years) Risk Factors: OCP use, anabolic steroids, (glycogen storage disease types I and III) Sx: RUQ pain, but usually there are no symptoms because it is often an incidental finding on imaging Malignant transformation in 10% of patients Rx: DC the OCP, serial imaging and AFP, +/- resection (esp. if > 5cm)

9. Hepatic Angiosarcoma - Malignant endothelial neoplasm in liver


Risk Factors: vinyl chloride, arsenic

1 A young man presents with ataxia and tremors. He has brown pigmentation in a ring around the periphery of his cornea. What treatment should he receive?

. An 80-year-old woman comes to your clinic because her family is concerned about her yellowing skin. Exam reveals yellowing of the skin including the palms and soles but no scleral icterus. What question could you to ask the patient that would most likely identif the cause of yellowing?

/-a 20-year-old man contracts inuenza then presents with an idiopathic hyperbilirubinemia. What is the most likely cause?

Wilson disease? What is the treatment for Wilson disease? (FAII p333) (FAQ p364)

What is the classic triad of symptoms in hemochromatosis? What lab tests are used to diagnose hemochromatosis? What is the treatment for hemochromatosis? (FAI I p334) (FAQ p364)

, What are the ri

E<pr^i;]Ki

:|u ' -^.yKoi;v>-,'.<i^i-i' i

Copyright 2012. Doctors In Training.com. LLC. All Rights Reserved.

What cocktail of medications is commonly taken by p. cirrhosis?

What is the cause of chronic granulomatous disease? What are the consequences of chronic granulomatous disease? (FAI I p2l4) (FAQ p238)

Which immunoglobulin isotype is associated with eosinophilia? (FAI I p206) (FAQ p2 Which cytokine is associated with differentiation of eosinophils? (FAI I p208) (FAQ p

Biliary Anatomy, Physiology, and Jaundice


R: Chapter 18 H: Chapters 303, 311 Biliary structures (FAII p3l4) (FAI2 p343) Bilirubin (FAII p32l) (FAI2 p350) Bile (FAII p320) (FAI2 p350) Physiologic jaundice (FAII p332) (FAI 2 p362) DDx of hyperbilirubinemia (FAII p332) (FAI2 p362)

4. What is the overall differential diagnosis for hyperbilirubinemia? Unconjugated Hyperbilirubinemias Increased bilirubin production Conjugated Hyperbilirubinemias Impaired transport

Biliary epithelial damage Impaired bilirubin uptake and storage

Decreased UDP-GT activity

Intrahepatic biliary obstruction

Extrahepatic biliary obstruction

Copyright 2012. Doctors In Training.com. LLC. All Rights Reserved

What is the fate of bilirubin after it is con

id secreted into the Gl tract?

What enzyme is functioning suboptimally in newborns with physiologic jaundice? (FAII p332) (FAQ p362) What is the difference between direct bilirubin and indirect bilirubin? (FAI I p32l) (FAQ p350) What are some of the intrahepatic and extrahepatic causes of biliary obstruction?

Biliary Tract Disease


R: Chapter 18 H: Chapters 303, 311 Biliary tract disease (FAII p334) (FAI2 p365) Gallbladder pathologic terminology Gallstones (FAII p335) (FAI2 366) Clonorchis sinensis (FAII pl63) (FAI2 pl79) Cholecystitis (FAII p335) (FAI2 p366)

9. Gallbladder Pathology Terminology:


Cholelithiasis - Gallstones (hint lith = stone) Cholecystitis - Inflammation/infection of the gall bladder (-rtis = inflammation) Cholangitis - Inflammation/infection of the biliary tree Choledochol'rthiasis - Gallstones in the bile ducts

A patient presents complaining of pain in the right upper quadrant that he can point to with one nger, the area is tender to light touch, and pain is worsened when the patient is asked to raise his arms above his head. What is most likely this patient's problem? What is the difference between primary biliary cirrhosis and primary sclerosing cholangitis? (FAII p334) (FAQp365)

What is Reynold's pentad for cholangitis? (FAI I p335) (FAQ p366)

(FAII p335) (FAQp366)

Copyright 2012. Doctors In Training.com, LLC. All Rights Reserved.

rm-Up
What cytokine(s) matches the following statement? (FAI I p208) (FAQ p230) Promotes B cell growth and differentiation Produced by Thi cells Produced by Th2 cells Secreted by helper T cells and activates macrophages Pyogens secreted by monocytes and macrophages Enhances synthesis of IgE and IgG Enhances synthesis of IgA Released by virally infected cells Which vitamin deciency results in gum bleeding, bruising, anemia and poor wound heal What clinical presentation would lead you to suspect lead toxicity? (FAII p350)(FAI2p38l)

Large Intestine Basics, Cecum, and Appendix


R: Chapter 17 H: Chapters 300, 350 COA: Chapter 2 Embryology of the large intestine - Hirschsprung disease (FAII p328) (FAI2 p357) Anatomy of the large intestine Introduction to Enterobacteriaceae Appendicitis (FAII p327) (FAI2 p356) Carcinoid tumor (FAII p330) (FAI2 p359)

What is the classic prese iir.'iMif.'if-.l. pendicitis? (FAI I p327) (FAQ p356)

50-year-old man complains of diarrhea. On exam, his face is plethoric and a heart murmur is detected. What is diagnosis? (FAI I p330) (FAQ p359)

What is the fundamental problem in Hirschsprung disease? (FAI I p328) (FAQ p357)

What is the most abundant bacteria in the colon of most individuals?

Copyright 2012. Doctors In Training.com. LLC. All Rights Reserved

Ascending, Transverse, and Descending Colon


H: Chapter 91 R: Chapters 7, 17 Colonic polyps (FAII p329) (FAI2 p358) Colorectal cancer (FAII p329) (FAI2 p359) Molecular pathogenesis ofCRC (FAII p330) (FAI2 p359) Multistep carcinogenesis (R p308, gure 7-40)

A 40-year-old woman presents with having to defecate 4 times a day for several months along with a constant low-grade abdominal pain that is somewhat relieved by defecation. Colonoscopy is normal. What is the most likely diagnosis?

Which pathological lesions of the colon are considered precursors to malignancy? Which colon pathology has the most malignant potential? (FAI I p329) (FAQ p358)

What are the risk factors for colon cancer? (FAI I p329) (FAQ p359)

Copyright 2012. Doctors In Training.com. LLC. All Rights Reserved.

uestion Warm-Up
I. Which type of hypersensitivity is responsible for the following clinical problem? Asthma Tuberculosis skin test I (FAI I P2IO)(FAI2p233) Allergies, anaphylaxis, and hay fever ABO-blood type incompatibility Poison ivy Eczema Goodpasture syndrome An alcoholic develops a rash, diarrhea, and altered mental status. What is the v'rtamin deciency? 3. Which class of antibiotics inhibits prokaryotic DNA topoisomerase? (FAII p70) (FAQ p72)

Distal Large Intestine


H: Chapter 297 R:Chopter 17 Diverticular disease (FAII p327) (FAI2 p356) Volvulus (FAII p328) (FAI2 p3S7) Fecal matter

What is the classic presentation for diverticulosis? (FAI I p327) (FAQ p356)

Which portion of the colon is most commonly affected by volvulus? (FAII p328) (FAQp357)

Where in the distal Gl tract does endodermal tissue make an abrupt transition to tis derived from surface ectoderm? (FAI I p3l3) (FAQ p342)

Copyright 2012. Doctors In Training.com. LLC. All Rights Reserved.

Anus
H: Chapter 295 Pectinate line (FAII p3l3) (FAI2 p342) Hemorrhoids Proctitis Ulcerative colitis (FAII p326) (FAI2 p355)

What problem is most closely associated with the following statement? Most common cause of acute RLQ pain I 50-year-old female presents with pruritus without jaundice, lab reveals (+) AMA Most common cause of acute LLQ pain j A patient with Gl bleeding has buccal pigmentation j 60-year-old female with rheumatoid arthritis and no alcohol history presents with fatigue and right abdominal pain, lab studies reveal high levels of ANA and ASMA, elevated serum IgG levels, and no viral serologic markers Colonoscopy reveals very friable mucosa extending from the rectum to the distal transverse colon Most common cause of RUQ pain Liver biopsy on a 23-year-old female with elevated levels of LKM-I antibodies, no alcohol history, and no viral serologic markers reveals inltration of the portal and periportal area with lymphocytes Diarrhea, fever, and abdominal cramps following a course of antibiotics Fatal disease of unconjugated bilirubin resulting from a complete lack of UDPGT activity Nonfatal disease of unconjugated bilirubin resulting from low levels of UDPGT activity Elevated levels of serum ferritin and increased transferrin I Alpha-fetoprotein levels >I000 pg/m1 Elevated serum copper, decreased se elevated 24-hr urinary copper Liver disease + lung emphysema ERCP reveals alternatinoplasmin, and

Copyright 2012. Doctors In Trainmg.com. LLC. All Rights Reserved.

the symptoms of hyperthyroidism in addition to the following ndings? (FAI I p298) (FAQ P325)(Physp9l6) Extremely tender thyroid gland Palpation of single thyroid nodule Palpation of multiple thyroid nodules Recent study using IV contrast dye (iodine) Eye changes: proptosis, edema, injection What are the differences between oligodendroglia and Schwann cells? (FAII p396) (FAQ p434-435) (RpQ58, 1282)

Describe the sensory innervation of the tongue. (FAI I pi 29) (FAQ pi 37) (Phys p647)

Immunology Basics
R: Chapter 6 H: Chapter 314 Primary lymphoid organs: bone marrow and thymus (FAII p20l) (FAI2 p223) (Phys Fig. 34-1, p435) Secondary lymphoid organs: lymph nodes, spleen (FAII p200-20l) (FAI2 p222-223) Mucosa associated lymphoid tissue (MALT) (H p2675) Lymph node histology (FAII p200) (FAI2 p222) (R pi89) Lymph drainage (FAII p200) (FAI2 p222) (COA p44, Figl.27) Innate vs. adaptive immunity (FAII p202) (FAI2 p224) (R pl84) Tlymphocytes (FAII p344) (FAI2 p375) (R pi86)

<1HM

4. To which lymph nodes do the sigmoid colon and the rectum drain? (FAII p200) (FAQp222)

5. Where can B cells and T cells be found in the lymph nodes? (FAI I p200) (FAQ p222)

Which MHC are found in T helper cells? Which MHC are found on cytotoxic T cells? (FAI I p344) (FAQ p226)

Copyright (:) 2012. Doctors In Training.com, LLC. All Rights Reserved.

Antigen Presentation
R: Chapter 6 MHC I and II (FAII p202) (FA 12 p224) (R pl90) HLA associated with diseases (FAII p202) (FAI2 p223) (R pl93, table 6-1) Dendritic cells (FAII p344) (FAI2 p374) (R pl87) Langerhans cell histiocytosis (FAII p360) (FAI2 p394) (R p63l)

7. Dendritic Cells (FAI I p344) (FAQ p374) (R pi87)


Covered with long membranous extensions that resemble the dendrites of nerve cells All display both class I and II MHC, B7 family of co-stimulatory molecules, CD80 and CD86 All also have CD40 which can influence T cells byway of interaction with a complementary ligand Purpose: antigen capture at one location and presentation at another location. This is accomplished by migration to lymph nodes for presentation at T cells When acting as peripheral sentinels (immature or antigen-capturing state), antigens are captured by 3 different mechanisms: - Phagocytosis - Receptor mediated endocytosis - Pinocytosis Immature sentinels make a transition into a new type of cell (antigen-presenting state) that can travel into circulation and into a cell that has maximal capacity to present to Th cells (increased expression of class II MHC and co-stimulatory molecules) Different types: - Langerhans (from immature epithelial tissue): in epidermal layers of skin (= Cutaneous-associated lymphoid tissue, CALT) - Interstitial (from immature nonepithelial, interstitial tissue): in interstitial spaces of virtually all organs (except brain) - Monocyte-derived (from monocytes (which also give rise to macrophages)): migrated from bloodstream into tissues * migrate from tissues to blood and lymph and lymph nodes - Plasmacytoid derived (from plasmacytoid): APCs in the innate immune response 8. Follicular Dendritic Cells Do not arise from bone marrow like the dendritic cells Do not express class II MHC do not present antigen to Th cells Exclusively reside in lymph follicles Important in the maturation and diversification of B cells

WI!ffXw5!TihTIIR'SI8ilf3rWitW Langerhans cell histiocytosis? (FAI I p360) (FAQ p394)

What molecules are expressed on the surface of antigen presenting dendritic cells? (FAII p344) (FAQ p374) What are 3 cell types that are known for presenting antigens to T cells? (FAI I p204) (FAQ p226)

Copyright 2012, Doctors In Training.com, LLC. All Rights Reserved.

arm-Up
What hematologic changes would you expect to see in a patient without a function; spleen? (FAI I p20l) (FAQ p223) (R p633)
r

Where can you nd nicotinic acetylcholine receptors in the body? (FAI I p235) (FAQ p262) (Phys p733) (GG p255-258)

What structures are derived from the branchial pouches? (FAI I pQ9) (FAQ pl37)

Differentiation of Tcells (FAII p203) (FA p225) (R pl95, Fig. 6-11) Helper Tcells (FAII p204) (FAI2 p226) (R pi86) Thymus (FAII p20l) (FAI2 p223) (R p635) T cell activation (FAII p204) (FAI2 p226) (R pl95) Cytotoxic Tcells (FAII p205) (FAI2 p227) (R p207) Regulatory T cells (FAI2 p227) NK cells (FAII p202) (FAI2 p224) (R pl88)

4. Which cytokines are proauce> l f J S i . ^ ftl^ioi^v (FAII p204) (FAQ p226)

^ W M ^ S i . c ^ i ' o l W l . t i - r. ^ l V i ' i S l r o r a L l t - c & | l , \ i l l ; - ; ' . '

. Which cytokines inhibit Thi cells? Which inhibit Th2 cells? (FAI I p204) (FAQ p226)

6. How do cytotoxic T cells kill virally-infected and neoplastic cells? (FAI I p205) (FAQ p

Copyright 2012. Doctors In Training.com. LLC. All Rights Reserved.

Monocytes and Macrophages


R: Chapter 6 Phys: Chapter 33 Monocytes (FAII p343) (FAI2 p373) (Phys p426) (H p2654) Macrophages (FAII p343) (FAI2 p373) (R pi88) Interferon-gamma (FA p208) (FAI2 p230) (R pi95) Acute phase cytokines (FA p208) (FAI2 p230) (R p6l, 193, 200) Spleen (FAII p20l) (FAI2 p223) (R p632)

7. Monocytes (FAI I p343) (FAQ p373) (Phys p426) (H p2654) Leave bone marrow for blood circulate 8 hours to mature * migrate into tissues to become macrophages Blood, alveoli, intestines - macrophages Connective tissue - histiocytes Liver - Kupffer cells Kidney - mesangial cells Brain - microglia Bone - osteoclasts

Which cytokine more than any other should be known as the macrophage activating cytokine? (FAI I p343) (FAQ p378) (R pi95)

What name is given to monocytes in other tissues of the body? (Phys p426-427) Blood, alveoli, intestines Connective tissue Liver Kidney Brain Bone What are the acute phase cytokines that are produced by macrophages? (FAI I p209) (FAQ p231) (R p203) (Phys p426-431)

. What is the function of macrophages in the spleen? (FAI I p20l) (FAQ p223) (R p633) (Phys p427)

What name is given to RBCs partially digested by splenic macrophages? (FAI I p348) (FAQ p338) (R p645) '

Copyright 2012. Doctors In Training.com. LLC. All Rights Reserved.

Question WarmiTsl HiimS tvmSi i [] ikvsI WSM Slv^uil Mil iPwTlT^^r-'l! IW il*] QMHa ^J [*yi L* C*m*j I [afl I

Describe the usefulness of the dexamethasone suppression test. (FAI I p296) (FAQ p323) (Phys p935)

What enzyme of the adrenal steroid synthesis pathway is inhibited by ketoconazole? (FAI I p29l) (FAQ P3I8) (Phys p922-933)

B Cells and Antibodies


R: Chapter 6 H: Chapter 112 Phys: Chapter 34 B lymphocytes (FAII p344) (FAI2 p374) (R pl87) Review of cell surface proteins (FAII p209) (FAI2 p23l) (H p2690) Plasma cells (FAII p344) (FAI2 p374) (H p936) Antibody structure and function (FAII p205) (FAI2 p227) Immunoglobulin isotypes (FAII p206) (FAI2 p228) (H pi010) Immunization - Active vs. passive immunity (FAII p209) (FAI2 p232) (Phys p433, 442) - Viral vaccines (FAII pl64) (FAI2 pl80) - Antigen type and memory (FAII p206) (FAI2 p228) (Phys p434, 438) - Vaccines (FAII pl39) (FAI2 pISO) Autoantibodies (FAII p2l2) (FAI2 p235) (R p208) Amyloidosis (FAII p224) (FAI2 249) (R p249) (H p945)

4. V(D)J Recombination - Highlights (FAI I p205) (FAQ p227)


Rearrangements of the DNA segments named variable (V), diversity (D), and joining (J) compose the coding regions for each specic antigen receptor on B and T cells The rearrangement process begins with breaks in the dsDNA at Re. (RSSs) that ank the V, D, and J coding regions V(D)J recombination is initiated by the recombination activating gene complex (RAG I and 2). RAG I and RAG 2 protein recognizes the RSSs Mutations in either of the RAG genes in mice causes an inability to initiate V(D)J rearrangements and an arrest of B and T cell development

Copyright 2012. Doctors In Training.com. LLC. All Rights Reserved.

(FAI I p206) (FAQ p228) (R pi87) (Phys p438) Associated with allergies because it is bound by mast cells and basophils and causes them to degranulate and release their histamine Comprises 70-75% of the total immunoglobulin pool Present in large quantities on the membrane of many B cells Crosses the placenta and, additionally, confers immunity to neonates in the rst few months of life Can occur as a dimer Largely conned to the intravascular pool and is the predominant early antibody frequently seen in the immune response to infectious organisms with complex antigens Distributed evenly between the intravascular and extravascular pools The predominant immunoglobulin in mucoserous secretions such as saliva, colostrum, milk, tracheobronchial secretions, and genitourinary secretions Can be a pentamer Which cellular organelles are particularly important to plasma cell function? (FAI I p344) (FAQ p374) (H p936)

What initiates recombination of VDJ sequences? (FAI I p205) (FAQ p227) (R pi86) (Phys p437)

Cytokines and Immunosuppressants


GG: Chapters 35, 37, 62 R: Chapter 6 Important cytokines (FAII p208) (FAI2 p23l) (R pl93,200) Interferon mechanism (FAII p208) (FAI2 p23l) (R p2l6) Recombinant cytokines and clinical uses (FAII p2l6) (FAI2 p240) (GG pl022) Transplant rejection (FAII p2IS) (FAI2 p239) (R p226) Immunosuppressants (FAII p2l5-2l6) (FAI2 p239-240) (GGpl006) Therapeutic antibodies (FAII p2l7) (FAI2 p24l) (GGpl023) Iniximab (FAII p338) (FAI2 p369. 432) (GG pl827) 8. Sirolimus MOA: binds FKBP-Q intracellular protein inhibition of mTOR (mammalian target of rapamycin) inhibition of T-cell proliferation 9. Thalidomide Uses - immunosuppression (SLE, organ transplant), anti-angiogenic Mechanism of Action - affects TNFfJ Toxicity - phocomelia (prior use as a sedative during pregnancy)

Copyright 2012. Doctors In Training.com. LLC. All Rights Reserved.

Quiz
0. What cytokine(s) matches the following statement? (FAI I p208) (FAQ p230) Produced byThi cells Produced by Th2 cells Involved in growth and activation of eosinophils Secreted by helper T cells and activates macrophages Inhibits macrophage activation Pyogenes secreted by monocytes and macrophages Inhibits production of Thi cells Inhibits production of Th2 cells Mediate cell growth I Promotes Binammation and differentiation Enhances synthesis of IgE and IgG Enhances synthesis of IgA Released by virally infected cells Supports growth and differentiation of bone marrow stem cells Supports T cell proliferation, differentiation, and activation What drugs are composed of antibodies against TNF? (FAI I p393) (FAQ p369) (GG pl82/

Which immunosuppressant matches the following statement? (FAI I p208-209, 391-392) (FAQ p230-23l, 392-393) Precursor of 6-mercaptopurine May prevent nephrotoxicity with mannitol diuresis Antibody that binds to CD3 on T cells Antibody that binds IL-2 receptor on activated T cells Inhibits inosine monophosphate (IMP) dehydrogenase Inhibits calcineurin resulting in the loss of IL-2 production and blockage of T cell differentiation and activation Binds FK-binding protein (FKBP) leading to loss of IL-2 production Binds FKBP 12 leading to inhibition of mTOR and T cell proliferation Used for lupus nephritis Metabolized by xanthine oxidase, therefore increasing allopurinol toxicity

Copyright 20I2. Doctors In Training.com. LLC. All Rights Reserved.

What enzyme is responsible fortRNA charging? What enzyme catalyzes peptide bon. formation? (FAI I p74) (FAQ p77) (Phys p34)

Which cytokines are secreted by the two different types of helper T cells? (FAI I p208) (FAQ p226) (R p206, Fig. 6-19)

What substances are utilized by natural killer cells to induce apoptosis in other cells? (FAI I p202) (FAQ p224, 231) (R pi88)

Complement, Hypersensitivity, and Granulocytes


R: Chapter 6 rd Phys: Chapter 34 >^. H: Chapter 314 '> Complement (FAII p207) (FAI2 p229) (R p63) (Phys p438) (H p2665) 'J Hypersensitivity (FAII p2IO and FAI2 p2ll) (R p208,2l2) (H p2722) 00 - Autoantibodies (FAII p2l2) (FAI2 p235) (R pl97) Granulocytes I - Eosinophils (FAI I p343) (FAI2 p373) (R p72) (Phys p430) - Mast cells (FAII p343) (FAI2 p373) (R p72) J - Basophils (FA 11 p343) (FA 12 p3 73) (Phys p423) (Phys p431)

I 4. Paroxysmal Nocturnal Hemoglobinuria (PNH) (FAI I p352) (FAQ p384) (R p652)


Hemosiderinuria Chronic intravascular hemolysis Thrombosis Dx. Ham's test

5. Erythroblastosis Fetalis (FAI I p353) (FAQ p385) Maternal antibodies to fetal RBC antigen The most common antibody is Rh-D In Rh(-) moms, dose anti-Rh-D immunoglobulin at 28wks, at any traumatic event (MVA), and within 3days of delivery Clinical features in the neonate: - Anemia due to hemolysis of RBC by maternal Ab - Jaundice * possible kernicterus - Hydrops fetalis (generalized fetal edema) - IU death

Copyright () 2012. Doctors In Traming.com, LLC. All Rights Reserved.

Quiz
6. Which type of hypersensitivity is responsible for the following clinical problem; Poststreptococcal glomerulonephritis k(FAIAsthmaI) (FAQ p233) (R pl97) - I p2l Rheumatic fever Tuberculosis skin test Allergies, anaphylaxis, and hay fever Polyarteritis nodosa Serum sickness ABO blood type incompatibility Poison ivy Eczema Contact dermatitis Goodpasture syndrome'

. . patient suffers from recurrent Neisseria infections. What complement proteins are decient? (FAI I p207) (FAQ p229, 236) (R p235)

8. A 45-year-old female complains of malar rash and arthritis. The presence of which antibodies are specic for SLE? (FAI I p2Q) (FAQ p234-235) (R p2l7, 220-221)

w'hich complement is responsible for neutrophil chemotaxis? (FAI I p208) (FAQ p230) (Phys p439)

Immunodeciencies
R: Chapter 6 Immunodeciencies (FAII p2l3-2l4) (FAI2 p237-237)

Copyright 2012. Doctors In Training.com. LLC. All Rights Reserved.

Immunodeciencies
Bruton Agammaglobulinemia
X-linked (Boys) B cell deficiency * defective tyrosine kinase gene low levels of all immunoglobulins Recurrent Bacterial infections after 6m X-linked Immunodeciencies Wiskott-Aldrich Bruton Agammaglobulinemia Chronic Granulomatous Disease (+/-) Hyper-IgM syndrome (3 types) tlgM, ilg - X-linked no CD ligand - AR"noCD40 - -NEMO deciency

"u c

c q;
<) L

Thymic aplasia (DiGeorge)

^=

3rd and 4th pouches fail to develop 4 - No thymus no T cells - No parathyroids * low Ca2+ tetany Congenital defects in heart / great vessels Recurrent viral, fungal, protozoal infections 90% have a chrom 22ql I deletion (detect with FISH)

TJ O D

Wiskott
Aldrich Immunodeciency Thrombocytopenia and purpura Eczema Recurrent pyogenic infections
No IgM v. capsular polysaccharides of bacteria Low IgM, high IgA X-linked

E E
"D C

Severe Combined Immunodeciency (SCID)


> o n

0) CL >>.

Defect in early stem cell differentiation Can be caused by at least 7 different gene defects: - Adenosine deaminase deciency Last defense is cytotoxic NK cells Presentation triad: 1) Severe recurrent infections - Chronic mucocutaneous Candidiasis - Fatal or recurrent RSV, VZV, HSV, measles, u, parainuenza - PCP pneumonia 2) Chronic diarrhea 3) Failure to thrive No thymic shadow on newborn CXR Chronic Mucocutaneous Candidiasis T cell dysfunction v. C. albicans Rx: ketoconazole

Ataxia-Telangiectasia

IgA deficiency Cerebellar ataxia, and poor smooth pursuit of moving target w/ eyes Telangiectasias of face > 5yo t cancer risk lymphoma & acute leukemias Radiation sensitivity (try to avoid x-rays) +/- tAFP in children > 8m Average age of death: 25 y/o

-h c

< u

E E o U
>Q.

ol _ o
z
D

Selective Immunoglobulin Deciencies IgA deficiency is most common - Most appear healthy - Sinus and lung infections - 1/600 European descent - Associated with atopy, asthma - Possible anaphylaxis to blood transfusions and blood products IL-Q receptor deciency Mycobacterial infections

/3B&L

l"S>i

[10]

Copyright ( 2012. Doctors In Traming.com. LLC. All Rights Reserved.

Immunodeciencies (continued)
Phagocyte Deciencies
Chronic granulomatous disease Chediak-Higashi syndrome Job syndrome Leukocyte adhesion deficiency syndrome

Chronic

Granulomatous

Disease

(CGD)

=^

Lack of NADPH oxidase activity ^impotent phagocytes # Susceptible to organisms with catalase (S. aureus, coli, Klebsiella spp., Asperg?//us spp., Candida spp.) -^

Dx:
No

(-)

nitroblue

tetrazolium

(NBT)

dye

r\
I O O

yellow to Prophylactic INPV

blue-black oxidation TMP-SMX also helpful

Chediak-Higashi
1) 2) 3) Recurrent Defective LY S T gene

Syndrome
(lysosomal triad: tract albinism and skin infections disorders transport)

4*I O _^ >*" 3

Defective phagocyte lysosome giant cytoplasmic granules in PMNs are diagnostic (~)
Presentation Partial respiratory Neurologic

n >

Job Syndrome 3 Hyperimmunoglobulin E syndrome -r D e f i d e n t I N F v P M N s f a i l t o r e s p o n d t o c h e m o t a c t i c s t i m u l i ( C 5 a , LT B 4 ) * < High levels of IgE and Eosinophils fD Presentation triad: ^ 1) Eczema 5

2) Recurrent cold Staph, aureus abscesses (think of biblical Job with boils) !.
3) Course facial features: broad nose, prominent forehead ("frontal bossing"), deep set eyes, and <" "dough/' skin Also common to have retained primary teeth resulting in 2 rows of teeth

D
Leukocyte Adhesion Deciency Syndrome Abnormal integrins inability of phagocytes to exit circulation Delayed separation of umbilicus

C D

Q_ > =>

>"

Copyright 2012. Doctors In Training.com. LLC. All Rights Reserved.

[II]

What would you expect to see in a patient with Wiskott-Aldrich syndrome; (FAII P2I4) (FAQP238)(Rp235)

What is the cause of chronic granulomatous disease? What infections are these individuals susceptible to? (FAI I p2l4) (FAQ p238) (R p234)

young child presents with tetany from hypocalcemia and candidiasis resulting from immunosuppression. What cell type is decient in this patient? (FAI I P2I4) (FAQ p238) (R p234)

A young child has recurrent lung infections and granulomatous lesions. What is the defect in neutrophils? (FAII P2I4) (FAQ p238) (Rp23l-233)

,, mother brings in her 2-year-old child who has had multiple viral and fungal infections and is found to be hypocalcemic. Which of the 3 types of germ cells (ecto-, endo-, and mesoderm) gives rise to the missing structure in this child? (FAII p89, 213) (FAQ pl37, 237)

A child has an immune disorder in which there is a repeated Staph abscesses. It is found that the neutrophils fail to respond to chemotactic stimuli. What is the most likely diagnosis? (FAI I P2I4) (FAQ p237-238)

Copyright c 2012. Doctors In Training.com. LLC All Rights Reserved.

est ion Warm-Up


What disorders are associated with the following antibodies? Anti-endomysial Anti-thyroglobulin Anti-smooth muscle p2Q) (FAQ p235)

An 18-year-old female has moderate generalized abdominal pain, normal WBC, and n fever. She has paresthesias in her lower extremities. What is her diagnosis?

v viiat is the mechanism and time-frame of chronic transplant rejection? (FAII P2I5) (FAQ p239)

DNA Basics
Phys: Chapter 3 Chromatin structure (FAII p66) (FAI2 p68) Nucleotides (FAII p67) (FAI2 p68) (Phys p27) De novo pyrimidine and purine synthesis (FAII p68) (FAI2 p69) Orotic aciduria (FAII p68) (FAI2 p69) Purine salvage deciencies (FAII p69) (FAI2 p70)

What is the rate-limiting step in purine synthesis? In pyrimidine synthesis?

What are the sources of carbons in the formation of purines? What are the carbon sources in pyrimidine synthesis

'hat accounts for the positive charge of histones? What accounts for the negative large of DNA?

How many adenine residues are found in a molecule of DNA if one strand contains A=2000, G=500, C=I500, and T=I000?

. What stand of DNA nucleotides opposes this DNA strand: 5'-ATTGCGTA-3'?

Copyright 2012. Doctors In Trainmg.com. LLC. All Rights Reserved.

DNA Interactions
Phys: Chapter 3 Mutations in DNA (FAII p69) (FAI2 p7l) DNA replication (FAII p70) (FAI2 p72) DNA repair (FAII p7l) (FAI2 p73) DNAIRNA/protein synthesis direction (FAII p74)

9. Label the players on the following simplied model of DNA replication.

^tnmmimi
Topoisomerase Leading strand

m
@ P
Lagging strand ' ;mentosum - hypersensitivity to UV light 1000 x increased risk of skin cancers "+5J3 - sensitivity to ionizing radiation, immunodeficiency, ataxia beginning at I -2 yrs - hypersensitivity to sunlight, leukemias and lymphomas are common, average age of

10. DNA Repair Defects

cancer onset is 25 HCTirirmgP polyposis Colorectal Cancer (HNPCC)

Copyright 2012. Doctors In Training.com. LLC. All Rights Reserved.

Which medication matches the following statement? Inhibits ribonucleotide reductase Inhibits dihydrofolate reductase Inhibits thymidylate synthase Inhibits inosine monophosphate dehydrogenase Inhibits PRPP synthetase What are the characteristic features of orotic aciduria?

ow does UV light damage DNA

What eukaryotic DNA polymerase matches the following description? Replicates lagging strand, synthesizes RNA primer Repairs DNA Replicates mitochondrial DNA leading strand DNA

Copyright 2012. Doctors In Training.com. LLC. All Rights Reserved.

cMJ'roJfVT7BWW77n
What gene complex EfSntS^JEWrahiiiR development?

What are the different types of collagen, and where can they be found in the body? (FAII p79) (FAQp82)

What medication inhibits the cellular sodium potassium ATPase? What is its clinical application? (FAI I p79) (FAQ p82)

RNA Basics
Phys: Chapter 3 Types of RNA (FAII p72) (FAI 2 p74) (Phys p3l) Start and stop codons (FAII p72) (FAI2 p74) (Phys p3l, Table 3-1) Functional organization of the gene (FAII p72) (FAI2 p75) Regulation of transcription (Phys p35) - Regulation of gene expressions (FAII p72) (FAI2 p75) - DNA binding motifs - lac operon (FAI2 p76)

4. Regulation of Transcription
Operon - structural genes that are transcribed + promoter region + all regulatory regions Transcription Factors - must bind to the promoter region C-25 Hoeness/Pribnow/TATA box and -75 AAT box) in order for transcription to take place - Common structural motifs: leucine zipper, helix-loop-helix, helix-tum-helix, zinc nger rator region - binds repressor (stops transcription, see lac operon), or inducer (starts transcription), located between the promoter region and start site its: Enhancer region and Repressor region - increase or decrease the rate of transcription when bound by protein factors; location may be close to, far from, or within the promoter
region

Lac operon example 5. Termination of Prokaryotic RNA Transcription Recognition of termination region in DNA (p -independent mechanism) - GC rich DNA GC same strand binding forms stem-loop in RNA (hairpin appearance) pause in RNA polymerase * subsequent weak RNA bonds (uracil rich region) separation of RNA polymerase Rho (p) factor of E. coli (an RNA dependent ATPase)

Copyright 2012. Doctors In Training.com, LLC. All Rights Reserved.

6. Helix-Loop-Helix

nSfteSk B2W 4 ^
7. Helix-Turn-Helix (Tryptophan Repressor Protein)

8. Other DNA Binding Motifs Zinc Finger Motif


Leucine Zipper Protein
C4d * unwound X^J

Aw

^^

. R>gnrtKwi

Copyright 2012. Doctors In Training.com. LLC. All Rights Reserved.

9. Operon Control Mechanisms


Some regulatory proteins can act as either transcription activators or repressors depending on their exact binding site on DNA.

RNA polymerase

Lambda repressor Promoter

Transcription is activated by lambda repressor

Operator

r
Operator Promoter

Transcription is repressed by lambda repressor

lac Operon
(Controls expression of p-galactosidase)
lacZ aene

CAP-blndlng RNA-polymerase ,,. .. site binding site (Promoter) RNA synthesis

Excess lactose + Absent glucose Stimulate lac operon to make p-galactosidase


Excess glucose Excess lactose lac repressor Excess glucose Absent lactose

Operon OFF b/c CAP not bound Operon OFF both b/c lac repressor bound and b/c CAP not bound Operon OFF b/c lac repressor bound

Absent glucose Absent lactose

RNA polymerase

Absent glucose Excess lactose

RNA transcribed

Operon ON

j4-j The prokaryotic cell derives its energy from glucose metabolism. In environments of low glucose concentration, the cell -p has the capability to make -galactosidose, an enzyme that cleaves lactose into glucose and galactose, thereby making r*-r more energy available. Of course, if no lactose is present, then the costly activity of making p-galactosidase is use/ess. \y The cell uses CAP (catabolite activating protein) and the lac repressor to determine when to begin manufacturing the Q costly -galactosidase. CAP facilitates RNA polymerase binding and promotes $-galactosidase production, but is inhibited by excess glucose levels. The lac repressor cannot bind to the operator site and inhibit $-galactosidase production in the LLJ presence of lactose.

Copyright 2012. Doctors In Training.com. LLC. All Rights Reserved

IS What are the mRNA stop codons? (FAI I p72) (FAQ p74)
What is the characteristic sequence of the promoter region? What does a mutation in the sequence cause? (FAI I p72) (FAQ p74)

12. What amino acid frequently has more coding sequences in the mRNA than are represented in the peptide that is created from that mRNA?

13. What structural motifs allow for proteins to bind to DNA?

Transcription
Phys: Chapter 3 RNA polymerases Termination of prokaryotic RNA transcription (FAII p75) (FAI2 p78) RNA processing (FAII p72) (FAI2 p75) Splicing ofpre-mRNA (FAII p73) (FAI2 p76) Introns vs. exons (FAII p73) (FAI2 p77)

Session Qui
TrWSftiSff l(l laraMSft

is hnRNA processed before it leaves the nucleus? (FAI I p72) (FAQ p75)

What is the difference between an intron and an exon? (FAI I p73) (FAQ p77)

17. How is transcription of the lac operon regulated? (FAQ p76)

Copyright 2012. Doctors In Training.com. LLC. All Rights Reserved.

What are the mRNA stop codons? (FAI I p72) (FAQ p74)

What substance inhibits RNA polymerase II in eukaryotes causing liver damage? (FAI I p72) (FAQ p75)

Which amino acids are modied by the Golgi apparatus? (FAI I p77) (FAQ p80)

tRNA and Protein Synthesis


Phys: Chapter 3

tRNA (FAII p74) (FAI2 p77) (Phys p32)


Protein synthesis (FAII p75) (FAI2 p78) (Phys p33) Post-translational modications (FAII p75) (FAI2 p78)

4. What enzyme matches amino acids to tRNA? (FAI I p74) (FAQ p77)

5. What antibiotics are inhibitors of prokaryotic protein synthesis at the 50S ribosome? (FAII p75, 187) (FAQ p 78, 208)

x What antibiotics are inhibitors of prokaryotic protein synthesis at the 30S ribosomal subunit? (FAI I pi87) (FAQ p208)

Protein modications
Phys: Chapter 3 Cells that make extracellular proteins - Fibroblasts

- Hepatocytes
- Plasma cells Extracellular protein - Collagen (FAII p79) (FAI2 p82) - Collagen synthesis and structure (FAII p79) (FAI2 p83) (R pl2ll) - Ehlers-Danlos syndrome (FAII p80) (FAI2 p83) (R pl45) - Osteogenesis imperfecta (FAII p80) (FAI2 p83) - Alport syndrome (FAII p80) (FA12 p84) - Elastin (FAII p80) (FA12 p84)
Copyright 2012. Doctors In Traming.com. LLC. All Rights Reserved.

f^TTT of Session Quiz


(FAN P79)(FAI2p82)

What are the clinical features of osteogenesis imperfecta? (FAI I p80) (FAQ p83)

In what disorder is there an abnormal breakdown of elastin? (FAI I p80) (FAQ p84)

Which type of collagen is abnormal in Alport syndrome? Which is abnormal in Ehlers-Danlos? (FAI I p80) (FAQ p83)

Copyright 2012. Doctors In Traimng.com. LLC. All Rights Reserved.

What are the ve 2s of a Meckel diverticulum? (FAI I p327) (FAQ p357)

What effect will stimulating the following receptors have on heart rate? (FAI I p236) (FAI P263) Pi p2
M2

What is the treatment for Zollinger-Ellison syndrome? (FAI I p296) (FAQ p33l)

Glycolysis and Gluconeogenesis


Phys: Chapter 67 Rate-limiting enzymes involved in glucose metabolism (FAII p95) (FAI2 plOI) Glycolysis/ATP production (FAII p97) (FAI2 pl03) (Phys p8l2) Hexokinase vs. glucokinase (FAII p97) (FAI2 pi03) Glycolysis regulation, key enzymes (FAII p98) (FA12 pi04) Regulation by F2,6-bisP (FAII p98) (FAI2 pl04) Gluconeogenesis, irreversible enzymes (FAII plOI) (FAI2 pl06) (Phys 817)

Copyright 2012. Doctors In Training.com, LLC. All Rights Reserved.

4. Label the following enzymes and substrates of glycolysis and gluconeogenesis. glucose or fructose

Glycolysis aJp^hO
- - *
A TP ADP

f
jdihydroxyacetone-P

Gluconeogenesis

1,3-blphosphogiycerate ADPA TP< 3-phosphoglycerate 2-phosphoglycerate

GTP GDP

ATpZ^c3D0 1?b pyruvate


5. Glycolytic enzyme deciency
Clinical presentation: hemolytic anemia, due to inability to maintain Na+-K*" ATPase RBC swelling/lysis Common cause: pyruvate kinase deficiency, phosphoglucose isomerase deficiency

CO

n
X

rn

Q c
n
O in

Copyright 2012. Doctors In Training.com. LLC. All Rights Reserved.

[II]

PFK-I and FBP-2


6. A muscle biopsy on a patient of yours reveals elevated glycogen levels, elevated fructose 6-phosphate, and decreased pyruvate. What enzyme deciency do you suspect most?
Deficiency in PFK-1

7. What enzymes are responsible for increasing and decreasing the intracellular levels of fru-2,6-bisP?
Phosphofructokinase-2 (PFK-2) increases levels of fru-2,6-bisR Fructose bisphosphatase-2 (FBP-2) decreases levels of fru-2,6-bisP.

8. Describe the process by which low levels of insulin and high levels of glucagon (fasting/ starvation) result in less activation of PFK-I by fructose-2,6-bisphosphate (which results in less glycolysis and more conservation of energy).
low Insulin: glucagon ratio high insulin: glucagon ratio (after a meal)

^ (PFK-2 Is phosphofructokInase-2) Adenylate Cyclase (FBP-2 is fructose bisphosphatase-2) AT P cAMP

active Phosphorylated PFK-2/FBP-2 complex < j ^ PFK-2/FBP-2 complex (dephospherylated) active EB2 * \ active PFK-2 Inactive PFK-2 AOP AT P . Inactive FBP-2

(nJi^T)
In short:

PFK-2

fructose-2rfi-b1sphosphata fructoses-phosphate fnictoso--phosphata '" ' fructose-2rf-blsphosphato (less activation of PFK-1 by(ru-2,6-blsP)

High glucagon more active PKA active FBP-2 less fru-2,6-bisP less active PFK-I less glycolysis Low glucagon (high insulin) less active PKA active PFK-2 > more fru-2,6-bisP ^ active PFK-I

<L) more glycolysis

O u

o
I

9. True or false? In the liver, high levels of glucagon result in elevated cAMP levels. True

<sj- 10. True or false? Having an excess of fructose-2,6-bisphosphate results in more fructosey 1,6-bisphosphate. LU True, by way of allosteric activation of PFK-I. I ^ II. How will levels of fructose-2,6-bisphosphate change in the liver and in the muscle CJ during a sympathetic ght or ight response? ^ Fru-2,6-bisP levels will decrease in the liver (* less activation of PFK-I * less glycolysis) Fru-2,6-bisP levels will increase in the musde ( activation of PFK-1, sugar where you need it)

r 121o p y r i g h t L J C

2012,

Doctors

In Training.com,

LLC. All

Rights

Reserved.

Quiz
What irreversible enzymes are involved in gluconeogenesis.

What effect do glucagon and insulin have on glycolysis? (FAI I p98) (FAI2 pi04)

What is the clinical consequence of a glycolytic enzyme deciency?

What are the differences between hexokinase and glucokinase? (FAI I p97) (FAI2 pi03)

Glycogen
Phys: Chapter 67 H: Chapter 362 Glycogen (FAII pl09) (FAI2 pll4) (Phys p8ll) Glycogenolysislglycogen synthesis (FAII pi 10) (FAI2 pi 15) (Phys p8ll) Glycogen storage diseases (FAII pllO) (FAI2 pllS) (R pISS) (H p3l98, 3501) Glycogen regulation by insulin and glucagon/epinephrine (FAII pl09) (FAI2 pll4) (Physp8ll)

16. Glycogen Breakdown (Glycogenosis)

a -1,4 glycosidic linkage to glycogen chain


ATP, glucose-6-P, glucose

Glycogen Phosphorylase
l X ( V, McArdle)

glucose-1-P
i

a-1,6 glycosidic linkage glucose-6-P


i

|j Phopho-gluco-mutase

g 1,6-glucosidase
X(lll,Cori's)

Glucose-6-phosphatase
r X(l, Von Gierke)

glucose

Copyright . > 2012. Doctors In Training.com, LLC. All Rights Reserved.

311gI M>1^H1 L'l i K9J>lr


What is the rate-limiting enzyme for the followin Glycolysis Gluconeogenesis Glycogenesis Glycogenosis

Which enzyme converts glucose to glucose-6-phosphate? (FAI I p97) (FAI2 pi03)

Which enzyme converts glucose-6-phosphate to glucose? (FAI I plOI) (FAI2 pl06)

Which glycogen storage disease matches the following phrase? (FAI I pi 10) (FAQ pi 15) Glycogen phosphorylase deficiency Glucose-6-phosphatase deficiency Lactic acidosis, hyperlipidemia, hyperuricemia (gout) a-1,6-glucosidase deficiency a-1,4-glucosidase deficiency Cardiomegaly Diaphragm weakness leading to respiratory failure Increased glycogen in liver, severe fasting hypoglycemia Hepatomegaly, hypoglycemia, hyperlipidemia (normal kidneys, lactate, and uric acid) Painful muscle cramps, myoglobinuria with strenuous exercise Severe hepatosplenomegaly, enlarged kidneys

Copyright ici 20I2. Doctors In Training.com. LLC. All Rights Reserved.

estion Warm-Up
I. What are the different RNA polymerases in eukaryotes?

mlnmnSBn

ii i which order of elimination (zero or rst) would you see a linear decrease in the plas, concentration of a substance when plotted against time? (FAI I p233) (FAI2 p260)

What is the equation for Gibbs free energy?

Potential Fates of Pyruvate


Phys: Chapters 67-73 Energy, ATP (FAII p97) (FAI2 pi03) Pyruvate metabolism (FAII p99) (FAI2 plOS) Lactic acid cycle (Cori cycle) Alanine cycle (Transport of ammonium by alanine and glutamate) (FAII pl05) (FAI2 pi 10) Oxaloacetate gluconeogenesis Acetyl CoA

4. Cori Cycle
ANAEROBIC GLYCOLYSIS Muscle/RBCs Blood GLUCONEOGENESIS

glucose 2ATP 2 pyruvate


lactate dehydrogenase

glucose 12ATP 2 pyruvate


lactate dehydrogenase

6ATP 2 lactate 2 lactate

Copyright <$) 2012. Doctors In Traimng.com. LLC All Rights Reserved.

Alanine Cycle 5. Outline the basic processes of transamination and oxidative deamination involved in the interorgan transport of nitrogen. excretion in kidneys Muscle protein Serum Uver *ureaN . H > glutamlne -^ glutamate a - ketoglutarate + alanine pyruvate

amino adds a ketoadds a - ketoglutarate glutamate -^-t*- glutamlnerate glut pyruvate alanine

X X

N H

glucose 4-

glucose

6. Why are alanine and glutamine found in such high concentrations in the blood? They are the two major carriers of nitrogen from tissues. 7. What is generally involved in transamination? Transfer of the amino group of an amino acid to a-ketoglutarate to form glutamate The remaining deaminated amino acid is a keto-acid (such as pyruvate) that is used in energy metabolism 8. What enzyme catalyses the above reaction? Aminotransferase 9. How are aminotransferases named? By donor of the amino group (alanine aminotransferase converts alanine to pyruvate and forms glutamate)

o
q; 10. In addition to substrates, what is required by all aminotransferases? I ij Pyridoxal phosphate (a derivative of vitamin )
LO

What are the two most important aminotransferase enzymes? What reaction do they catalyze? alanine + a-ketoglutarate < glutamate + pyruvate glutamate + oxaloacetate - d-ketoglutarate + aspartate

u o
CD

[16]

Copyright 2012. Doctors In Training.com. LLC. All Rights Reserved.

Quick Quiz
12. What are the possible products of pyruvate? (FAI I p 99) (FAQ pi05)

13. What are the two main nitrogen transporters in the blood? (above)

anaerobic ATP are yielded I p97) (FAI2 metabolism? How are many are yielded fro [How manymetabolism? (FAIfrom aerobic pi03)

Pyruvate Dehydrogenase, TCA and ETC


Phys: Chapters 67-69 Pyruvate dehydrogenase complex (FAII p99) (FAI2 pl04) Pyruvate dehydrogenase deciency (FAII p99) (FAI2 pl05) TCA cycle (Krebs cycle) (FAII p99) (FAI2 plOS) (Phys p8l3) Universal electron acceptors (FAII p97) (FAI2 pl03) Electron transport chain and oxidative phosphorylation (FAII plOO) (FA/2 pl06) (Phys p8l4)

15. Electron Transport Chain (ETC)


Cytosol
-yxxr iy-r<xxicx*r]rirxyxyyyx~zxx'xxyr*x^x~xx

*er mitochondrial membrane jcooci syxoxcaxaxox rmembrane space

Complex I NADH reductase


V Coenzyme /^H e

Complex III Cytochrome

Complex

IV

mitochondrial embrane

,.e. ,,J
Cytochrome)

Cytochrome

(Succinate A ^dehydrogenase/

" /* "
1/20j+2H- HaO lowlH']

j + H* NAD

Complex II / Succinate Fumarate

Intermembrane space Mitochondrial matrix-

Copyright 2012. Doctors In Training.com, LLC. All Rights Reserved.

=fiL**I*T1?-"ML What are the major regulatory enzym /del (FAII p96)(FAI2pl02)

Arrange the following molecules from most exergonic with loss of phosphate to least exergonic with loss of phosphate: adenosine monophosphate, adenosine triphosphate, phosphoenolpyruvate

. .. ,at substances are known to inhibit the complexes of the electron transport chain? (FAII plOO) (FAI2 pl02)

What substances can increase the permeability of the inner mitochondrial membrane thereby decreasing ATP synthesis but increasing heat generation? (FAI I plOO) (FAI2 plO

Copyright >) 2012, Doctors In Training.com, LLC. All Rights Reserved.

Question Warm-Up
What are the features of Plummer-Vinson syndrome? (FAI I p322) (FAI2 p35l)

Explain why methanol is such atoxic substance. (FAI I p246) (FA12 pi00, 270)

A woman commonly develops intense muscle cramps and darkening of her urine after exercise. What is her diagnosis?

HMP Shunt
Phys: Chapter 67 HMP shunt (pentose phosphate pathway) (FAII plOI) (FA/2 pl07) (Phys 816) Respiratory burst (oxidative burst) (FAII pi02) (FAI2 pi07) Glucose-6-phosphate dehydrogenase deciency (FAII pl02) (FAI2 pl08)

What is the rate limiting step of the pentose phosphate pathway? (FAI I p95) (FAI2 pK

lich tissues of the body use the pentose phosphate pathway? (FAI I pl02) (FAI2 pl(

j. Explain why a deciency of the enzyme that is the rate-limiter for the HMP shunt can result in hemolytic anemia? (FAI I pi02) (FAI2 pi07)

Fructose, Galactose, and Lactose


Phys: Chapter 67, 84 Disorders of fructose metabolism (FAII pi 03) (FAI 2 pi 08) Disorders of galactose metabolism (FAII pl03) (FAI 2 pi 08) Lactase deciency (FAII pl04) (FAI2 pl09) (R p797) Lactate (Phys pi033)

Copyright 2012. Doctors In Training.com. LLC. All Rights Reserved.

'tl' 4vi m-l-i-i-i [*] i M.#TT


What are the symptoms of classic galactosemia? (FAI I pi03) (FA12 pi09)

What disorder is caused by a deciency of the following enzyme? (FAII pl03)(FAI2pl08, 109)
Galactokinase Aldolase B Lactase Galactose-1-phosphate uridyltransferase Fructokinase

Copyright @ 2012. Doctors In Training.com. LLC. All Rights Reserved.

What substances are well known for causing hemolytic anemia in patients with G6PD deciency? (FAI I pi02) (FAI2 pl08)

"l

**-

r~r~\

What are the 4 important pharmacokinetic equations? (FAI I p232) (FA12 p259)

What is seen histologically in patients with celiac sprue? (FAI I p324) (FAI I p353)

Ketones
R: Chapter 24 Feeding, Fasting, and Exercise Phys: Chapter 84 H: Chapter 75

Fuel usage in feeding vs. fasting states (FAI I pi 13) (FAI2 pi 18) 4. What fuels are produced and used in the post-absorptive period?
Produced -

Used - Muscles, brain, and other tissues use predominantly.

5. When does gluconeogenesis begin in the post-absorptive period? When does it become fully active?
Begins hours after the last meal Fully active when glycogen stores are depleted ( hours after last meal)

6. How does the pattern of fuel production and usage change in early starvation (24 hours after the last meal)? Produced -

Used- Brain uses predominantly - Muscles and other tissues use some.

.but predominantly.

Copyright Si 2012. Doctors In Trainmg.com. LLC. All Rights Reserved.

7. In intermediate starvation (48 hours after the last meal), how does the pattern of fuel production and consumption change?
Produced -

Used - Brain uses predominantly but also some - Muscles and other tissue use predominantly but also some

8. What metabolic scenario favors the synthesis of ketone bodies? 9. True or false? Ketone bodies can be used by all body tissues including the brain. 10. What is the pattern of fuel utilization and production in prolonged starvation (5 days after last meal)? Produced -

Used- Brain uses predominantly - Muscles and other tissue use predominantly but also some

Comparing an overnight fast to a 3 day fast, what percentage of energy comes from glucose and from ketone bodies?
Overnight - % from glucose (2/3 from glycogen breakdown, 1/3 from gluconeogenesis)

% from ketone bodies


3day% from ketone bodies (Vi are 8-hydroxybutyrate, J4 acetoacetate)

_% from glucose (most from gluconeogenesis)

. What is the primary energy source in a patient that has not eaten in two days? (FAII pi 13) (FAI2pi 19) . What hormone stimulates the storage of lipids in the fed state? (FAI I pi 13) (FA12 pi 19) . What is the rate limiting enzyme in ketone body synthesis? (FAI I pi 12) (FAI2 pi 18) 5. Which ketone body is metabolized by muscle and brain tissue? (FAI I pi 12) (FA 12 pi 18)

Copyright (O 2012. Doctors In Training.com, LLC. All Rights Reserved.

Ethanol and Caloric Deciencies


H: Chapter 392 R: Chapter 9 Ethanol metabolism (FAII p94) (FAI2 plOO) (R p4l2) Ethanol hypoglycemia (FAII p94) (FAI2 pi00) Malnutrition (kwashiorkor and marasmus) (FAII p94) (FAI2 plOO) (R p428)

lb

'i*liM*JnTA

A stressed physician comes home from work, consumes 7 or 8 shots of tequila in rapid succession before dinner, and becomes hypoglycemic. Why did she become hypoglycemic? (FAI I p94) (FAQ pi00)

What enzymes are used to metabolize alcohol? (FAI I p94) (FA12 pi00)

18. What are some of the hallmark features of kwashiorkor? (FAI I p94) (FAI2 plOO)

How does the brain utilize ketone bodies? (FAI I pi 12) (FAI2p 119)

Copyright 2012. Doctors In Training.com, LLC. All Rights Reserved.

What are the 3 most com

id cancer? (FAI I p298) (FAI2 p326)

What are the side effects of oriistat?

What are the arterial branches off of the celiac trunk? (FAI I p3l2) (FAI2 p340)

Lipid Transport
Phys: Chapter 68 H: Chapter 356 Lipid transport, key enzymes (FAII pi 14) (FA12 pi20) (H p3l45) Major apolipoproteins (FAII pi 14) (FAI2 pl20) (Phys p82l) (H p3l45) Lipoprotein functions (FAII pi 15) (FAI2 pl2l) (H p3l45) Familial dyslipidemias (FAII pi IS) (FAI2 pl2l) (R pl47) (H p2l48) Abetalipoproteinemia (FAII pi 15) (FAI2 pl2l) (H p3l53) Hyperlipidemia signs (FAII p269) (FAI2 p294)

Copyright 2012, Doctors In Trainmg.com. LLC. All Rights Reserved.

4. Lipid transport
Dietary Fat Exogenous

Endog

Intestine

Into lymph, through thoracic duct,

then into blood

Dietary Fat Exogenous


cholesterol, bile acids

Intestine

cholesterol

endocytosls

B >
chylomicrons Into lymph. chylomicron through thoracic duct, remnants then into blood lipoprotein lipase j

<v

T triglycerides and olesterol phospholipids

V CEPT f
lipoprotein lipase

hepatic triglyceride lipase (HGTL)

Copyright 2012. Doctors In Training.com. LLC. All Rights Reserved.

What deciency causes IslHI BI!m!raOTSls5!aH!15 Which apolipoprotein matches the following statement? (FAI I pi 14) (FAI2 pi20) Activates LCAT Mediates chylomicron secretion Mediates VLDL secretion Binds to LDL receptor Cofactor for lipoprotein lipase Mediates uptake of remnant particles When looking at a slide of RBCs you notice RBCs that look spiny like a medieval mace. What is likely to be decient in this patient? (FAI I pi 15) (FAI2 pl2l)

Fatty Acids and Cholesterol


Phys: Chapter 68 Fatty acid synthesis (FAII pi 12) (FAI2 pi 18, 119) (Phys p824) Fatty acid degradation (FAII pi 12) (FAI2 pi 17) Omega-3 and omega-6 fatty acids (H p3l60) Cholesterol synthesis (FAII pi 13) (FAI2 pi 19) (Phys p826)

What is the rate-limiting enzyme for the following metabolic pathway? Hexose monophosphate pathway Fatty acid synthesis 3-oxidation of fatty acids Ketone body synthesis Cholesterol synthesis 9. Which group of medications inhibits the rate limiting enzyme of cholesterol synthesis? (FAII pll3)(FAI2plOI)

Where in the cell would you nd the following enzymatic processes taking place? (FAII p95) (FAQ pi00) Fatty acid degradation Fatty acid synthesis Glycolysis TCA cycle l chain (oxidative phosphorylation)

Copyright g! 2012. Doctors In Training.com. LLC All Rights Reserved.

e s t i o n Wa r m A 2-year-old girl has an increase in abdominal girth, failure to thrive, and skin/hair depigmentation. What is her diagnosis? (FAI I p94) (FAI2 plOO) What enzyme is decient in the following diseases? (FAI I pl03-l04) (FAI2 pi09) Fructose intolerance Essential fructosuria Classic galactosemia What structures run through the cavernous sinus? (FAI I p4l8) (FA12 p458)

Amino Acid Basics


Phys: Chapter 69 Amino acids (FAII pl04) (FAI2 pi 10) (Phys p83l) - Essential amino acids (Phys p834) - Ketogenic vs. glucogenic - Amino acid claims to fame - Acidic vs. basic Amino acid derivatives (FAII pl06) (FAI2 pllO) Catecholamine synthesis (FAII pl07) (FAI2 pi 12)

What are the essential

ids? (FAI I pl04)(FAI2pM0)

What amino acid is a precursor to the following molecule? (FAI I pl06) (FAQ pi 11) Histamine Porphyrin, heme NO GABA (a neurotransmitter) S-adenosyl-methionine (SAM)

Creatine
6. Which amino acids have the following structure?

What is the following compoi

COOH

H,NC

r
I

COOH

/
/ C H

Copyright 2012. Doctors In Training.com. LLC. All Rights Reserved.

Nitrogen Metabolism
Phys: Chapter 69 Urea cycle (FAII pl05) (FAI2 pi 10) (Phys p835) Ornithine transcarbamoylase deciency (FAII pi05) (FAI2 pill) Hyperammonemia (FAII pl05) (FAI2 pill) (H p2527) Lactulose (GG pi332)

Sess
Compare carbamoyl phosphate synthetase I to carbamoyl phosphate synthetase II pl05)(FAI2plll)

What is the mechanism of action of lactulose? (FAI I pl05) (FAI2 pi 11)

What is the most common urea cycle disorder? (FAI I pi 05) (FAI2p III)

Copyright 2012. Doctors In Traming.com, LLC. All Rights Reserved.

(FAI2pll2)

I. What are the byproducts of MAO and COMT enzymatic activity on dopamine, norepinephrine, and epinephrine? (FAI I pi07) (FA12 pi 12)

, What substrate and cofactor is required for the generation of GABA? (FAII pl06)(FAI2pll2)

Amino Acid Disorders


H: Chapter 364 PKUfFAII pl07)(FAI2pll2) Alkaptonuria (FAII pl08) (FAI2 pi 12) Albinism (FAII pl08) (FAI2 pi 12) Homocystinuria (FAII pl08) (FAI2 pi 13) S-adenosyl-methionine (FAII p92) (FAI2 p98) Cystinuria (FAII pl08) (FAI2 pll3) Maple syrup urine disease (FAII pl08) (FAI2 pll3) Hartnup disease (FAII pl08) (FAI2 pll3)

Quick Quiz
JSrSfcREliiSliwiraSTeif What is the diagnosis?

A patient with PKU should have diet low in phenylalanine. What other dietary modications should a patient with PKU make?

A middle-aged man has dark spots on his sclera and has noted that his urine turns bk when left sitting for a period of time. What is the diagnosis?

What is the underlying cause of maple syrup urine disease? (FAI I pl08) (FAI2 pi 13)

Copyright <) 2012, Doctors In Tr.iining.coin. LLC. All Rights Reserved.

E l f ,

What are the differences between c L'MllU'J (FAI2plOO-IOI)


CPSI

etase (CPS) I and CPS II? CPSII

Location
Pathway Nitrogen source

A 45-year-old male alcoholic gets blistering lesions in sun-exposed areas especially the dorsum of the hands. He also has hypertrichosis of the face. What is the diagnosis? (FAI2 p386)

What is the rate-limiting enzyme of heme synthesis? (FAI I p354) (FA 12 p386)

Calcium, Iron, and Zinc


R: Chapter 9 H: Chapter 357 Calcium (FAII p293) (FAI2 p320) Iron - Iron in the human body - Iron deciency (FAII p349) (FAI2 p380) (R p659) - Iron poisoning (FAII p224) (FAI2 p249) - Hemochromatosis (FAII p334) (FAI2 p364) (R p86l) Zinc (FAII p94) (FAI2 p99) (Phys p856) (R p439, Table 9-10)

4. Iron Proteins (FAI I p353) (FA12 p385)


Ferritin Iron-protein complex (Ferric acid and apoferritin) Cellular storage protein for iron Acute phase reactant Transferrin Protein that binds ferric molecules and transports them through plasma. Synthesized in the liver t Vi = 8 days Increased in iron deficiency

Copyright 2012, Doctors In Training.com, LLC. All Rights Reserved.

uick Quiz
is iron bound in blood?

What has been the most common clinical scenario of excess iron ingestion in the last 3 decades? (FAI I p224) (FAI2 p249)

7. What are some of the clinical effects of zinc deciency? (FAI I p94) (FAI2 p99)

8. What are some of the signs of hypocalcemia? (FAI I p464) (FAQ p464)

Trace Minerals and Toxic Metals


R: Chapter 9 Overview of elements Trace minerals Copper Toxic metals - Lead poisoning (FAII p350) (R p406) - Mercury poisoning (R p407) - Arsenic (R p408) - Cadmium (R p408)

Copyright v':: 2012. Doctors In Training.com, LLC. All Rights Reserved.

9. Overview of Elements
1

H
3 MnMn

He
4

Li
11

Be 90l
12

rrfroQMt 7

B
Mil!. 11

C Si
J 2
- O U -

N
IS

10

Ne

Na Mg
19

Al

p
3]

s
liuur urw
14

16

CI Ar
fcrypon

K
m c i

Ca
M

Sc Ti
M 5 5 4 0
_9iHi.

It

22

22

roMxui tacmotn

Cr Mn Fe Co Ni Cu Zn Ga Ge A s Se stm _&ZL
-wwi.
45
44 47 eaauM 41

2S

26

29

Jl

28

mn Mkn

Br

ffW-

Kr
S4

R Sr b
ss
tiaaet 87 56 S7-70

Y *
71

Zr

Nb Mo Tc Ru Rh Pd A g Cd my
tanttUi 73

In
I

49

St

Sn Sb Te
92

Xe A t Rn
urn

7$

75

77

79

7 9

Cs Ba
J.HV

Lu
ttwoncun 10]

Hf Ta
104 105

W Re OS
100 bcMun 107

89.102

_no_

Fr Ra * * Lr

,'wn TumrT -JgU2_ 1M 109

lr

Pt A u Hg
m
iiHMkjn 112

TI

Pb
11 4

Bi

Po

Rf Db s g Bh Hs Mt Uun Uuu Uubj W1


CCOA

Uuq

Lanthanide series

5 7

St

6 9

-tkt"v

C O

?CC=4CM

6 1

MTOUH

62

uceen

6]

easaan

tOMCT

6 4

65

mCMn r o n u i - i5555T-I 6 67 69

69

70

La
89

Ce
won 90

Actinlde series

Pr Nd Pm Sm Eu Gd Tb Dy Ho '" l 1MM 1W ,M1 (SM (reUttnw uanwv occuim cMonn


144 M IH9

Er Tm Yb
100 11 0 102
(W*

9 1

A c Th ran

Pa aiiM

u mn

9 2

9)

94

9 S

96
1341 1

97

99

9 9

Np Pu A m Cm B k Cf Es Fm Md No no run ma OT C ma inn Bfl m'l

10. Trace Minerals Mineral Lithium Beryllium Boron Chromium Manganese Cobalt Nickel e n Copper I. Mercury Poisoning Accumulates in the kidney and brain Acrodynia - peeling of the fingertips Abdominal pain Common sources: shark, swordfish, old thermometers, batteries Source Function Deciency Excess

/&i$y

u o
C D

[32]

Copyright 20I2. Doctors In Training.com. LLC. All Rights Reserved.

hypersensitivity? (FAI I p2IO) (FAI2 p233) What are some of the risk factors for esophageal cancer? (FAI I p323) (FAI2 p35l) What enzyme is decient in Lesch-Nyhan syndrome? What is the treatment? (FAII p69) (FAI2 p70)

Vitamins D, K, and A
R: Chapter 9 Phys: Chapter 71 Vitamin D (FAII p93) (FAI2 p99) (R p433) (Phys p855) Vitamin K(FAII p94) (FAI2 p99) (R p438, Table 9-9) (Phys p855) Vitamin A (FAII p90) (FA 12 p95) (R p431) (Phys p853) o

Vitamin D (FAI I p93) (FA12 p99) Tj 4, How does the biological form of vitamin D exert its actions?
It interacts directly with target cell DNA to selectively stimulate or repress gene expression.

True or false? Vitamin D is obtained by the body from both sun exposure and diet. True, up to 8096 of required vitamin D can be derived from sunlight (at least 15 minutes daily), and the remaining amount must be ingested.

. 6.

e n

How does vitamin D help maintain adequate plasma levels of calcium? Increases calcium uptake in the intestine (via increased expression of calcium binding protein) Stimulates PTH-dependent reabsorption of calcium in the distal tubules Stimulates bone resorption when necessary (along with PTH)

_Q

D 7 . What are the steps in the metabolism of vitamin D?


Gut absorption (D2) or skin synthesis (D3) Binding to plasma a I-globulin (D-binding protein) and transport to liver Conversion to 25-hydroxyvitamin D (AKA 25-hydroxycholecalciferol) by 25-hydroxylase in liver Conversion of 25-hydroxycholecalciferol to 1,25-dihydroxycholecalciferol by a I-hydroxylase in the kidney What is the precursor of vitamin D3 in the skin? 7-dehydrocholesterol What other names have been given to D2, D3, and 1,25-dihydroxycholecalciferol? D2 - ergocalciferol (ingested form derived from plants) D3 - cholecalciferol (formed in sun-exposed skin) l,25(OH)2D - calcitriol (the active form of vitamin D)
Copyright 2012, Doctors In Training.com, LLC. All Rights Reserved.

10. When prescribing vitamin D replacement, which form is preferred? Over the counter D3 (cholecalciferol) 11. What are the names for vitamin D deciency in adults and in children? Adults Children 12. Explain how vitamin D deciency brings about the clinical symptoms seen in osteomalacia and rickets? Lack of vitamin D hypocalcemia increased levels of PTH - Mobilization of caldum from bone - Decreased renal calcium excretion - Increased renal excretion of phosphate hypophosphatemia impairment of bone mineralization 13. What is the basic derangement in both rickets and osteomalacia? Excess in unmineralized bone matrix 14. What are some of the common clinical manifestations of rickets? Bow-legged Lumbar lordosis Pectus carinatum (pigeon chest) - protrusion of the sternum and ribs Rachitic rosary- overgrowth of cartilage or osteoid tissue at the costochondral junction C O 15. What problem is encountered with a vitamin D intake 10-100 times the US RDA? Q Hypercalcemia 16. In what disease does vitamin D toxicity result from excess macrophage generation of 25-hydroxy-vitamin D? Sarcoidosis

Vitamin K (FAI I p94) (FAI2 p99) 17. What is the principle role of vitamin K? &i' modication of various where it serves as a ' coenzyme in the carboxylation of certain glutamic add residues present in these proteins Sy 18. For what protein synthesis is vitamin K dependent upon? qProteins C & S, prothrombin, and clotting factors II, VII, IX, and X (2,7,9, and 10) j" 19. What characterizes vitamin K deciency in adults and children? H-" Hemorrhagic disease |jj 20. Why are newborns particularly prone to vitamin K deciency? What prophylactic ^ measures can be taken to reduce the incidence of this problem? ^ Newborns do not have microbes in the gut to produce vitamin K, and the mother's milk only provides ^ l/5th of their need. Q It is recommended that newborns receive a single IM dose (Img) of vitamin K at birth. ^
ZJ

21. What pharmacologic agents can cause vitamin K deciency with long-term use? q" Coumadin X. Anticonvulsants CL Antibiotics ( loss of gut bacteria) j 22. What characterizes vitamin K toxicity? Hemolytic anemia and jaundice in an infant
Copyright 2012. Doctors In Training.com. LLC. All Rights Reserved.

[35]

Vitamin A (FAI I p90) (FAQ p95) 23. What are the different forms of vitamin A? Retinol, retinal (both used by the body) P-carotene (deaved in intestine to yield 2 molecules of retinal) Retinoic add (cannot be reduced, unusable by the body) 24. Retinol esters are stored on the liver until needed by the body. How are they transported in the body when needed? Via plasma retinol-binding protein (RBP) 25. In what broad metabolic functions is vitamin A necessary? Vision Growth Reproduction (cf spermatogenesis, 9 prevent fetal resorption) Maintenance of epithelial cells (especially mucous secreting cells) 26. en For what Acne disorder is and topical retinoic acid useful? ^

psoriasis

""")

X> 27. What retinoid derivative can betaken orally for the treatment of acne? ^ X Isotretinoin (Accutane) ^ C <
rO en

28. What are the signs of vitamin A deciency? Night blindness

C Xerophthalmia (pathologic dryness of the conjunctiva and cornea) corneal ulceration and blindness

.(wrinkling, douding of cornea) C ( d r y, s i l v e r - g r a y p l a q u e s o n t h e b u l b a r c o n j u n c t i v a )

^ 29. What are the signs of hypervitaminosis A (vitamin A toxicity)? ^> Headache, nausea/vomiting, stupor ^ _Q O **Q ro
i

/^s

Skin Liver-enlarged Bone Increase

dry (and and in

and possibly joint intracranial

pruritic cirrhotic) pain pressure

>

'**> ^ ^^ 30. True or false? Lack of vitamin A in pregnant women has the potential for causing ^ congenital malformations. False, excessive vitamin A in pregnant women has the potential for causing congenital malformations

yincluding: LU X 31. In which patient populations is vitamin A supplementation a particularly bad idea? Pregnancy increased risk of teratogenic damage Smokers increased risk of lung cancer 00

""*,

U O

[36]

Copyright 2012. Doctors In Training.com, LLC. All Rights Reserved.

What vitamin in excess can cause hypercalcemia? What are the symptoms of vitamin A toxicity? (FAI I p90) (FAQ p95) In which common chronic diseases is vitamin D supplementation particularly essential?

Vitamins C, E, and Other Antioxidants


R: Chapter 9 Phys: Chapter 71 Vitamin C (FAII p93) (FAI2 p98) (R p437) (Phys p855) Vitamin E (FAII p93) (FAI2 p99) (R p438. Table 9-9) (Phys p8S5) Other antioxidants

Vitamin C (FAI I p93) (FA12 p98) 36. What is the main metabolic reaction that vitamin C is involved in? 7^
Hydroxylation of and residues spongy vessels (bleeding Anemia into of collagen gums hemorrhages joint spaces) f ~^ |vj i j

37. What are the major signs of vitamin C deciency (scurvy)? rn


Sore, Loose teeth Fragile blood Swollen joints Impaired wound healing

Vitamin 38. What

E is

(FAI I the primary

p93) (FA12 p99) function of vitamin E?

i j

CO

Antioxidant - prevention of the nonenzymatic oxidation of cell components (especially on red blood [ i cells) by molecular oxygen free radicals

39. What is another name for vitamin E? 40. What is associated with vitamin E deciency?
Spinocerebellar degeneration ataxia Peripheral neuropathy and proximal muscle weakness .-

41. Mutations in what gene results in the autosomal recessive hereditary vitamin E deciency? >> Alpha-tocopherol transfer gene protein !
i

42. True or false? Vitamin E supplementation can help prevent the development of Alzheimer disease?
False, studies have not yet shown that vitamin E supplementation can reduce the risk of Alzheimer , disease; however, studies have shown that vitamin E supplementation at doses of < 1,100 IU daily do ~r' increase one's risk of "all-cause" mortality. (Ann Int Med 2005; 142:37-46).
Copyright^-; 2012. Doctors In Training.com. LLC. All Rights Reserved. L

[38]

Copyright 2012. Doctors In Traming.com. LLC. All Rights Reserved.

tion Warm-Up

In what disease would you see the following antibodies? (FAI I p2l2) (FAI2 p235) Anti-mitochondrial
Anti-TSH receptor Anti-centromere Anti-basement membrane Anti-neutrophil

What is the most common salivary gland tumor? What is the second most common salivary gland tumor? What is the most common location for a salivary gland tumor? (FAII p32l)(FAI2p350)

Folic Acid, Vitamin BI2, and Vitamin B6


Phys: Chapter 71 Water soluble vitamins Folic acid (FAII p92) (FAI2 p97) (Phys p854) Vitamin 6/2 (cobalamine) (FAII p92) (FAI2 p97) (Phys p854) Vitamin B6 (pyridoxine) (FAII p9l) (FAI2 p96) (Phys p854)

Folate (B9) and BI2 (FAII p92) (FAQ p97) 4. In what metabolic reaction(s) is folic acid (B9) involved?
Synthesis of purines (A, G) and thymine (T)

5. What is the biologically active form of folic acid? 6. What is the intracellular storage form of folic acid (folate)?
N-methyl folate

7. What are the characteristics of folic acid deciency? Growth failure Megaloblastic anemia 8. What is megaloblastic anemia? A type of macrocytic (large RBC) anemia characterized by an elevated number of megaloblasts in the marrow 9. Deciency in which two vitamins may cause megaloblastic anemia? Folate and BI2
Copyright 2012. Doctors In Traimng.com, LLC. All Rights Reserved.

10. How can you determine if a megaloblastic anemia is caused by folate or BI2 deciency? BI2 deficiencydecreased serum BI2 "^ Folate defidency - decreased serum folate (both may have decreased RBC-folate) "^
(note that megaloblastic anemia can be caused by a deciency in both at the same time) ^

11. How much folate should a sexually-active woman of childbearing age take in order to ^ prevent neural tube 0.4 mg of folic add a day (4 grams if at high-risk for neural tube defects) 12. defects? ^

What results from an excess of folate? Why? """i BI2 deciency results because this vitamin is used in making tetrahydrofolate. /*^

13. True or false? BI2 deciency can cause a deciency in folate. True

/^Jl\

14. What percentage of people in the US is estimated to have low serum folate levels? ~ 15-20% 15. What is found in the center of the corrin ring of cobalamin (vitamin BI2)? ^ Cobalt e^ 16. In what metabolic reactions is vitamin BI2 involved? ' Homocysteine & methyl-THF ^ methionine & THF (required for SAM to function (FAI I p92) (FAI2 p98) ^ Methylmalonyl CoA (coenzyme A) succinyl CoA ^ 17. What is seen in BI2 deciency (most often due to a failure to absorb this vitamin rather than its absence from the diet)? <2
/ %.

Pernicious anemia: > Megaloblastic anemia ^ t - CNS symptoms (myelin degeneration in the dorsal and lateral tracts of the spinal cord) sensory .m, .p problems (i.e. pins-and-needles) ' ^> - Atrophy of stomach fundic glands ( achlorhydria) and replacement of the gastric epithelium with ^ <U mucus-secreting goblet cells that resemble those lining the large intestine (intestjnalization) -. ~c\ - Homocystinuria and methylmalonic acid in urine

yfHBk

18. What is another Subacute combined degeneration

name

for

B12

neuropathy?

**)

,rt 19. What is usually the cause of malabsorption of B12? p> Autoimmune destruction of gastric cells responsible for the synthesis of intrinsic factor (which is ' necessary for the absorption of B12)
PO

***)
/*%

20. Where is BI2 absorbed in the Gl tract? 2L Distal ileum


111 ^

-X 21. Name two malabsorption problems of the distal ileum that cause BI2 deciency. ^ y ^ 22. IM Crohn's What disease is the injection and treatment of for celiac sprue ^ ^ ^

B12 deciency? cyanocobalamin

[40]

Copyright 2012, Doctors In Training.com. LLC. All Rights Reserved.

Vitamin B6 23. What is the biologically active form of B6 (pyridoxine)?


Pyridoxal phosphate

24. What is the metabolic function of pyridoxal phosphate?


Coenzyme for numerous enzymes including those of and deaminations) metabolism (transaminations

25. What drug can lead to a deciency in B6 as well as B3? (used for tuberculosis) 26. What are the clinical ndings of B6 deciency?
Same as riboavin deciency + hyperirritability, and peripheral neuropathy

Quiz
7. A patient presents with convulsions and irritability, these symptoms in this patient? lat vitamin deciency is causing

I. What type of anemia can be caused by folate or BI2 deciency? (FAI I p92) (FAQ p9/

Where is BI2 absorbed into the circulation? (FAI I p92) (FAQ p97)

Vitamins B5, Niacin, Riboavin, Thiamine, and Biotin


Phys: Chapter 71 Vitamin B5 (pantothenate) (FAII p9l) (FAI2 p95) (Phys p855) Vitamin B3 (niacin) (FAII p9l) (FAI2 p96) (Phys p8S3) Vitamin B2 (riboavin) (FAII p9l) (FAI2 p95) (Phys p854) Vitamin BI (thiamine) (FAII p9l) (FAI2 p95) (Phys p853) Biotin (FAII p93) (FAI2 p98)

30. What vitamins have the following names:


Thiamine Retinol, retinal Pantothenic acid Pyridoxine a-tocopherol Folic acid Niacin Riboavin Ascorbic acid Cobalamin B i A B s B E B* B 3 B . C B

hint The Rich Never Pan Pyrite Filled Creeks. B. Bi B3 Bs B* B* Be

Copyright is) 2012. Doctors In Training.com. LLC All Rights Reserved

Vitamin BI (thiamine) 31. What is the functionally active form of thiamine (BI)? 32. In what reactions does thiamine pyrophosphate have a role? ^ Pyruvate acetyl CoA Pyruvate dehydrogenase ^ a-Ketoglutarate succinyl CoA (TCA) a-ketoglutarate dehydrogenase R i b o s e 5 - P < - - g l y c e r a l d e h y d e 3 - P ( H M P ) T r a n s k e t o l a s e ^ 33. How is thiamine deciency diagnosed? By an increase in erythrocyte transketolase activity observed upon addition of thiamine 34. What two syndromes are associated with thiamine (BI) deciency? In what ^ populations are these usually seen? ~ Beriberi (dry and wet) Where polished rice is the major component of the diet Wernicke-Korsakoff syndrome Chronic alcoholism (hint BerI BerI) "^ 35. What characterizes dry beriberi? Nonspecific peripheral with myelin degeneration To e - d r o p , wrist-drop, and Musde weakness, hyporeflexia,

foot-drop areflexia

"^ ^
S%\

36. What characterizes wet beriberi? P e r i p h e r a l v a s o d i l a t i o n h i g h - o u t p u t p e r i p h e r a l e d e m a Cardiomegaly 37 What are the clinical characteristics of Wernicke-Korsakoff syndrome? * Ocular disturbances and
CO
/nis

nystagmus

'

Gait

ataxia

M e n t a l d y s f u n c t i o n ( c o n f u s i o n , a p a t h y, l i s t l e s s n e s s , a n d d i s o r i e n t a t i o n ) ^
/"^\

C Korsakoff psychosis- retrograde recall, inability to acquire new information, and rd

> 38. What exactly is confabulation? ^ -Si The invention of ctitious detail about supposed past events (often to disguise an inability to remember "^ -3 past events) /^ oo Vitamins B2, B3, B5, and B6 1 ^
/Witj

Jx)
i

(j) 39. What are the two biologically active forms of riboavin (B2)? What is the role of these molecules?

no

Both are cofactors for oxidation-reduction reactions.

_ 40. What are some of the symptoms associated with riboavin deciency? ^ ijj ~f~ Cheilosis/angular stomatitis (^J Glossitis Dermatitis (smooth, purple tongue) ^ *

o
CO

41. What is cheilosis? What is angular stomatitis? Cheilosis - fissuring of the corners of the mouth Angular stomatitis - inflammation of the comers of the mouth
Copyright 2012. Doctors In Training.com, LLC. All Rights Reserved.

[42]

42. In what patient population is angular stomatitis particularly common? Denture wearers 43. What nutrient deciencies are associated with cheilosis, glossitis, and stomatitis? Iron, riboavin, niacin, folate, and BI2 44. What are the biologically active forms of niacin (B3)? What is the role of these molecules? Nicotinamide adenine dinucleotide (NAD+) Nicotinamide adenine dinucleotide phosphate (NADP+) Both are cofactors for oxidation-reduction reactions. 45. Which amino acid required for the generation of niacin? 46. What disease is caused by niacin deciency? What are the symptoms of this disease?

hint niadn (B3); NADH yields 3 ATP; 3 Ds 47. Why might pellagra be seen in a population that eats primarily corn? can be metabolized to form niacin, and corn is lacking in tryptophan. 48. How is niacin (at a dose of lOOx the RDA) affective in treating type lib hyperlipoproteinemia? Inhibits lipolysis in adipose tissue less circulating free fatty acids less fatty acids to liver * less VLDL made and less LDL produced

rjrj

49. What is the main side effect of high dose niacin treatment of dyslipidemia? How can it O be prevented? I m The peripheral vasodilation (ushing) of high-dose niadn can be lessened by taking aspirin with niadn. 50. What is the role of pantothenic acid (B5) in metabolism? uj It is a component of coenzyme A (CoA) which functions in the transfer of acyl groups. 51. True or false? There is no deciency disease for pantothenic acid True Biotin 52. What (FAI is I the p93) metabolic (FAI2 role of p98) biotin?

(B5). r~5 C O O c"

^ (D Apoenzyme in carboxylation reactions (hint buy-a-tin of C02) ^ 7+

53. What can cause a deciency in biotin? ^ The glycoprotein avidin which is found in egg whites prevents absorption of biotin (with a normal diet 3 Ui 20 egg whites per day are required to induce a deciency). Antibiotic use (gut bacteria make biotin for us)
Copyright 2012. Doctors In Traimng.com. LLC. All Rights Reserved.

[43]

MfSTSSBRr
1 Increased RBC fragility ' Dermatitis, cheilosis, glossitis Peripheral neuropathy, glossitis Hemorrhagic disease Neural tube defects Dermatitis, diarrhea, dementia Megaloblastic anemia Pernicious anemia Bitot spots, keratomalacia, xerophthalmia Osteomalacia Rickets Which vitamin matches the following statement? Can be used to treat acne and psoriasis Used in oxidation/reduction reactions Used in carboxylation reactions Involved in the hydroxylation of prolyl residues Requires intrinsic factor for absorption Deficiency may result from kidney disease . Used by pyruvate dehydrogenase and a-ketoglutarate dehydrogenase Given prophylactically to newborns Can be used to elevate HDL and lower LDL Deficiency can be caused by isoniazid use Cobalt is found within this vitamin Critical for DNA synthesis

Copyright 2012. Doctors In Training.com. LLC. All Rights Reserved.

u est ion Warm-Up


Which lysosomal storage disease matches the following statement? (FAI I pill) (FAQ pi I. Accumulation of GM2 ganglioside Associated with renal failure Accumulation of dermatan sulfate Deciency in hexosaminidase Which antivirus) matches the following statement? (FAI I p 195-196) (FAQ p2l6-217) Inhibits CMV DNA polymerase Used in treatment for chronic hepatitis C Blocks viral penetration and uncoating Treats both inuenza A and B Second-line for CMV retinitis What is the function of MacConkey's agar? (FAI I pl38) (FAQ pl48)

Genetic Laboratory Techniques


R: Chapter 5 PCR (FAII p8l) (FAI2 p84) (R pl74) Blotting procedures (FAII p8l) (FA/2 p85) (R pl78) Microarrays (FAII p8l) (FAI2 p8S) Enzyme-linked immunosorbent assay (ELISA) (FAII p8l) (FAI2 p85) FISH (FAII p82) (FAI2 p85) (R pl79) Cloning methods (FAII p82) (FAI2 p86) Gene expression modications (FAII p82) (FAI2 p86) Karyotyping (FAII p82) (FAI2 p86) (R pi58) Electrophoresis

Copyright'S 2012. Doctors In Trainmg.com. LLC. All Rights Reserved

Gel electrophoresis
Cathode

Molecular weight

Anode

4. If well I contains DNA sample A, well 2 contains DNA sample B, and well 3 contains DNA sample C; then what can you say about wells 4, 5, 6, 7, and 8?

5. DNA sequences

Cathode

DNA sequence patient A

Molecular weight DNA sequence patient B

Anode
Copyright 2012. Doctors In Trainmg.com. LLC. All Rights Reserved.

ick Quiz

7. What type of test uses a known antigen to discern the presence of an antibody? (FAQ pS

What type of test is performed in order to diagnose chromosomal imbalances? (FAII p82) (FAQp86)

Inheritance
R: Chapter 5 Genetic terms (FAII p83) (FAI2 p87) Hardy-Weinberg population genetics (FAII p84) (FAI2 p88) Imprinting (FAII p84) (FAI2 p88) (R pl7l) - Prader-Willi syndrome - Angelman syndrome Pedigrees Modes of inheritance (FAII p85) (FAI2 p89)

9. What is the frequency of the BB phenotype and the Bb phenotype if the frequency of allele B is 70%? 10. Prader-Willi Syndrome (FAI I p84) (FAQ p88)
Deletion of proximal portion of chrom I5ql l-ql3 from paternal origin Presents in infancy: hypotonia, poor feeding, characteristic facial features (almond shaped eyes, downward turned mouth) Sx: hyperphagia, obesity, short stature (partial GH deficiency), MR, behavior disorders (tantrums, skin-picking, OCD), hypogonadotrophic hypogonadism * genital hypoplasia, osteoporosis, delayed menarche Dx: confirmed with FISH (fluorescence in-situ hybridization) Rx: limit access to food, GH if short stature

Copyright C) 2012. Doctors In Training.com. LLC. All Rights Reserved

II. The numbers in this diagram indicate the age of disease presentation. What is the name of this phenomenon?

a a
28l

'62

6 UiCL ML6
42 45

"27

c5~^

12. What is the likelihood that child X will have the genetic mutation?

13. If the shaded boxes indicate a phenotypic expression of a genetic mutation, what is the name given to this
to

m "c
(J
CO

< u y

phenomenon?

"8 1

<T5

IE

o
CO

14. Mitochondrial Inheritance Defects


Mitochondrial myopathies (ragged-red muscle fibers seen on biopsy) Leber hereditary optic neuropathy Leigh syndrome (subacute sclerosing encephalopathy)
Copyright 2012. Doctors In Training.com, LLC. All Rights Reserved.

[48]

of Session Quiz
15. What is the probability that a female heterozygous for an X-linked disease will pass it o to her son?

16. What is the probability that a female heterozygous for an X-linked disease that mates wttr a normal male will have a carrier daughter?

What is the probability that a female carrier of an X-linked disease will have a child wit that disease assuming she mates with a normal male?

If aa symbolizes a recessive disease, what is the likelihood that parents Aa and Aa will have a phenotypically normal child?

Cystic brosis is an autosomal recessive disorder. Two parents that are heterozygous cystic brosis have a normal, non-affected child. What is the probability that the child homozygous normal?

Upon examination of a pedigree, you note that both males and females are affected w a disease in every generation. What type of genetic disease is this?

LI. What is the frequency of the Aa genotype and the AA genotype if the frequency of allel A is 0.95?

If 49% of a particular population is homozygous for a curly hair gene that is dominant j straight hair gene, what percentage of the population has curly hair?

5. Frequency of CFTR mutation X in patients with cystic brosis is 0.1. Cystic brosis car brosis patients mutation X or mutation Y in Y? caused by eitherare homozygotes for mutationthe CFTR gene. What percentage of c)

24. Two patients have the same mutation on chromosome 15 but have different phenotyr. expressions. One patient received the mutation from the father while the other recei< the mutation from the mother What is this an example of?

Copyright 2012. Doctors In Trainmg.com. LLC. All Rights Reserved.

3 Quest/on Warm-Up
1. What is the rate-limiting enzyme for the following metabolic pathway? (FAII p95) (FAplOI) (Phys p8l7) Glycolysis Gluconeogenesis TCA cycle Glycogen synthesis Glycogenosis 2. Which of the oral agents used in the control of type II diabetes has the following characteristics? (FAI I p304-333) (FAI2 p330) (GG pl255, 1268-1269) Lactic acidosis is a rare but worrisome side effect Most common side effect is hypoglycemia MOA: closes K* channel on * cells depolarization Ca2"*" influx insulin release MOA: inhibits d-glucosidase at intestinal brush border MOA: agonist at PPARy receptors 3. What three mineral compounds can be used to treat esophageal reux? (FAII p338) (FAQ p368) (GG pl3l6)

Coagulation Basics
R: Chapter 4 Phys: Chapter 36 H: Chapter 116 Coagulation, complement, and kinin pathways (FAII p345) (FAI2 p376) (R p64) Coagulation cascade components (FAII p346) (FAI2 p346) (R pi 18) (H p986) Bradykinin (R p65) Coagulation disorders (FAII p35S) (FAI2 p387) (H p973) Hereditary thrombosis syndromes leading to hypercoagulability (FAII p356) (FAI2 p388) (R pl22)

Quick Quiz
4. Which coagulation factors are dependent on vitamin K for synthesis? (FAII p345) (FAQ p376) ~T

m
5. What coagulation factor is decient in hemophilia A? f^j (FAI I p355) (FAQ p387) (R pi43) (Phys p458)

I
6. What
(FAI

coagulation
I p346)

factor
(FAQ

is

decient
p376)

in

hemophilia
p458)

B?

r\
q

(Phys

7. What are some of the effects of bradykinin on the body? r/q (FAI I p345) (FAQ p376) (Phys 199-200) -,-,

8. What is the clinical consequence of a deciency in either protein C or protein S? (FAI I p356) (FAQ p388) (Phys 457-458) (H p987) '

m p. O

en

Copyright 2012. Doctors In Training.com. LLC. All Rights Reserved.

[I]

Agents Affecting Clotting Factors


GG Chapter 30 H: Chapter 118 Heparin (FAII p362) (FAI2 p395) (GG p853) (H p992) Lepirudin, bivalirudin (FAII p362) (FAI2 p395) (H p997) Argatroban, dabigatran Low molecular weight heparins Warfarin (FAII p362) (FAI2 p395) (GG p860) Heparin vs. warfarin (FAII p362) (FAI2 p396) Rivaroxaban Thrombolytics (FAII p363) (FAI2 p396)

!MW:rll
(Phys p459-460)

What lab value is used to monitor the following medications: heparin, warfarin, enoxaparin? (FAI I p356-36l)(FAQ p388-396) (Phys p460-46l) (H p462-263)

What is the treatment for heparin-induced thrombocytopenia? (FAI I p356) (FAQ p388, 396) (R pQ3, 668-669)

What are ve hereditary thrombosis syndromes? (FAI I p356) (FAQ p388) (R pQ2) (H p462)

Copyright ;=' 2012. Doctors In Training.com. LLC. All Rights Reserved.

on Warm"
What are the vitamin K dependent clotting factors? (FAI I p345) (FAQ p376-387) (H p46l.

-,, ,at are some examples of substances eliminated at a constant rate (zero-order elimination)? (FAI I p233) (FAQ p233)(H p35)

'. What disorder might you suspect in a patient with a perianal stula? What study would y< order to further conrm your suspidon? (FAI I p326) (FAQ p355-356) (COA p396, Fig B3.24)

Erythrocyte Basics
R: Chapters 3, 14 Phys: Chapters 32, 35, 83 Embryologic origins of bone marrow (Phys p4l4) (R p590) Fetal erythropoiesis (FAII pi24) (FAI2 pl3l) (Phys plOI9) RBC basics (FAII p342) (FAI2 p372) (Phys p4l3) Blood groups (FAII p345) (FAI2 p375) (Phys p445) Erythroblastosis fetalis (Phys p447, 1024)

What allows RBCs to change shape as they pass through vessels? (FAI I p342) (FAQ p372)

Where does fetal erythropoiesis take place? In which adult bones does erythropoiesis place? (FAI I pl24) (FAI2

What is the life-span of an RBC (which is the maximum time to expect foreign RBCs to in a patient after a blood transfusion)? (FAI I p342) (FAQ p372)

What name is given to immature erythrocytes? (FAI I p342) (FAQ p378)

Copyright @ 2012. Doctors In Traimng.com. LLC. All Rights Reserved.

RBC Pathology
R: Chapter 14 H: Chapters 57, 104 Pathologic RBC forms (FAII p348-349) (FAR p378) Polycythemia (FAII p36l) (FAI2 p394) (R p609) Heme synthesis, porphyrias, and lead poisoning (FAII p354) (FAI2 p386) (R p660)

8. What are the different forms of hemoglobin (Hb)? (H p852)


Hemoglobin HbA HbA2 HbAlc HbF Hb Gower HbS HbC Hb Bart's HbH Where Found 97% of normal hemoglobin 2% of normal hemoglobin Poorlyrcontrolled diabetes Fetal hemoglobin Embryonic hemoglobin Sickle cell hemoglobin Hemoglobin C disease Severe Cf-thalassemia Severe O-thalassemia Structure 02 02 02 02 02 B2-glucose 02 Y2 (gamma chains replace beta chains)
t> 3

02 Bs2 (glu > val in chain) 02 3C2 (glu lys in * chain) Y4 (no alpha chains) B4 (no alpha chains)

What are some of the different causes of polycythemia? (H p456-457)

. What pathologic form of RBC would you see in the following diseases? Lead poisoning G6PD deficiency DIC Abetalipoproteinemia Asplenia What is the rate-limiting enzyme in heme synthesis? (FAI I p354) (FAQ p386)

What is the structure of HbH? What disease results in HbH production? What is the structure of Hb Bart's? What disease results in Hb Bart's production? (FAI I p349) (FAQ p38l) (R p652)

. Does HbF have more or less afnity for 2,3-bisphosphoglycerate than adult hemoglobin? (H p852)

Copyright 2012. Doctors In Training.com. LLC. All Rights Reserved.

(FAI2p386,395)(Rp406)

Which antibiotic matches the following description? (FAI I p68,70,75) (FAQ p78,204) Inhibits 50S peptidyltransferase (FAI I p75) (FAI2 p78) Binds 50S, blocking translocation (FAII p75) (FA 12 p78) Bind 30S, preventing attachment of tRNA (FAI I p74) (FAQ p77) Inhibits prokaryotic RNA polymerase (FAI I pl9l)(FAI2 p2l2) Inhibits prokaryotic topoisomerase (FAI I p70) (FAI2 p2l I) Inhibits prokaryotic dihydrofolate reductase (FAI I p68) (FA12 p2IO) What is the most likely scenario that a person would receive toxic exposure to arsenic? (Rp408)

Microcytic Anemia
H: Chapter 103 R: Chapter 14 Anemias (FAI2 p380) Microcytic anemias (FAII p349) (FAI2p 380-381) (R p640) - Iron deciency - Alpha thalassemia - Beta thalassemia - Lead poisoning - Sideroblastic anemia Distinguishing iron deciency anemia from anemia of chronic disease Lab values in anemia (FAII p353) (FAI2 p385) (R p640)

What test can be used to diagnosis beta-thalassemia minor? (FAII p349)(FAI2p38l)(Hp65l)

>. What lab ndings allow you to distinguish iron deciency anemia from a microcytic, hypochromic anemia resulting from thalassemia? (FAII p349) (FAQ p 380-38l)(H p650-65l)

tat should you rule-out in a male over 50 with new onset iron deciency anemia? (FAI I p349) (FAQ p359) (R p308)

Copyright 2012. Doctors In Training com. LLC. All Rights Reserved.

Macrocytic and Nonhemolytic, Normocytic Anemias


R: Chapter 14 H: Chapters 57, 103 Macrocytic anemia (FAII p350) (FAI2 p382) - Folate deciency - 8/2 deciency Nonhemolytic, normocytic anemia (FAII p35l) (FA12 p383) - Anemia of chronic disease (R p662) - Aplastic anemia (R p662)

L* J ^ 14->7 [JlKfj'li

A patient is diagnosed with a macroJHiSliitSSiwnfciiErTiS giving folate alone? (FAII p92) (FAQ p97) (Rp657-659) (H p870-87l)

irfHB

What is the cause of anemia given the following statement? (FAII p350-353) (FAQp380-388) (H p454) Microcytic anemia + swallowing difficulty + glossitis Microcytic anemia + > 3.5% HbA2 Megaloblastic anemia not correctable by BI2 or folate Megaloblastic anemia along with peripheral neuropathy Microcytic anemia + basophilic stippling Microcytic anemia reversible with B6 HIV positive patient with macrocytic anemia Normocytic anemia + red urine in the morning Normocytic anemia and elevated creatinine What are the causes of aplastic anemia? (FAI I p35l) (FAQ p383) (R p663)

Copyright 2012. Doctors In Trajning.com. LLC. All Rights Reserved.

I. What are the 3 different mechanisms cells employ to break down proteins? (FAI I p75) (FAQ p78) (Phys 34-38,41)

Which medication used in the treatment of HIV is known for causing bone marrow suppression? (FAI I p350) (FAQ p382)

What is the treatment for lead poisoning? (FAI I p350) (FAQ p38l) (GG pl873-l874)

Intrinsic Hemolytic Anemia


H: Chapters 57, 104, 106 R: Chapter 14 Normocytic, normochromic anemias: types of hemolysis (FAII p350) (FAI2 p382) (H p454, Fig. 57-17) Intrinsic hemolytic normocytic anemia (FAII p352) (FAI2 p382) (H p879)

A child anemic since birth has now been cured with splenectomy. What is the disease? (FAII p352) (FAQp384)

What ndings are associated with hereditary spherocytosis? (FAI I p352) (FAQ p384) (Phys p420) (R p642)

lat is the difference between the hemoglobin S defect and the hemoglobin C defect? (FAI I p352) (FAQ p384) (Phys p420) (H p855-857)

What can be seen in erythrocytes in patients that do not have a functional spleen? (FAI I p349) (FAQ p379) (Phys p4l9)

Copyright 2012. Doctors In Training.com, LLC. All Rights Reserved.

/ ^ v \

Extrinsic
H: R: Extrinsic hemolytic

Hemolytic
Chapter Chapter normocytic anemia (FAII p353)

Anemias
106 14 (FAI2 p385) (R p653-6S4)

^
"*> "^ ^

8.

Coomb's

(+)

(FAI

p353)

(FAI2

p385)

RBC agglutination with the addition of antihuman antibody because RBCs are coated with immunoglobulin or complement proteins ^ Direct Coomb's ( D AT ) m^ Prepared antibodies are added to a patient's washed RBC to detect the presence of ^ immunoglobulins already present on the RBC (Using an antibody to detect an antibody.) Positive in: hemolytic disease of the newborn, drug-induced autoimmune hemolytic anemia, ' hemolytic transfusion reactions "*) Indirect Coomb's ^ Patient's serum is incubated with normal RBC to detect for the presence of antibodies a Positive when: antibodies present to foreign blood (used to test blood prior to transfusion, screening for maternal antibodies to a fetus'blood)

9.

Cold

Agglutinins

(FAI

p353)

(FAQ

p385)

Antibodies against RBCs that interact more strongly at low temps (4C) than at body temp "^ Nearly always ^ Occur regularly in infections with /^\ Problems/disease occurs when there is drculation to a cold extremity IgM binds RBC antigen c o m p l i m e n t f i x a t i o n ^ M A C l y s i s ( a n d o p s o n i z a t i o n p h a g o c y t o s i s ) i

10.

Warm

Agglutinins

(FAI

p353)

(FAQ

p385)

*,

Antibodies that react against RBC protein antigens at body temperature ^ Nearly always ^ Seen in I) 2)

3
C C

3)
4) ^

i^?\

^ -

LU

X
X

LU

[8]

Copyrighc <Q 2012. Doctors In Tratning.com. LLC. All Rights Reserved.

d of Session Quiz
What is the difference between a warm agglutinin and a cold agglutinin? (FAI I p353) (FAQ p385) (R p653)

What are schistocytes? (FAI I p353) (FAQ p378) (R p645-646)

'3. What are two protozoal diseases that can cause hemolytic anemia? (FAII p353) (FAQ p382, 384)

In what hematologic disorder would you nd the following abnormal tests/ndings? Ham's test
DEB test Heinz bodies Basophilic stippling Osmotic fragility test

An 11-year-old child presents with a chronic non-healing ulcer on his foot and imaging sh< a small calcied spleen. What drug can improve his symptoms? (FAI I p386) (FAQ p422

Copyright -TO 2012. Doctors In Training.com. LLC. All Rights Reserved.

What enzyme catalyzes peptide bond formation during protein synthesis? (FAII p75) (FAQp78)

2 What conditions are associated with target cells? (FAI I p348) (FAQ p379) (R p649)

What name is given to anemia resulting from mechanical destruction of erythrocytes due aortic stenosis or prosthetic heart values? (FAI I p353) (FAQ p382) (R p654)

Platelet Basics
R: Chapter 4 Megakaryocytes (R p625) Platelets (FAII p342) (FAI2 p372) (R pll7) Platelet plug formation (FAII p346) (FAI2 p377) (R pi 17) Thrombogenesis (FAII p347) (FAI2 p377) (R pl2l)

4. Platelet Stimulation (FAI I p346) (FAQ p377)


Adhesion (endothelial damage, vWF, Gplb) Activation - Secretion of ADP, PDGF, serotonin, brinogen, lysosomal enzymes, thromboxane A2, calcium, thrombin - thrombin: fibrinogen fibrin - thromboxane A2 vasoconstriction and platelet aggregation Aggregation of platelets via Gpllb/llla

5. Von Willebrand Factor (R pi 17, 670)


Basics: several subunits linked by disulfide bonds, synthesized by endothelial cells and megakaryocytes Major functions: - Complexes with and stabilizes factor VIII (deciency-* t PTT) - Platelet adhesion to vessel wall and other platelets (deciency -* t bleeding time)

Platelet Disorders
I H: Chapter 115 LO Platelet disorders (FAII p355) (FAI2 p387) LU

Copyright 2012. Doctors In Training com. LLC. All Rights Reserved.

UIZ What is the cause of ITP?

BlSEBItiSvEma

What is the defect in Bemard-Soulier disease? (FAI I p356) (FAQ p387) What is the life span of a platelet (which is also the maximum life of platelets after a transfusion)? (FAI I p342) (FAQ p372) What molecule is expressed on the surface of a platelet after it becomes activated? (FAI I p346) (FAQ p377)

Other Bleeding Disorders


H: Chapters 115, 116 Mixed platelet and coagulation disorders (FAII p356) (FAI2 p388)

Antiplatelet Medications
GG: Chapter 30 H: Chapter 118 Aspirin (FAII p363) (FAI2 p397) (R p59) Clopidogrel, ticlopidine (FAII p363) (FAI2 p397) (GGp869) Abciximab (FAII p364) (FAI2 p397) (GGp870)

NSAIDs inhibit the production of which substance important in platelet aggregation?

After a normal spontaneous vaginal delivery, the new mom bleeds profusely from her vagina and later from her gums. What abnormal lab values would you suspect? What is the mechanism of action of the following drugs? (FAI I p362-364) (FAQ p395-397) Streptokinase Aspirin Clopidogrel Abciximab Tirofiban Ticlopidine Enoxaparin Eptifibatide What lab ndings are indicative of disseminated intravascular c" ' j: <DIQ? (FAI I p356) (FAQ p388) (R p673)
Copyright"? 2012. Doctors In Tr.iining.com. LLC. All Rights Reserved.

F / i i^i A [ J i V/ f if 11B * J > 1


What neoplasms are associated with AIDS? (FAI I pl7v What is the most common inherited bleeding disorder? (FAI I p356) (FAQ p388) (R p670) What cancer is associated with Hashimoto thyroiditis? (FAI I p298) (FAQ p325) (R p6l3)

Lymphoma
R: Chapter 13 H: Chapter 109 Leukemia vs lymphoma (FAII p357) (FAI2 p389) (R p598) Leukemoid reaction (FAII p357) (FAI2 p389) (R p595) Lymphoma - Hodgkin vs. non-Hodgkin lymphoma (FAII p357) (FA12 p389) - Reed-Stemberg cells (FAII p357) (FAI2 p389) - Hodgkin lymphoma (FAII p3S7) (FAI2 p389) - Non-Hodgkin lymphoma (FAII p358) (FAI2 p390)

4. What hematologic disease matches the following statement?


Most common lymphoma in US Diffuse large B-cell lymphoma Reed-Stemberg cells Hodgkin lymphoma

Particularly associated with EBV Burkitt and Hodgkin Lymphomas A s s o c i a t e d w i t h l o n g t e r m c e l i a c d i s e a s e I n t e s t i n a l T- c e l l l y m p h o m a Lymphoma equivalent of CLL Small lymphocytic lymphoma "Starry-sky pattern" due to phagocytosis of apoptotic tumor cells Burkitt lymphoma Associated with Sjogren syndrome, Hashimoto thyroiditis, Marginal cell MALToma and H. pylori

Compare the age distribution of those affected by Hodgkin lymphoma to those affected non-Hodgkin lymphoma. (FAI I p357) (FAQ p389)

What is the most common type of non-Hodgkin lymphoma in adults? In children? (FAII p358) (FAQp389)

What clinical presentation might lead you to suspect a patient may have lymphoma? (FAII p357) (FAQp389, 390)

Copyright () 2012, Doctors In Training.com. LLC All Rights Reserved.

Leukemias
R: Chapter 13 H: Chapters 109, 110 Leukemias (FAII p3S9-360) (FAI2 p392) (R p600) Auer bodies (FAII p360) (FAI2 p393) (R p623) Chromosomal translocations (FAII p360) (FAI2 p93)

8. Acute Leukemia (R p600)


Rapid onset and rapidly progressive Over 5096 myeloblasts (AML) or lymphoblasts (ALL) in the bone marrow Numerous blast (immature) cells (> 2096 blasts) Often associated with pancytopenia (anemia, bleeding tendency, infection)

ALL Philadelphia chromosome may be seen (poor prognosis) Most common in children and young adults Males > females, whites > blacks B cell types more common that T cell 3 morphologic variants and 5 phenotypic variants Bone pain is common Most have the enzyme terminal deoxynucleotide transferase (TdT) Very good prognosis in children (up to 9096 remission) PAS(+) Difficult to diagnose on blood smear (others can be diagnosed with smear)

AML Philadelphia chromosome rarely seen Characteristic Auer rods 8 different morphological classications Most all are CD 13/33 (+) Usually nonspecific esterase (+) myeloid cells Median age of onset is 50 PAS(-) Associated with numerous risk factors

9. 8 types of AML
MO undifferentiated Ml minimal differentiation/maturation M2 with differentiation/maturation M3 acute promyelocytic M4 myelomonocytic /myelomonoblastic M5 pure monocytic /monoblastic M6 erythroleukemia M7 megakaryocyte

m rn
ON

n
D O C D

10. AML risk factors


Radiation, benzene, or alkylating agents (such as in Hodgkin lymphoma treatment) Myeloproliferative disease, myelodysplastic syndrome, or aplastic anemia Down syndrome, Fanconi syndrome, or Bloom syndrome

5 2

o
Copyright 2012. Doctors In Training.com. LLC. All Rights Reserved.

Cl

[13]

11. ALL type by morphology


LI-small blasts L2- large blasts with prominent nucleoli

ALL types by phenotype


common (= LI)

null (= LI) T(=L2) B(=L3)

L3- large blasts with cytoplasmic vacuoles

12. Chronic Leukemia (R p603)


Insidious onset and gradual progression (months to years) Mature cells (rather than blasts) (< 596 blasts) Can be either myeloid (CML) or lymphoid (CLL) Associated with hepatosplenomegaly and lymphadenopathy Prominent infiltration of bone marrow by leukemic cells and peripheral WBC counts may be high
/5%

CLL Most common adult leukemia seen in western countries Males > females, whites > blacks Adults over age 50 9596 have B cell markers (rather than T cell) 1096 progress to ALL Characteristic smudge cells and Autoimmune hemolytic anemia Tends to be indolent

CML May progress to AML (80%) or ALL (2096) Numerous basophils and PMNs are LAP (-) Adults ages 25-60 Hyperplasia of all 3 cell lines (granulocytic, erythroid, and megakaryocytic) but granulocyte precursors predominate Philadelphia chromosome (t 9;22) is always present Fatigue, abdominal pain, splenomegaly, bleeding tendency
/""tv\

Chronic Myeloproliferative Disorders


H: Chapters 109, 110 R: Chapter 13

o _ o

go
CO L .

Chronic myeloproliferative disorders (FAII p36l) (FAI2 p394) Polycythemia vera (R p609) CML (FAII p359) (FAI2 p393) (H p9l4) (R p627)

< D U

Multiple Myeloma
H: Chapter III R: Chapter 13 Multiple myeloma (FAII p3S8) (FAI2 p39l) (R p609) (H p937) MGUS(FAIIp3S9)(Rp6ll)

c rd

vO LU

[14]

Copyright 2012. Doctors In Training.com. LLC All Rights Reserved.

ess/on Quiz
13. A patient with anemia, hypercalcemia, and bone pain receives a bone marrow biopsy what may plasma cells (large, round, off-center nucleus). What is the diagnosis, and twhich revealsbe found on urinalysis?

What form of leukemia matches the following statement? Most common leukemia in children Most common leukemia in adults in US AML associated with Down syndrome AML that are CDI3 and CD33 (+) Characteristic Auer rods Myelodysplastic syndromes have a tendency to progress to Myeloproliferative disorders may progress to Greater than 2096 blasts in marrow Leukemia with more mature cells and less than 596 blasts AML that is CD 41 and CD 61 (+) PAS (+) acute leukemia Commonly presents with bone pain Leukemia equivalent of Burkett lymphoma Numerous basophils, splenomegaly, and negative for leukocyte alkaline phosphatase (LAP) Most common neonatal leukemia Always positive for the Philadelphia chromosome (t 9;22) Only AML that is CD 13 and CD 33 (-) Acute leukemia positive for peroxidase Solid sheets of lymphoblasts in marrow PAS (-) acute leukemia Always associated with the BCR-ABL genes

Copyright 2012. Doctors In Training.com. LLC All Rights Reserved.

JJMWMiIW*
Which protozoa are responsible for the following diseases? (FAI I pl6l-l62) (FAQ pl75-l77) (R p335) Chagas disease Protozoal vaginitis Malaria Birth defects Bloody diarrhea Foul-smelling diarrhea, flatulence, bloating

After bone marrow transplantation, a patient suffers from dermatitis, enteritis, and hepa What disease process is occurring? (FAI I p2l5) (FAQ p239) (R p228) Which regions of the brain are induded in the limbic system? (FAI I p399) (FAQ p437) (Phys p7l4)

Genetics of Cancer
R: Chapter 7 Cancer epidemiology (FAII p230) (FA12 p255) (R p270) Review of cell cycle (FAIIp76) (FAI2 p79) - Roles of cyclins, CDKs, and CDK inhibitors (R p285, Table 7-29) - Checkpoints (R p286) Basic molecular basis of cancer (R p276) Tumor suppressor genes (FAII p228) (FA/2 p253) (R p286) Oncogenes (FAII p227) (FAI2 p253) (R p279)

4. Molecular Basis of Cancer (R p276)


Malignant transformation (carcinogenesis) is fundamentally due to nonlethal genetic damage. Malignant tumors are derived from clonal expansion of a single precursor cell. The four main targets of genetic damage are: - Proto-oncogenes - Tumor suppressor genes - Genes that regulate apoptosis - DNA repair genes Malignant transformation is a multistep process, resulting from multiple mutations. The individual cells in a single tumor have varying degrees of malignant potential.

5. Retinoblastoma (R p288)
'A of cases are bilateral (both eyes), and all bilateral cases are inherited point mutations. The other Va of cases are unilateral (one eye); most of these are sporadic mutations. Overall, sporadic mutations account for about 6096' of the mutations in the Rb gene, and these are always unilateral. In other words, 4096 of retinoblastomas are inherited (which results in either bilateral or unilateral retinoblastoma). In order for the mutation to occur, there must be mutations to both alleles (AKA "two-hits"). - In the heritable form, one of those hits comes from the parent gene, and the other hit arises sporadically. - In the somatic/sporadic form, both hits arise sporadically.
Copyright 9 2012. Doctors In Training.com. LLC. All Rights Reserved.

6. p53(Rp290) Acts through p2l to cause cell-cycle arrest Involved at the Gl/S checkpoint and G2/M checkpoint Causes apoptosis by inducing the transcription of pro-apoptotic genes (such as BAX) Mutations in this gene allow the cell to progress through the checkpoint despite the presence of DNA damage/mutations 7. Proto-oncogenes Normal cellular genes that regulate cell proliferation and differentiation that can become oncogenes

8. Oncogenes Genes that promote autonomous cell growth in cancer cells by promoting cell growth in the absence of normal mitotic signals Oncoproteins produced from these genes are devoid of important regulatory elements 9. RAS oncogene (R p282) Mutations in RAS is the most common oncogene abnormality in human tumors 15-20% of all human tumors contain mutated versions of RAS proteins K-RAS mutation colon, lung, and pancreatic tumors (Kolon, panKreatic) H-RAS mutation bladder and kidney tumors (Hematuria) N-RAS mutation melanomas, hematologic malignancies

What type of cancer is associated with the following tumor suppressor genes? (FAI I p228) (FAQ p253) Rb DPC p53 APC WTI BRCAI and BRCA2 What cancers are associated with a mutation of the K-RAS oncogene? (R p282)

Among men and among women, compare the most common cancers and the most common cancers causing mortality. (FAI I p230) (FAQ p256)

Copyright <t) 2012. Doctors In Trainmgcom. LLC All Rights Reserved.

Cancer Risk Factors


R: Chapter 7 Geographic and environmental (R p272) Chemical carcinogens (FAII p229) (FAI2 p2S5) (Rp274, Table 7-3)(Rp309-3ll, table 7-10) Radiation carcinogenesis (R p3l I) Age (R p273) Genetic predisposition (R p273) (R p27S, Table 7-4) DNA repair defects (R p302) Diseases associated with neoplasms (R p276) (FAII p227) (FAI2 p252) Oncogenic microbes (FAII p229) (FAI2 p254) (R p3l2-3l6)

13. Chemical carcinogens (R p272-274)


Arsenic lung cancer, skin cancers, hemangiosarcoma Benzene: leukemia, Hodgkin lymphoma Beryllium: lung cancer (possibly) Cadmium: prostate and lung cancer (possibly) Chromium compounds: lung cancer Nickel sulfide: lung cancer, upper airway cancers Radon: lung cancer

Radiation carcinogenesis (R p311-312.) 14. What are the most common cancers associated with ionizing radiation?

15. Which cancers are associated with UV radiation exposure? Which type of UV radiation is most problematic? (R p3l2)

16. Inherited Predisposition to Cancer (R p273-276)


Gene Predisposition Retinoblastoma, osteosarcoma FAP/colon cancer Li-Fraumeni syndrome Breast cancer, ovarian cancer MEN Ha and lib, papillary thyroid cancer

Rb APC P53
BRCAI. BRCA2

o
z n
-<

RET

17. What diseases are associated with DNA-repair defects? (R p302)

o r~ o a
i
CO
U </>

18. Oncogenic Bacteria and Helminths


Strep, bows K pylori' Schistosoma haematobium
Oonorchis sinensis"*

n"

Copyright 2012. Doctors In Training.com. LLC. All Rights Reserved.

[3]

What neoplasms are associated with the following conditions? (FAI I p227) (FAQ p252) Hashimoto thyroiditis Down syndrome Plummer-Vinson syndrome Tuberous sclerosis Ataxia-telangiectasia Paget disease of bone

Which neoplasm is associated with the following? (FAI I p229) (FAQ p255) Nitrosamines Asbestos Naphthalene Arsenic EBV HPV Schistosoma haematobium

Copyright 2012. Doctors In Traimng.com, LLC. All Rights Reserved.

ueszion warmWhich nucleus of the hypothalamus ts the following description? (FAI I p399) (FAQ p437) Considered the "master clock" for most of our circadian rhythms Regulates the parasympathetic NS Regulates the sympathetic NS Produces antidiuretic hormone (ADH) to regulate water balance Mediates oxytocin production Regulates the release of gonadotropic hormones (i.e., LH and FSH) What organism is transmitted by the following vector? Freshwater snail Ixodes tick Reduviid bug Anopheles mosquito Sandfly Tsetse fly What drugs should not be given to sulfa allergic patients? (FAI I p246) (FAQ p246)

Neoplastic Progression
R: Chapter 7 Overview of neoplastic progression (FAII p225) (FAI2 p2S0) (R p278) -plasia denitions (FAII p226) (FAI2 p25l) (R p262) Tumor grade vs. stage (FAII p226) (FAI2 p2SI) (R p322) Tumor nomenclature (FAII p226) (FAI2 p25l) (R p260) (R p263, Table 7-1) Tumor differences (FAII p226) (FAI2 p25l) (R p262)

'iilfW:

What type of tumor matches the following description? (FAI I p226) (FAQ p25l)
Benign tumor of epithelium Malignant tumor of blood vessels Benign tumor of bone Malignant tumor of smooth muscle

What unique enzyme is normally absent in somatic cells, but is active in stem cells and cancer cells? (R p297) What are the proangiogenic cytokines? (R p298)

Copyright 2012. Doctors In Training.com. LLC. All Rights Reserved.

Downstream Effects of Neoplasia


R: Chapter 7 Mets to brain (FAII p230) (FAI2 p2S6) Mets to liver (FAII p230) (FAI2 p256) Mets to bone (FAII p230) (FAI2 p256) Cachexia (FAI2 p252) (R p320) Paraneoplastic effects of tumors (FAII p229) (FAI2 p255) (R p32l) Serum tumor markers (FAII p228) (FAI2 p254) (R p326, table 7-12) Psammoma bodies (FAII p230) (FAI2 p255)

(FAII p228) (FAQ p254) Hepatocellular carcinoma (Hep B and C patients) Ovarian cancer Pancreatic cancer Melanoma Colon cancer Astrocytoma What is the most common cause of hypercalcemia? What cancers may cause hypercalcemia? (FAI I p229) (FAQ p255)

What neoplasm is most commonly responsible for the following paraneoplastic syndrome? (FAI I p229) (FAQ p255) ACTH * Cushing syndrome Erythropoietin * polycythemia ADH SIADH Which cancers metastasize to bone? (FAI I p230) (FAQ p256)

Which cancers metastasize to the brain? (FAI I p230) (FAQ p256)

Which cancers metastasize to the liver? (FAI I p230) (FAQ p256)

Copyright @ 2012, Doctors In Trainmg.com. LLC. All Rights Reserved.

Which cranial nerve is responsible for the following actions? (FAI I p416) (FAQ p456) (COA p1054) Eyelid opening Taste from anterior 2/3 of tongue Head turning Tongue movement Muscles of mastication Balance Monitoring carotid body and sinus chemo- and baroreceptors

What CD surface marker ts the following statement? (FAI I p209) (FAQ p23l) (R pi901 Displayed only by helper T cells Displayed only by cytotoxic T cells (and suppressor T cells) Found on all T cells (except NK cells) Used to ID B cells Found on all NK cells and binds the constant region of IgG Inhibits complement C9 binding Endotoxin receptor found on macrophages

3. What surface molecule on platelets binds brinogen? What surface molecule binds von Willebrand factor? (FAI I p346) (FAQ p376-377) (R p 670)

Cancer Prevention
H: Chapter 82
Risk factor reduction (primary prevention) (H p655) Cancer chemoprevention (H p656) Screening and early detection (secondary prevention) (H p659)

4. What are the four most important lifestyle factors that impact cancer risk? (H p655)

5. What are the screening recommendations for the following cancers? (H p66l)
Cancer Breast cancer Cervical cancer Prostate cancer Colon cancer US Preventive Services Task Force (USPSTF) Recommended Screening

Copyright 2012. Doctors In Training.com. LLC All Rights Reserved.

.jTTf?:cm
What is currently the m< cancers? Lung cancer
! Cervical cancer Renal cell carcinoma Breast cancer Colon cancer

KidSJLfJffLfJlMMIiSigl^^

What is the only denitive way to prevent ovarian cancer?

Which male patients should be screened for prostate cancer with a serum PSA? (H p66l)

Host Defense Against Cancer


R: Chapter 7 Tumor antigens (R p3l6) Antitumor effector mechanisms (R p3l8) Immune surveillance and escape (R p3l9)

9. What types of antigens can the immune system use to distinguish tumor cells from healthy cells? (Rp316-318)
Products of mutated genes (ex: RAS oncogene, BCR-ABL proteins) Over-expressed normal cellular proteins (ex: tyrosinase products in melanomas) Tumor antigens produced by oncogenic viruses (ex: HPV, EBV) Oncofetal antigens (ex: OfP, CEA) Altered surface glycoproteins and glycolipids (ex: CA 125, CA 19-9) Cell type-specific differentiation antigens (ex: CD20)

10. What immune cells are involved in killing cancer cells? (R p318) CD8+ cytotoxic T lymphocytes
Natural killer (NK) cells Macrophages

11. What mechanisms to cancer cells use to evade the immune system? (R p319-320)
Selective outgrowth of antigen-negative variants Reduced expression of MHC molecules Lack of co-stimulation Immunosuppression (TGF-|3) Antigen masking (mucopolysaccharides, glycocalyx) Apoptosis of the cytotoxic T cells (FasL)
Copyright 2012. Doctors In Training.com. LLC. All Rights Reserved.

12 What is the primary immune system cell type involved in host tumor immune surveillaa (Rp3l6)

What enzyme present in melanoma generates peptides that are targets for host T cells? (RP3I7)

How can tumor cells evade attack from cytotoxic T cells? (R p319)

How might tumor cells induce apoptosis of T cells that would antagonize them? (R p320)

Copyright 2012. Doctors In Training.com. LLC. All Rights Reserved.

What are the symptoms of inhibiting parasympameTic a (FAI I p239) (FAQ p265) (GG p225,242)

Which cancers are assodated with the following tumor markers? (FAI I p228) (FAQ p254) (R p327, Table 7-12) PSA . CEA AFP . p-hCG CA 125 . SIOO Elevated alk phos What medication is used to treat the following parasitic infection? (FAI I p 160-163) (FAQ PI75-I79) (GG PI8I8) Gardia, Entamoeba, Trichomonas Plasmodium vivax or ovale Most ukes and tapeworms

Cancer Drugs I
GG: Chapter 60 Cancer drugs - cell cycle (FAII p364) (FAI2 p398) Antineoplastics (FAII p364) (FAI2 p398) Cancer drugs (FAII p365-366) (FAI2 p399) - Antimetabolites (FAII p365) (FAI2 p399) - Antitumor antibiotics (FAII p 366) (FAI2 p400)

How does the mechanism of action of methotrexate differ from 5-uorouracil? (FAII p365) (FAQp399)

low does the body metabolize 6-mercaptopurine? (FAI I p365) (FAQ p399)

Which anticancer drug is also often used in rheumatoid diseases as well as ectopic pregnancies? (FAI I p365) (FAQ p399)

For which cancers is dactinomycin particularly useful? (FAI I p366) (FAQ p400)

Copyright 2012. Doctors In Training.com. LLC. All Rights Reserved.

Cancer Drugs 2 (FAII p365-366) (FAI2 p399-40l)


GG: Chapter 60 Alkylating agents (FAII p366) (FAI2 p400) Microtubule inhibitors (FAII p367) (FAI2 p40l) Other cancer drugs (FAII p367-368) (FAI2 p40l-402) Common chemotoxicities (FAI2 p402)

fjW-iMii
Which anticancer drug ts the following description? (FAI I p365-368) (FAQ p399-40l) Forms a complex between topoisomerase II and DNA Alkylates DNA, toxicity * pulmonary fibrosis Fragments DNA, toxicity * pulmonary fibrosis Blocks purine synthesis, metabolized by xanthine oxidase Cross-links DNA, nephrotoxic, ototoxic Nitrogen mustard, alkylates DNA (electrophile that binds DNA) - Folic acid analog that inhibits dihydrofolate reductase Prevents tubulin disassembly Intercalates DNA, produces oxygen free radicals, cardiotoxic DNA alkylating agents used in brain cancer Prevents tubulin assembly Inhibits thymidylate synthase decreased nucleotide synthesis SERMblocks estrogen binding to ER(+) cells Mechanism similar to antivirals acyclovir and foscarnet Mechanism similar to fluoroquinolones Mechanism similar to trimethoprim Monodonal antibody against HER-2 (erb-B2) Free radical-induced DNA strand breakage Inhibitor of PRPP synthetase Reversible with leucovorin Treatment for choriocarcinoma Treatment for AML - Treatment for CML Prevents breast cancer Treatment for testicular cancer Applied topically for AKs and basal cell cancers Treatment for childhood tumors (Ewing sarcoma, Wilms tumor, rhabdomyosarcoma) - Inhibits ribonucleotide reductase SE of hemorrhagic cystitis Antibody against Philadelphia chromosome What are the potential side effects of prednisone use? (FAI I p367) (FAQ p40l)

Copyright '3 2012. Doctors In Training.com. LLC. All Rights Reserved.

raTfffliwwwn
What G protein dasses do the following receptors stimulate? (FAI I p236) (FAQ p263)

Which anticancer drug ts the following description? (FAI I p365-367) (FAQ p399-400) Prevents breast cancer Treatment for testicular cancer Treatment for childhood tumors (Ewing sarcoma, Wilms tumor, rhabdomyosarcoma) Inhibits ribonucleotide reductase SE of hemorrhagic cystitis Antibody against Philadelphia chromosome What is the rate-limiting step in purine synthesis? In pyrimidine synthesis? (FAII P68)(FAI2p69)

Studies
H:

and
Chapter

Diagnostic
3

Te s t s
p52) T p52)

(
( j ~~j

Types of studies (FAII pSO) (FAI2 Clinical trial (FAII p50) (FAI2 Drug development (GG p7, II) Meta-analysis (FAII pSO) (FAI2 p58) Evaluation of diagnostic tests (FAII p5l)

(FAI2

p53)

4. What is the sensitivity, specicity, positive and negative predictive value using antibodies <. to X to detect disease X? p Autoimmune
Present

Disease
Absent

j
l

> TJ

a a.
I

800

200

Copyright^ 2012. Doctors In Training.com. LLC. All Rights Reserved.

lSBS^!m!9KSSS!tlS9MSfSs!s
predictive value? (FAI I p5l) (FAQ p53)

6. A physician is looking for risk factors for pancreatitis. He interviews 100 hospitalized of study is this? A group of people that smoke and that do not smoke is followed over twenty years. Every two years, it is determined who develops cancer and who does not. What type of study is this? A certain screening test has a 1% false-negative rate. What is the sensitivity of the test7 I patients with pancreatitis and 100 hospitalized patients without pancreatitis. What type

Application of Test Data


H: Chapter 3 Prevalence vs. Incidence (FAII p52) (FAI2 p54) Odds ratio vs. relative risk (FAII p52) (FAI2 p54) 9. Attributable Risk (AR) (FAI I p52) AR incidence of disease in the exposed group - incidence of disease in the unexposed group Example: In a population of sexually-active people, 30% have HPV infection. In a population of people who are not sexually active, only 5% have HPV infection. The attributable risk of sexual activitytoHPVis25%.

10. Number Needed to Treat (NNT)


NNT = l/absolute risk reduction Number of patients you would need to treat in order to save/effect one life Important number to help determine if a drug should be used or is cost effective Example: If out of 10,000 patients that took t-PA during a STEMI, 100 were saved by the t-PA, then the NNT is 100. In other words, you would need to treat 100 patients in order to save/effect I life. 11. What is the absolute risk reduction and number needed to treat in the following example: In a study where 100 patients received medication Z to prevent the development of diabetes and 200 patients did not receive the medication, 10 patients in the experimental group developed diabetes and 40 patients in the control group developed diabetes?

Copyright 2012. Doctors In Training.com, LLC. All Rights Reserved.

Session Quiz
to treat? (FAI I p52) (FAQ p54)

The small town of Mickey City (pop. 8,000) is immediately adjacent to factories whet asbestos products are produced. During the past year, the prevalence of mesothelio. has been 16 cases. In the town of Donaldville (pop. 6,000) 15 miles upwind of (and theoretically safely distant from) Mickey City, there was a prevalence of 3 cases during same year. What is the relative risk of mesothelioma for the population of Donaldville

A new glucose test arrives and you decide to see how well it works. There is a stand substance provided that has 90 mg/dL of glucose. Your repeated measurements of th substance reveal the following values: 54, 56,55, 54, 53, 56, 55, and 54. What can you say about the precision and accuracy of your new glucose test?

The prevalence of varicella in population A is 2 times the prevalence of varicella in population B. The incidence is the same in populations A and B. What can be assum about the disease duration in population A versus population B?

Copyright () 2012. Doctors In Tmining.com. LLC. All Rights Reserved.

Which hormones share a common alpha subunit?

What is the difference between a case-control study, a cohort study, and a clinical trial? Which studies use odds ratios, and which use relative risks? (FAI I p50) (FAQ p52)

Which neoplasm is most commonly responsible for the hormone paraneoplastic syndrome (FAII p229) (FAQp255)
ACTH Cushing syndrome PTH-related peptide hypercalcemia Erythropoietin polycythemia ADH SIADH

Bias and Error


Bias (FAII pS3) (FA12 p56) Statistical distribution (FAII p53) (FAI2 p57) Statistical hypotheses (FAII p54) (FAI2 p57) Error types (FAII p54) (FAI2 p57) Power (FAII p54) (FAI2 p58)

Examples of Bias:
Selection bias - The referral centers for a trial of a new anticancer drug have more patients with end stage disease than early stage, so more patients with end stage disease are referred for the trial than early stage disease. erkson's bias - Studies performed on patients that have been hospitalized... type of selection bias ecall bias - Parents of autism patients having a more detailed recall of events and illnesses in their child's rst two years of life compared to parents of healthy controls. Sampling bias - A study performed in China may not be generalizable to the US population. Late-look bias - Sending a survey out to people diagnosed with a fatal illness 5 years after diagnosis will preferentially sample those with a low grade disease (or few comorbidities) Procedure bias - The positive benefit of a new drug during a study simply may have been due to the fact that study participants were required to attend clinic monthly, where they received extra disease education and counseling compared with the controls. Confounding bias - Are asbestos miners more likely to have cancer because they mine asbestos or because they are more likely to smoke? Lead-time bias - While test PSA-xyz may detect prostate cancer before it is detected by a traditional PSA, early detection using PSA-xyz does not increase cancer survival compared to traditional PSA. Pygmalion effect - An orthopedic surgeon investigator who finds statistically significant benefit of arthroscopic surgery when compared to non-invasive therapeutic strategies. A chiropractor-led study that nds signicant benet of the effects of cervical manipulation when compared to traditional medicine strategies. Hawthorn effect - When studying the effects that infection control education has on physicians, the investigator notes that both the experimental and the control groups improve their hand hygiene.
Copyright 2012. Doctors In Training.com, LLC. All Rights Reserved.

show no benet. What type of error is found in study X? (FAI I p54) (FAQ p57) . Study Y shows that aspirin administration during an Ml offers no improvement in patie morbidity or mortality. What type of error is present? (FAI I p54) (FAQ p57) '. What is the difference between the mean, median, and mode? (FAI I p53) (FAQ p57)

(FAII p53) (FAQ p57) In a randomized, placebo-controlled study of a new blood pressure medication, member of the study group received their supply of pills at a monthly clinic visit at which the blooc pressure was checked, and at which the clinic nurses were available to answer questions and to instruct the patients on statistical distribution has a positive skew? I What does it mean to say that a the importance of controlling blood pressure through diet exercise, and weight loss. Members of the control group received their supply of placebc pills in the mail each month. After 6 months, the study group's mean systolic blood pressure had decreased 11 points from baseline, compared to a 2-point decrease in the control group. What type of bias may have contributed to the difference in outcomes between the two groups?

Condence Interval
Standard deviation vs. standard error of the mean (FAII p54) (FAI2 p57) Condence interval (FAII p55) (FAI2 pS8) t-test vs. ANOVA vs. X2 (FAII pSS) (FAI2 p58) Correlation coefcient (FAII p55) (FAI2 p58)

10. Condence Interval


Dr M: "My study shows that drug X reduces the risk of prostate cancer by 10%." Student P: "Does drug X actually reduce the risk of prostate cancer by 10% or was it a flawed study?" Dr M: "There is honestly no way that I can know if my study of 1,000 patients reflects the reality of the entire population. Also, there may be some error somewhere in my study." Student P: "So, do you have any confidence at all that what you found in your study reflects reality?" Dr M: "I am 95% confident that drug X truly does reduce the risk of prostate cancer by somewhere in the interval of 8-12%." Student P: "Where did you come-up with 8-12%?" Dr M: "Using the standard deviation of the data from the study and the sample size of the study, I determined what the standard error of the mean is. I then took the standard error of the mean from my study and multiplied it by 1.96. That number turned out to be 2, so I can be 95% condent that drug X truly does reduce the risk of prostate cancer by 10% +/- 2." Student P: "Where did you come up with 1.96?" Dr M: "When you want a confidence interval of 95%, you always use 1.96." Student P: "What if you wanted a confidence interval of 90% or 99%?" Dr M: "For 90% I would use 1.645 x SEM, and for 99% I would use 2.57 SEM."
Copyright';) 2012. Doctors In Training.com. LLC. All Rights Reserved.

What is the equation for determining the condence interval?

12. In a study of USMLE scores at a particular medical school, the mean score is 230 and the standard deviation is 20. Knowing that the sample size is 100, calculate the 95% condence interval.

13. What is the 99% condence interval in this same study?

14. In a study of USMLE scores at a particular medical school, the mean score is 230 and the standard deviation is 20. Knowing that the sample size is 16, calculate the 95% condence interval.

In a normal Gaussian curve, what percentage of the sample population falls I standard deviation, 2 standard deviations, and 3 standard deviations? (FAI I p54) (FAQ p57)

tat is the equation for determining the condence interval?

In a study of diabetic patients on drug Meca-leca-liver, the average patient HgbAIC after 3 months is 8.0 and the standard deviation is 05. Knowing that the sample size is 10,000, calculate the 99% condence interval.

Copyright 2012. Doctors In Training.com, LLC. All Rights Reserved.

What are the clinical manifestations of Addison disease? What is the cause of Addison disease? (FAI I p296) (FAQ p324)

A heart failure patient is newly diagnosed with cancer and is being evaluated for chemotherapy. Which chemotherapeutic agent should be avoided in this patient?

Assuming a normal Gaussian distribution forthe results of a particular test, an average vali of 35, and SD of 4, what percentage of people will be in the interval between 31 and 43? (FAIIp53)(FAI2p57)

Disease Prevention: Pediatrics and Obstetrics


Disease prevention (FAII p55) (FAI2 p58) Prevention in childhood Immunizations Prevention in obstetrics

4. Car Seats
< I yr and < 20 lbs * Infant seat in back seat facing backwards I -4 yr and > 20 lbs * sits in back seat but still in car seat, now have option of facing forward Once forward-facing car seat is outgrown (4yrs and 40 lbs) booster seat in the back Keep in booster seat until the belt fits correctly (usually 4' 9" and 8-12 yr) then belted with a lap/ shoulder belt in the back seat until 13 years of age

What is the current

k'*M

illi'iW

I '

[TiTTiI=V4A

ik!=

meningitis vaccin.

When is it important for a patient with a tetanus prone wound to receive a tetanus vaccine?

7. Which adult patient populations should receive a pneumonia vaccine?

At what ag

@?'<ajmfe|s=!fl? ^iCTa}^ '"HsSSf'-^I^

Copyright 2012. Doctors In Trnining.com. LLC. All Rights Reserved.

Prevention of Adult Diseases


H: Chapter 4 Prevention in women Prevention in CAD Prevention in DM Prevention in patients over age 50 Reportable diseases Leading causes of death in the US by age

9. Important Preventive Measures


Diabetes Serum glucose levels, HbAIC, urine microalbumin, serum lipids, BP, foot exams, dilated eye exam, inuenza and pneumococcal vaccines HIV and syphilis screening, Hepatitis B vaccine, GC/Ch screen, Pap smear, HPV screen and vaccine, counsel on STDs and condom use Address quitting at each encounter, avoid vitamin A supplementation, avoid OCPs in women over age 35, abdominal ultrasound in males 65-75 to r/o AAA, inuenza and pneumococcal vaccines

High-risk sexual behavior

Smoking

What are the current indications on the herpes zoster vaccine?

At what age should patients begin receiving screening colonoscopies?

When should a woman begin to receive regular mammograms?

At minimum, what four medications should a patient with a prior Ml due to atherosclerosis be taking?

What is the target HgbAIC for every diabetic patient?

Copyright 2012. Doctors In Training.com, LLC. All Rights Reserved.

What are the common side effects of B-blockers? Which patient populations should use caution when taking B-blockers? (FAI I p242) (FAQ p269) A small study of USMLE test takers for a particular school revealed scores of 225,225,2, 229,230,240, and 250 with the average score being 232. What is the mean, median, ar mode for these values? Would this create a positively-skewed curve or a negatively-skew curve? (FAI I p53) (FAI2p57) !, A patient of yours develops hypercalcemia from the most common primary tumor arisin; within bones in adults. What lab ndings would you suspect in this patient? (FAIIp358)(FAI2p39l)

Physiological and Psychological Changes


H: Chapters 70-72 Changes in the elderly (FAII p6l) (FAI2 p63) Suffering Grief (FAI I p6l) (FAI2 p63) Kiibler-Ross grief stages (FAII p6l)

4. What are the Kiibler-Ross grief stages? Denial Anger Bargaining Grieving (depression) Acceptance Stages may occur in any order, and more than one stage may be present simultaneously. 5. At what point does grief/bereavement become pathological? Grief becomes pathological when any of the following are found: Depression criteria met for at least 2 weeks after the first 2 months following the loss Generalized feelings of hopelessness, helplessness, worthlessness, and guilt Suicidal ideation Distressing feelings do not diminish in intensity by 6 months Inability to move-on, trust others, and re-engage in life by 6 months

uick Quiz

What is the general rule in prescribing new medications to elderly patients? What are the Kubler-Ross grief stages? (FAI I p6l)

Copyright '.:> 2012. Doctors In Training.com. LLC. All Rights Reserved.

Diseases and Resources for the Elderly


H: Chapters 4, 70-72 Diseases and medications common in the elderly Prevention in patients over age 65 Medical equipment commonly needed by the elderly Resources to aid in caring for the elderly

What are the 3 leading causes of death in patients over age 65?

10. Which patients should be screened for an abdominal aortic aneurysm using an abdominal ultrasound?

I. At what age should every geriatric patient have had a DEXA bone density scan?

A 60-year-old businessman complains of a lack of successful sexual contacts with women and a lack of ability to reach full erection. One year ago, he had a heart attack. What might be the cause of his problem?

Copyright 2012, Doctors In Trainmg.com. LLC. All Rights Reserved.

I. What patient populations are particularly susceptible to mucormycosis? (FAI I pl58) (FAQ pi > Chromosomal analysis of a leukemia patient reveals the presence of the Philadelphia chromosome t(9;22). What is the treatment7 . Which tumor marker would you use to follow the following cancer? (FAI I p228) (FAQ p
Ovarian cancer
Hepatocellular cancer (Hep B and C patients)

Pancreatic cancer Melanoma


Colon cancer Astrocytoma

Healthcare System - Part I


Medicine is a science. Healthcare is a business. Healing is an art. What is the world without insurance?
Physicians and hospitals can charge whatever they want to patients. Patients say they will pay anything to get their health back, and often lack other options for obtaining health care, due to deciency of local options or urgency of time. Patients sometimes incur excessive unforeseen medical expenses. Patients often are unable to compensate healthcare providers for the services rendered. Healthcare providers are less willing to take risks on establishing new healthcare institutions.

Healthcare Terms You Need To Know - Table #1


Premium Co-pay Deductible The amount the insured person has to pay the insurance company (usually paid monthly). The amount the insured person pays at the time of service (e.g., $30 for a clinic visit or $15 for a particular drug) The amount an insured person must pay "out-of-pocket" before the health insurance begins to pay.

6. Health Insurance - Private Pay Insurance - Third-party payer


Party Patient Financial Duty - Monthly premiums - Co-pays - Deductible Risk Paying more to insurance than what is received in medical care Reward (1) Financial benet if the cost of medical care exceeds what is paid to insurance

Insurance

Health care expenses of patient beyond what the patient is required to pay
i

(2) Peace of mind allows the individual to focus on other things Paying more for a patient's Financial benet if the patient medical care than what is pays more than the cost of received nancially from medical care the patient

Copyright 2012. Doctors In Training.com. LLC. All Rights Reserved.

%> !$ /

7. What problems came about when health insurance was introduced?


In short, everyone tries to get the biggest slice of the finandal pie now in the hands of the hearth insurance companies. Health insurance companies leverage their volume of subscribers to drive down the reimbursement to medical care providers. Providers still provide care because they agreed to do so. Hearth care providers and hospitals counter by increasing the cost of medical care. Insurers pay because they agreed to do so. Patients demand more and more care in order to "get what they have already paid for." Insurance companies ended up receiving too little funds from patients to pay the rising costs of health care.

8. Healthcare Terms You Need To Know - Table #2


Pre-existing condition Lifetime maximum Network A condition that a patient is known to have that is not covered by health insurance. The maximum amount that an insurance company agrees to pay, as specied in the plan that is purchased. The group of healthcare providers that has agreed to a reduced payment in order to have access to a larger number of patients. High deductible plan that covers the patient in the event of excessive medical expenses

Major medical

9. Health Maintenance Organization (HMO)


PCP is the gatekeeper to more specialized care. In order for medical expenses to be covered, the provider has to be "in-network".

10. Preferred Provider Organization (PPO)


There is no gatekeeper to the specialist. Patient can see whomever they want; however; the cost is higher for "out-of-network" medical care.

11. What problems came about when HMOs and PPOs were introduced?
The HMO and PPO drive down the payments to physicians and hospitals. As physicians receive less compensation, the quality of visit is decreased in order to generate quantity of visits. The patient pays more and more for less and less medical care. In short, the third party payers drove down the cost of care. The physidans counter by decreasing the quality of care. The patient loses. Physicians that are unsatisfied with the new arrangement exit the system and establish new systems.

[12]

Copyright 2012. Doctors In Training.com, LLC. All Rights Reserved.

12. Newly-established systems established by physicians


Minor emergency clinics Cash-only clinic Heavily utilized by patients without any third-party coverage Usually offers no primary or secondary prevention Physician refuses to take any third-party payment, essentially returning back to the old patient-physician system Patient may or may not carry additional "major-medical only" Sliding scale clinic coverage A patient pays a nominal fee based on income. This is usually run by a charity with a volunteer physician in order to minimize costs. Physician charges an annual fee (e.g., $l200/yr) for a specically designated set of services (e.g., unlimited visits, or 4 visits a year) Physician offers unique services not covered by insurance but desired by afuent patients (usually cosmetic in nature) in order to increase clinic revenue (e.g., Botox injections) In an effort to exclude third-party systems and regain control of their own payments, physician groups join clinics together, build their own hospitals, and run their own surgery centers.

Concierge medicine

Boutique medicine

Group physician networks - Clinic systems - Physician hospitals - Day-surgery centers

Quick Quiz
13. What is the difference between a premium, co-pay, and deductible? 14. Which usually offers patients a greater variety of physicians to choose from: HMO or PPO? 15. Why might a 63-year-old patient with newly diagnosed multiple myeloma have a difcult time obtaining insurance? What hope might this patient have of medical care in the future?

m O
m

n o

Q
16. Healthcare Terms You Need to Know - Table #3
Formulary Utilization Management Resource-Based Relative Value Scale The medications for which insurance companies will pay. Generics are preferred. Evaluation of the appropriateness, necessity, and efciency of health care services. A patient pays a nominal fee based on income. This is usually run by a charity with a volunteer physician in order to minimize costs.

3
n m

17. Health Plan - Managed Care Organizations (MCO) - Health Care Organizations
Organization that attempts to maximize quality of care and minimize the cost of care. Uses techniques such as encouraging patients and physicians to choose less costly care, controlling inpatient admissions and lengths of stay, and emphasizing preventive medicine. Accomplished through a designated utilization management person or department.

Copyright 2012. Doctors In Training.com. LLC. All Rights Reserved.

[13]

18. What about those without the nancial resources for healthcare? Medicare Medicaid Children's Health Insurance Plan (CHIP) City-, county-, or state-funded health networks Federally-funded teaching hospitals Federally-funded VA hospitals Universal health care 19. How are physicians compensated? Fee-for-service Payment is provided for a specied service Fixed payment for a period of time or number of patients Specied amount, regardless of work performed Payment is increased if a physician meets preestablished targets Surgical procedure Clinic visit (e.g., 99211-99215) Inpatient visit (e.g., 99231 -99233) ER shift/Minor emergency shift Concierge practice HMO Universities Hospital administration Base salary Increased pay by the health care organization for meeting certain criteria such as preventive medicine targets (vaccines, colonoscopy, HgbAlc) A funding entity may agree NOT to pay a physician for complications such as leaving a catheter in place, nosocomial infections, and surgical site infections

Capitation

Salary

0&1

Pay for performance

Nonpayment for performance

No payment is made for a complication that could have been avoided

20. Current Problems


"Young Invincibles" These are people (usually under 30) that tend to not have many health problems and tend not to purchase insurance. In order for a third-party payer to afford to care for an ailing patient at a low premium cost to that patient, healthy patients must be present in the system to bear the burden of the additional expense. Excessive preexisting conditions. There are some patients that are unable to get any health insurance at a reasonable cost because of their preexisting conditions. The provider does not want to take on the high risk of providing care for the patient. Freedom of choice. Patients that need others to carry the burden of their healthcare expenses are at odds with those that want to be free to not have to carry that burden. The US government would like everyone to carry the burden via the "Individual Mandate" in the PPACA. Others argue that this violates their freedom. Excessive complexity. All this complexity leaves patients at a loss to understand where they need to go for healthcare. Additionally, this complexity requires hiring individuals to manage the complexity which drives up the cost of healthcare. Everyone still wants the biggest possible piece of the pie. Patients want the highest-costing care.

^^

LU

Physidans want the highest reimbursements. Insurance providers want the largest prots. Drug companies want the largest possible prots. Physician shortage. Many physicians now see better opportunities elsewhere and are leaving full-time medicine. Older physicians are tired of the complexity and will leave practice if reimbursements decrease. The population continues to increase and the percentage of geriatrics in the population is also increasing. Insufficient government funds. It will be very difficult for the government to pay for Medicaid and for Medicare for the baby boom generation.
Copyright 2012. Doctors In Training.com. LLC. All Rights Reserved.

/SIT,

[14]

21.Emerging Solutions
Government intervention to regulate the system. Patients taking responsibility for their own health via the internet, OTC medication options, and complementary and alternative medicine. Patients seeking US-trained physicians practicing in developing countries where the cost of care is low. New generic medications that help lower the cost of health care. Cooperative healthcare teams that help reduce the need for costly hospitalization. More mid-level practitioners to meet the rising demand A four-level healthcare system: Level Level 1 (Sufciently-Funded) Burden carried by: Public Capabilities Minimal screening and prevention Major medical provided, but not the highest quality Lowest cost health insurance plans Minimal screening and prevention Good quality medical care Moderate cost health insurance plans Good screening and prevention Good quality medical care Highest cost health insurance plans Private pay Highest quality medical care Examples Charity clinics ERs VA hospitals County hospitals Minor-emergency clinics Most hospitals in the US Most clinics that accept insurance Personal PCP Most hospitals in the US Any clinic that accepts insurance Exclusive private hospitals in large cities Executive physicals Concierge medicine Boutique medicine

Level 2 (Well-Funded)

Balance of physicians, patients, and business

Level 3 (Greatly-Funded)

Balance of physicians, patients, and business

Level 4 (Abundantly-Funded)

Benefactors, employers, and wealthy patients

What is the difference between being paid by capitation and fee-for-service?

What must a hospital submit to a third-party payer in order to receive payment for services?

lat department in a hospital oversees the maximization of the quality of care while minimizing the cost of care?

Copyright 2012. Doctors In Training.com. LLC. All Rights Reserved.

I. What type of thyroid cancer matches the following: Second most common type of thyroid cancer (10%) Activation of receptor tyrosine kinases Hashimoto thyroiditis is a risk factor Cancer arising from parafollicular C cells I* Most common type of thyroid cancer (70-75%)

tementi

2. Metastasis to the brain, liver; and bone commonly come from which locations? (FAII p230) (FAQp256) Brain Liver Bone

What changes in sleep patterns and sexual anatomy are seen in the elderly? (FAII p6l) (FAQ p63)

Ethical Principles
H: Chapter 9 Core ethical principles (FAII p57) (FAI2 p59) Informed consent (FAII p57) (FAI2 p59) Exceptions to informed consent (FAII p57) (FAI2 p59) Consent for minors (FAII p57) (FAI2 p59) Decision-making capacity (FAII p57) (FAI2 p60) Advance directives (FAII p57) (FA12 p60) Condentiality (FAII p58) (FAI2 p60) Exceptions to condentiality (FAII p58) (FAI2 p60) Malpractice (FAII p58)

A 68-year-old man is diagnosed with incurable glioblastoma multiforme. His family asks you, the doctor; not to tell the patient How do you handle the situation? A patient had made it clear to you on previous visits that if something should happen to him that he would not want surgery. The patient now comes to you with a condition that requires surgery, and because of his condition, he is incapable of providing for you his present feelings on the matter: His friend relays to you that the man has told him numerous times that he does not want surgery. His wife later shows up and tells you to do everything you can do to save her husband's life, induding surgery. What should you do? What should you do if a minor requests birth control during a clinic visit in the absence of her parents? (FAI I p59)(FAI2p6l) A 36-year-old female has a rst degree relative who had breast cancer at age 40. You recommend a mammogram, but she refuses to have one because she says it is too pain

Copyright 2012, Doctors In Training.com. LLC. All Rights Reserved.

Ethical Scenarios
Ethical situations (FAII p59) (FAI2 p6l)

8. You have to deliver bad news to a patient of yours who you have just discovered to have lung cancer The patient is a 52-year-old male in marketing that has smoked I pack a day the bad news the patient replies, "How can this be? I'm a healthy guy. I eat right, watch my weight, and exercise regularly." What is the most appropriate next response? I for You past yourself attracted to regularly, and maintains a healthyyou handle you situation the nd 25 years, exercises your 24-year-old patient. How do diet. After the disclose

,. A 72-year-old man in the hospital with an Ml refuses to take his aspirin on the ground that it makes him feel "funny". What do you do?

II. How should you handle a situation where you smell alcohol on a physician's breath that physician is practicing medicine?

Copyright 2012, Doctors In Training.com. LLC. All Rights Reserved.

Question
Rectum External anus T estes Scrotum What are the leading causes of death in ages 1-14 and in ages 25-64? (FAI I p56)

. Which anticancer drug ts the following description? (FAI I p2l 6) (FAQ p399) Fragments DNA, toxicity pulmonary fibrosis Blocks purine synthesis, metabolized by xanthine oxidase Folic acid analog that inhibits dihydrofolate reductase Prevents tubulin disassembly DNA alkylating agents used in brain cancer SERMsblock estrogen binding to ER(+) c

Diaphragm and Respiration


Phys: Chapter 37 Diaphragm structures (FAII pS03) (FAI2 p560) (COA p308) Embryology of the diaphragm Diaphragmatic hernia (FAII p3l6) (FAI2 p345) (COA p3l7) Controls of the diaphragm (Phys p505-SI3) Muscles of respiration (FAII p504) (FAI2 p560)

lick Quiz
What structures traverse the diaphragm, and at what vertebral levels do they pass throuj

Which molecule has a greater direct effect on respiration at the brainstem, oxygen or hydrogen ion? (Phys 506)

By what mechanism does oxygen effect respiration? (Phys p509)

What is the difference between a hiatal hernia and an indsional hernia after an abdominal surgery? (FAI I p3l6) (FAQ p345)

Copyright @ 2012. Doctors In Training.com. LLC. All Rights Reserved.

Respiratory Tree and Alveoli


COA: Chapter I Phys: Chapter 39 Respiratory tree (FAII p502) (FAI2 p558) Bronchopulmonary segments (FAII p503) (COA pi 14) Lung relations (FA 11 p503) (FAI2 p560) Pneumocytes (FAII p502) (FAI2 p559) Gas exchange barrier (FAII p502) (FAI2 p559) (Phys p49l)

E H
What histological change takes place in the trachea of a smoker? A patient in the ER is having anaphylaxis. You make an incision beneath thyroid cartilage to establish airway. What structure was cut?

I What cell type proliferates during lung damage?

What amniotic uid measurement is indicative of fetal lung maturity?

A young woman has infertility, recurrent URIs, and dextrocardia. Which of her proteins is defective?

Copyright 2012. Doctors In Training.com. LLC. All Rights Reserved.

estfon Wi
(FAII p204) (FAQp226) 2. Compare the leading causes of death in ages 15-24 to those in ages 25-64? (FAI I p56)

Which protozoal organism matches the following statement? (FAII Pl60-l6l)(FAI2pl75-l76)


Diarrhea in campers and hikers Itchy vaginitis Sandfly is the vector Ixodes tick is the vector Anopheles mosquito is the vector Sodium stibogluconate is the treatment Suramin or melarsoprol is the treatment Maltese cross seen in RBCs Treat with metronidazole Severe diarrhea in AIDS patients

Copyright 2012, Doctors In Training.com. LLC. All Rights Reserved.

Lung Volumes and Hemoglobin


Phys: Chapters 37, 40 Important lung products (FAII p504) (FA/2 p56l) Lung volumes (FAII pS04) (FAI2 p56l) (Phys p469) Determination of physiologic dead space (FAII p505) (FAI2 pS62) Lung and chest wall (FAII pS05) (FAI2 p562) Hemoglobin (FAII p505) (FAI2 p562) Hemoglobin modications (FAII p506) (FAI2 p563) Oxygen-hemoglobin dissociation curve (FAII pS06) (FAI2 p563) (Phys p498)

4. Label the following diagram of lung volumes.

5
V)

c o

3 + c

u
c
rcj C~

>/\aa/

1 i'

-*

m.

V1
Time i

_ o

GO

o E < u

5. What substances are known for causing methemoglobinemia?

E I
(N
<

o
D o _ [4]
Copyright 2012. Doctors In Training.com, LLC. All Rights Reserved.

lick Quiz
6. Label the following diagram of lung volumes

The following lung volumes are obtained from an elderly smoker FRC 5.0L, IRV I.5L, 2.0L, VC 3.5L What is his total lung capacity?

If a lung collapses, what happens to the intrathoracic volume? (FAI I p505) (FAQ p562

What is the treatment for methemoglobinemia? (FAI I p506) (FAQ p563)

Pulmonary Circulation and Pulmonary Hypertension


COA: Chapter I H: Chapter 250 Pulmonary circulation (FAII p507) (FAI2 p564) (COA pi 16) Pulmonary vascular resistance (PVR) (FAII p508) (FAI2 pS64) Pulmonary hypertension (FAII p507) (FAI2 p564) (H p2076) Treatment for pulmonary hypertension (H p2078) - Bosentan (FAII p5l6) (FAI2 pS77) (GG pi 10) (H p2079)

10. Primary Pulmonary Hypertension (FAI I p507) (FAQ p564)


AKA Idiopathic Pulmonary Arterial Hypertension Associated with abnormalities in BMPR2: Bone Morphogenetic Protein Receptor type II Associated with HIV and Kaposi sarcoma (HHV-8) More common in women, average age is 36
Copyright 2012. Doctors In Training.com. LLC. All Rights Reserved.

ension? (FAI I p507) (FAQ p564)

What are some the secondary causes of pulmonary hypertension? (FAI I p507) (FAQ p564)

What are some of the treatment options available for pulmonary hypertension? | (FAII p5l6)(FAI2p577)

is the mechanism of action of bosentan? (FAI I p5l6) (FAQ p577)

Copyright 2012, Doctors In Training.com, LLC. All Rights Reserved.

uestion Warm-Up
I. What is the characteristic sequence of the promoter region? What does a mutation i the sequence cause? (FAI I p72) (FAQ p75)

2. What are the pathological characteristics of the arteries in pulmonary hypertension? (FAII p507) (FAQp564)

3. What is the rate-limiting enzyme for the following metabolic pathway? Urea cycle Hexose monophosphate pathway Fatty acid synthesis p-oxidation of fatty acids
Ketone body synthesis Cholesterol synthesis Bile acid synthesis Heme synthesi

Oxygenation
Phys: Chapters 39,40 Oxygen content of blood (FAII p508) (FAI2 p565) Alveolar gas equation (FAII p508) (FAI2 p565) Oxygen deprivation (FAII p508) (FAI2 p565)

Copyright 2012. Doctors In Training.com, LLC. All Rights Reserved.

4. What controls the amount of oxygen the body receives?


Factor Inspired air Air quality Upper airway Bronchial tree diameter Effector 02 and C02 concentrations Smoke, NOx, ozone Obstructed or open Histamine, inammation Parasympathetics Lung diffusion barrier Surface tension Respiratory rate and depth Pulmonary circulation Number of RBCs Concentration of hemoglobin in RBCs Type of hemoglobin Ability of 02 to dissociate from Hgb Local vasodilation Sympathetics Thickness Surfactant Diaphragm, phrenic nerve Prostaglandins Blood ow Erythropoeitin Nutrients (iron, BI2, folate) Prostaglandins Transcription, translation CO2 , acidosis, temperature 2,3-DPG Prostaglandin, adenosine, lactic acid, serotonin, bradykinin, histamine Thromboxane Source of effector Mankind, nature, altitude Society, industry, cigarettes Environment Immune system Vagus nerve, brainstem Adrenals Immune system Type II pneumocytes Carotid body, aortic body Stress

Kidney, bone marrow Gl tract RBCs in bone marrow Local O2 demand and usage

Local occlusion

Platelets

What is the normal value for the A-a gradient? What might an elevated A-a gradient indicate? (FAI I p508) (FAQ p565)

A 42-year-old woman with broids is chronically tired. What is the most likely diagnosis, and what changes have occurred in the oxygen content and saturation?

'. Patient is shown to have hypoxia and CXR reveals an enlarged heart What is the most likely cause of hypoxia?

Copyright (>:> 2012. Doctors In Training com. LLC. All Rights Reserved.

Ventilation
Phys: Chapter 40 VIQ mismatch (FAII p509) (FA12 p566) COl transport (FAII p509) (FAI2 p566) Response to exercise (FAII p509) (FAI2 p567)

WBS&aSM
obstruction? During blood ow obstruction?

How is CO2 transported from the tissues to the lungs? (FAI I p509) (FAQ p566)

How do CO2 levels in circulation change during exercise? (FAI I p509) (FAQ p566)

Copyright 2012. Doctors In Training.com. LLC. All Rights Reserved.

In what disorder is there an abnormal breakdown of elastin? (FAI I p80) (FAQ p84)

What is the name of the genetic syndrome that ts the following description? Alcoholics with BI deficiency and neurologic defects Abnormal collagen type I synthesis Heinz bodies Musty/mousy odor, albinism, MR, eczema Hyperextensible skin, loose joints, bleeding tendency Hypoglycemia, jaundice, cirrhosis Bloating, cramps, osmotic diarrhea Dark brown urine, organs, and connective tissue: benign disease Multiple fractures and blue sclerae 3. What questions are asked during the 4 clinical phases of drug development? (FAII p50) (FAQ p52) Phase IPhase IIPhase IIIPhaselV-

Altitude, Aviation, and Space


Phys: Chapter 43 High altitude - Response to high altitude (FAII p509) (FAI2 p567) - Effects of low oxygen pressure on the body (Phys p527) Aviation - Effects of acceleratory forces on the body in aviation and space physiology (Phys p53l) Space - Articial climate in the sealed spacecraft (Phys p533) - Weightlessness in space (Phys p533)

Copyright 2012. Doctors In Training.com, LLC. All Rights Reserved.

How does the body compensate for hypoxia at high altitude? (

now much does the H&H change in a person that has acclimatized to a hypoxic environment for weeks? (Phys p529)

At what positive G force does visual "black-out" occur? Why does this occur? (Phys p531'

How is the body affected by a prolonged stay in space at zero gravity? (Phys p533)

Deep Sea and Hyperbarics


Phys: Chapter 44 Effect of high partial pressures of individual gases on the body (Phys p535) Scuba diving (Phys p539) Special physiologic problems in submarines (Phys p540) Hyperbaric oxygen therapy (Phys p540)

Tf

* y = + > 11

What physiologically is taking place in decompression sickness? (Phys p538)

What are the symptoms of decompression sickness? (Phys p538)

m For what conditions is hyperbaric oxygen therapy particularly helpful? (Phys p540)

Copyright 2012. Doctors In Training.com. LLC. All Rights Reserved.

Which immunosuppressant matches the following statement? Precursor of 6-mercaptopurine May prevent nephrotoxicity with mannitol diuresis Antibody that binds to CD3 on T cells Antibody that binds IL-2 receptor on activated T cells Inhibits inosine monophosphate dehydrogenase Inhibits calcineurin loss of IL-2 production blockage of T cell differentiation and activation Metabolized by xanthine oxidase, therefore allopurinol increases its toxicity What is the treatment for homocystinuria? What is the treatment of choice for the following protozoa? (FAI I pl6l) (FAQ pl75-l76) Trichomonas vaginalis Trypanosoma cnm Plasmodium vwax Leishmania donovani Entamoeba histolytica M f W, :

PE, DVT, and COPD


^ H: Chapters 260, 262 R: Chapter 15 Embolus types (FAII p5IO) (FAI2 p567) (R pl25-l27) lr, Deep venous thrombosis (FAII pSIO) (FAI2 p568) P E -^ Obstructive lung disease (COPD) (FAII p5IO) (FAI2 p 568) (H p2l5l) Obstructive vs. restrictive lung disease (FAII p5l2) (FAI2 p568-569)

q- 4. Virchow's triad (FAI I p5IO) (FAQ p568)

Q_ Stasis: post-op, long trips, cast, pregnancy Hypercoagulability: sickle cell, polycythemia, CHF, estrogen excess, smoking ' Endothelial damage: fracture, post-op, postpartum LU 0_ 5. DVT (FAI I p5IO) (FAQ p568) i Sx: swollen foot/ankle (unilateral), pain, Homan's sign (pain with ankle dorsiflexion), palpable cord '-*) Prevent: sq heparin bid-tid, SCDs/compression stockings, long-term warfarin Rx: heparin until warfarin therapeutic 6. Pulmonary Embolism (PE) . Sx: pleuritic CP, SOB, cough, hemoptysis (rare), fever, tachypnea, tachycardia, AMS/confusion Studies: elevated d-dimer, DVT on LE US, usually nl CXR, large Aa gradient on ABG, EKG changes (SIQ3T3 - wide S in lead I, large Q and inverted T in lead III), CT scan, V/Q scan, gold I standard = pulm angiogram Q_ "Saddle embolus" = death Rx: if massive PE then consider thrombolysis, but usually just heparin/warfarin

11 2J

Copyright 6) 2012. Doctors In Training.com. LLC All Rights Reserved.

ick Quiz DBSRUn


. patient suffers a stroke after incurring multiple long bone fractures in a skiing accident What caused the infarct?

A patient with a recent tibia fracture and no history of COPD or asthma is shown to haw hypoxia. CXR is normal. What is the cause of the hypoxia, and what disease process does mimic?

How does the emphysei sindeden

king differ from the emphysema caused by al.

Asthma
H: Chapter 254 R: Chapter 15 GG: Chapter 36 Asthma medication (FAII p5l5) (FAI2 p576) Expectorants (FAII p5l6) (FAI2 pS77) (GGplOS7) HI blockers (FAII p5l4) (FAI2 pS75) (GG p9l8)

II. What is the differential diagnosis for eosinophilia?

mi

12. Antihistamines
Drug Cyproheptadine Promethazine Chlorpheniramine Hydroxyzine Meclizine Use Appetite stimulant Nausea, vomiting OTC allergy/cold Sedation, itching Vertigo

Copyright2012. Doctors In Training.com. LLC. All Rights Reserved.

responsible for anaphylaxis in this patient? What asthma medication ts the following statement (FAI I p5l5) (FAQ p576) Inhaled treatment of choice for chronic asthma Inhaled treatment of choice for acute exacerbations Narrow therapeutic index, drug of last resort Blocks conversion of arachidonic acid to leukotriene Inhibits mast cell release of mediators, used for prophylaxis only Inhaled treatment that blocks muscarinic receptors Inhaled long-acting 02 agonist Blocks leukotriene receptors

A patient has an extended expiratory phase. What is the disease process?

Copyright 2012. Doctors In Training.com. LLC. All Rights Reserved.

estion Wa
What are the two HIV envelope proteins and the drugs that interfere with them? (FAII pi73) (FAQ pi92) A patient has a genetic disease in which the treatment includes protein restriction to prevent mental retardation, ketoacidosis, and death. What is the diagnosis? 3. What pathology ts the following high-yield statement? (FAI I p5l8-520) (FAQ p580-582)
Opacities seen on x-ray on both sides of the carina Dermatitis, diarrhea, dementia, possibly death Greenish rings around the periphery of the iris " Elastic skin, joint hypermobility 1 Enlarged, hard, left supraclavicular lymph node

Restrictive Lung Diseases and ARDS


H: Chapters 261, 268 R: Chapter 15 Restrictive lung disease (FAII p5ll) (FAI2 p569) Acute respiratory distress syndrome (ARDS) (FAII p5ll) (FA12 p570) Neonatal respiratory distress syndrome (FAII pSIl) (FAI2 p569)

4. ARDS (FAI I p5ll) (FAI2 p570)


Shock, infection, toxic gas inhalation, aspiration, high [O2], pancreatitis, heroin OD i Inammatory cells / mediators and oxygen free radicals I Damage to endothelial or alveolar epithelial (type I) cells ; Diffuse Alveolar Damage (DAD) and Hyaline Membrane Disease (HMD)

. A preterm infant has difculty breathing. An x-ray reveals diffuse ground glass appearani with air bronchograms. What is the diagnosis, and what could have prevented this condition?

> patient develops AFIDS from an occupational inhalation of nitrogen dioxide. What histologic change is seen in a patient recovering from ARDS?

7. What are common causes of ARDS?

Copyright 2012. Doctors In Training.com. LLC. All Rights Reserved.

Pneumoconiosis and Sarcoidosis


H: Chapter 261 R: Chapter 15 Pneumoconiosis (FAII p5ll) (FAI2 p569) Sarcoidosis (FAII p386) (FAI2 p42l) Brief review: Wegener granulomatosis, bleomycin, busulfan, amiodarone Sleep apnea (FAII p5l2) (FAI2 p570) Lung-physical ndings (FAII p5l2) (FAI2 p57l)

8. Coal Workers Pneumoconiosis (CWP)


Anthracosis (mild) - black pigment in lung Simple CWP - small fibrotic lung nodules Complicated CWP - progressive massive fibrosis

9. Sarcoid
Granulomas

RA Uveitis (eye) Erythema nodosum (tibial) Lymphadenopathy (hilar, bilateral) Idiopathic NotTB Gamma globulinemia (ACE increase)

10. H&E of lung biopsy from a plumber shows elongated structures with clubbed ends in tissue. What is the diagnosis and what is he at increased risk for?

I. What are the manifestations of sarcoidosis? (FAI I p386) (FAQ p42l)

What do patients with silicosis need to be worried about7 (FAI I p5l I) (FAQ p569)

CXR shows pleural effusions. What are the clinical ndings?

A tall, thin male teenager has abrupt onset dyspnea and left-sided chest pain. Percussion on the affected side reveals hyperresonance, and breath sounds are diminished. What is the diagnosis?

Copyright 2012. Doctors In Training.com. LLC. All Rights Reserved.

m-Up
(FAQ p346)

What is the triad of Kartagener syndrome? What is the underlying defect?

3. What pathology ts the following high-yield phrase? Gout + mental retardation + lip-biting Lack of Gpllb/llla defect in platelets * prolonged bleeding Anti-histone antibodies Psammoma bodies (FAI I p230) (FA12 p255) Lytic bone lesions on x-ray

Lung Cancer
H: Chapter 89 R: Chapter 15 Lung cancer (FAI I p5l3) (FAI2 p57l) Pancoast tumor (FAII p5l3) (FAI2 p572)

A patient of yours develops bronchogenic lung cancer but has never smoked. He is a coal miner Exposure to what substance has put him at risk for developing lung cancer?

What complications can arise from lung cancer?

What are the 3 most common locations of lung cancer mets?

Copyright 2012. Doctors In Training.com. LLC All Rights Reserved

Lung Infections
R: Chapter IS H: Chapters 257, 258 Pneumonia (FAII pSI4) (FAI2 pS73) Common causes of pneumonia (FAII pi 76) (FAI2 pi 96) Lung abscess (FAII p5l4) (FAI2 pS73) Pleural effusions (FAII pSI4) (FAI2 p574)

7. Which infectious agent ts the following description?


Common cause of pneumonia in immunocompromised patients Most common cause of atypical / walking pneumonia Common causative agent for pneumonia in alcoholics Can cause an interstitial pneumonia in bird handlers Often the cause of pneumonia in a patient with a history of exposure to bats and bat droppings Often the cause of pneumonia in a patient who has recently visited South California, New Mexico, or West Texas Pneumonia assodated with "currant jelly" sputum Q fever Associated with pneumonia acquired from air conditioners Most common cause of pneumonia in children I-year-old or younger Most common cause of pneumonia in the neonate (B-28d)

to C Most common cause of pneumonia in children and young adults (including college students, military .Q recruits, and prison inmates)

tj

0*1 Common cause of pneumonia in patients with other health problems Most common cause of viral pneumonia "O Causes a wool-sorter's disease (a life-threatening pneumonia) rd Endogenous flora in 20% of adults V; Common bacterial cause of COPD exacerbation U Common pneumonia in ventilator patients and those with cystic fibrosis

u
I ^

kj Pontjac fever O0 J

8. Pleural Effusions - Causes ' Transudate CHF

Exudate infections, Cancer Uremia tissue TB

Pneumonia, Cirrhosis syndrome Connective

Z <

Nephrotic PE Ruid overload

disease

z o z _J
D
Q_

[18]

Copyright 2012. Doctors In Training.com. LLC. All Rights Reserved.

sion Quiz
9. Examination of a lung at autopsy reveals a peripheral lesion with caseous necrosis. What. the diagnosis?

10. A 30-year-old comatose man on ventilatory support in the ICU develops an infection and dies. Autopsy reveals a pus-lled cavity in his right lung. What is the likely etiology?

A 55-year-old man who is a smoker and heavy drinker presents with a new cough and u like symptoms. Gram stain shows no organisms; silver stain of sputum shows gram-negati rods. What is the diagnosis?

CXR shows collapse of middle lobe of right lung and mass in right bronchus; patient has history of recurrent pneumonias. What is the diagnosis?

What infectious agent is the cause of pneumonia based on the following lab test? Gram (+) cocci in clusters Gram (+) cocci in pairs Gram (-) rods in 80-year-old Gram (+) coed in neonate Gram (-) rods in neonate

Copyright 2012. Doctors In Training.com. LLC. All Rights Reserved.

( FA I I p Q 4 ) ( FA Q p l 3 l ) ( l What adult structures are derived from the 3rd, 4th, and 6th aortic arches? (FAII pQ7) (FAQ pl35) What cell wall inhibitor matches the following statement? (FAII pi85-187) (FAQ p205-207) (GG pl497) Next step in treatment of otitis media if resistant to amoxicillin Prophylaxis against bacterial endocarditis Increases the nephrotoxicity of aminoglycosides Sufficient for the treatment of syphilis Single dose treatment for gonorrhea

Overview and Ventricle Development


R: Chapter 12 H: Chapter 224 P: Chapters 23, 83 Heart embryology (FAII pl23) (FAI2 pl30) (R p538, Fig. I2-3)(H pl809) Truncus arteriosus (FAII pl23) (FAI2 pl30) Interventricular septum development (FAII pl23) (FAI2 pl30) (R p540. Fig. 12-4) (H pl923) 4. Outline the pathway by which the heart tube forms the atria of the four-chambered heart. Tube grows, elongates, and folds into an S-shape atrial chamber lays posteriorly in S, and ventricular chamber lays anteriorly in S atrial chamber grows and incorporates superior vena cava and pulmonary vein septum primum forms * septum secundum forms incompletely (leaving a hole called foramen ovale) and cell death in septum primum forms a hole called ostium secundum 5. What divides the right and left atria? Septum primum and septum secundum 6. How is blood shunted from the right atrium to the left atrium in an embryo? Through the foramen ovale (of the septum secundum) and ostium secundum (of the septum primum) 7. Outline the pathway by which the ventricles and their outow tracts are separated. Ventricular chamber lays anteriorly in the S-shaped heart tube muscular ventricular septum forms which begins to divide the ventricles Truncoconical swellings (ridges) of the truncus arteriosus meet, fuse, and zip (both superiorly and inferiorly) in a 180 degree turn to form the spiral septum (AKA aorticopulmonary septum) Inferior portion of spiral septum meets with muscular ventricular septum to divide the ventricles and form the aorta and pulmonary arteries 8. Describe how the ventricles are remodeled in order to form the atrioventricular valves, q~ Myocardium erodes * ventricles enlarge as a result > residual mesodermal tissue becomes fibrous 0^9 and forms chordae tendineae Formation of papillary muscles and AV valves
Copyright @ 2012. Doctors In Training.com. LLC. All Rights Reserved. L

9. Name 6 different truncoconical (spiral) septum defects. Fenestrae


Ventricular septal defect (VSD) Tetralogy of Fallot Persistent truncus arteriosus Transposition of the great vessels (RV aorta, LV" PA) Dextrocardia

Which embryologic structure of the heart gives rise to the following aduft structure? Ascending aorta and pulmonary trunk Coronary sinus SVC Smooth parts of the left and right ventricle Smooth part of the right atrium Trabeculated left and right atrium Trabeculated parts of the left and right ventricle What structure divides the truncus arteriosus into the aortic and pulmonary trunks? What is the cellular origin of this structure?

Atrial Development and Fetal Circulation


H: Chapter 224 >> P: Chapters 23, 83 CxO F

O Interatrial septum development (FAII pl24) (FAI2 pl3l) O Fetal circulation (FAII pi25) (FA 12 pi32) >^

_q Q. What are the 3 possible causes of an atrial septal defect? , 13. What structure grows to close the opening/canal between the atrial chamber and ventricular chamber into two smaller openings?
y

<C 14. What genetic abnormality is commonly associated with endocardial cushion defects?

essio
Which fetal vessel has the highest oxygenation? What causes the ductus arteriosus to dose? (FAI I p 125) (FAQ p 132) Q5)(FAI2pl32)

Copyright 2012. Doctors In Training.com. LLC. All Rights Reserved.

What germ layer gives rise to the following adult structures? (FAI I pi 19) (FAQ pi26) Retina Salivary glands Pancreas Muscles of the abdominal wall Thymus Spleen Aorticopulmonary septum Anterior pituitary Posterior pituitary Bones of the skull Cranial nerves What are the clinical uses for 1st, 2nd, 3rd, and 4th generation cephalosporins? (FA pi86) (FAQp 206) (GG pl497)

What are the classic symptoms of carcinoid syndrome? (FAI I p302) (FAQ p33l) (R p787) (H p3062)

Right-to-Left Shunts
R: Chapter 12 H: Chapter 236 Congenital heart disease (FAII p267) (FAI2 p 292) Eisenmenger syndrome (FAII p267) (FAI2 p 292) (R p540) Tetralogy of Fallot (FAII p267) (FAI2 p 292) (R p542) (H pl926) D-transposition of great vessels (FAII p268) (FAI2 p 293) (R p543) (H pl927)

What are the components of the tetralogy of Fallot? (FAI I p267) (FAQ p292)

Explain how the great vessels are attached in a transposition of the great vessels. (FAII p268) (FAQp293)

Copyright ..) 2012. Doctors In TrAining.com. LLC All Rights Reserved.

Left-to-Right Shunts and Other Anomalies


R: Chapter 12 H: Chapter 238 VSD (R p54l) ASD (R p54l) Patent ductus arteriosus (FAII p268) (FAI2 p 293) (R p54l) Coarctation of the aorta (FAII p268) (FA12 p 293) (R p544) Congenital cardiac defect associations (FAII p269) (FAI2 p 294)

7. Ebstein's anomaly
Associated with maternal lithium use Tricuspid leaflets are displaced into right ventricle, hypoplastic right ventricle, tricuspid regurg or stenosis 80% have a patent foramen ovale with a R to L shunt Dilated right atrium causing increased risk of SVT and WPW Physical exam: widely split S2, tricuspid regurgitation

A 45-year-old male presents with a BP of 160/90 on the right arm and 170/92 on the left arm. There are no palpable pulses in the feet/ankle. What problem does this patient most likely have?

Describe blood ow thru a PDA.

What heart defect is associated with the following disorder? (FA p269) (FAQ p 294) Chromosome 22q 11 deletions Down syndrome Congenital rubella Turner's syndrome Marian's syndrome What problems are offspring of diabetic mothers at higher risk for?

Copyright 2012. Doctors In Trnning.com. LLC. All Rights Reserved.

Question Warm-Up
I. What irreversible enzymes are involved in gluconeogenesis? (FA plOI) (FAQ pi07) (Phys p8l6)

What cranial nerves innervate the tongue in the following ways? (FAI I pi29) (FAQ pl37) (COA p94l) Taste in the anterior 2/3 Taste in posterior 1/3 (main innervation) Motor Sensation in the anterior 2/3 Sensation in the posterior 1/3 Which tumors arise centrally in the lung and are linked to smoking? Which tumors arise peripherally in the lung and are less linked to smoking (if at all)? (R p72l)

Cardiac Output Variables


Phys: Chapters 20, 21 Cardiac output (FAII p254) (FA/2 p28l) Cardiac output variables (FAII p255) (FAI2 p282) Cardiac drugs: sites of action (FAII p283) (FAI 2 p308) Preload and afterload (FAII p255) (FAI2 p282)

4. What is the pulse pressure in a patient with systolic BP of 150 and a MAP of 90?

5. What is the basic equation for cardiac output? What is the Fick prindple?

What medications affect a heart's contractility? (FAI I p255) (FAQ p282)

,at ionic changes affect a heart's contractility? (FAI I p255) (FAQ p282)

How can the myocardial oxygen demand be decreased in circumstances where the he; ischemic? (FA p255) (FAQ p282)

Copyright 2012. Doctors In Training.com. LLC. All Rights Reserved.

Charting Cardiac Output


Phys: Chapters 9, 20, 21 Starling curve (FAII p255) (FAI2 p28l) (Phys pi 10) Ejection fraction (FAII p255) (FAI2 p282) (Phys 106) Resistance, pressure, ow (FAII p256) (FA12 p282) (Phys 158) Cardiac and vascular function curves (FAII p256) (FAI2 p282) (Phys. pl68, 15-1)

What factors affect stroke volume?

What is the heart ejection fraction?

Copyright fc) 2012. Doctors In Training com, LLC. All Rights Reserved.

armI. What is the most common type of TE stula? (FAI I pl3l) (FAQ pi39) (R p765) What are the major regulatory enzymes of citric acid cycle? (FAI I p96) (FAQ pi02) (Phys p8l3)

What infections are caused by Chlamydia species? What is the treatment for most Chlamydia infections? (FAI I pl56) (FAQ p 169) (GG pl529)

Heart Failure Pathophysiology


Phys: Chapter 22 R: Chapter 14 H: Chapter 234 Normal pressures (FAII p266) (FAI2 p29l) CHF(FAII p274) (FAI2 p300) (R pS33) Renin-angiotensin-aldosterone system (FAII p462) (FAI2 p5IO) (Phys p260)

4. CHF Compensation
. l.V contractility! I 4, Cardiac output -fc? Sympathetic _w ^LV contractility \w^ ^ activation ^ f Renal Na' and H2O reabsorption 1* Systemic venous pressure, ^preload Peripheral edema Pulmonary venous g 4. rv output congestion

*1* Renin-angiotensin-_ aldosterone

5. What are the signs of right-sided heart failure? (FAI I p274) (FAQ p300)

What are the signs of left-sided heart failure? (Phys p259)

7. How does poor cardiac output result in an increase in aldosterone? (FAI I p274,462) (FAQ p300,5I0) (Phys 260)

Copyright 20I2. Doctors In Training.com. LLC. All Rights Reserved.

Heart Failure Medications


GG: Chapters 27, 28 H: Chapter 234 CHF medications - Chronic treatment (FAI2 p300) - Acute treatment - Nesiritide (FA 12 p309) (GG p696) Cardiac glycosides (FAII p283) (FAI2 p309) (Phys p259) (GG p838)

8. Cardiac Glycosides (Digoxin)

Ion Exchange

Free Ca'

, 1. t

What medications are used to treat chronic heart failure?

medications are used to treat acute heart failure?

What is the mechanism of action of the cardiac glycosides (digoxin, digftoxin)?

Copyright J.) 2012. Doctors In Training.com, LLC. All Rights Reserved.

Question Warm-U
Which Rickettsial species has properties unique from the other Rickettsial organisms. What are those unique properties? (FA p 154-155) (FAQ pi 67-168) (R p7l4)

What is the classic presentation of congenital pyloric stenosis? (FAII pl3l) (FAQpl39)(Rp766)

Explain why a deciency of the enzyme that is the rate-limiter for the HMP shunt can result in hemolytic anemia. (FA pi02) (FAQ pi07) (Phys 816-817, Fig. 67-8)

Capillaries and Edema


Phys: Chapters 16, 25 R: Chapter 11 Capillary uid exchange (FAII p266) (FAI2 p29l) (Phys pi84) (R pi 11-113)

4. Illustrate the Starling forces affecting capillaries. (Phys pi84)

How do the following circumstances impact the Starling forces of uid movement throu capillaries? (FAI I p266) (FAQ p291) Heart failure Liver failure Renal failure Infections and toxins Nephrotic syndrome Lymphatic blockage Bums Diuretic administration IV infusion of albumin or clotting factors Venous insufciency

Copyright; 3 2012. Doctors In Training.com. LLC. All Rights Reserved.

Capillaries and Shock


Phys: Chapter 24 R: Chapter 4 Shock (FAII p224) (FAI2 p249) (R pl29-l32) Central lines (H: 2218) Femoral region (FAII p3l4) (FApl2 p343) (COA p204, Fig 2.15)

6. How do SVR and CO change in the following types of shock? (FAI I p224) (FAQ p245)
SVR Hypovolemia Heart failure Sepsis/anaphylaxis Neurogenic CO Rx

7. Locations of central lines:


The femoral is the easiest site with the least risk; however, it cannot stay in place more than 5-7 days due to risk of infection. The subclavian (SC) is preferred by some because it is easy to find, can remain longer (3-4 weeks), and is not as uncomfortable to the patient; however, it has the highest risk of pneumothorax. Not such a great choice in those with COPD (barrel-chested) or lung tumors. The internal jugular (IJ) is a good choice because there are good landmarks, can remain a long time (3-4 weeks); however, there it can be a little more uncomfortable to the pt, and there is a risk of puncturing the carotid or causing pneumothorax. Do not place a Left IJ without discussing it with staff, due to a greater risk of perforating the left SC vein due the angle at which the left IJ and left SC meet. catheter preferred sites: right IJ > right SC > left SC > left IJ

8. How does the vascular resistance and stroke volume differ in hypovolemic shock compared to septic shock?
Vascular resistance Hypovolemic shock Septic shock

What are some of the causes of cardiogenic shoe OTCTiPi^l . How is the skin of a patient different in cardiogenic shock compared to septic shock? What other severe problems often coexist with septic shock in ICU patients? What serum lab marker is helpful in assessing the appropriate oxygenation of tissues?

Copyright 2012. Doctors In Training.com, LLC. All Rights Reserved.

What is the primary energy source in a patient that has not eaten in two days? (FAII pi 13) (FAQ pi 18) When placing a femoral venous catheter, while palpating the pulsatile femoral artery, where is the femoral catheter placed in relation to the artery? (FAI I p3l4) (FApQ p344) (COA p204, Fig 2.15) 1. Where in the cell would you nd the following enzymatic processes taking place? Fatty acid degradation Fatty acid synthesis Glycolysis TCA cycle I (FAII p95) (FAQ pi00) (oxidative phosphorylation) Electron transport chain Gluconeogenesis Protein synthesis (RER) Steroid synthesis (SER) KUrea cycle Heme synthesis

Atria and Ventricles


Phys: Chapters 9, 20 Cardiac Cycle (FAII p257) (FAI2 p284) (Phys pl08, Fig. 9.8) - Pressure volume loop (Phys pi06-109) - Pressure!time: aorta, LV, LA (Phys pi 10-111)

When does isovolumetric contraction take place? How does an increase in afterioad affect the stroke volume of the heart assuming contractility remains the same? (FAI I p257) (FAQ p284) What impact does an increase in contractility have on stroke volume assuming preloi and afterioad remain constant? (FAI I p257) (FAQ p284)

Heart Sounds and Jugular Waves


Phys: Chapter 9 Cardiac cycle (FAII p257) (FAI2 p284) - Heart sounds

- Splitting (FAII p258) (FAI2 p284)


- jugular pulse -ECG
Copyright (<' 2012. Doctors In Training.com. LLC. All Rights Reserved.

What protozoal/helminth matches the following statement? (FAII pi60-163) (FAI2pl75-l76) (Rp336) Contracted by eating undercooked fish and causes an inflammation of the biliary tract Most common protozoal infection in US Cause of Chagas disease Most common helminthic infection in the US Snail host, "swimmers itch" Diarrhea in campers and hikers Transmitted in raw meat or infected cat feces What effect does stress have on adipocytes? (Phys p929)

.at are the 4 obligate aerobic bacteria? (FAI I pl38) (FAQ pi49)

Systolic Murmurs
H: Chapter 227 Phys: Chapter 23 Auscultation of the heart (FAII p258) (FAI 2 p284) (Phys p26S, Fig. 23-2, 23-3) Heart murmurs (FAII p259) (FAI2 p285) (Phys p267) 4. What heart sounds are considered benign when there is no evidence of disease? Split SI Split S2 on inspiration S3 heart sound in a patient <40yo Early, quiet systolic murmur 5. What are the most common causes of aortic valvular stenosis?

Copyright 2012. Doctors In Training.com, LLC. All Rights Reserved.

Diastolic Murmurs
H: Chapter 227 Phys: Chapter 23

Crescendo-decrescendo systolic murmur best heard in the 2nd-3rd right interspace close to the sternum Early diastolic decrescendo murmur heard best along the left side of the sternum Late diastolic decrescendo murmur heard best along the left side of the sternum Pansystolic (AKA holosystolic or uniform) murmur best heard at the apex and often radiates to the left axilla Late systolic murmur usually preceded by a mid-systolic click Crescendo-decrescendo systolic murmur best heard in the 2nd-3rd left interspaces close to the sternum Pansystolic (AKA holosystolic or uniform) murmur best heard along the left lower sternal border and generally radiates to the right lower sternal border Rumbling late diastolic murmur with an opening snap Pansystolic (AKA holosystolic or uniform) murmur best heard at the 4th-6th left intercostal spaces Continuous machine-like murmur (in systole and diastole) High-pitched diastolic murmur associated with a widened pulse pressure

10. Know the classic descriptions of heart murmurs. What is the murmur heard best in left lateral decubitus position?

An 80-year-old man presents with a systolic crescendo-decrescendo murmur: What, the most likely cause?

Copyright (5)2012. Doctors In Training.com. LLC. All Rights Reserved.

armWhy do the kidneys retain uid in heart failure patients? (FAI I p464) (FAQ P5I2) (Phys p259)

Which hereditary hyperbilirubinemia matches the following statement? (FAI I p333) (FAQ p363) (R p84l-842) - Mildly decreased UDPGT Completely absent UDPGT - Grossly black liver Responds to phenobarbital - Treatment includes plasmapheresis and phototherapy Asymptomatic unless under physical stress (alcohol, infection) What is a disulram-like reaction? What drugs cause a disulram-like reaction? (FAI I p94) (FAQ plOO) (GG p643-644)

Action Potentials
Phys: Chapters 9, 10 H: Chapters 228, 258 GG: Chapter 29 Cardiac myocyte physiology (FAII p260) (FAI2 p288) (Phys pl02, Fig. 9-5) Ventricular action potential (FAII p260) (FAI2 p286) (Phys pi 15, 118) Pacemaker action potential (FAII p26l) (FAI2 p287) (Phys pll5-U9)

Copyright 2012. Doctors In Trainmg.com. LLC. All Rights Reserved.

Antiarrhythmics
GG: Chapter 29 Na+ channel blockers (FAII p284) (FAI2 p3ll) (GG p840) ^-blockers (FAII p285) (FAI2 p3ll) (GGp830) K1" channel blockers (FAII p285) (FA/2 p3ll) (GG p843) Ca2* channel blockers (FAII p285) (FAR p3l2) (GGp830) Other antiarrhythmics (FAII p286) (FAI2 p3l2) (GG p842)

What is the mechanism of action of each class of anti

Which antiarrhythmic has the side effect of cinchonism? (FAI I p284) (FAQ p3l I)

What are the potential side effects of amiodarone use? (FAI I p285) (FAQ p3l I)

To which dass of antiarrhythmics does each of the following agents belong? (FAII p284-286)(FAI2p309-3Q) Sotalol Propranolol Bretylium Quinidine Verapamil Procainamide Lidocaine Diltiazem

Copyright 2012. Doctors In Training.com, LLC. All Rights Reserved.

Identify the hepatitis B status of the following patients based on their hepatitis B serol markers. (FAII pl72) (FAQ pl9l) (Rp843) HepBsAg Negative Negative Positive Positive Negative HepBsAb Positive Negative Negative Negative Positive HepBcAb Positive Positive Positive IgM Positive IgG Negative Status

With what type of congenital heart defect would increasing afterioad be benecial? (FAI I p267-268) (FAQ p292-293)

What drugs are known for inhibiting cytochrome P450? (FAI I p245) (FAQ p273) (GG pQ9)

Atria to AV conduction
Phys: Chapter 10 Phys: Chapter 11 H: Chapter 232 Electrocardiogram (FAII p262) (FAI2 p288) Axis ECG tracings (FAII p263-264) (FAI2 p289-290) - Atrial brillation (H pi881) - Atrial utter (H pi885) Problems with the AV conduction (H pl888) - AV block

- Mobitz type I
- Mobitz type II - Third degree Wolff-Parkinson-White syndrome (FAII p263) (FAI2 p289)

Copyright 2012. Doctors In Traming.com. LLC. All Rights Reserved.

4. ECG Axis
Common conditions that result in axis deviation:
Left axis deviation
. Inferior wall myocardial infarction

-i5o aVR

aVL -30=

Left anterior fascicular block . Left ventricular hypertrophy (sometimes) . LBBB . High Diaphragm Right axis deviation . Right ventricular hypertrophy Acute right heart strain (ex. massive pulmonary embolism) . Left posterior fascicular block . RBBB . Dextrocardia

II +60 +90 aVF

'2o

The net electrical signal (cardiac axis) will fall within the shaded region in normal cardiac physiology.

Sinus Bradycardia

B5HMB
M 1 1 M B M B B
Normal P wave Normal QRS complexes

SPIBIMIWIM^I; BPM < 60

Paroxysmal Supraventricular Tachycardia

Copyright is) 2012. Doctors In Traimng.com. LLC. All Rights Reserved.

n
5. What are the two different types of second degree AV block? How do they differ? 6. How does the cause of a narrow QRS complex differ from the cause of a wide QRS complex? 7. What is the ECG axis given the QRS deections in the following scenarios? Positive in lead I, positive in lead II Positive in lead I, negative in lead III Negative in lead I, positive in lead III
- Positive in lead I, negative in aVR

.. Why is warfarin anticoagulation important in patients with chronic atrial brillation? (FAII p263) (FAQp289)

Ventricular Arrhythmias
Phys: Chapter 23 H: Chapter 23 Ventricular arrhythmias (H pi890) Torsades de pointes (FAII p262) (FAI2 p288) (H pl89l, Fig 233-9) Ventricular brillation (Phys pl49-l5l)

9. Drugs that prolong the QT interval Anti-infection: macrolides, chloroquine Antipsychotics: haloperidol, risperidone Anti-HIV: HIV protease inhibitors (-navirs) Anti-arrhythmics: class IA (quinidine) and class III (K1' channel blockers such as sotalol)
Ventricular Premature Beats ,

Monomorphic VentrkularTachycardia

Ventricular Fibrillation

Copyright 2012. Doctors In Training.com. LLC. All Rights Reserved.

What is the treatment for ventricular brillation? (FAI I p264) (FAQ p289)

What is the treatment for ventricular tachycardia when there is no pulse?

What is the hallmark of athird degree heart block? (FAI I p264) (FAQ p289)

What drugs are known to prolong the QT interval, increasing the likelihood of torsades in those at risk?

Copyright 2012. Doctors In Traming.com. LLC All Rights Reserved.

What is the difference between primary biliary cirrhosis and primary sclerosing cholangitis? (FAI I p334) (FAQ p365) (R p867, 869)

. ,ow does increasing the diameter of a vessel by two times affect the resistance of the vessel? (FAI I p256) (FAQ p282) (Phys pi64)

3. What substances are known teratogens? List as many as you can recall. (FAI I pQO) (FAQ pQ7) (R p450, Table 10-2)

Physiology of BP Regulation
Phys: Chapters 18, 19,20 Maintenance of mean arterial pressure (FAII p265) Baroreceptors and chemoreceptors (FAII p265) (FA/2 p290) Physiology of vasoconstriction - Smooth muscle contraction (FAII p378) (FAI2 p4l3) Review of organs involved in BP regulation (Phys p204, 213, 220)

4. Physiology of Vasoconstriction
Sildenafil * bradykinin *ACh *alpha-2-agonist histamine serotonin * shear stress (Viagra) cGMP phosphodiesterase myosin activated myosin phosphatase myosin-P04 actln > contraction activates Guanylyl Cyclase myosin lightchain kinase myosin calmodulin-Ca cAfV dtrulline and NO Endothelial cell * Nitrates L-arginine * Ca-channel * Epinephrine LVj blockers Prostaglandin E2 Smooth muscle cell

ased intracellular Icium L-arginine

Copyright 2012. Doctors In Training.com. LLC. All Rights Reserved.

5. Organs involved in BP regulation


Organ or Tissue Atria Aorta Carotid sinus Medulla Adrenal glands - zona glomerulosa Adrenal glands - zona fasciculata Adrenal glands - medullary chromafn cells Liver JGA of kidneys Lungs Kidney glomerulus Kidney tubules Local tissue effectors Mediator ANP Aortic baroreceptor Carotid body baroreceptor Control of sympathetic NS Aldosterone Cortisol Epinephrine and NE Angiotensinogen Renin (angiotensinogen angiotensin I) ACE (angiotensin I * angiotensin II) GFR impacted by multiple variables Impacted by angiotensin II, ADH, aldosterone NO, Prostaglandins, serotonin, histamine, bradykinin, adenosine, acidosis, lactate, O2, CO2, K1"

6. Outline the mechanism by which the kidneys regulate BP.

.. .at substances act on smooth muscle myosin light-chain kinase? How does this effect blood pressure?

Describe the chain of events in which hypotension causes a reex tachycardia.

Copyright 2012. Doctors In Training.com. LLC. All Rights Reserved.

Hypertension
H: Chapter 247 Phys: Chapter 19 Circulation through organs (FAII p265) (FAI2 p29l) Autoregulation (FAII p266) (FAI2 p29l) Hypertension (FAII p269) (FAI2 p294) Left-sided hypertensive heart disease (R p559) Aortic dissection (FAII p270) (FA/2 p295)

Know the current standards for HTN for Step I: http://www. nhlbi. nih.gov/guidelines/hypertension/phycard. pdf

Session
What BP values mark the diagnosis of hypertension? What values (FAII p269) (FAQ p294)

What would you most suspect the cause of hypertension to be in a patient with the foil clinical dues? Paroxysms of increased sympathetic tone: anxiety, palpitations, diaphoresis Age of onset between 20 and 50 Elevated serum creatinine and abnormal urinalysis Abdominal bruit BP in arms > legs Family history of HTN Tachycardia, heat intolerance, diarrhea Hyperkalemia Episodic sweating and tachycardia Abrupt onset in a patient younger than 20 or older than 50, and depressed serum K*" levels Central obesity, moon-shaped face, hirsutism Normal urinalysis and normal serum K4" levels Young individual with acute onset tachycardia Hypokalemia - Proteinuria

11. What chest x-ray nding is a possible sign for aortic dissection? (FAI I p270) (FAQ p295)

What category of BP medications is preferred in the treatment of aortic dissection?

Copyright 2012. Doctors In Training.com. LLC. All Rights Reserved.

Which group of medications inhibits the rate limiting enzyme of cholesterol synthesis? (FAI I pi 13) (FAQ plOI, 119) (Phys p827-829)

What are the primary mechanisms of action of the different classes of antiarrhythmics? Class I Class II Class III Class IV Name 8 different indirect cholinergic agonists and state the use for each. (FAII p238) (FAQ p265)

Antihypertensive Agents
GG: Chapters 25, 26, 27 ACE inhibitors (FAII p475) (FAI2 p527) CO Hydralazine (FAII p280) (FAI2 p306) <D Minoxidil (GG p78l) . ^ Calcium channel blockers (FAII p280) (FAI2 p306) Nitroglycerin, isosorbide dinitrate (FAII p280) (FAI2 p306) 4 . M i n o x i d i l ^ Mechanism of action: opens potassium channels and hyperpolarizes smooth muscle, resulting in >>, relaxation of vascular smooth muscle Therapeutic use: severe hypertension (topical application for hair loss) Toxicity: hypertrichosis, hypotension, reflex tachycardia, fluid retention/edema

' 5. Calcium Channel Blockers (CCB)


Dihydropyridine CCB: nifedipine, amlodipine, felodipine, nicardipine, nisoldipine - Mechanism of action: act on vascular smooth muscle to cause vasodilation - Therapeutic use: hypertension, angina, vasospasm (Prinzmetal's angina, Raynaud phenomenon) esophageal spasm, migraine prophylaxis I s - Toxicity: peripheral edema, ushing, dizziness, constipation, reex tachycardia ) Non-dihydropyridine CCB: verapamil, diltiazem - Mechanism of action: block calcium-channels at pacemaker cells (FAII p286) (FAI2 p3l2) . - Therapeutic use: hypertension, angina, arrhythmias - Toxicity: cardiac depression, AV-block, ushing, dizziness, constipation

Copyright 2012. Doctors In Training.com. LLC. All Rights Reserved.

[JTTrJ 6. Which antihypertensive class or drug ts the following side effect? First dose orthostatic hypotension Ototoxic (especially with aminoglycosides) | j Hypertrichosis Cyanide toxicity ! Dry mouth, sedation, severe rebound HTN j Bradycardia, impotence, asthma exacerbation j | j Reex tachycardia | | j

Cough Avoid in patients with sulfa allergy Possible angioedema

j Possible development of drug-induced lupus j Hypercalcemia, hypokalemia l While on an ACE inhibitor, a patient develops a cough. What is a good replaceme drug, and why doesn't it have the same side effects? (Phys p520)

Which calcium channel blockers can be used to treat both hypertension and tachyarrhythmia?

Copyright'.-) 2012. Doctors In Training.com. LLC. All Rights Reserved.

Choice of Antihypertensive
GG: Chapters 25,26, 27 Phys: Chapter 19 Antihypertensive therapy (FAII p280) (FAI2 p306) (GGp772-784) Malignant hypertension treatment (FAII p280) (FAI2 p306) (GG p73)

9. For each comorbid condition, what antihypertensives would be considered rst, and which antihypertensives should be avoided (either used with caution, or absolutely contraindicated)? (H p2054, Table 247-8)
Hypertension plus...

CHF

Initial Therapy Options Diuretics > ACE inhibitor/ARB P-blocker Aldo antagonist

Avoid P-blocker (in acute decompensated CHF or cardiogenic shock) CCB P-blocker

DM
Post-MI/CAD

ACE inhibitor/ARB Thiazide Thiazide ' P-blocker ACE inhibitor/ARB CCB (as needed for angina) Nitrates (as needed for angina)

C/)

Atrial brillation Bradycardia Renal insufciency

Diltiazem/verapamil (for rate control) Diltiazem/verapamil P-blocker ACE inhibitor/ARB (for proteinuria) ACE inhibitor/ARB (may increase creatinine) K*"-sparing diuretics ACE inhibitor/ARB Ct-blocker Propranolol Loop diuretic Thiazide Hydralazine Methyldopa Labetalol Dihydropyridine CCB CCB P-blocker Thiazide Thiazide ACE inhibitor/ARB

C O

C C D

(U Ql Renal artery stenosis

c
<

BPH
Hyperthyroidism Hyperparathyroidism Osteoporosis Gout Pregnancy

or: D U

CO

o
Q c
<

Migraines Essential tremor

Propranolol

[26]

Copyright 2012. Doctors In Training.com, LLC. All Rights Reserved.

Which antihypertensives are particularly benedal to heart failure patients? (FAII p280) (FAQp306)

What categories of antihypertensives are considered rst line in hypertensive patients wh diabetes mellitus? (FAI I p280) (FAI2 p306)

Which antihypertensives are safe to use in pregnancy?

A 25-year-old pregnant woman in her 3rd trimester has a normal BP when standing and sitting. When supine, her BP drops to 90/50. What is the diagnosis?

Copyright g! 2012. Doctors In Training.com. LLC. All Rights Reserved

regurgitation. (FAI I p259) (FAI2 p285)

Which 2 bacteria are well known for being obligate intracellular bacteria? Why can't these bacteria replicate extracellularly? (FAI I pl39) (FAQ pi50)

What marker of inammation produced by the liver and within atherosclerotic plaques i a strong predictor of Ml risk?

Atherosclerosis and Coronary Artery Disease (CAD)


R: Chapters II, 12 H: Chapters 243, 244, 245 Arteriosclerosis (FAII p269-270) (FAI2 p294) (R p496) Coronary arteries (FAII p254) (FAI2 p280) (R p547. Fig. 12-12) Ischemic heart disease (FAII p270) (FAI2 p295) (R p545) Causes of chest pain (R p546)

Coronary Arteries Anterior View

Posterior View

Lippincott Williams & Wilkins Atlas of Anatomy 2008 Wolters Kluwer Health. Inc. All rights reserved.

Copyright 2012. Doctors In Training.com. LLC. All Rights Reserved.

4. Pathogenesis of Atherosclerosis (R p498-504) Endothelial injury leads to increased vascular permeability, leukocyte adhesion and thrombosis Accumulation of lipoproteins: occurs in the vessel wall and is mostly LDL Monocyte adhesion to the endothelium: subsequent migration of the monocytes into the intima and then transformation of these cells into macrophages and foam cells. Platelet adhesion Factor release: activated platelets, macrophages and vascular wall cells; induces smooth muscle recruitment Smooth muscle cell proliferation and ECM (extracellular matrix) production Lipid accumulation: occurs extracellularly and within macrophages and smooth muscle cells

5. Complications of Atherosclerosis Based on Location of Plaque (R p504-507)


Smaller vessels can become occluded and then compromise distal tissue perfusion Ruptured plaque can embolize atherosclerotic debris and cause distal vessel obstruction or can lead to acute vascular thrombosis (stroke) Destruction of the underlying vessel wall can lead to aneurysm formation, with secondary rupture and thrombosis 6. Abdominal Aortic Aneurysm (R p507) Caused by atherosclerotic plaque compressing the underlying media Nutrient and waste diffusion is compromised Media degenerates and necroses, leading to arterial wall weakness More common in men over 50 and in smokers Presents as a pulsating mass in the abdomen s-\ Consequences include: rupture leading to fatal hemorrhage, embolism from atheroma, obstruction of a *^ branch vessel and impingement on an adjacent structure (ureter) -rj

D
7.

Deadly
Aortic

Causes

of Ml

Acute

Chest
aortic angina

Pain

Q
<C ^

dissection (or Unstable Te n s i o n

dissecting

aneurysm)

O
pneumothorax C_

70
What is the most likely cause of chest pain in the following scenarios? to ST segment elevation only during brief episodes of chest pain i Patient is able to point to localize the chest pain using one finger ^ Chest wall tenderness on palpation c* Rapid onset sharp chest pain that radiates to the scapula fD Rapid onset sharp pain in a 20-year-old and assodated with dyspnea O Occurs after heavy meals and improved by antacids $ Sharp pain lasting hours-days and is somewhat relieved by sitting forward (D Pain made worse by deep breathing and/or motion O Chest pain in a dermatomal distribution r^' Most common cause of non-cardiac chest pain
Copyright 2012. Doctors In Training.com. LLC. All Rights Reserved.

[29]

What are the most common locations for atherosclerosis, and what disorders result from plaques in these locations?

A patient with poorly-managed HTN has acute, sharp substernal pain that radiates to the back and progresses over a few hours. Death occurs in a few hours. Diagnosis?

During a high school football game, a young athlete collapses and dies immediately. What type of cardiac disease did he have?

Antianginal Therapy and Lipid-Lowering Agents


GG: Chapters 27, 31
Antianginal therapy (FAII p28l) (FAI2 p307) (GG p76l) Lipid-lowering agents (FAII p282) (FAI2 p308) (GG p884)

essio
Which lipid-lowering agent matches the following description? (FAI I p282) (FAQ p307-308) 1 SE: facial ushing SE: elevated LFT myositis s, SE: Gl discomfort, bad taste ' Best effect on HDL Best effect on triglycerides/VLDL Best effect on LDL/cholesterol Binds C diff. toxin 13. A 50-year-old man starts on lipid-lowering medication. Upon his rst dose, he develops a rash, pruritus, and diarrhea. What drug is he taking?

\. How can the ushing reaction of niadn be prevented?

Copyright 2012. Doctors In Training.com. LLC All Rights Reserved.

Histamine Porphyrin, heme NO GABA (a neurotransmitter) S-adenosyl-methionine (SAM) Creatine What is the basic equation for cardiac output? What is the Fick principle? (FAI I p254) (FAI2 p28l)

What is the mechanism of action of lactulose? (FAI I pi05) (FA12 pi 11)

Ml Pathophysiology
R: Chapter 12 H: Chapters 243-245 Evolution of an Ml (FAII p27l) (FAI2 p297) Serum diagnosis of an Ml (FAII p272) (FAI2 p298) Type of infarcts (FAII p272) (FAI2 p298)

4. Evolution of an Ml
Timeframe 0-4 hrs Gross Features Microscopic Features Risks and Other Findings

4-24 hrs

1-3 days

3-10 days

10 days-8 wks

> 2 months

Copyright (;) 2012. Doctors In Training.com. LLC. All Rights Reserved.

(FAQ p298)

Which coronary artery is most commonly occluded in an Ml? (FAI I p27l) (FAQ p297)

Describe the evolution of an Ml.


Timeframe | Gross Features | Microscopic Features | Risks and Other Findings

Copyright Gl 2012, Doctors In Training.com. LLC. All Rights Reserved.

Diagnosis and Treatment of Ml


H: Chapters 243-245 GG: Chapters 25, 26, 27 ECG diagnosis of Ml (FAII p272) (FAI2 p298) Ml complications (FAII p272) (FAI2 p298) Ml treatment

8. Evolution of Myocardial Infarction


i r Normal
-i 1 r~

Acute

Hours

Days later

Weeks later

4^.

P
ST elevates ST elevated R wave decreases Q wave appears T wave inverts Q wave deepens ST normal T wave inverted ST normal T wave normal Q wave persists

9. EKG Changes with Ml (Always obtain a previous EKG for comparison!)


ST segment elevation of at least I mm in 2 contiguous leads T wave inversion NewLBBB New Q waves (at least I block wide or 1/3 height of the total QRS complex)

10. Arteries - Walls - EKG Leads


Artery Left anterior descending Left circumex Right coronary Right coronary Wall Perfused Anterior wall Lateral wall Inferior wall* Posterior wall EKG Leads V1-V4, V5 aVL, V5, V6 II, III, aVF R precordial EKG: V4

n
>

g n c
I

* Always obtain a right-sided EKG (VI-V6 on right chest) in inferior wall Ml if ST segment elevation in V4, then posterior right ventricle also affected. This indicates a "right-sided Ml"" Fluids and avoid NTG.

i
5
[33]

Copyright 20I2. Doctors In Training.com. LLC. All Rights Reserved.

. What are the two most common complications after an Ml?

What would cause the following ndings after an Ml? (FAI I p272) (FAQ p298) Cardiac tamponade Severe mitral regurg NewVSD Stroke What is Dressier syndrome? (FAI I p272) (FAQ p298)

-. Which ECG leads will show evidence of ischemia in an anterior wall Ml?

". Which ECG leads will show evidence of ischemia in an inferior wall Ml?

16. What medications would you thinkto prescribe in an ER patient diagnosed with Ml?

Copyright 2012. Doctors In Training.com. LLC. All Rights Reserved.

What does a relative risk less than I indicate? (FAI I p54) (FAQ p57)

A screening test under investigation uses a lab marker SCSQ to identify early small ce and squamous cell lung cancer. In a population of 100 smokers over age 60, 20 test positive. Of the 20 that tested positive, 5 actually had either of the two lung cancers, those that did not test positive, it was determined that 5 had either of the two cancer What is the sensitivity, specicity, PPV and NPV of this study? (FAI I p53) (FAQ p57)

What are normal BPs in the right and left ventricles?

Cardiomyopathies and Endocarditis


R: Chapters 6, 12 H: Chapter 238 Cardiomyopathies (FAII p273) (FAI2 p299) Myocarditis (R p578) (H pi953-1961) Bacterial endocarditis (FAII p275) (FAI2 p30l) (R p566) Libman-Sacks endocarditis (FAII p275) (FA p30l) (R p220, Fig6-34) 4. Myocarditis (R p578) (H pi953) Generalized inflammation of the myocardium (not resulting from ischemia) Most common cause in US; H i s t o : d i ff u s e i n t e r s t i t i a l i n f i l t r a t e o f w i t h m y o c y t e

What are the typical signs and symptoms of endocarditis? Fever, chills, weakness, anorexia New regurgitation heart murmur or heart failure - Mitral valve is most common - Tricuspid is most common in IV drug users septic pulmonary infarcts Splinter hemorrhages in fingernails Osier's nodes (painful red nodules on finger and toe pads) Janeway lesions (erythematous macules on palms and soles) Roth spots (retinal hemorrhages with clear central areas) Signs of embolism: brain infarct * focal neuro defects, renal infarct * hematuria, splenic infarct abdominal or shoulder pain Systemic immune reaction: glomerulonephritis, arthritis

Copyright 2012. Doctors In Training.com. LLC. All Rights Reserved.

What are the differences between acute and subacute bacterial endocarditis? An IV drug user presents with chest pain, dyspnea, tachycardia, and tachypnea. What is ma., likely the cause? A patient in a MVA presents with chest pain, dyspnea, tachycardia, and tachypnea. What is the most likely cause? A post-op patient presents with chest pain, dyspnea, tachycardia, and tachypnea. What is the most likely cause? A young girl with congenital valve disease is given penidllin prophylactically. In the ER, bacterial endocarditis is diagnosed. What is the next step in her management7

w> Other Cardiac Pathology


-p: R: Chapter 12 q H: Chapters 237-239 rd Rheumatic heart disease (FAII p276) (FAI2 p302) (R p565) Q_ Pericarditis (Rp58l)(Hpl97l) U Cardiac tamponade (FA 11 p2 76) (FA 12 p302) .ftJ Syphilitic heart disease (FAII p276) (FAI2 p302) XJ Cardiac tumors (FAII p277) (FAI2 p303) (R p583) (H pl98l) rd

{J II. What are the diagnostic criteria (Jones criteria) for rheumatic fever?
L_ If evidence of a group A Strep infection, there is a high probability of acute rheumatic fever if a ~ patient has two major criteria and one minor criterion.

- Joints (migratory polyarthritis) - Heart (pancarditis) - Nodules (subcutaneous) - Erythema marginatum (serpiginous skin rash) - Sydenham chorea (chorea of the face, tongue, upper-limb) Minor criteria: arthralgia, fever, elevated ESR or CRP, prolonged PR interval on EKG

12. Kussmaul's sign vs. pulsus paradoxus:


Kussmaul's sign Event Mechanism Disease JVD with inspiration Decreased capacity of RV Constrictive pericarditis tamponade Pulsus paradoxus Decreased SBP by more than 10 mmHgwith inspiration Decreased capacity of LV Cardiac tamponade pericarditis

Copyright 3 2012. Doctors In Training.com, LLC. All Rights Reserved.

sion
Under what drcumstances might you see pulsus paradoxus?

tat are the Jones criteria for the diagnosis of acute rheumatic fever?

What are the common causes of restrictive cardiomyopathy?

16. What heart pathology ts the following statement7 I Diffuse myocardial inammation with necrosis and mononuclear cells Focal myocardial inammation with multinucleate giant cells Fever + IVDA + new heart murmur Chest pain and course rubbing heart sounds in patient with Cr of 5.0 Tree-barking of the aorta Child with fever, joint pain, cutaneous nodules 4 weeks after a throat infection ST elevations in all EKG leads Disordered growth of myocytes EKG shows electrical alternans

Copyright 2012. Doctors In Training.com. LLC. All Rights Reserved.

What organisms are known for causing endocarditis? (FAI I p275) (FAQ p30l)

In a study of 100 people, your study results show an average of 500 with a standard deviation of 50. Calculate the 95% condence interval. (FAQ p54) (FAQ p57)

Vascular Disorders I
R: Chapter II H: Chapter 326 Varicose veins (FAII p277) (FAI2 p303) Raynaud phenomenon (FAII p277) (FAI2 p303) Sturge-Weber disease (FAII p277) (FAI2 p305) Vascular tumors (FAII p279) (FAI2 p305) Small-vessel vasculitis (FAII p277-278) (FAI2 p304-305) - Microscopicpolyangiitis (FAII p277) (FAI2 p304) - Wegener's granulomatosis (FAII p277) (FA12 p304) - Churg-Strauss syndrome (FAII p277) (FAI2 p304) - Henoch-Schonlein purpura (FAII p278) (FAI2 p305)

29
Which disorders are commonly discovered in patients with Raynaud phenomenon? (FAIIp277)(FAI2p303)(Rp5l8) What local skin problems may arise from venous insufciency resulting in varicose veins? (Rp5l8)

What are the characteristic features of Henoch-Schonlein purpura? (FAI I p278) (FAQ p303)

Copyright 2012. Doctors In Training.com. LLC. All Rights Reserved.

Vascular Disorders 2
R: Chapter II H: Chapter 326 Medium-vessel vasculitis (FAII p278) (FAI2 p304) - Polyarteritis nodosa (FAII p278) (FAI2 p304) - Kawasaki disease (FAII p278) (FA12 p304) - Buerger disease (FAII p278) (FAI2 p304) Large-vessel vasculitis (FAII p278-279) (FAI2 p304) - Temporal arteritis (FAII p279) (FAI2 p304) - Takayasu arteritis (FAII p278) (FAI2 p304)

n Quiz
diagnose temporal arteritis?

What are the characteristic features of Kawasaki disease?


Which type of vasculitis ts the following high-yield characteristics? Weak pulses in upper extremities Necrotizing granulomas of lung and necrotizing glomerulonephritis Necrotizing immune complex inammation of visceral/renal vessels | Young male smokers Young Asian women Young asthmatics i Infants and young children; involved coronary arteries | Most common vasculitis | Associated with hepatitis B infection I Occlusion of ophthalmic artery can lead to blindness Perforation of nasal septum j Unilateral headache, jaw claudication Which vascular tumor ts the following description? (FAI I p279) (FAQ p305) Benign, raised, red lesion about the size of a mole in older patients Raised, red area present at birth, increases in size initially then regresses over months to years Lesion caused by lymphoangiogenic growth factors in an infected HIV patient Polypoid red lesion found in pregnancy or after trauma j I Benign, painful, red-blue tumor under ngernails Cavernous lymphangioma associated with Turner syndrome Skin papule in AIDS patient caused by Bartonella

Copyright 2012. Doctors In Training.com, LLC. All Rights Reserved.

What should I do ve days prior to the test?


I would split your Step I book reference such as First Aid into 4-5 equal sections. Since FA contains some 600 pages of material, each section will have about 120 pages. Over 4-5 days review your text entirely by going through about 120 pages per day. If you go through 10 pages an hour; it should take you 9-12 hrs per day to do this. As you come across a detail that you did not recognize before or have not seen before, write it down. At the end of each day and at the beginning of the next day, review those details you have written. The small details in FA are often overlooked and may be on your test If you dedde to continue with a question bank during these 5 days, I wouldn't do more than 25-50 questions a day. Concentrate on your knowledge base. At the end of Day 5,1 would do the following: - Review all of the unfamiliar details written from the previous 5 days - Review all the questions in the DIT study guide, the "denitely know" topics highlighted during the course, and the 3-5 star topics highlighted during the course. - Decide what information is going to be put on your marker board right before the test and memorize that information. - Consider taking a 150 NMBE test online (free or $45). Since these tests are slightly easier than the real thing rt will boost your condence a Ifttle. Since there are no answers given, you won't waste time going through the answers. You can expect 3-4 of those questions to be on your actual test It will allow you to regain familiarity with the actual testing format and make sure you've gone through the tutorial so you can skip it on your test day and add that time to your break time. - Skim through FA again at your leisure to review random topics, but pay particular attention to the "denitely know" and 3-5 star topics - Stop studying at least 2 hours before bedtime to make sure you get a good night's sleep. By going through this 5-day program, you will be able to say and know that you've reviewed everything in the 5 days before the test

What should I put on my markerboard prior to the start of the test?


Don't write on your markerboard for more than 5 minutes before you start your test Put whatever you want but you may want to consider the following - Developmental milestones - The 4 pharmacokinetic equations - Error square - Sens, spec, PPV, NPV, OR, RR equations and square - Lung volume diagram

Test Day Tips


Bring a cooler with ice, water; Gatorade, or juice. Pack a lunch. Bring some fruits and snacks. (You may not be able to predict what you're going to want to eat so it's better to bring too much than too little.) Eat ligt, not heavy. Consider getting out in the sun and/or stretching during your breaks. j-. Bring a light sweater or sweatshirt in case it's cold. Q_ Don't forget your ID and USMLE pass. | Take your breaks when you need them/however you want them (for example: 2 sections break s_ 2 sections * break I section break * 2 sections). Some breaks may need to be longer than < others. Don't be afraid to take a small 5 minute bathroom break Q Expect 5-10 questions in each section that you have never seen before. If you expect this, then it won't I freak you out when it happens (and it will happen). (jO Bring your own watch to keep track of your break time! M-* Consider answering 10 practice questions prior to going into the test center for "warm-up" (but don't ' look at the answers in case you are incorrect).

[40]

Copyright 2012, Doctors In Training.com. LLC. All Rights Reserved.

stio
Which exotoxin works via the following mechanism? (FAI I pl43) (FAQ pl54) Inactivates EF-2 Actives Gs Disables Gi Bacterial adenylate cyclase (no G protein involved) Blocks GABA and glycine

Which immune deciency matches the following descriptions? (FAI I p2l3) (FAQ p23" Anaphylaxis on exposure to blood products with IgA Course facial features, abscesses, eczema Thrombocytopenia, purpura, infections, eczema Delayed separation of the umbilicus Neuro defects, partial albinism, recurrent infections Describe how the murmur of mitral regurgitation is different than the murmur of aor regurgitation. (FAI I p255) (FAQ pFA282)

Bone Histology and Pathology


R: Chapter 26 Bone histology (R pl206, Fig. 26-1) Bone formation (FAII p379) (FAI2 p4l4) Osteoclasts (Rpl206, 1216) Osteoblasts (R pl207) Achondroplasia (FAII p379) (FAI2 p4l4) Osteoporosis (FAII p379) (FAI2 p4l4) Bisphosphonates (FAII p392) (FAI2 p490) Osteopetrosis (FAII p379) (FA/2 p4l4) Osteomalacialrickets (FAII p379) (FAI2 p4l4) Osteitis brosa cystica

Copyright'.'^ 2012. Doctors In Training.com, LLC. All Rights Reserved.

4. Osteoporosis Treatment
Sfejp smoking, stop alcohol consumption, stop steroids (if able), avoid PPI and H2-blockers if possible Exercise (aerobic, resistance, and weight bearing) EaJLrJsk assessment at home Vitamin D supplementation Calcium supplementation Bisphosphonates (inhibit osteoclastic resorption, 4. fracture risk by 40-50%) ETH (anabolic agent that stimulates osteoblasts) for moderate-severe OP treatment: Teriparatide decreases hip fracture rate by 53%, duration of therapy not to exceed 2yrs, must use bisphosphonate after stopping PTH to maintain BMD Other Treatments Worth Knowing Testosterone replacement for men with low testosterone (relatively un-researched but theoretically a good idea; OP with testosterone deciency should still be treated with bisphosphonates. Conjugated estrogen (WHI trial hip fx reduction of 33%) USPSTF recommends not using estrogen for the sole purpose for fx reduction risk due to risk of other SE (Ml and CVA). May use with bisphosphonate. Accelerated bone loss is seen after withdrawal of estrogen therapy. SERM: Raloxifene decreases vertebral fx by 40% in women with OP, no effect on risk of non-vertebral
CO

i_

fx risk, reduces risk of breast CA. Calcitonin (Miacalcin) - inhibits osteoclasts, dosed nasally or SC/IM, not as effective as bisphosphonates; in fact, overall effectiveness is questionable Combination therapy (i.e., bisphosphonate + Raloxifene)

"^ 2

L-J 5. Osteitis brosa cystic (AKA von Recklinghausen disease of bone)


^ A bony manifestation of an endocrine disorder O Can be caused by: CD - Hyperparathyroidism

High PTH ^ high serum calcium, low serum phosphate, high alkaline phosphatase

fti - Type IA pseudohypoparathyroidism (AKA Albright Hereditary Osteodystrophy) V PTH resistance at the renal tubules low serum calcium and high phosphate O * L o w c a l d u m h i g h P T H CQ High PTH excess osteoclastic activity "Brown tumors" in bone which are cystic spaces lined by I osteodasts lled with brous stroma and blood DEXA scan reveals low bone mineral density, but the mechanism of bone loss is different from OP

z err
LU

Q Q
<

CfT

[2]

Copyright 2012. Doctors In Training.com. LLC. All Rights Reserved.

UIZ 6. Where does new bone formation take place in growing long bones?

What cells types are most similar osteoclasts?

Which bony disease ts the following description? Reversible when vitamin D is replaced Excess osteoclastic activity results in disorganized bony architecture Bone is replaced by broblasts, collagen, and irregular bony trabecule Soft bones due to defective mineralization of osteoid Failure of bone resorption thickened and dense bones Genetic deciency of carbonic anhydrase II What are the risk factors for osteoporosis? (FAI I p379) (FAQ p379)

What measures can be taken to prevent osteoporosis?

11. What bone disorder results from excess PTH?

Paget Disease and Bone Tumors


R: Chapter 26 Paget disease (FAII p380) (FAI2 p4l4) Polyostotic brous dysplasia (FAII p380) (FAI2 p4l5) Lab values in bone disorders (FAII p380) (FAI2 p4!5) Primary bone tumors (FAII p38l) (FAI2 p4l6)

Copyright 2012. Doctors In Training.com, LLC. All Rights Reserved.

1111 ii i u ic iuiiuwii rg Lduic. inru i pjou; i^r/-\iz p^iD/

Serum Ca2+ Serum Phos Alk Phos Paget Disease Osteomalacia/rickets Osteitis brosa cystica Osteoporosis Osteopetrosis

PTH

. Which primary bone tumor ts the following description? (FAI I p38l-382) (FAQ p 415-416) Most common malignant primary bone tumor of children Most common malignant primary bone tumor in adults Most common benign bone tumor 11 ;22 translocation Soap-bubble appearance on x-ray Onion-skin appearance of bone May actually be a hamartoma Codman's triangle on x-ray Associated with Gardner

Copyright 2012, Doctors In Training.com. LLC All Rights Reserved.

armI. What signs and symptoms would lead you to suspect rheumatic heart disease? (FAI I p276) (FAQ p302)

When does implantation of the ovum take place? (FAI I p482) (FAQ p536) What nerve is damaged when a patient presents with the following symptom (upper extremity)? (FAII p374-375) (FAQ p409-4IO) Claw hand Wristdrop Scapular winging Unable to wipe bottom Loss of forearm pronation Weak lateral rotation of arm Loss of arm and forearm flexion Loss of forearm extension Unable to raise arm abt

Basics and Lower Extremity Joints


COA: Chapters 3, 5 R: Chapter 26 Joint basics Types of joints Chondrocytes (R pi227) Hip - Anterior hip dislocation - Osteoarthritis (FAII p382) (FAI2 p4l7) Knee - Unhappy triad I knee injury (FAII p370) (FAI2 p405) - Patellofemoral syndrome - Prepatellar bursitis (housemaid's knee) Anklelfeetltoes - Ankle sprain - Plantar fasciitis - Gout

Copyright 2012, Doctors In Trainnig.com. LLC. All Rights Reserved.

A football player who was kicked in the leg suffers from a damaged medial meniscus. What else is likely to have been damaged?

What ligaments are typically injured during an ankle sprain?

What is the leading cause of knee pain in patients younger than 45?

7. What is the other name for housemaid's knee?

Upper Extremity Joints


COA: Chapter 6 Shoulder - Rotator cuff muscles (FAII p37l) (FAI2 p405) - Shoulder dislocation - Shoulder separation - Subacromial bursitis (H: Chapter 337) - Adhesive capsulitis Elbov/ - Repetitive elbow trauma (FAII p376) (FAI2 p4ll) - Epicondylitis - Ulnar nerve injury - Tear of biceps tendon Hands and wrist - Bones of the hand - Carpal tunnel syndrome - Dupuytren contracture

8. What is the difference between a dislocated shoulder and a separated shoulder?


Dislocated - head of humerus rotates out of glenoid cavity Separated - clavicle separates from acromion and coracoid process of the scapula

9. What structures can be damaged in an anterior shoulder dislocation?


Axillary nerve and posterior circumflex artery Supraspinatus tendon Anterior glenohumeral ligaments and glenoid labrum separation from the articular surface of the anterior glenoid neck (AKA Bankart lesion) Posterolateral humeral head defect (due to abrasion against the anterior rim of the glenoid) - (AKA Hill-Sack lesion)
Copyright 2012. Doctors In Training com. LLC. All Rights Reserved.

10. Carpal Bones

n
EHS^Hfft ctions? (FAI I p

Lippincott Williams & Wilkins Atlas of Anatomy 2008 Wolters Kluwer Health. Inc. All rights reserved.

Which muscles of the rotator cuff are responsible fo (FAQ p 405) Initial 15 degrees of arm abduction Lateral rotation of arm Medial rotation of arm

What elbow injury is common in tennis players? What elbow injury is common in golfers? (FAII p376)

Which antibiotic class is known to increase the risk of tendon rupture in adults?

A patient comes to the clinic complaining of anterior shoulder pain that radiates down into the forearm. On examination, you notice a swelling of the biceps muscle. What the most likely diagnosis?

Copyright 2012. Doctors In Training.com. LLC. All Rights Reserved.

What portion of the brachial plexus is injured in Erb-Duchenne Palsy? What are the symptoms? (FAI I p375) (FAQ P4I0)

What is the clinical appearance of intemuclear ophthalmoplegia? With what disorder is it commonly associated? (FAI I p424) (FAQ p464)

Which lipid-lowering agent matches the following description? (FAI I p282) (FAQ p308) SE: facial flushing SE: elevated LFTs, myositis SE: Gl discomfort bad taste Best effect on HDL Best effect on triglyceridesA/LDL Best effect on LDL/cholesterol \nds C.diff. toxin

Osteoarthritis and Treatment


e n
H: Chapter 332 R: Chapter 26 GG: Chapter 34

<D Osteoarthritis (FA11 p382) (FAI2 p417) $ Aspirin (FAII p39l) (FAI2 p429) OJ NSAIDS (FAII p39l) (FAI2 p429) (H p2835, Table 332-1) . ! COX-2 inhibitors (FAII p392) (FAI2 p430) Q Acetaminophen (FA 11 p392) (FA 12 p430)

What is the mechanism of treating acetaminophen overdose? (FAI I p392) (FAQ p43<

What is the classic feature of osteoarthritis? (FAI I p382) (FAQ p43l)

By what mechanism do NSAIDS cause renal disease? (FAI I p39l) (FAQ p430)

Copyright 2012, Doctors In Training.com, LLC. All Rights Reserved.

Other Joint Diseases


H: Chapters 331, 333, 334 R: Chapter 26 GG: Chapter 34 Rheumatoid arthritis (FAII p383) (FAI2 p4l8) Sjogren syndrome (FAII p383) (FAI2 p4l8) Gout (FAII p 384) (FAI2 p4l9) Gout drugs (FAII p392) (FAI2 p4l9) (GG p994) Pseudogout (FAII p384) (FAI2 p4l9) (H p2839) Infectious arthritis (FAII p384) (FAI2 p420) (H p2842)

joints and Achilles tendon, and exquisite pain in the metatarsophalangeal joint of his ri big toes. Biopsy reveals needle-like crystals. What is the diagnosis?

What medications are used in the treatment of an acute gout exacerbation?

What drugs can be used in the treatment of chronic gout? (FAI I p393) (FAQ p431)

tat are the classic symptoms of Sjogren syndrome? (FAI I p383) (FAQ p4l8)

Copyright 2012. Doctors In Training.com. LLC. All Rights Reserved.

A 75-year-old man presents with acute knee pain and swelling. An X-ray reveals absenc of erosion of the joint space and calcium deposits in the menisci. What is the diagnosis, and what would you nd on aspiration of the joint? (FAI I p384) (FAQ p4l9) (H p2839)

What are the common locations for tophi in gout patients? (FAI I p384) (FAQ p4l9)

What is the mechanism and time-frame of acute transplant rejection? (FAI I p2l5) (FAQ p239) (R p228)

Systemic Rheumatic Diseases


H: Chapters 318-323 Seronegative spondyloarthropathies (FAII p385) (FA12 p42l) TNF-alpha inhibitors (FAII p393) (FAI2 p432) Systemic lupus erythematosus (FAII p385) (FAI2 p42l) Polymyalgia rheumatica (FAII p386) (FAI2 p42l) Scleroderma (FAII p387) (FAI2 p423)

4. Systemic Lupus Erythematosus = 4/11


Skin - Malar rash - Discoid rash - Photosensitivity - Oral ulcers -itis - Arthritis (non erosive, 2 joints) - Serositis (pleuritis, pericarditis) - (+)ANA Disorders - Renal (proteinuria, cellular casts) - Neuro (seizures, psychosis) - Heme (hemolytic anemia, leukopenia, lymphopenia, thrombocytopenia) - Immune (antiphospholipid Ab (+), anti-dsDNA, anti-Smith, false (+) VDRL)

Copyright 2012. Doctors In Training.com, LLC. All Rights Reserved.

ick Qui
5. What drugs are known for causing drug-induced lupus? (FAI I p385) (FAQ p4_.

lat are the manifestations of CREST scleroderma? (FAI I p387) (FAQ p423)

A patient has difculty swallowing, distal cyanosis in cold temperatures, and anticentromere antibodies. What else would you expect to see in this patient?

A patient presents with photosensitivity, arthritis, renal disease and recurrent oral ulcers is taking primaquine and NSAIDS. What type of check-up should she be receiving twice aye.

30-year-old woman presents with a low grade fever, a rash across her nose that gt worse when she is out in the sun, and widespread edema. What blood test would > orderto conrm your clinical suspicion?

Muscle Disorders
R: Chapter 26 H: Chapter 335 Muscular dystrophies (FAII p87) (FAI2 p9l) (R pl268) Polymyositisldermatomyositis (FAII p386) (FAI2 p422 ) (R pl270) Neuromuscular junction diseases (FAII p386) (FAI2 p422) (R pl270) Fibromyalgia (H p2849)

10. Fibromyalgia
Excess muscular tenderness in II of 18 particular sites Chronic generalized pain, fatigue, sleep disturbances, HA, cognitive difficulty, mood disturbances 30% also have depression and/or anxiety Pharm treatment - FDA-approved: pregabalin, milnacipran - Traditional (non FDA-approved): amitriptyline, low dose analgesic, uoxetine Non-pharmacological treatment: reassurance that it is a real disease and that it is benign, exercise and stretching, sleep, relaxation techniques, stress reduction

Copyright 2012. Doctors In Training.com. LLC. All Rights Reserved.

. What is the cause of Duchenne muscular dystrophy? (FAI I p87) (FAQ p9l)

What is the function of the protein that is absent in Duchenne muscular dystrophy? (FAII p87)(FAI2p9l)

13. What are some of the characteristics of polymyositis that distinguish it from polymyalgia rheumatic? (FAI I p386) (FAQ p422)

hat drug category is often used in the treatment of myasthenia gravis? (FAII p386) (FAQp422)

Which cancer is particularly associated with Lambert-Eaton syndrome? (FAII p386) (FAQ p422)

Copyright 2012. Doctors In Training.com. LLC. All Rights Reserved.

What are the symptoms of normal pressure hydrocephalus? (FAI I p408) (FAQ p448)

,n which genetic disorder would you nd acanthocytosis of RBCs and excess lipids in enterocytes? (FAI I pi 15) (FAQ pQI)

What disorder is associated with the following mutation? (FAI I p86-87) (FAQ p 90-9P
FGF receptor 3 FMRI gene

Derm Basics and Derm Path Part I


H: Chapters S3, 54 Epidermis layers (FAII p370) (FAI2 p404) Epithelial cell junctions (FAII p370) (FAI2 p404) Nonepithelial cell junctions Fibroblasts (R pi02) Melanocytes (R pi 168) Derm terminology (FAII p387) (FAI2 p423) Derm path part I (FAII p388-389) (FAI2 p424-427)

4. Nonepithelial/Nonjunctional Adhesion Mechanisms Cadherins ICAMs Integrins Selectins 5. Hemidesmosome, cadherin, integrin, ICAM-I. Which joins only cells of the same type and does not attach cells to the basement membrane? 6. Fibroblasts
Collagen Glycosaminoglycans Reticular and elastic fibers Glycoproteins Stimulated by tissue damage 7. Melanocytes Melanin-producing cells Located in the stratum basale (bottom layer) of the epidermis Responsible for skin color

Copyright 2012. Doctors In Traimng.com. LLC. All Rights Reserved.

8. Energy Exchange That Takes Place at the Skin


UV light is used to generate vitamin D Melanocytes displace UV light as heat Bilirubin is converted Dendritic cells process antigens Mechanical energy is converted to electrical energy by nerve endings

9. Atopic dermatitis (eczema) Treatment:


Switching to a moisturizing soap (Dove, Aveeno) and adding an OTC emollient may be all that is needed for maintenance and mild cases Calcineurin inhibitors: tacrolimus (Protopic) or pimecrolimus (Elidel) Topical Steroids Antibiotics for open lesions (cover Staph, aureus and Strep, spp.) Antihistamines Leukotriene inhibitors (Montelukast) - theoretical efficacy supported by weak studies UV light therapy Systemic steroids (l-2mg/kg/d in children then taper) only in severe cases and only for short duration For very severe cases, consider methotrexate, cyclosporin, azathioprine

10. Acne
Pathophysiology Hyperkeratosis Sebum overproduction Propionibacterium acnes proliferation Inammation Treatment

A pregnant patient comes to the clinic complaining of new dark pigmentation on her face. What is the most likely diagnosis? (FAI I p388) (FAQ p424)

What are some of the hallmark features of necrotizing fasciitis? What organism causes this infection? (FAI I p388) (FAQ p425)

What organism causes painless white patches on the tongue that cannot be scraped off? (FAII p388) (FAQp425) What is a lesion of excessive collagen deposition especially around the face, ears, neck, or upper chest called? (FAI I p387) (FAQ p423) What organism is responsible for acne vulgaris?

Copyright 2012, Doctors In Trainmg.com, LLC. All Rights Reserved.

Derm Path Part 2


H: Chapter 52

Derm path part 2 (FAII p388-389) (FAI2 p424-427)


Psoriasis (H p395) Blistering disorders Skin cancers (FAII p390) (FAI2 p428)

16. Psoriasis Treatment Options Treatment Options (effectiveness commonly wanes, so rotate therapies about q-year) Topical steroids - NEVER give oral steroids to someone with psoriasis! Calcipotriene (vitamin D3 analog that inhibits epidermal cell proliferation) - Most worldwide prescribed single-agent treatment for psoriasis Tazarotene (Tazorac) (topical retinoid * normalizes keratinocyte proliferation) Coal tar (suppresses DNA synthesis) - Brown tar (liquor carbonis detergens, LCD) comes in moisturizers, soaks, or ointments - Black tar (crude coal tar) comes as 2%, 5%, or 10% (FDA ruled 5% or less not carcinogenic) - Use with UV therapy (Goeckerman therapy) for mod-severe psoriasis - Must be left on for at least 4 hours. Stains clothes and sheets. - It is messy (purple-brown staining of skin (reversible) and everything), irritating, and moderate efcacy - Absorption occurs within I hour may shower an hour after application Salicylic acid (keratolytic used to remove excess scale) i l IV -t-hpt-apy- for patients with more than 10% (refer to a Dermatologist) e (acitretin) - Use if > I0%BSA - CBC, LFTs, Lipids qlmx 3 then q3m - Absolute contraindication in pregnancy!
: iniprtinnc; into dermk

Etanercept: Anti-TNF agent approved for use in mod-severe psoriasis ' Others agents (if refractory to Soriatane or UV light): oral retinoids, methotrexate, cyclosporine

17. Which skin disorder matches the following statement? Pruritic, purple, polygonal papules Life threatening rash with bullae Pruritus associated with asthma Pruritic vesicles associated with celiac disease Allergy to nickel Thickened scar esp. around face/chest Antibodies against epidermal basement membrane Antibodies against cell-cell adhesions Parakeratotic scaling Keratin-filled cysts Sand-paper; predisposition to squamous cell cancer Skin rash and proximal muscle weakness Honey-crusting lesions common about the nose and lips Hyperkeratosis and koilocytosis Histology shows palisading nuclei

Copyright 3 2012. Doctors In Training.com. LLC. All Rights Reserved.

u est ion warmWhat are the Kubler-Ross stages of grief? (FAI I p6l) What physiologic changes are sensed by peripheral chemoreceptors? By cen. chemoreceptors? (FAI I p265) (FAQ p290)

Which oncogene is associated with colon cancer? Which is associated with small cell li cancer? (FAI I p227) (FAQ p253)

Psychology Basics
8roin Organization - Amygdala (Phys p7l9) - Brainstem neurotransmitter changes with disease (FAII p442) (FAI2 p485) Freud's structural theory of the mind (FAII p439) Psychology Basics - Personality - Life stages - Maslow's hierarchy of needs - Suffering, hope, helplessness - Review of stress - List of stressful events by degree of stress induced

4. Erikson's Stages of Development (1968) ^


m

> *

Stage in Life Infant Toddler Preschooler School-Age Child Adolescent Young Adult Middle-Aged Adult Older Adult

Ages Birth- 18m 18m - 3yrs 3yrs - 5yrs 6yrs - 12 yrs 12yrs - 18yrs 18 yrs - 35 yrs 35 yrs - 55 to 65 yrs 55 to 65 yrs - Death

Psychosocial Crisis Basic Trust vs. Mistrust Autonomy vs. Shame Initiative vs. Guilt Industry vs. Inferiority Identity vs. Role Confusion Intimacy vs. Isolation Generativity vs. Self-absorption Integrity vs. Despair

1
>
k >

Maslow's Hierarchy of Needs (1943)


1) Physiological Needs 2) Safety Needs 3) Love and Belonging 4) Esteem 5) Self-Actualization
Copyright () 2012. Doctors In Training.com. LLC. All Rights Reserved.

fMBBHRH
What psychosocial crisis are most 25-year-olds working through? What is the order of human needs as described by Maslow? What neurotransmitter changes do you see with the following diseases? (FAII p442) (FAQp485) Anxiety disorders Depression Mania Alzheimer disease Huntington disease Schizophrenia itjmi^jjiajijjjj^jy

Learning and Therapy


Phys: Chapter 58 Stages of learning Types of therapy counseling 10. Four Stages of Learning 1) Unconscious Incompetence 2) Conscious Incompetence 3) Conscious Competence 4) Unconscious Competence 11. Bloom's Taxonomy of Learning Objectives 1) Knowledge 2) Comprehension 3) Application 4) Analysis 5) Synthesis 6) Evaluation 12. Cognitive Behavioral Therapy Treatment tools: - Joumaling - Challenging beliefs - Mindfulness - Relaxation Treatment goals: - Recognize unrealistic perceptions and maladaptive behavior patterns - Replace negative thoughts/behaviors with images/beliefs/actions that facilitate recovery
Copyright C'j 2012. Doctors In Training.com. LLC All Rights Reserved.

13. Psychoanalysis
Analysis of dreams, fantasies, and associations, as well as the verbal and physical expression of thoughts Therapist helps the client recognize and confront the inner conflicts responsible for symptoms and behavior problems

14. Axis
I) Psychiatric disorders II) Intellectual disabilities and personality disorders III) General medical condition IV) Psychosocial V) Global Assessment of Function

Which portion of the brain is responsible for delayed gratication?

16. Which portion of the brain is responsible for memory?

'hat term applies to the growth of new neurons?

Copyright 2012. Doctors In Tmintng.com. LLC. All Rights Reserved.

What are some of the clinical effects of zinc deciency!

What is the cause of anemia given the following statement? (FAI I p349-353) (FAQ p380-385) Microcytic anemia + swallowing difficulty + glossitis Microcytic anemia + > 3.5% HbA2 Megaloblastic anemia not correctable by Bl2 or folate Megaloblastic anemia along with peripheral neuropathy Microcytic anemia + basophilic stippling Microcytic anemia reversible with B6 HIV positive patient with macrocytic anemia Normocytic anemia + red urine in the morning Normocytic anemia and elevated creatinine

Vhat type of cancer is associated with the following tumor suppressor genes? (FAII p228) (FAQp253)

APC WTI BRCAI and BRCA2

Infants and Early Childhood


Regress/on in children (FAII p44l) Child abuse (FAII p44l) (FAI2 p484) Child neglect (FAII p44l) (FAI2 p484) Infant deprivation effects (FAII p44l) (FAI2 p484) Depression in infants - Anaclitic depression (FAII p44l) Mental retardation Trichotillomania Childhood and early-onset disorders (FAII p442) (FAI2 p484) ADHD medications (FAII p452) (FAI2 p497) (GG p303) 4. What are the most common causes of mental retardation? Down syndrome, Fragile X syndrome, Rett syndrome Congenital hypothyroidism Fetal alcohol syndrome and other prenatal toxin exposures (lead, mercury, valproate) Perinatal hypoxia or infection Postnatal causes: trauma/abuse, CNS hemorrhage, hypoxia (e.g., near-drowning), toxins, psychosocial deprivation, malnutrition, intracranial infection, and CNS malignancy

Copyright 2012. Doctors In Training.com. LLC. All Rights Reserved.

5. Trichotillomania
Compulsive nervous hair pulling More common in young girls Examination reveals unusual patterns of broken hairs of varying length "wire brush" feel Treatment: Education Cognitive behavioral therapy fluoxetine or clomipramine

6. Stimulants Used in ADHD


Methylphenidate (Ritalin) and Dextroamphetamine (Adderall) - Amphetamine, increases pre-synaptic norepinephrine vesicle release Atomoxetine (Strattera) - Norepinephrine reuptake inhibitor Other clinical uses of stimulants - Narcolepsy (modanil), obstructive sleep apnea an excessive daytime sleepiness, major depressive disorder

When a young monkey is separated from its mother; it becomes withdrawn, socially isolc and grooms poorly, ft is thought that this behavior is the monkey equivalent of what hun problem?

A 4-year-old girl complains of pain in her genitalia. On exam, a discharge is noticed and a smear of the discharge shows N. gonorrhoeae. How was she infected?

What name is given to chronic compulsive hair pulling common in young girls?

What stimulant medications are commonly used in the treatment of ADHD?

Adolescence
H: Chapter 79 Pervasive developmental disorders (FAII p442) (FAI2 p485) Depression in children Bullying Eating disorders (FAII p449) (FAI2 p493) (H p636) Gender identity disorder (FAII p449) (FAI2 p493)

II. What are some of the characteristic features of autism?


"Living in his own world" Symptoms evident prior to age 3 Lack of responsiveness to others, poor eye contact, absence of social smile Impairments in communication, language delay, repetitive phrases Peculiar repetitive, ritualistic habits (e.g., spinning around, hand flapping) Fascination with specific, seemingly mundane objects (vacuum cleaners, sprinklers) Usually below normal intelligence
Copyright 2012. Doctors lnTraining.com, LLC. All Rights Reserved.

12. What are the characteristic features of Rett disorder? Females with normal birth and development for the rst 6 months of life Stage I - Deceleration of head growth between ages 5 months and 4 years - Developmental arrest between 6-18 months - Social withdrawal and communication dysfunction Stage II - Loss of acquired purposeful hand skills between ages 6-30 months - Stereotypic hand movements usually midline (e.g., washing, wringing, squeezing) and constant during waking hours - Gait and truncal apraxia between ages I -4 years Stage III - Pseudostationary phase beginning between 2-10 years - Some improvement in behavior, hand use, and communication skills - More prominent motor dysfunction and seizures Stage IV - Onset after age 10 - Increased rigidity, bradykinesia, reduced mobility Eating Disorders (FAI I p449) (FAQ p493) 13. What electrolyte changes are seen in patients with excessive purging overtime? 14. Binge Eating Disorder Purging is not present Expression of deeper psychological problems Many actually have negative feelings toward food 15. Compulsive Eating Disorder Purging is not present Type of obsessive-compulsive disorder Much time is spent thinking and fantasizing about food

Which childhood psychiatric disorder matches the following statement7 Females only, loss of previously acquired purposeful hand skills between 6-30 months Impairments in social interactions, communications, play, repetitive behaviors Impairment in social interaction (but not avoidance), no language delay Stereotyped hand movements Ignoring the basic rights of others Characterized by hostility, annoyance, vindictiveness, disobedience, and resentfulness Multiple motor and vocal tics Impulsive and inattentive 7-year-old that avoids going to school to stay home with parent What is the difference between binge eating disorder and compulsive eating disorder? A 15-year-old girl of normal height and weight for her age has enlarged parotid glands bi no other complaints. The mother condes she found laxatives in the daughter's closet.

Copyright 2012. Doctors In Training.com, LLC. All Rights Reserved.

Ebwwr^wi

What is the difference between a warm agglutinin and a cold agglutinin? (FAI I p353) (FAQ p385) What are the equations for sensitivity; specicity, positive predictive value, and negativ predictive value? (FAI I p5l) (FAQ p53)

Delirium, Dementia, and Alcohol


H: Chapters 25, 371, 392 Orientation (FAII p443) (FAI2 p485) Delirium (FAII p443) (FAI2 p486) (H pl96) Dementia (FAII p443) (FAI2 p486) (H p3300) Dissociative disorders (FAII p444) (FAI2 p488) Substance dependence (FAII p449) (FAI2 p494) (H p3526) Substance abuse (FAII p449) (FAI2 p494) (H p3526) Substance withdrawal (FAII p449) Stages of change (FAII p450) (FAI2 p494) Signs and symptoms of substance abuse (FAII p450) (FAI2 p495) - Alcohol, barbiturates, benzos (GG p6S6-657) - Alcoholism (FAII p45l) (FAI2 p496) (H p3549) - Delirium tremens (FAII p45l) (FAI2 p496)

4. Stages of Behavioral Change


1) Precontemplation: +/- awareness of problem, no thought of changing 2) Contemplation: develops a desire to change 3) Preparation: intention to change 4) Action: begins to change behavior consistently 5) Maintenance: consistently changed behavior for over 6 months

5. What is the difference between alcohol abuse and alcohol dependence? (FAII p449) (FAQ p494)
Alcohol abuse - repeated alcohol use despite recurrent adverse consequences Alcohol dependence - at least 3 of the following: tolerance; withdrawal; alcohol abuse; inability to quit, cut-back, or regulate use; great amount of time spent obtaining, using, or recovering

6. What medications are effective in helping to prevent relapse in recovering alcoholics? is the tried-and-true best relapse prevention

Acamprosate (Campral)

Copyright 2012. Doctors In Traimng.com. LLC. All Rights Reserved.

exam. When asked what he would do if he smelled smoke in a movie theater; he replie that he would yell "re." When asked what a table and chair have in common, he replie that they both have 4 legs are made of wood. The family reports that for the past year someone has had to stay with him at all times for his own safety and that he stays awake all day and sleeps well at night. What is the most likely diagnosis in this patient? A 72-year-old patient of yours is brought into the clinic by his family because of strange behaviors over the past week. She has been very agitated recently, takes many naps during the day, occasionally urinates on herself and has had a poor appetite. It is difcult for you to administer the MMSE because she is unable to focus her attention on the questions. What is the most likely diagnosis? What are the stages of behavioral change?

Substance Abuse
H: Chapter 393 GG: Chapters 18, 24 Signs and symptoms of substance abuse (FAII p450) (FAI2 p495) Opioids (H p3552) (GG p658) Heroin addiction (FAII p45l) (FAI2 p496) (H p649) Opioid analgesics (FAII p430) (FAI2 p47l) (GG p48l) - Butorphanol (FAII p430) (FAI2 p472) (GG p509) - Tramadol (FAII p430) (FAI2 p472) (GGp508) Amphetamines and cocaine (GG p66l) Caffeine and nicotine (GG p657) PCP (GG p666) LSD (GG p665) Marijuana (GG p663)

Copyright 2012. Doctors In Training.com. LLC All Rights Reserved.

What drug intoxication or withdrawal is causing the following symptoms in the followir patients? Post-op constipation and/or respiratory depression Severe depression, headache, fatigue, insomnia/hypersomnia, hunger Pinpoint pupils, N/V, seizures Belligerence, impulsiveness, nystagmus, homicidal ideations, psychosis Headache, anxiety/depression, weight gain Anxiety/depression, delusions, hallucinations, flashbacks Euphoria, social withdrawal, impaired judgment, hallucinations Rebound anxiety, tremors, seizures, life-threatening Anxiety, piloerection, yawning, fever, rhinorrhea, nausea, diarrhea

What weak opioid agonist is commonly used in chronic pain control prior to prescribir stronger opioids? (FAI I p430) (FAQ p472)

v i iat is the treatment for a patient that is overly intoxicated with each of the followin substances? (FAI I p450) (FAQ p495) 1 Alcohol Benzodiazepines |

Copyright 2012. Doctors In Training.com. LLC. All Rights Reserved.

Which neoplasm is associated with the following statement! Nftrosamines Asbestos Naphthalene Arsenic EBV HPV Schistosoma haematobium

What is the cause of ITP? (FAI I p355) (FAQ p387)

What is the mechanism of action of the following drugs? (FAI I p362-364) (FAQ p396397) Streptokinase Aspirin Clopidogrel Abciximab Tirofiban Ticlopidine Enoxaparin Eptifibatide

cd Psychosis and Schizophrenia


H: Chapter 391 GG: Chapter 16 Hallucination vs. illusion vs. delusion vs. loose association (FAII p443) (FAI2 p486) Hallucination types (FAII p444) (FAI2 p487) Delusional disorder (FAII p444) (FAI2 p487) Anatomy and physiology of schizophrenia Schizophrenia (FAII p444) (FAI2 p487) (H p3542) Keeping "schizo-"straight (FAII p449) (FAI2 p493) Antipsychotics (FAII p453) (FAI2 p498) (GG p428) (H p3544, Table 391-14) Atypical antipsychotics (FAII p4S3) (FAI2 p498) (GG p428) (H p3544, Table 391-14)

Copyright 2012. Doctors In Training com, LLC. All Rights Reserved.

Schizotypal Schizophrenia Schizoaffective - Schizoid Schizophreniform What are some of the positive symptoms of schizophrenia? What are some of the neg symptoms? Categorize the following antipsychotics in the appropriate category as neuroleptics (low, moderate, or high potency) or atypical antipsychotics: olanzapine, thioridazine, quetiapine, molindone, chlorpromazine, haloperidol, uphenazine, loxapine, risperidone, thiothixene, triuoperazine, clozapine, aripiprazole
iMMlMBUBtl

What are the clinical features of neuroleptic malignant syndrome? How is it treated?

You are on-call and receive a call from a nurse asking if she can give some sleep median or diphenhydramine to an elderly patient with mild dementia who is currently hospitalizt

Mania and Bipolar Disorder


H: Chapter 391 GG: Chapter 16 Manic episode (FAII p445) (FAI2 p488) Hypomanic episode (FAII p44S) (FAI2 p488) Bipolar disorder (FAII p445) (FAI2 p488) (H p3539) Lithium (FAII p454) (FA/2 p499) (GG p445)

i[1im.9lllr4

9. What are the criteria for the diagnosis of mania?

I, What is the milder form of bipolar disorder?


11. What mood stabilizers are used in the treatment of bipolar disorder?
B Y- T l ^ K ^ ^

.e potential

BBI5igB^(oi!iiiii!iii'^

Copyright 2012. Doctors In Trnining.com. LLC. All Rights Reserved.

What clinical presentation might lead you to suspect a patient may have lymphoma? (FAI I p357) (FAQ p389)

What form of leukemia matches the following statement? Most common leukemia in children Most common leukemia in adults in US Characteristic Auer rods Greater than 20% blasts in marrow Leukemia with more mature cells and less than 596 blasts PAS (+) acute leukemia - Always positive for the Philadelphia chromosome (t 9;22) Acute leukemia positive for peroxidase - Solid sheets of lymphoblasts in marrow Always associated with the BCR-ABL genes What histological change takes place in the trachea of a smoker?

Major Depressive Disorder (MDD)


H: Chapter 391 Major depressive episode (FAII p445) (FAI2 p489) (H p3536) Atypical depression (FAII p446) (FAI2 p489) Postpartum mood disturbances (FAII p446) (FAI2 p489) Electroconvulsive therapy (ECT) (FAII p446) (FAI2 p489) Risk factors for suicide completion (FAII p46) (FAI2 p490)

Copyright"; 2012. Doctors In Training.com. LLC. All Rights Reserved.

ick Quiz
What are the criteria for the diagnosis of major depressive episode?

at is atypical depression?

What is the milder form of depression?

A patient mentions that he has had thoughts of suicide. What questions should you to help determine how high-risk he is?

A patient tries to commit suicide by slitting her wrists. After she is appropriately managed in the ER, what question would you ask the patient in order to determine her level of commitment of trying to take her own life?

A 28-year-old woman has symptoms of mild depression for 6 years. What's the diagnosis?

Antidepressant Medications
GG: Chapter 15 SSRIs (FAII p4S5) (FAI2 p500) (GGp405) SNRIs (FAII p455) (FAI2 pSOO) (GGp405) Tricyclic antidepressants (FAII p455) (FAI2 pSOO) (GG p405) Monoamine oxidase inhibitors (FAII p455) (FAI2 p500) (GGp405) Atypical antidepressants (FAII p456) (FAI2 p50l)

Copyright 2012. Doctors In Traimng.com. LLC. All Rights Reserved.

Two months after the loss of her spouse, a 42-year-old female is having trouble eating, concentrating and sleeping. She sleeps only 2-3 hours each night. What would you do f this patient? What is the mechanism of action of the following medication class? SSRISNRITCAMAOIBenzodiazepinesBarbitu ratesTypical neurolepticsAtypical neurolepticsCategorize the following antidepressants as either an SSRI, TCA, MAOI, NDRI, or SNFU. (FAII p455-456) nortriptyline, selegiline, bupropion, mirtazapine, uvoxamine, doxepin, phenelzine, uoxetine, clomipramine, imipramine, amitriptyline, nefazodone, milnacipran, desipramine, sertraline, venlafaxine. paroxetine, tranylcypromine, duloxetine, citalopram, desvenlafexine, trazodone SSRI NDRI TCA MAOI

Tetracyclic

13. Which antidepressant matches the following statement? SE priapism Lowers the seizure threshold Works well with SSRIs and increases REM sleep Appetite stimulant that is likely to result in weight gain Can be used for smoking cessation Can be used for bedwetting in children 14. What are the symptoms of TCA overdose? 15. What are the symptoms of serotonin syndrome?

Copyright 2012. Doctors In Training.com. LLC. All Rights Reserved.

uestion Warm-Up
What are the signs of vitamin A deciency?
What are the major signs of vitamin C deciency (scurvy)?
If \ty$jiTar.(i!l^

Anxiety Disorders
H: Chapter 391 Panic disorder (FAII p446) (FAI2 p490) (H p3529) Specic phobias (FAII p446) (FAI2 p490) (H p3533) Obsessive-compulsive disorder (FAII p447) (FAI2 p490) (H p3535) Post-traumatic stress disorder (FAII p447) (FAI2 p49l) (H p3534) Generalized anxiety disorder (FAII p447) (FAI2 p49l) (H p3530) - Buspirone (FAII p454) (FAI2 p 499) (GGp349) Malingering (FAII p447) (FAI2 p49l) Factitious disorder (FAII p447) (FAI2 p49l) Somatoform disorders (FAII p447) (FAI2 p49l) (H p354l)

.ViM^f^^'^fcifn-i.-cl-S-.l^i^l Irdlfr >','E(ia 'Sr: ^Ir?- 'V.'^lrilnTon

because of claustrophobia. What can you do to help this patient? A young woman that is anxious about receiving her rst pap smear is told to relax and imagine going through the steps of the exam. What does this process exemplify? 6. A woman has ashbacks about her boyfriend's death one month ago following a hitand-run accident. She often cries and wishes for the death of the culprit. What is the diagnosis? 7. A nurse has episodes of hypoglycemia. Blood analysis reveals no elevation in C-proteir What is her diagnosis? . . 8. What somatoform disorder matches the following description?. Unexplained pain Patient with normal anatomy is convinced a part of their anatomy is abnormal Unexplained loss of sensory or motor function (tests and PE are negative) Unexplained complaints in multiple organ systems False belief of being pregnant I* Unwavering belief by the patient that she has a specic disease (despite medical reassurance)

Copyright 2012. Doctors In Training.com. LLC. All Rights Reserved.

Ego Defense Mechanisms and Personality Disorders


H: Chapter 391 Anxiety self-treatment - Ego defense mechanisms (FAII p440) (FAI2 p482) - Alcohol - Escapism (video games, TV) Personality disorders (FAII p448) (FAI2 p492-493) (H p3542) Review of psychiatric pharmacology (FAII p452) (FAI2 p497)

9. Which mature defense mechanism ts the following statement?


Voluntarily choosing not to think about a piece of bad news Indiana Jones using humor to express feelings of discomfort Arsonist donates money to the fire department Using one's aggression to succeed in business ventures Realistically planning for future inner discomfort Consdously postponing an inner conflict until after a big project is completed Redirecting impulses to a socially favorable object

10. Which immature defense mechanism ts the following statement?


Not acknowledging a piece of bad news, as though it was never said Involuntary withholding of a feeling from consdence awareness A veteran that can describe horric war details without any emotion A child abuser was himself abused as a child Man yells at his family when he has a bad day at work A doset homosexual hates homosexuals because of the way they "make" him feel Using intellectual processes to avoid affective expression; Dr Frasier Crane Belief that people are either all good or all bad Expressing aggression through passivity, masochism, and turning against self Believing an external source is responsible for an unacceptable inner impulse Changing ones character or personal identity to avoid emotional distress Returning to an earlier level of maturation to avoid the conict at the current maturational level (stressed children wet the bed)

Offering an explanation for an unacceptable attftude/belief/behavior >s. A thought that is avoided is replaced by an unconscious emphasis on the opposite
<D Converting mental conflicts into bodily symptoms X Temporarily inhibiting thinking but continuing to build more tension < . Avoiding interpersonal intimacy to resolve conflict and obtain gratification - Extreme forms can result in multiple personalities __ Chronically giving into an impulse to avoid tension from an unexpressed unconscious wish (tantrums)

u>
CO Q_

[16]

Copyright 2012. Doctors In Traimng.com, LLC. All Rights Reserved.

11. What personality disorder ts the following statement?


Excessive need to be taken care of, submissive and clinging behavior, low self-confidence, fears of separation and losing support Grandiosity, feels he is entitled to things, lack of empathy Suicide attempts (* 1596 mortality), unstable mood and behavior, sense of emptiness and loneliness, impulsiveness Distrustful, suspicious, litigious Lifelong voluntary social withdrawal, no psychosis, emotional expression is limited (restricted range of affect) Feelings of inadequacy, hypersensitive to rejection or criticism, socially inhibited, shy Constant mood of unhappiness and pessimism Odd appearance, thoughts, and behavior no psychosis; social awkwardness Controlling, perfectionistic, orderly, stubborn, indedsfve Criminality, unable to conform to social norms, disregard for others' rights Excessively dramatic emotional, and extroverted; sexually provocative behavior; unable to maintain intimate relationships

-u CO
- <

n
>

ON

Copyright 2012. Doctors In Training.com. LLC. All Rights Reserved.

[17]

Wlfnmi What defense mechanism ts the following statement. , Voluntarily choosing not to think about a piece of bad news Not acknowledging a piece of bad news, as though it was never said Involuntary withholding of a feeling from conscience awareness A veteran that can describe horrific war details without any emotion A child abuser was himself abused as a child Man yells at his family when he has a bad day at work
. A 65-year-old asks her husband to stay in the hospital overnight with her because she is afraid to be alone. What defense mechanism is she exhibiting? . Which defense mechanism underlies all other defense mechanisms?

. A 60-year-old man, admitted for chest pain, jumps out of bed and does 50 push-ups to show the nurses he has not had a heart attack What defense mechanism is he using? . A 40-year-old woman tells you during one of her ofce visits that she is in love with you. You refer her to someone else, and she attempts suicide. What type of personality disorder does this patient have? 17. A 30-year-old woman tells you during one of her ofce visits that you are the best she threatens to change doctors because you do not feel a particular lab test is necessary. (doctor she's ever had but several symmetrical cuts on her left forearmsubsequent visit, Additionally, you notice that your nurse is very disrespectful. On a which she attributes to cat scratches. What type of personality disorder does this person have?

and black feather boa. She also is wearing an excessive amount of lipstick, and you notice her having conversations with many of the other patients in the waiting room. What type of personality disorder do you suspect in this patient? A person demands only the best and most famous doctor in town. What is the personality disorder does this person have? What are the Cluster A personality disorders? I A 55-year-old woman comes to your disorders? What are the Cluster B personality ofce wearing all black including a black miniskirt 12. What are the Cluster C personality disorders?

Copyright 2012, Doctors In Training.com. LLC. All Rights Reserved.

uestion WarmWhat pathology ts the following phrase? Antiplatelet antibodies - Bamboo spine on X-ray ' Webbed neck, short stature j Painful, raised lesions on palms + fever Dry eyes, dry mouth, arthritis - Posterior cervical adenopathy Low serum ceruloplasmin p5l8-526) (FAQ p580-598)

What type of antipsychotic is often the rst line of treatment for psychosis? Which antipsychotic should be reserved for severe, refractory cases because of agranulocyto (FAII p453) (FAQp498)

Which skin disorder matches the following statement? (FAI I p388-389) (FAQ p424-427) Pruritic, purple, polygonal papules Pruritic vesicles assodated with celiac disease Thickened scar esp. around face/chest Parakeratotic scaling Keratin-filled cysts Skin rash and proximal muscle weakness Honey crusting lesions common about the nose and lips Hyperkeratosis and koilocytosis

Anatomy and Embryology


R: Chapter 20 Phys: Chapter 25 Anatomy - Kidney anatomy and glomerular structure (FAII p458) (FAI2 p504) - juxtaglomerular apparatus (jGA) - Ureters: course (FAII p458) (FAI2 p505) Embryology - Kidney embryology (FAII pl32) (FAI2 pl40) - Potter syndrome (FAII pi32) (FAI2 pi40) - Horseshoe kidney (FAII pl32) (FAI2 pl40) Fluid compartments (FAII p459) (FAI2 p505)

Copyright 2012, Doctors In Training.com. LLC. All Rights Reserved.

What artery prevents a horseshoe kidney from ascending in the abdomen?

A 40-year-old patient of yours weighs 100 kg. What is her estimated plasma volume? (FAI I p459) (FAQ p505) (FAQ p505)

What fundamental problem creates Potter syndrome? (FAII pl32) (FAQ pl40)

What cell type releases renin? (FAI I p463) (FAQ p5IO)

In what manner do the ureters cross the uterine artery which helps you identify these structures during pelvic surgery? (FAI I p458) (FAQ p5IO)

Glomerulus
Phys: Chapters 25-26 GG: Chapter 25 Glomerular ltration barrier (FAII p459) (FAI2 p505) Renal clearance (FAII p459) (FAI2 p506) Glomerular ltration rate (GFR) (FAII p459) (FA/2 p506) Effective renal plasma ow (ERPF) (FAII p459) (FA12 p506) Filtration (FAII p460) (FAI2 p5l2) Changes in glomerular dynamics (FAII p460) (FAI2 p507) Calculation of reabsorption and secretion rate (FAII p460) (FAI2 p507) Glucose clearance (FA 11 p460) (FA 12 p507) Amino acid clearance (FAII p460) (FAI2 p507)

9. How will the following changes affect RBF, GFR, and ltration fraction? (FAII p458) (FAI2p507)
GFR Constriction of the afferent arteriole Constriction of the efferent arteriole Dilation of the afferent arteriole Dilation of the efferent arteriole Increase in serum protein Ureter stone obstruction ACE inhibitors Indomethacin, Naprosyn, ibuprofen
Copyright 2012. Doctors In Training.com. LLC All Rights Reserved.

RBF

FF (GFR/RBF)

What effect will a renal stone that obstructs the ureter have on GFR and FF?

tat is the maximal serum glucose concentration at which glucose can be absorbed ii the tubules?

What vitamin deciency results from Hartnup disease? (FAI I p460) (FAQ p507)

What substances can be used to estimate GFR? What substances can be used to estimate renal plasma ow? (FAI I p459) (FAQ p506)

What is the equation for the renal clearance of any substance? (FAI I p459) (FAQ p50

Copyright 2012. Doctors In Trainmg.com. LLC. All Rights Reserved.

What agents can be used to treat osteoporosis.

A child exhibits proximal muscle weakness and enlarged calves. What is the disease a. how is it inherited? (FAI I p87) (FAQ p9l)

3. What are 4 types of epithelial cell junctions? What are 4 proteins involved in nonepithelial adhesion mechanisms? (FAI I p370) (FAQ p404)

Nephron Physiology
Phys: Chapters 27-28 GG: Chapter 25 Nephron physiology (FAII p46l) (FAI2 p508) Proximal tubule Relative concentrations along proximal tubule (FAII p462) (FAI2 p509) Thin descending loop ofHenle Thick ascending loop ofHenle Early distal convoluted tubule Collecting tubule

4. Proximal Tubule (rst half)

HC03carbonlc anhydrase

C02 + H20

3HC03" HCO3-, C02 + H20

Lumen

where X is glucose, amino acids, Pi, or lactate

Copyright 2012. Doctors In Training.com, LLC. All Rights Reserved.

5. Proximal Tubule (second half) CI" _ _ Na+

r T
Lumen Anion"

J| a-X 1

Interstitiu

CI".__J Na+ f 6. Proximal Tubule (anions and cations)


organic anion organic anion organic anion -' alpha-ketoglutarate

Lumen

Interstitii

organic cation

organic cation

7. Thick Ascending Limb (TAL)

Lumen

carbonic' anhydrasal

Interstiti

Ca++'

Copyright 2012. Doctors In Training.com. LLC. All Rights Reserved.

8. What class of drugs inhibits the Na72CI7K+ symporter in the thick ascending limb? (FAI I p473)

9. Early Distal Tubule

Lumen

H20

10. What determines how much water is reabsorbed in the distal tubules and the collecting ducts? 11. What two types of cells compose the collecting duct and the last segment of the distal tubule? What do they do?
Principle cells Intercalated cells -

12. What are the two types of intercalated cells? in .y 13. What class of diuretic directly affects principle cells?
i

.z> 14. What affect does aldosterone have on the intercalated cells and principle cells of the O collecting duct?
! ' Intercalated cells-

Principle cells15. What drug antagonizes aldosterone's action on the principle cells of the collecting duct, , thereby promoting Na+ excretion and inhibiting K+ excretion? 16. What are the critical steps involved in excreting dilute urine?
Dilution of fluid in the thick ascending segment (to 100 mOsm/kg H2O) as solute is reabsorbed and water remains in lumen (due to the impermeability of water in the thick ascending limb) The absence of ADH renders the distal tubule and cortical collecting duct impermeable to water Tubular fluid is diluted even more as solute is removed from the tubular fluid in the distal tubule and cortical collecting duct but water remains Because of the low fluid osmolality in the collecting duct and the slight permeability of the medullary collecting duct to urea, urea enters the tubule from the medullary interstitium thereby keeping the osmolality of the medullary interstitium low
Copyright 2012, Doctors In Training.com. LLC. All Rights Reserved.

17. What are the critical steps involved in excreting concentrated urine? 1) Dilution of uid in the thick ascending segment (to 100 mOsm/kg H2O) as solute is reabsorbed and water remains in lumen (due to the impermeability of water in the thick ascending limb) - The reabsorption of solute without water in the thick ascending limb helps to increase the osmolality in the interstitium 2) The presence of ADH renders the distal tubule and collecting duct permeable to water - As water leaves the tubular uid to an interstitium of high osmolality, the osmolality of the tubular uid increases (and equals the osmolality of the interstitium) 3) The presence of ADH increases the permeability of the last portion of the medullary collecting duct to urea - As the tubular uid has a high concentration of urea (due to the reabsorption of water in the initial segments of the collecting duct and impermeability of those segments to urea), urea enters the interstitium as it goes down its concentration gradient from the last portion of the medullary collecting duct - As urea leaves the last portion of the medullary collecting duct (to go into the interstitium) and enters the loop of Henle (from the interstitium), it becomes more and more concentrated within the interstitium thereby increasing the osmolality of the interstitium - This high osmolality serves to concentrate the urine in the collecting ducts (which are permeable to water which allows the uid in the cortical collected duct to achieve the same osmolality as the uid in the medullary interstitium)

What segment of the renal tubule matches the following statements? (FAI I p46l) (FAQ p508) Reabsorbs 67% of the fluid and electrolytes filtered by the glomerulus Segment responsible for concentrating urine Site of secretion of organic anions and cations - Always impermeable to water Permeable to water only in the presence of ADH Site of the Na+/2C17K+ co-transporter Site of isotonic uid reabsorption Site responsible for diluting urine Only site where glucose and amino acids are reabsorbed Water reabsorption in the Loop of Henle What are the two main cell types of the collecting duct? 20. What affect does aldosterone have on the intercalated cells and principle cells of the Intercalated cells Principle cellsWhat class of drugs inhibits the Na+/2Q"/K+ symporter in the thick ascending limb? (FAII p473) (FAQ p524) I collecting duct?

Copyright 2012. Doctors In Training.com. LLC. All Rights Reserved.

Diuretics
GG: Chapter 25 Phys: Chapter 31 Site of action (FAII p473) (FA 12 p524) Mannitol (FAII p474) (FAI2 p526) Acetazolamide (FAII p474) (FAI2 p526) Furosemide (FAII p474) (FAI2 p526) Ethacrynic acid (FAII p474) (FAI2 p525) Hydrochlorothiazide (FAII p474) (FAI2 p526) K*-sparing diuretics (FAII p47S) (FAI2 p526) Diuretics: electrolyte changes (FAII p475) (FAI2 p527)

What type of diuretic is the following drug? (FAI I p473-475) (FAQ p524-526) Triamterene Mannitol Acetazolamide Metolazone Hydrochlorothiazide Chlorthalidone Bumetanide Furosemide Spironolactone Amiloride Chlorothiazide To r s e m i d e Ethacrynic acid

. What diuretic or class of diuretic would be most useful in the following situation? (FAI I p474-475) (FAQ p524-526) Acute pulmonary edema Idiopathic hypercalciuria ( calcium stones) Glaucoma Mild to moderate CHF with expanded ECV In conjunction with loop or thiazide diuretics to retain K+ Edema associated with nephrotic syndrome Increased intracranial pressure Mild to moderate hypertension Hypercalcemia Altitude sickness Hyperaldosteronism What is the site of action of mannitol? (FAI I p474) (FAQ p525)

What is the site of action of the thiazides? (FAI I p474) (FAQ p474)

A patient with heart failure exacerbation needs medical diuresis but has a sulfa allergy. What diuretic can be used?

Copyright 2012. Doctors In Trainmg.com, LLC. All Rights Reserved.

ues
A patient with recent kidney transplant on cyclosporin for immunosuppression requii an antifungal agent for candidiasis. What antifungal agent would result in cyclosporin toxicity? What is the most common cause of the following? (FAI I p529) (FAQ p594-596) Hypoparathyroidism Metastatic disease to brain Lysosomal storage disease
Myocarditis

For what type of information is the following thalamic nucleus a relay station? Ventral posterior lateral Lateral geniculate Ventral posterior medial Ventral anterior

Renal Metabolic Basics


R: Chapter 20 Phys: Chapters 29-30 H: Chapter 45, 47 Kidney endocrine functions (FAII p463) (FAI2 pSIl) Hormones acting on kidney (FAII p464) (FAI2 p5l2) Potassium shifts (FAII p464) (FAI2 pSI3) Electrolyte disturbances (FAII p464) (FAI2 pSI3) Acid-base physiology (FAII p465) (FA/2 p5l4)

4. Plasma osmolality = 2[Na+]Piasma + [glucose]/!8 + [BUNJ/2.8

5. Potassium Shifts (FAI 1 p464) (FAQ p5l2)


K* shift out of cells Hyperkalemia Low insulin Beta-blockers Acidosis Digoxin Cell lysis (i.e. leukemia) IC1" shift into cells Hyperkalemia Insulin 8-agonist Alkalosis Cell creation proliferation

Copyright 2012. Doctors In Training.com. LLC. All Rights Reserved.

for status asthmaticus?

What factors/substances cause hyperkalemia?

8. What factors/substances cause hypokalemia?

A patient presents with hypertension, hypokalemia, metabolic alkalosis, and low plasma renin. What is the diagnosis, and how do you treat it?

What are the actions of angiotensin II? (FAI I p462) (FAQ p5IO)

Which electrolyte disturbance ts the following presentation? (FAI I p464) (FAQ p5l3) Correcting too rapidly may result in central pontine myelinosis Peaked T waves Tetany Arrhythmias Decreased deep tendon reflexes Flattened T waves, U waves on EKG

pH and ADH
Phys: Chapters 29-30 H: Chapter 45, 47 Acidosis/alkalosis (FAII p463) (FAI2 p5l3) Renal tubular acidosis (RTA) (FAII p466) (FAI2 p5l4) Diabetes insipidus (FAII p300) (FAI2 p300) SIADH (FAII p300) (FAI2 p328) Demeclocycline (FAII p305) (FAI2 p333)

Copyright '> 2012. Doctors In Training.com. LLC. All Rights Reserved.

Normal Ranges (use for the question below) pH 7.35-7.45 pCC>2 35-45 mmHg p02 75-105 mmHg HCCV 22-28 mEq/L

(45 x 2 = 90) (45/2 = 22.5)

12. Determine what is wrong in patients with the following lab values:
1 H ' HC03- I pC02 i Type of Acid-Base Disorder

(Please note that the above values only reect high and low values and may not accurately reect values in appropriate compensatory mechanisms.)

mi' MT k^T:+> i [ 11 M.*i 'lr 13. How does acidosis/alkalosis affect extracellular K+ concentrations?

Determine what is wrong in a patient with the following lab values: HCOr Problem pCOl pH 3 5 4 2 750 7.33 1 3 2 8 7.20 1 8 40 7.66 3 6 30 7.47 1 4 2 2 7.10 1 5 50 What are the causes of acidosis with an elevated anion gap? (FAI I p465) (FAQ p5l4)

ostic features of diabetes

Ml p300)(FAI

Copyright :) 2012. Doctors In Traimng.com. LLC. All Rights Reserved.

What nerve is damaged when a patient presents with the following symptom (upper extremity)? (FAI I p374) (FAQ p4IO) Scapular winging Loss of forearm pronation Cannot abduct or adduct fingers Weak lateral rotation of arm Unable to abduct arm beyond 10 degrees What are the most common causes of hypocalcemia?

Label the following Gram positive algorithm.

Gram (+) Algorithm

Copyright 2012. Doctors In Training.com, LLC. All Rights Reserved.

Nephritic Syndrome
R: Chapter 20 H: Chapter 283 Nomenclature of glomerular disorders (FAII p466) (FAI2 p5l5) Nephritic syndrome (FAII p467) (FAI2 p5l6)

What are the dening features of nephritic syndrome? (FAI I p467) (FAQ p467)

Why is it important to treat Strep throat in children or in those that could transmit St pyogenes to a child? Which glomerular disease would you suspect most in a patient with the following ndings: (FAI I p466-468) (FAQ P5I5-5I8)
Granular pattern of immune complex deposition; LM: hypercellular glomeruli Linear pattern of immune complex deposition Deposition of IgG, IgM, IgA, and C3 in the mesangium Anti-GBM antibodies, hematuria, hemoptysis Nephritis, deafness, cataracts Crescent formation in the glomeruli Wire-loop appearance

Nephrotic Syndrome
R: Chapter 20 H: Chapter 283 Nephrotic syndrome (FAII p468) (FAI2 p468)

Copyright;-) 2012. Doctors In Trainmg.com. LLC. All Rights Reserved.

Which glomerular disease would you suspect most in a patient with the following ndin (FAI I p466-468) (FAQ P5I5-5I8) Most common nephrotic syndrome in children IF: granular pattern of immune complex deposition; LM: diffuse capillary thickening Kimmelstiel-Wilson lesions (nodular glomerulosclerosis) Most common nephrotic syndrome in adults EM: loss of epithelial foot processes Nephrotic syndrome associated with hepatitis B Nephrotic syndrome associated with HIV EM: subendothelial humps and tram-tack appearance LM: segmental sclerosis and hyalinosis Purpura on back of arms and legs, abdominal pain, IgA nephropathy Apple-green birefringence with Congo-red stain under polarized light EM: spiking of the GBM due to electron dense subepithelial deposits

Glomerular histology reveals multiple mesangial nodules. This lesion is indicative of what disease?

A teenager presents with nephrotic syndrome and hearing loss. What is the disease?

A 4-year-old boy presents with facial edema and proteinuria. What is the appropriate treatment?

Copyright C 2012. Doctors In Tr.iimng.com. LLC. All Rights Reserved.

armA patient taking lisinopril complains of new onset, constant coughing. To what medication class should this patient be switched?

What pathology is associated with the following high-yield phrase? (FAI I p518-522) ( p580-584) "Worst headache of my life" Waxy casts in urine Neuropathy + AV nodal block Port-wine stain in the ophthalmic division of the trigeminal nerve Urethritis, conjunctivitis, arthritis Painless jaundice To what drug category does the following drug belong? Azathioprine Probenecid Primaquine Cefprozil Lamivudine Tobramycin Losartan dinavir 6-mercaptopurine Rofecoxib Carmustine Doxycydine Timolol Methotrexate Cimetidine Meoquine

Other Renal Pathology I


R: Chapter 20 H: Chapters 94, 285, 287, 288 Casts in urine (FAII p466) (FAI2 pSIS) Kidney stones (FAII p469) (FAI2 p5l9) Renal cell carcinoma (FAII p469) (FAI2 p520) Wilms tumor (FAII p469) (FAI2 p520) Transitional cell carcinoma (FAII p470) (FAI2 p520) Pyelonephritis (FAII p470) (FAI2 pS2l) Drug-induced interstitial nephritis (FAII p470) (FAI2 p470)

4. Under what circumstances would you see the following type of cast? (FAII p466) (FAI2p5l5)
RBC cast WBCcast Bacterial cast Epithelial cell cast Waxy cast Fatty cast Granular cast

Copyright 2012. Doctors In Training.com. LLC. All Rights Reserved.

5. Acute Interstitial Nephritis (AIN) (FAII p470) (FAQ p52l)


Results in acute renal failure (ARF) Classic presentation: fever, rash, eosinophilia, and azotemia Most common cause is drug-induced (NSAIDs, PCN/cephalosporins (esp. methicillin)), sulfonamides (e.g. TMP-SMX, furosemide), ciprooxacin, cimetidine, allopurinol, PPIs, indinavir, mesalamine) Rx: 2 weeks of corticosteroids

Other Renal Pathology 2


R: Chapter 20 H: Chapters 279-282, 284 Diffuse cortical necrosis (FAII p470) (FAI2 p52l) Acute tubular necrosis (FAII p470) (FAI2 p52l-522) Renal papillary necrosis (FAII p470) (FA 12 p522) Acute renal failure (FAII p47l) (FAI2 p522) Consequences of renal failure (FAII p47l) (FAI2 p522) Renal cysts (FAII p472) (FAI2 pS23)

10. Causes of ATN (FAI I p47l) (FAQ p523)


Drugs: aminoglycosides, cephalosporins, polymyxins Radiograph contrast dye (prevent with N-acetylcysteine, fluids, NaHC03) Rhabdomyolysis/myoglobinuria - Due to muscle breakdown from seizure disorder, cocaine, or crush injuries - Findings: 4+ blood in urine, no RBC on urine cell count, renal failure, elevated CPK

Copyright 2012. Doctors In Trainnig.com. LLC. All Rights Reserved.

ssion Quiz
A patient reports a long-term history of acetaminophen use. What is she at increase risk for?

2 What changes will be seen in a basic metabolic panel in a patient with renal failure? (FAII p47l) (FAQp523)

A CT scan reveals massively enlarged kidneys bilaterally. What is the diagnosis?

14. Renal pathology rapid review: (FAI I p53l) (FAQ p592-596) Most common tumor of the urinary tract system Most common renal malignancy of early childhood (ages 2-4) Histologic appearance of renal cell carcinoma Histological appearance of chronic pyelonephritis Fever + rash + hematuria + eosinophilia Cancer associated with Schistosoma haematobium Treatment for cystine kidney stones

Copyright 2012, Doctors In Training.com. LLC. All Rights Reserved.

I. What personality disorder ts the following statement? (FAI I p447-448) (FAQ p492-493) 1 Excessive need to be taken care of, submissive and clinging behavior j Low self-condence, fears of separation and losing support ' Grandiosity, feels he is entitled to things, lack of empathy j Suicide attempts ( 15% mortality), unstable mood and behavior ' Sense of emptiness and loneliness, impulsiveness Odd appearance, thoughts, and behavior; no psychosis; social awkwardness Controlling, perfectionistic, orderly, stubborn, indecisive Criminality, unable to conform to social norms, disregard for others' rights What pathology matches the following statements? Antiepfthelial cell antibodies Anti-basement membrane Cough, conjunctivitis, coryza, fever Councilman bodies Green/yellow pigment just within the corneoscleral margin Anticentromere antibodies Dementia + eosinophilic inclusions in neurons Anti-dsDNA antibodies (ANA antibodies) What are the different causes of post-op fever? (FAI I pi78) (FAQ pl98)

Reproductive Anatomy
R: Chapters 21. 22 H: Chapters 346, 347 Male reproductive anatomy (FAII p479) (FAI2 p53l) (H p30IO) Female reproductive anatomy (FAII p478) (FAI2 p530) (H p3028) Female reproductive histology (FAII p478) (FAI2 p530) Innervation of the male and female genitals (COA p4ll) Pudendal nerve block (COA p433) Autonomic innervations of the male sexual response (FAII p479) (FAI2 p53l) Gonadal venous drainage (FAII p478) (FAI2 p530) Inguinal canal (FAII p3l5) (FAI2 p344) Hernias (FAII p3l6) (FAI2 p345)

Copyright <<s 20I2. Doctors In Training.com, LLC. All Rights Reserved.

What cell type lines the uterus? (FAI I p478) (FAQ p530) What ligament of the female pelvis matches the following description? (FAII p478) (FAQ p478) Connects the cervix to the pelvic side wall Connects the ovaries to the lateral pelvic wall Connects the uterus, fallopian tubes and ovaries to the pelvic side wall Contains the uterine vessels Contains the ovarian vessels What is the landmark used for the pudendal nerve block? (COA p433) To where does testicular cancer rst metastasize? (FAI I p478) (FAQ p530)

Reproductive Embryology
COA: Chapter 3 H: Chapter 349 Genital embryology (FAIIpl33) (FA12 pi42) SRYgene (FAII p486) (FAI2 pl42) Bicornuate uterus (FAII pl33) (FAI2 pl4l) Male/female genital homologues (FAII pl34) (FAI2 pl42) Congenital penile abnormalities (FAII pl34) (FAI2 pl43) Descent of testes and ovaries (FAII pl34) (FAI2 pl43) (COA p205)

8. Exstrophy of the bladder - congenital gap in the anterior bladder wall and abdominal wall in front of it exposure of the bladder interior to the outside world

What structures develop from the mesonephric duct system? (FAI I pl32) (FAQ pl4l-l42) 10. What is the male homologue to the following female structure? (FAI I pl34) (FAQ pl43) Labia minora Bartholin glands Urethral and paraurethral glands (of Skene) What gene product comes from the SRY gene that underlies male genital development? (FAII p486) (FAQ pl42) i* Vestibular bulbs

4) (FAQ pi
Copyright @ 2012. Doctors In Training.com. LLC. All Rights Reserved.

What is the most common: (FAI I p528) (FAQ p580) (H pl929, Table 237-1) Cause of DIC Heart murmur Coronary artery involved in thrombosis Cause of death in lupus patients Congenital heart anomaly What are the positive symptoms of schizophrenia? What are the negative symptoms schizophrenia? (FAI I p444) (FAQ 487,493) (H p3542)

What is the WAGR complex? (FAI I p469) (FAQ p520) (R p440,479)

Androgens
H: Chapter 346 Phys: Chapter 80 Seminiferous tubules (FAII p480) (FAI2 p532) Spermatogenesis (FAII p48l) (FAI2 p533) Regulation of spermatogenesis (FAII p48l) (FA/2 p533) Control of reproductive hormones (Male) (FAII p497) (FAI2 p553) Androgens (FAII p482) (FAI2 p534) Testosterone (methyltestosterone) (FAII p497) (FAI2 p553) Antiandrogens (FAII p498) (FAI2 p553) (GG pl204) Androgen insensitivity syndrome (FAII p486) (FA/2 p540) (H p305l) S-alpha reductase deciency (FAII p486) (FAI2 p540) Pseudohermaphroditism (FAII p486) (FAI2 p539) True hermaphrodite (FAII p486) (FAI2 p540) (H p3050)

Copyright 2012. Doctors In Training.com, LLC. All Rights Reserved.

How many carbon mole (FAI I p29l) (FAQ p292) (Phys p980 Fig. 80-8) What is the difference between androgen insensrtivity and 5-alpha-reductase deciency? (FAII p486) (FAQ p540) What is the role of Sertoli cells and Leydig cells in spermatogenesis? (FAII p48l)(FAI2p48l,533) ,'. What reproductive pathology matches the following statement? Female with short stature + no Barr body Chromosomal XXY Chromosomal XO Presence of ovaries, but male genitalia Unable to generate DHT Both ovarian and testicular tissues are present Webbing of the neck Male with Barr body in PMNs Ambiguous genitalia until puberty, then masculinization

Testes Pathology
H: Chapters 96, 346 Epididymitis (R p986) Torsion (R p987) (H pl097) Cryptorchidism (FAII p495) (FAI2 p55l) (R p984) Testicular germ cell tumors (FAII p496) (FAI2 p552) (R p987) Testicular non-germ cell tumors (FAII p496) (FA12 p552) (R p992) Tunica vaginalis lesions (FAII p496) (FAI2 p552) (R p993)

8. Epididymitis
Inflammation of the epididymis Dx: support of the testes some relief Rx <35 = GC/Chlamydia - Ceftriaxone IM then Doxycycline xlOd Rx >35 or h/o anal intercourse = Enterobacteriaceae Fluoroquinolone xl0-l4d 9. Testicular Torsion Twisting of the spermatic cord ischemia Dx: support of testis no relief; US Rx: surgical detorsion with bilateral orchiopexy within 6hrs

10. Cryptorchidism (FAI I p495) (FA p55l)


Failure of testis to descend into scrotum Usually unilateral Descent usually complete in I st year of life 35x increased risk of malignant tumor in the undescended testicle (usually a germ cell tumor)
Copyright <n 2012, Doctors In Traimng.com. LLC. All Rights Reserved.

d of Session Quiz
(FAII p495)(FAI2p55l)

What testicular tumor is described by the following statement? Composed of cytotrophoblasts and syncytiotrophoblasts May present initially with gynecomastia Elevated AFP Elevated B-hCG Most common testicular tumor Most common testicular tumor in infants and children up to 3yrs of age Most common testicular tumor in men over age 60 Histologic appearance similar to koilocytes (cytoplasmic clearing) Histologically may have alveolar or tubular appearance sometimes with papillary convolutions Composed of multiple tissue types Histologic endodermal sinus structures (Schiller-Duval bodies) 25% have cytoplasmic rod-shaped crystalloids of Reinke Androgen producing and associated with precocious puberty

Copyright 2012. Doctors In Traming.com. LLC All Rights Reserved.

What pathology is associated with the following key wo. (FAII p518-519) (FAQp580-584) Bilateral hilar lymphadenopathy Cherry-red spot on macula Slapped cheeks rash on child Organism associated with dog or cat bite Facial muscle spasm upon tapping the cheek Cough, conjunctivitis, coryza Nephritis, hearing loss, cataracts When performing a lumbar puncture for anesthesia administration, where is the anesthesia dosed? Where is CSF found? (COA p 505-506)

What is the female homologue to the following male structure? (FAI I pi34) (FAQ pi42) (H p3049, Fig 349-3) Scrotum Prostate gland Glans penis Corpus spongiosum Bulbourethral glands Vent

60 Penis

-pr Phys: Chapter 80 X COA: Chapter 3 +=> H: Chapter 48 rd 0_ Sexual dysfunction (FAII p6l) (FAI2 p63) (H p374) nj Erectile dysfunction (COA p427) (H p375) -= Sildenal, vardenal (FAII p499) (FAI2 p556) (H p377) Penile pathology (FAII p496) (FAI2 p552) - Bowen disease (R p983) ! - Bowenoid papulosis (R p496) ~~ Condyloma acuminatum (R p983) Balanitis (R p982) Q 4. Erectile Dysfunction Failure to (atherosclerosis) b* Failure to initiatefill(psychogenic, endocrinologic or neurogenic) Failure to store adequate blood volume within the lacunar network Diabetic, atherosclerotic and drug related causes account for >80% of cases of ED in older men

Q_ 5. Condyloma acuminatum Benign genital wart Caused by HPV 6 and 11


Copyright 2012. Doctors In Training.com. LLC. All Rights Reserved.

lick Quiz
What is the mechanism of action of sildenal? (FAI I p499) (FAQ p556)

What are the side effects of sildenal? (FAI I p499) (FAQ p556)

What organism is commonly implicated in balanitis?

Prostate
H: Chapter 95 R: Chapters 21, 22 Prostatitis (FAII p494) (FAI2 p550) (R p993) (H p239l) Benign prostatic hyperplasia (FAII p495) (FAI2 pSSI) (R p994) Tamsulosin (FAII p499) (FAI2 p556) (GGp307) Prostatic adenocarcinoma (FAII p495) (FAI2 p55l) (R p996) (H p799) 9. BPH - Benign Prostatic Hyperplasia (R p994) Present in 80% of men over age 80 y/o Diagnosis based on the following symptoms: sensation of incomplete voiding, increased urinary frequency (less than q2hrs), straining to void, intermittent or weak urine stream, urgency, nocturia (at least 2-3 times a night) Palpable prostate size may not correlate with degree of obstruction or symptom severity Alternative Medicine: Isoflavones, saw palmetto Medical Intervention - Nonselective a-blockers: Doxazosin, Prazosin, and Terazosin Decrease prostate smooth muscle tone immediate improvement in urine flow SE: dizziness, postural hypotension, fatigue, asthenia. To reduce SE, dose qHS and titrate dose upward slowly overtime (weekly) Tamsulosin (Flomax) (selective a-1Ablocker) - fewer SE than non-selectives, has no antihypertensive effects - 5a-reductase inhibitors: Finasteride, Dutasteride slowly reduces dihydrotestosterone levels 20% decrease in prostate volume over 3-6m SE: decreased libido, ejaculatory disorder, impotence Surgical Intervention

Copyright 2012, Doctors In Trainmg.com. LLC. All Rights Reserved.

TFii
1 How does utamide differ from nasteri use? (FAI I p498) (FAQ p 554)

A 55-year-old man undergoing treatment for BPH has increased testosterone and decreased DHT as well as gynecomastia and edema. What is his medication?

kmvjilu are the diagnostic symptoms of BPH? (FAI I p495) (FAQ p55l)

Copyright 2012. Doctors In Trainmg.com. LLC. All Rights Reserved.

Question Warm-Up
i?IH|ll<'|["<*/-|iliV/*vm*JK*t*jlM<Wllll^JPJHSJlkif*lfSli

What nerve damage causes carpal tunnel syndrome? (FAI I p372) (FAQ p407) (R pi266) To what drug categories do the following drugs belong? Cholestyramine Clarithromydn Venlafaxine n l a f a x i n e Ve Famotidine Fluphenazine Phentolamine Captopril Trazodone Carteolol Fluvoxamine 6-mercaptopurine Selegiline Terbinafme Clozapine

Female Reproductive Cycle


Phys: Chapter 81 H: Chapter 347 Control of reproductive hormones (female) (FAII p497) (FAI 2 p535) Estrogen (FAII p482) (FAI2 p535) (GGp 11 6 3 ) Progesterone (FAII p483) (FAI2 p535) Menstrual cycle (FAII p483) (FAI2 p536) Ovulation (FAII p484) (FAI2 p537) Oogenesis (FAI I p484) (FAI2 pS37) Pregnancy (FAII p484) (FAI2 p537) hCG(FAII p485) (FAI2 p538) [ ~D { [ . ) ( I
l

4. Outline the hormone sequence of the female reproductive cycle. '. (FAI I p483) (FAQ p536) (Phys p995) ( FSH follicle maturation production of estradiol production of LH surge ovulation and *_ production of progesterone (along with estradiol) inhibition of FSH and LH production decline of _^ corpus luteum no production of estradiol and progesterone loss of FSH inhibition increase in FSH (repeat step I) ' 5. Corpus Luteum (Phys p995) Formed after ovulation, produces progesterone and estrogen in the luteal phase q Lifespan 13-14 days (luteal phase) ' If BhCG from the placenta is present, the lifespan will extend to 6-7 weeks until the placenta is able to produce its own progesterone. 6. What are the layers of the endometrium? Which layers are shed during menstruation? ~D

r~

7. Which hypothalamic nucleus is involved in ovulation?

8. When does the basal body temperature increase occur in relation to ovulation?

Copyright 2012. Doctors In Trainmg.com. LLC. All Rights Reserved

What is the two-cell theory of estradiol production? (FAI I p482) (FAQ p535)

What are the target cells of LH? What cells respond to FSH? (FAI I p48l) (FAQ p534)

State whether the following statement describes estrogen or progesterone. (FAII p482-483) (FAQp535) Production of thick mucus that inhibits entry of sperm into the uterus Induces LH surge Uterine smooth muscle relaxation Follicle growth Maintenance of pregnancy Hepatic synthesis of transport proteins Withdrawal leads to menstruation

When does 3-hCG appear in the urine during pregnancy? (FAI I p483) (FAQ p537)

Hormonal Birth Control and Menopause


H: Chapters 347, 348 GG: Chapter 40 Hormonal birth control - Oral contraception (FAII p499) (FAI2 p555) (Phys plOOl) (GG pi 185) - Estrogens (FAII p498) (FAI2 p554) (GG p 1163) - Progestins (FAII p498) (FAI2 p555) (GGpll87) - Medroxyprogesterone (GG p 1187) - lUDs Menopause (FAII p485) (FAI2 p538) (R pl027) Menopausal hot ashes Hormone replacement therapy (FAII p498) (FAI2 p555) (GG pi 175)

13. Medroxyprogesterone (Depo-Provera) (GG p 1187) IM injection dosed q3m Associated with bone mineral density loss especially if long-term, therefore not ideal for >2yr of use Good choice of contraception for patients with MR 14. Intrauterine Device (IUD) Copper (Paragard) 10 year Progesterone (Mirena) 5 year, prevents menstruation, used as Rx for menorrhagia Small risk of uterine rupture when placing Contraindicated if high risk of STD

Copyright 2012, Doctors In Training.com. LLC. All Rights Reserved.

15. Menopausal Hot Flashes (R pi027) (Phys p999)


Occur in 75% of menopausal women Presentation: starts in face/chest then generalizes, lasts 2-4min, associated with diaphoresis and palpitations, followed by chills and shivering May cause sleep disturbances Rx: Estrogen replacement > SSRI or SNRI (venlafaxine) > clonidine orgabapentin Herbal Rx: soy isoflavonesf, red cloverf, black cohoshf, vitamin E f potential estrogenic effect on the breast, much like estrogen replacement

16. What are the pros and cons of oral contraceptive pill use? (FAI I p499) (FAQ p554) (GG pi 187)

Why is p

What is the best option for birth control in a mentally retarded patient?

What hormonal changes are seen during menopause? (FAI I p485) (FAQ p536)

A 23-year-old female who is on rifampin forTB prophylaxis and on birth control (estrogen) gets pregnant Why?

Copyright 2012. Doctors In Training.com, LLC. All Rights Reserved

What pathology matches the following statement? Lens-shaped lesion on head CT


Common underlying cause of intussusception No milk production in the postpartum period Pigmented hamartomas in the iris Howell-Jolly bodies Cancer associated with asbestos Owl's eye inclusions Owl's eye nucleus Owl's eye protozoan

50-year-old male with new, unexplained skin yellowing and no other symptoms

In which causes of vaginal discharge/vaginitis will the pH be high? In which will the pH be low? (FAII pl8l)(FAI2p20l) What structures arise from the paramesonephric ducts? (FAII pl33) (FAQ pl4l) (H p3048)

Vaginal and Cervical Pathology


R: Chapter 22 H: Chapter 97 Female reproductive anatomy (FAII p478) (FAI2 p530) Vaginismus and vestibulitis (COA p434) Benign vulvar cysts - Bartholin gland cysts (COA p433) (R plOII) - Skene glands Vulvar intraepithelial neoplasia (VIN) and vaginal intraepithelial neoplasia (VAIN) (R plOI2, 1016) Vaginal carcinoma (FA II p492) (FAI2 p547) (R plOI6) Cervical pathology (FAII p488) (FAI2 p543 Endometriosis (FA p489) (FAI2 pS43) (R pl028)

4. Female Reproductive Anatomy

Lippmcou vvutiams & vvnKins Alias 01 Anatomy Vs/zuuo vvoncrs Muwer neaiui. inc. /mi ngnts reserved. Copyright 2012, Doctors In Training.com. LLC. All Rights Reserved.

5. Vulvar Intraepithelial Neoplasia (VIN) (RplOQ)


Very similar to CIN except vulvar location Grades I, II, III Associated with HPV (esp. 16, 18, and 31) Koilocytosis: squamous cell w/ perinuclear cytoplasmic clearing Precursor to vulvar carcinoma (FAI I p492) (FAQ p547)

What are the risk factors for cervical cancer? (FAI I p488) (FAQ p543)

What is the classic histological appearance of a vaginal epithelial cell infected with HP\ (FAII p488) (FAQ p543)

Who is at risk of clear cell adenocarcinoma? (FAI I p492) (FAQ p547)

Uterine Pathology
H: Chapter 97 R: Chapter 22 Endometrial proliferation (FAII p489) (FA12 p544) (Phys p995) - Endometrial hyperplasia - Endometrial carcinoma Myometrial tumors (FAII p489) (FAI2 p544) (R pl03l) - Leiomyoma - Leiomyosarcoma Leuprolide (FAII p497) (FA/2 p554) (GGpll22) Gynecologic tumor epidemiology (FAII p489) (FAI2 p544) (H p8l3)

What are the risk factors for endometrial carcinoma? What uterine pathology matches the following description? (FAI I p488) (FAQ p544, Excess unopposed estrogen is the main risk factor Menorrhagia with an enlarged uterus and no pelvic pain Pelvic pain that is present only during menstruation Diagnosed by endometrial biopsy in clinic Definitive diagnosis and treatment is by laparoscopy Menstruating tissue within the myometrium Malignant tumor of the uterine smooth muscle Most common gynecologic malignancy 1 n I A^ Se M 1' 5 ! ing of leuprolide affect its physiological impact7 (FAI I p497) (FAQ p5 S 'S t v " -' l 2

Copyright 2012. Doctors In Training.com, LLC. All Rights Reserved.

What organism is associated with the following statement? (FAI I pi54) (FAQ pi67) (R p358, Table 8-8) Cat scratch Dog/cat bite Cat feces Puppy feces Animal urine To what classes of medication do the following drugs belong? Primaquine Te m a z e p a m Saquinavir Desipramine Betaxolol Captopril Prazosin Busulfan Thiopental Moxifloxacin Tranylcypromine Sertraline Zanamivir Miconazole

Which defense mechanism ts the following description? (FAI I p440) (FAQ p482) (H p3529) Involuntary withholding of a feeling from conscience awareness A veteran that can describe horrific war details without any emotion A child abuser was himself abused as a child Underlies all other defense mechanisms May lead to multiple personalities Adult whining, bedwetting, crying

Ovarian Pathology
H: Chapter 97 R: Chapter 22 Ovarian histology (R pi041) Premature ovarian failure (FAII p489) (FAI2 p544) Most common causes of anovulation (FAII p489) (FAI2 p544) (R pl026) Polycystic ovarian syndrome (FAII p490) (FA12 p545) Clomiphene (FAII p498) (FAI2 p555) (GGpll79) Ovarian cysts (FAII p490) (FAI2 p545) (R pl038)

Copyright 2012. Doctors In Training.com, LLC. All Rights Reserved.

Quick Quiz
What is the underlying cause o treatment? (FAI I p490) (FAQ p545) A patent with polycystic ovarian disease is most at risk for developing which type of cance Under what circumstances would you expect to see an elevated LH? What drug would you give to inhibit prolactin secretion? (FAI I p290) (FAQ p3l7)

Ovarian Tumors
H: Chapter 97 R: Chapter 22 Ovarian non-germ cell tumors (FAII p492) (FAI2 p547) Ovarian germ cell tumors (FAII p49l) (FAI2 p546) (R pl047)

8. What are the 4 main categories of ovarian tumors? (FAI I p491 -492) (FAQ p546-547)
Epithelial (65% of ovarian tumors, 90% of ovarian cancers) Germ cell Stromal Metastatic (Gl, breast, endometrium)

9. What are the main types of epithelial cell ovarian tumors (which account for 65% of ovarian tumors and 90% of ovarian cancers)?
Serous, mucinous, endometrioid, clear cell, Brenner, mixed (hint My Med Students Consistently Beat Exams.)

10. What are the main types of germ cell ovarian tumors?
Teratoma, dysgerminoma, endodermal sinus, choriocarcinoma

11. What are the main types of stromal/sex cord ovarian tumors? Granulosa-theca cell, Sertoli-Leydig cell, broma 12. What ovarian tumor matches the following statement? (FA p49l-492) (FAQ p546-547)
Lined with fallopian tube-like epithelium Psammoma bodies Intraperitoneal accumulation of mucinous material Resembles bladder epithelium
arm rp{\ ti imnrc:

Multiple different tissue types Elevated B-hCG Produces AFP Ovarian tumor + ascites + pleural effusions Estrogen-secreting, leading to precocious puberty Call-Exner bodies Testosterone-secreting, leading to virilization
Copyright 2012, Doctors In Training.com. LLC. All Rights Reserved.

What ovarian tumor matches the following statement? (FAI I p490-492) (FAQ p546-547) Produces AFP Estrogen-secreting, leading to precocious puberty Intraperitoneal accumulation of mucinous material Testosterone-secreting, leading to virilization Psammoma bodies Multiple different tissue types Lined with fallopian tube-like epithelium Ovarian tumor + ascites + hydrothorax Call-Exner bodies Resembles bladder epithelium Elevated B-hCG What are the risk factors for ovarian cancer?

An obese woman presents with amenorrhea and increased levels of serum testosterone. What is the most likely diagnosis?

Copyright :. 2012. Doctors In Training.com. LLC. All Rights Reserved.

Question Warm-Up
What pathology ts the following high-yield phrase? Thyroid cells with optically clear nudei Anemia with hypersegmented neutrophils Branching rods on oral infection Eczema + recurrent infections + thrombocytopeni Hemosiderinuria + thrombosis What are the stages of behavioral change? (FAI I p450) (FAQ p494)

Which primary bone tumor ts the following description? (FAI I p38l) (FAQ p Most common malignant primary bone tumor of children Most common benign bone tumor 11 ;22 translocation Soap-bubble appearance on X-ray Onion-skin appearance of bone Codman's triangle on X-ray

Pregnancy Complications
H: Chapter 7 Gravidity and parity Twinning (FAII pl2l) (FAI2 pl28) Amniocentesis and chorionic villus sampling (CVS) Placental development (FAII pl22) (FAI2 pl29) Umbilical cord (FAII pl22) (FAI2 pl29) Hydatidiform mole (FAII p487) (FAI2 p540) Common causes of recurrent miscarriages (FAII p487) Infertility Physiologic changes in pregnancy

4. In monozygotic twins, twin placentation is determined by timing of egg division: Diamniotic / dichorionic placentation- division occurs prior to morula stage (within 3 days of fertilization) Diamniotic / monochorionic placentation- division occurs 4-8 days post-fertilization (blastocyst) Mono/mono (one placenta) occurs with division 8-12 days after fertilization (epiblast/hypoblast) - One yolk sac and 2 fetal poles = monoamnionicity - Cord entanglement = monoamnionicity Division at or after 13 days conjoined twins

Copyright 2012. Doctors In Training.com. LLC. All Rights Reserved

5. Amniocentesis
Performed at 15-17 weeks for genetic evaluation (in third trimester for lung maturity eval) Indications: - Evaluation of lung maturity (lecithin: sphingomyelin ratio a 2.5) - > 35-year-old (offered for eval of possible trisomy/genetic defects) - Abnormal maternal serum triple or quad screen - In Rh-sensitized pregnancy to detect fetal blood type or fetal hemolysis Risks: I -2% maternal/fetal hemorrhage, 0.5% fetal loss

6. Chorionic Villus Sampling (CVS)


Performed at 10-12 weeks Risks: 1% fetal loss, 1% inability to diagnose NTD, limb defects if done < 9 weeks

7. What are some of the normal physiologic changes that take place during pregnancy?
Cardiac output increases 30-50% Plasma volume increases 50%, RBC volume increases 30% BP decreases in early pregnancy nadir at 16-20wks return to pre-pregnancy levels by term Increased minute ventilation decreased PACO2 and PaC02, mild respiratory alkalosis CO2 transferred more easily from fetus to mother Increased procoagulation factors hypercoagulable state Increased GFR decreased BUN and Cr Normal TSH and free T4 Increased peripheral resistance to insulin (due to human placental lactogen) that worsens throughout pregnancy hyperinsulinemia, hyperglycemia, hyperlipidemia

>^ :8. A pregnant woman at 16 weeks of gestation presents with an test, and what is the 1 and hypertension. What abnormality might be seen on blood atypically large abdomen disorder?

A 15-year-old female patient of yours that normally comes with her parents presents alone this time. She states that she is sexually active but that she knows she is not pregnant because she has never menstruated. What would be the appropriate next step in managing this patient?

Monozygotic twins are delivered. One is pale and has a hematocrit of 15%, and the other is ushed with a hematocrit of 55%. What is the cause of these features?

Copyright 2012. Doctors In Trainmg.com, LLC. All Rights Reserved.

Labor and Delivery


H: Chapter 7 Pregnancy-induced hypertension (preeclampsia and eclampsia) (FAII p487) (FAI2 p54l) Pregnancy complications (FAII p488) (FAI2 p541, 542) Amniotic uid abnormalities (FAII p488) (FAI2 p542) OB safe medications Labor - Dinoprostone (FAII p499) (GGp952) - Ritodrinelterbutaline (FAII p499) (FAI2 pSSS) (GG p293) Pregnancy termination - Mifepristone (RU-486) (FAII p498) (FAI2 p555) (GG pi 184)

II. OB Safe Medications


Category A: nystatin (vaginal), Category B: acetaminophen, diphenhydramine, ondansetron, meclizine, PCN/ampicillin/cephalosporins/ piperacillin, macrolides (erythromycin, azithromycin), nitrofurantoin, metronidazole, H2-blockers/PPIs, calcium antacids, insulin, metformin, methyldopa, Category C: pseudoephedrine, promethazine, demerol/morphine/hydrocodone, nystatin (oral), hydralazine, nifedipine, labetalol, docusate sodium, heparin

Category A E 1 C C ) X

Description Safety established in human studies Presumed safety based on animal studies No human or animal studies show an adverse effect/uncertain safety Human risk, but benet may outweigh risk Contraindicated, risk clearly outweighs benet

What is the difference between placental previa, abruptio, and accreta? (FAII P488)(FAI2p54l-542)

A pregnant woman with previous c-section is at increased risk for what?

14. What are some of the conditions that can result in polyhydramnios? FAII p488) (FAQp542)

What is the mechanism of action of mifepristone? (FAI I p498) (FAQ p555)

What agents are commonly used as tocolytics? (FAI I p499) (FAQ p334,537)

Copyright Q 2012. Doctors In Trainmg.com. LLC. All Rights Reserved.

[20]

Copyright 2012. Doctors In Trainmg.com. LLC. All Rights Reserved.

Trisomies and Mental Retardation


R: Chapter 5 H: Chapter 62 Fragile X syndrome (FAII p87) (FAI2 p92) (R pi69) Autosomal trisomies (FAII p88) (FAI2 p93) - Down syndrome (FAII p88) (FAI2 p93) - Robertsonian translocation (FAII p89) (FAI2 p93) - Edwards syndrome (FAII p88) (FAI2 p93) - Patau syndrome (FAII p88) (FAI2 93) Mental retardation Overview of causes of mental retardation

us may nave uown synaromer (FAI I p88) (FAQ p93) What is the most common event that causes Down syndrome: meiotic nondisjunctio mosaicism, or Robertsonian translocation? (FAI I p88) (FAQ p93) What gene is affected in Fragile X syndrome? (FAI I p87) (FAQ p92) What are the clinical features of Fragile X syndrome? (FAI I p87) (FAQ p92)

Other Chromosomal Disorders


H: Chapters 61, 62 R: Chapter 5 Sex chromosome disorders (FAII p485) (FAI2 p485) (R pl64-l67) Cri-du-chat syndrome (FAII p485) (FAI2 p94) Williams syndrome (FAII p89) (FAI2 p94) 22qll deletion syndromes (FAII p89) (FAI2 p94) (R pi62)

What are the clinical features of Williams syndrome? (FAI I p89) (FAQ p94)

>, ,iat are the possible defects in cases of chromosome 22ql I deletion? (FAII p89) (FAQp94)

What are the distinguishing characteristics of Klinefelter syndrome compared to Turner syndrome? (FA p483) (FAQ p485)

Copyright 2012. Doctors In Training.com. LLC. All Rights Reserved.

(FAI I p498) (FAQ p555) (GG pi 179)


2. What nerve is damaged when a patient presents with the following upper extremity symptom? (FAI I p374) (FAQ p409) (COA p729) Loss of forearm pronation Loss of arm and forearm flexion Trouble initiating arm abduction Unable to raise arm above horizontal What pathology ts the following description? Smudge cell Port-wine stain in ophthalmic division of trigeminal nerve S3 heart sound Adrenal hemorrhage associated with meningococcemia Ferruginous bodies Subepithelial humps on EM Spike and dome on EM Myocyte disarray

Currant jelly stool


Sacroiliitis \dverse

Benign Breast Conditions


R: Chapter 23 Breast feeding (H p588, 756) (Phys pi023) Common breast conditions (FAII p494) (FAI2 p550) - Acute mastitis (FAII p494) (FAI2 p550) (R pi069) - Fibrocystic change (FAII p494) (FAI2 p550) - Fat necrosis (FAII p494) (FAI2 pSSO) - Gynecomastia (FAII p494) (FAI2 p550) (H p30l9) (R pl093) Benign breast tumors (FAII p492) (FAI2 p548) (R pl07l) - Fibroadenoma (FAII p492) (FAI2 p548) (R pl09l) - Intraductal papilloma (FAII p492) (FAI2 p548) (R pl072) - Phyllodes tumor (FAII p492) (FAI2 p548) (R pl092) - Adenoma (R pi071)

4. Benign Epithelial Lesions (FAI I p492,494) (FAQ p548)


Non-proliferative breast (brocystic) changes Fibrosis Cysts-_ Adenosis: fibroadenoma Proliferative breast disease without atypia Sclerosis adenosis Epithelial hyperplasia Complex sclerosing lesion -

Copyright 2012. Doctors In Training.com. LLC. All Rights Reserved.

uick Quiz
What are some of the causes of gynecomastia?

SHlSSsiR?n2 p550)

What organism is most commonly responsible for acute mastitis? (FAI I p494) (FAQ p550) What is the most common cause of breast lumps in women of childbearing age? (FAI I p494) (FAQ p550) What is the most common breast tumor in women under age 25? (FAII p492) (FAQp548) What is the classic presenting complaint in a patient with intraductal papilloma? (FAII p492) (FAQp548)

Breast Tumors and Cancer


R: Chapter 23 Malignant breast tumors (FAII p493) (FAI2 p549) SERAIS (tamoxifen and raloxifene) (FAII p498) (FAI2 p555) (GG pi 177) Anastrozolelexemestane (FAII p498) (FAI2 p555) (GG pl76l)

10. In Situ Breast Carcinoma (15-30%) Ductal carcinoma in situ (DCIS, Intraductal Carcinoma) (FAI I p493) (FAQ p549)
Comedocarcinoma Solid Cribiform Papilary Micropapillary Lobular carcinoma in situ (LCIS) -

Invasive Breast Cancer (FAI I p493) (FAQ p549) Inflammatory carcinoma (70-80%) Invasive ductal carcinoma Invasive lobular carcinoma Medullary carcinoma Tubular/cribiform carcinoma Mucinous carcinoma Papilary carcinoma Metaplastic carcinoma

Copyright 2012. Doctors In Training.com. LLC. All Rights Reserved.

What breast pathology ts the following description? (FAI I p492-494) (FAQ p548-550) Most common breast tumor in women under 25 Most common breast mass in postmenopausal women Most common breast mass in premenopausal women Most common form of breast cancer Small, mobile, firm mass with sharp edges in 24-yr-old woman Histological "leaf-like projections" Signet ring cells Loss of e-cadherin cell adhesion gene on chrom 16 Always ER(+) and PR(+) Commonly presents with nipple discharge Eczematous patches on nipple Multiple bilateral fluid-filled lesions with diffuse breast pain Firm, fibrous mass in a 55-yr-old woman A 58-year-old postmenopausal woman is on tamoxifen. What is she at increased risk of acquiring?

Copyright ir; 2012. Doctors In Training.com, LLC. All Rights Reserved.

Signet-ring cells Nutmeg liver Maternal elevations of AFP RBC casts in urine Currant-jelly sputum Dog or cat bite

Which glomerular disease would you suspect most in a patient w'rth the following ndin (FAII p468)(FAI2p5l7-5l8) IF: granular pattern of immune complex deposition; LM: diffuse capillary thickening IF: granular pattern of immune complex deposition; LM: hypercellular glomeruli IF: linear pattern of immune complex deposition IF: deposition of IgG, IgM, IgA, and C3 in the mesangium EM: subendothelial humps and tram-track appearance Nephritis, deafness, cataracts LM: crescent formation in the glomeruli LM: segmental sclerosis and hyalinosis Purpura on back of arms and legs, abdominal pain, IgA nephropathy EM: spiking of the GBM due to electron dense subepithelial deposits

For which organisms can the following antibiotic classes be used? (FAI I pi86, FAI I pi88) (FAQ p206, FAQ p209) uaitrjaOTHH-si

Micro by Systems I
H: Chapters 126, 257, 381 R: Chapters 15, 26. 28 Normal ora: dominant (FAII pl75) (FAI2 pl95) Common causes of pneumonia (FAII pl76) (FAI2 pl96) Common causes of meningitis (FAII pl77) (FAI2 pl97) CSF ndings of meningitis (FAII pl77) (FAI2 pl97) Osteomyelitis (FAII pl77) (FAI2 pl97)

Copyright'':-) 2012. Doctors lnTr.iining.com, LLC. All Rights Reserved.

4. Which infectious agent ts the following description? Common cause of pneumonia in immunocompromised patients Most common cause of atypical / walking pneumonia Most common fungal infection of the lung in the Texas/Gulf Coastal region Common causative agent for pneumonia in alcoholics Can cause an interstitial pneumonia in bird handlers Often the cause of pneumonia in a patient with a history of exposure to bats and bat droppings Fungal cause of pneumonia in a patient who has recently visited South California, New Mexico, or West Texas Pneumonia associated with "current jelly" sputum Q fever Associated with pneumonia acquired from air conditioners Most common cause of pneumonia in children I-year-old or younger Most common cause of pneumonia in the neonate (birth-28d) Most common cause of pneumonia in children and young adults (including college students, military recruits, and prison inmates) Common cause of pneumonia in patients with other health problems Most common cause of viral pneumonia Causes a wool-sorter's disease (a life-threatening pneumonia) Endogenous ora in 20% of adults Common bacterial cause of COPD exacerbation Common pneumonia in ventilator patients and those with cystic brosis Pontiac fever

CO

H
2

<> L

> LU

[2]

Copyright 2012. Doctors In Training.com. LLC. All Rights Reserved.

5. An adolescent presents with cough and rust-colored sputum. What does the gram of the sputum show?

'>. An older male patient has blood in his urine and renal stones. What organism is most likely responsible for this patient's stones?

What organism is the most common cause of osteomyelitis? (FAI I p!77) (FAQ pl97)

What organism would you suspect as the cause of a septic arthritis in a 20-year-old m (FAII pl77) (FAQ pi97)

(FAII pl76) (FAQpl96) 6w-l8y I8y-40y I What are the most common causes of pneumonia for the following patient populatior
40y-65y

Elderly 10. What ndings would help you distinguish bacterial meningitis from fungal meningitis an viral meningitis? (FAI I pl77) (FAQ pl97)

What CSF ndings would you see in a patient with TB meningitis? (FAII p 177) (FAQp 197)

Micro by Systems 2
H: Chapters 130-131 R: Chapter 22 Sexually transmitted diseases (FAII pl8l) (FAI2 p20l) Vaginal dischargelvaginitis Pelvic inammatory disease (FAII pl8l) (FAI2 p202) Fever in the post-op patient Nosocomial infections (FAII pl8l) (FAI2 p202) TORCHES infections (FAII pl79) (FAI2 pl99)

Copyright 2012. Doctors In Training.com. LLC. All Rights Reserved.

12. What are the features of congenital syphilis?


Earlv manifestations Crst 5 weeks of lifel Hepatosplenomegaly, elevated LFTs Hemolytic anemia, jaundice Rash followed by desquamation of hands and feet Snuffles (blood-tinged nasal secretions) Radiographic changes at birth: metaphyseal dystrophy and periostitis Hutchinson teeth (notching or blunting of the upper incisors) Saddle nose deformity Frontal bossing Saber shins (anterior bowing of the tibia)

13. Vaginal Discharge/Vaginitis


pH Physiologic discharge Candidiasis Trichomonas Bacterial vaginosis Historical Clues Wet Prep

14. Fever in the Post-Op Period


Wind Water Wound Walking Wein Wonder drugs

What are the TORCH infections? (FAI I pl78) (FAQ pl99)

50-year-old patient recovered from abdominal surgery performed 2 days ago and has had an internal catheter in place since that time. He now has a fever of 100 F. What is the most likely cause of his fever?

Which STD matches the following statement? (FAI I pl8l) (FAQ p20l) Clue cells
Painless genital ulcer Flagellated cells

Copyright S 2012. Doctors In Training.com, LLC. All Rights Reserved.

What are 5 examples of encapsulated bacteria? What test can be used to detect encapsulated bacteria? (FAII pl39) (FAQ pl49) What causes a steeple sign on X-ray? What causes a thumb sign on X-ray? What pathology matches the following statement? (FAI I p52l-53l) (FAQ p580-588) Signet ring cells in the ovary Signet rings in RBCs Smudge cell Spike and dome of glomerulus on EM Tram track of glomerulus on light microscopy Strawberry tongue Most common location of tophi

Drug Side Effects


GG: Chapter 4 Drug reactions (FAII p244-245) (FAI2 p27l-272)

4. What drugs have the following potential side effects? (FAI I p244) (FAQ p27l)
Anticholinergic SE (dryness, confusion) Coronary vasospasm Cutaneous ushing Dilated cardiomyopathy Torsades de pointes Agranulocytosis Aplastic anemia Thrombocytopenia Gray baby syndrome Hemolysis in G6PD-decient patients Thrombosis Pulmonary brosis Cough Focal to massive hepatic necrosis Hepatitis Pseudomembranous colitis Adrenocortical insufciency Gynecomastia Hot ashes Hypothyroidism
Copyright 2012. Doctors In Traming.com. LLC. All Rights Reserved.

5. What drugs have the following potential side effects? (FAI 1 p245) (FAQ p272)
Gingival hyperplasia Gout Osteoporosis Photosensitivity Stevens-Johnson syndrome Drug-induced SLE Tendon rupture/cartilage damage Fanconi syndrome Interstitial nephritis Hemorrhagic cystitis Dizziness, nausea, headache, vision changes, tinnitus Nephrogenic diabetes insipidus Seizures Tardive dyskinesia Disulram-like reaction Nephro + neurotoxicity Nephro + ototoxicity 1

What medication causes cardiotoxicity and bone marrow suppression? A patient tries to commit suicide by overdosing on digitalis. What is the most important step in the management of this patient? What drugs induce the P450 system, and what effect this will have on other drugs? What drugs inhibit the P450 system, and what effect this will have on other drugs? . An African-American male that goes to Africa develops anemia after taking prophylactic medicine for primary disease prevention. What enzyme is this patient decient in? A 65-year-old male patient taking multiple medications presents with gynecomastia. Which of the following meds is most responsible for this patient's gynecomastia? digitalis, cimetidine, or spironolactone . A patient presents with tinnitus, dizziness, headaches, and Gl distress. What drug is causing these symptoms? drug-indu

Copyright 2012, Doctors In Traming.com. LLC. All Rights Reserved.

Antidotes
GG: Chapter 4 Specic antidotes (FAII p243) (FAI2 p270)

End of Session Quiz


14. What are the antidotes to the following toxins? Acetaminophen Salicylates Amphetamines Anticholinesterases, organophosphates Antimuscarinic, anticholinergic agents B-blockers (or verapamil) Digitalis Iron (FAN p224) (FAI2 p270) Lead Arsenic, mercury, gold Copper; arsenic, gold Cyanide Methemoglobin Carbon monoxide Methanol, ethylene glycol (antifreeze) Opioids Benzodiazepines Tricyclic Antidepressants Heparin Warfarin t-PA, streptokinase Theophylline

Copyright 2012. Doctors In Training.com. LLC. All Rights Reserved.

What pathology matches the following statement? (FAI I p524-52_, Focal myocardial inflammation with multinucleate giant cells Eosinophilic cytoplasmic globules in liver near nucleus Desquamated epithelial casts in sputum "Onion-skin" periosteal reaction Pseudopalisading tumor cell arrangement Elevated serum uric acid What are the most common causes of meningitis in the following age ranges? (FAII pl77) (FAQ pl97)

3. What structures form Hesselbach's triangle? (FAI I p316) (FAQ p345)

A Autosomal Dominant Diseases Part I


^ R : Chapter 5 q Autosomal dominant diseases (FAII p86) (FAI2 p90)

e n

What genetic defect is responsible for achondroplasia? (FAQ p90)

A genetic defect in which proteins results in hereditary spherocytosis? (FAQ p90)

6. What are the Cs of Huntington disease? (FAQ p90)

Copyright 2012, Doctors In Training.com. LLC. All Rights Reserved.

Autosomal Dominant Diseases Part 2 and Trinucleotide Repeats


R: Chapter 5 Autosomal dominant diseases (FAII p86) (FAI2 p90) Neurobromatosis (FAII p427) (FAI2 p467) Von Hippel-Lindau disease (FAII p427) (FAI2 p467) Trinucleotide repeat diseases (FAII p88) (FAI2 p92)

What autosomal dominant disease ts the following statement? Associated with floppy mitral valve, dissecting aortic aneurysm, berry aneurysm Associated with mitral valve prolapse, liver disease, berry aneurysms Neural tumors and pigmented iris hamartomas Very strong assodation with colon cancer Ml before age 20 Hemangioblastomas of retina/cerebellum/medulla Increased MCHC, hemolytic anemia Bilateral acoustic neuromas Fadal lesions, seizure disorder, cancer risk Caudate atrophy, dementia Cystic medial necrosis of the aorta Defect of fibroblast growth factor (FGF) receptor 3

What are some of the disorders resulting from trinucleotide repeats? (FAQ p92)

Copyright () 2012. Doctors In Traimng.com. LLC. All Rights Reserved.

What high-yield path ts the following statement. Large "bull's eye" rash Strawberry tongue Resting tremor, rigidity Rash on palms and soles

p520-52l) (FAQ p582-583)

What are the toxic side effects of tricyclic antidepressant use? (FAI I p455) (FAQ p500) 3. In which glomerular disease would you expect to see the following changes? (FAI I p467-468) (FAQ p516-517) Anti-GBM antibodies (immunofluorescence) Kimmelstiel-Wilson lesions (light microscope) "Spike and dome" appearance (electron microscope) 'Tram track" appearance of subendothelial humps (electron microscope) umps (electron microscope)

Autosomal-Recessive Diseases and Lysosomal Storage Diseases


-j-, R : Chapter 5 ty Autosomal-recessive diseases (FAII p86) (FAI2 p9l) Lysosomal storage diseases (FAII pill) (FAI2 pi 16-117)

ss 4. Cystic Fibrosis
Blood Lumen

HC03 carbonic! anhydrasel HCO3

Copyright (9 2012. Doctors In Tratnmg.com, LLC. All Rights Reserved.

Lysosomal Storage Diseases (FAI I pi 11) (FAQ pi 16-117) 5. Which lysosomal storage disease is associated with renal failure?

6. What are the only two X-linked recessive lysosomal storage diseases? What is the method of inheritance of the others?

7. What is the most common lysosomal storage disease?

8. Which lysosomal storage diseases are associated with an early death (usually by age 3)?

9. Which lysosomal storage disease is a demyelinating disease?

10. How might corneal clouding and mental retardation help distinguish between the mucopolysaccharidoses?
Hurler syndrome: Hunter syndrome: Scheie syndrome:

l-cell

disease:

11. Which lysosomal storage disease is characterized by the following enzyme deciency? <
a-L-iduronidase Iduronate sulfatase Arylsuffatase A a-galactosidase A m ^ ^ "^

G a l a c t o c e r e b r o s i d a s e (* g a l a c t o c e r e b r o s i d e a c c u m u l a t i o n ) I p - g l u c o c e r e b r o s i d a s e ( g l u c o c e r e b r o s i d e a c c u m u l a t i o n ) Q )
Hexosaminidase fl>

Sphingomyelinase

(*

sphingomyelin

accumulation)

12. Characterized by an accumulation of GM2 ganglioside.

D
to

13.

Characterized

by

an

accumulation

ofdermatin

sulfate.

pj 0)
to

14. Which are particularly common among Ashkenazi Jews? }>

15.

Which

has

characteristic

"crinkled

paper

cytoplasm"?

Q_ \s
i

5*
16. What is the differential diagnosis for a cherry-red spot on the retina? Q_

Copyright 2012. Doctors In Trainmg.com. LLC. All Rights Reserved.

[II]

. What is the name of the ge

e the ts the following description? Macroorchidism and autism Endocardial cushion defects are common Recurrent pulmonary infections, steatorrhea Multiple fractures, easily confused with child abuse Associated with Alzheimer disease after age 35 Excess fibro-fatty tissue deposits amongst muscle

'8. What gene is defective in cystic brosis? (FAQ p9l)

What is the traditional test used to diagnose cystic brosis? (FAQ p9l)

X-Linked Recessive Disorders


TD & Chapter 5 Q J V X-linked recessive disorders (FAII p86) (FAI2 p9l) C

~T 20. Hint for X-linked diseases: Fabry's Tale: Duke the Muscular Hunter Brutally Lysed the X Albino Gopher without being aWAre it was a Fragile Hemophiliac!

Copyright <C 2012. Doctors In Trainmg.com. LLC. All Rights Reserved

Session Quiz
Classify the following disorders as either autosomal dominant autosomal recessive, o, X-linked recessive.
Glycogen storage diseases, fragile X, polycystic kidney disease (adult and infant), PKU, hereditary spherocytosis, Duchenne muscular dystrophy, familial adenomatous polyposis, Lesch-Nyhan, Bruton, Huntington, thalassemias, sickle cell, Wiskott-Aldrich, Von Recklinghausen, Von-Hippel Lindau, hemophilia, mucopolysaccharidosis, familial hypercholesterolemia, Marfan, cystic brosis, hemochromatosis, G6PD'-deciency Autosomal Dominant Autosomal Recessive X-Linked Recessive

Copyright <9 2012. Doctors In Training.com. LLC. All Rights Reserved.

What is the antidote for the following toxin? Copper, gold, arsenic Arsenic, mercury, gold t-PA, streptokinase Digitalis

WXmmmfSTtsm

What drugs have the following side effects? (FAI I p244-245) (FAQ p27l-272) Agranulocytosis Osteoporosis Pulmonary fibrosis Gynecomastia Photosensitivity Drug-induced lupus What pathology ts the following high-yield phrase? (FAI I p518-520) (FAQ p580-582) Hypertension + hypokalemia + metabolic alkalosis Fever + night sweats + weight loss Adrenal hemorrhage due to meningococcemia Blue sclera Cs of Huntington disease Hyperphagia, hypersexuality, hyperoralfty, hyperdocilfty Nystagmus, intention tremor; scanning speech Lower extremity purpura, arthralgias, renal disease

Pediatric Review Part I


Apgar score (FAII p60) (FAI2 p62) Cerebral palsy Low birth weight (FAII p60) (FAI2 p62) Review of neonatal infections and antibiotics SD IS Early developmental milestones (FAII p60) (FAI2 p62) Tanner stages of sexual development (FAII p60) (FAI2 p535) Puberty 4. What developmental structure matches the following description? Supplies oxygenated blood to the fetus Removes nitrogenous waste from the fetal bladder Fetal placental structure that secretes hCG Maternal component of the placenta Returns deoxygenated blood from the fetal internal iliac arteries 5. Sudden Infant Death Syndrome (SIDS) Usually occurs at 2-4 months old Usually occurs while infant is sleeping Maternal risk factors: low SES, age <20, drugs/cigarettes during pregnancy Infant risk factors: low birth weight, female, premature, prior sibling with SIDS Preventive measures: "back to bed," pacifier when sleeping, fan in the room
Copyright s> 2012. Doctors In Training.com. LLC. All Rights Reserved.

Development & Physiology (FAI I p60) (FAQ p62) 6. Complete the following chart of developmental milestones
3m 6m 12m 15m 18m 2y 3y 4y 5y Gross motor Rolls over Verbal Fine motor Hands together Self-care

1 -3 words Walk backward, run 4-cube tower Jump up Half understandable

Drinks from cup

Washes/dries hands

Copies square

7. A mother presents with her I-year-old child that can stand alone, has just learned to walk, and has a 5-word vocabulary. She would like to know if her child is developmentally normal and when she can begin toilet training. 8. A 2-year-old child speaks in short sentences of 2-3 words but cannot identify colors or recite his ABCs. The mother is concerned that he is not developing normally. What do you tell the mother?

9. A girl can speak in complete sentences, has an imaginary friend, and considers boys to be "yucky". How old is she?

10. Tanner Stages


Stage 1 Stage 2 Boy genitalia development Prepubertal Enlargement of scrotum and testes Enlargement of penis (length rst) Penis - growth in breadth and development of glans Testes - enlarge Stage 5 Scrotum - larger and darker Adult Girl breast development Prepubertal Bud with elevation of breast and papilla Areola enlarges Further enlargement Areola and papilla form a secondary mound above the level of the breast Pubic Hair Prepubertal Sparse long, slightly pigmented hair Darker, coarser, and more curled Adult hair in type but covering a smaller area

73

5
Ln

Stage 3 Stage 4

a ?
%

Mature - only papilla projects as areola recesses

Adult in type and quantity

Female development Breast development (I I) Growth spurt (12) Menarche (13) Male development Stage 2 (12) * Growth spurt (14-15)

2
[15]

Copyright 2012. Doctors In Training.com. LLC. All Rights Reserved.

What factors are taken i

What is the denition of low birth weight? What complications are associated with low birth weights?

What are the features of 21-hydroxylase deciency? (FAI I p29l) (FAQ p3l8)

Other than fragile bones, what are the features of osteogenesis imperfect? (FAI I p80) (FAI2p83)

What genetic defect results in Fragile X syndrome? (FAI I p87) (FAQ p92)

What might you see in a rst trimester ultrasound of a fetus with Down syndrome? (FAII p88) (FAQ p93)

lat are the features of Williams syndrome? (FAI I p89) (FAQ p94)

. Based on the following milestones, how old are the following children? Jumps up, 6 cube tower, eats with spoon, 2-3 word sentences Regards face, responds to sound, not yet able to roll over Stands with support, 1-3 words, stranger anxiety, drinks from a cup
'Rides tricycle, understandable sentences, plays board-games

. A 16-year-old female patient presents with amenorrhea. It is later discovered that this patient lacks a uterus and uterine tubes, and there are two round structures in the midline just superior to the labia majora. What is most likely the cause of this patient's amenorrhea?

. A child presents with cleft-lip. Which embryonic process failed?

ow does the presentation of a branchial cleft cyst (FAI I pi 27) (FAQ pi 36) differ fro that of athyroglossal duct cyst? (FAI I pl30) (FAQ pl38)

Copyright 2012. Doctors In Training.com, LLC. All Rights Reserved.

Pediatric Review Part 2


Diagnosing disorders of sex hormones (FAII p486) (FAI 2 p539) Kallmann syndrome (FAII p486) (FAI2 p540) Review of ethics - consent for minors Review of pediatric infections already discussed - Otitis media Review of pediatric psychiatric disorders Review of pediatric neoplasms

22. Neoplasms in Children


ALL Astrocytoma Neuroblastoma Hemangioma Wilms tumor Hepatoblastoma Retinoblastoma Rhabdomyosarcoma Ewing sarcoma Osteogenic sarcoma Lymphoma (lymphoblastic) Teratoma

73
<

m
Ln

r?
n"

c D

Copyright 2012. Doctors In Training.com. LLC. All Rights Reserved.

[17]

111 r i M-i^a w i K9i' i F 23. Congenital most commons: Most common cause of early cyanosis Most common cause of late cyanosis Most common cause of primary amenorrhea Most common chromosomal disorder Most common cause of congenital mental retardation Second most common cause of congenital mental retardation Most common lethal genetic disease of Caucasians Most common cause of congenital malformations in US

. What are some of the main distinguishing features in autistic disorder, Rett disorder, and Asperger syndrome? (FAI I p442) (FAQ p485)

. vVhat pathology is associated with the following statement? (FAI I p53l) (FAQ p596) Most common testicular tumor Most common tumor of infancy Most common primary cardiac tumor in children 26. Complete the following chart of developmental milestones (FAI I p60) (FAQ p62) Gross motor Rolls over Verbal Fine motor Hands together Self-care

Jump up

Half understandable

Washes/dries hands

Copyright :; 2012. Doctors In Training.com. LLC. All Rights Reserved.

The Wisdom and Art of Medicine

70
<

m
m
O N

31
D

>

Copyright

2012.

DoctorslnTraining.com.

LLC.

All

Rights

Reserved.

rL "i9i ' J

"Realize how much time there is, [and] how long the day is. You have sixteen waking hours, three or four of which at least should be devoted to making a silent conquest of your mental machinery.... A few hours out of the sixteen will sufce, only let them be hours of daily dedication.... The failure to cultivate the power of peaceful concentration is the greatest single cause of mental breakdown."
Sir William Osier

Listen to the Exhortation of the Dawn! Look to this Day! For it is Life, the very Life of Life. In its brief Course lie all the Varieties and Realities of your Existence: The Bliss of Growth, The Glory of Action, The Splendour of Beauty; For Yesterday is but a Dream And Tomorrow is only a Vision; But Today well lived makes Every Yesterday a Dream of Happiness, And every Tomorrow a Vision of Hope. Look well therefore to this Day! Such is the Salutation of the Dawn!

[20]

Copyright 2012. Doctors In Training.com. LLC. All Rights Reserved.

Founder and Chief Educator Brian Jenkins, MD and our other MD physician educators teach the Doctors In Training Step I Review Course. All of the Doctors In Training physician lecturers pull insight from their own clinical experience and pair this with their extensive knowledge of the USMLE and COMLEX. This vital combination of clinical experience and Basic Science knowledge allows our physicians to teach students the material needed not only to perform well on board exams, but also to be well-rounded physicians and make the world a better place.

Michael Mclnnis, MD

The Doctors In Training 2012 Step I Review Course, taught by Doctors in Training Chief Educator, Brian Jenkins, MD, and additional MD educators focuses on high-yield information in an active-learning format. This course format keeps medical students engaged and maximizes retention through the use of quick quizzes, entertaining learning components and mnemonics. The 2012 course is broken down into approximately 30-45 minute video lecture segments, which allow students to easily digest the high-yield testing material. With success around the world, over 14,000 medical students have utilized the Doctors In Training Step I Review Course from over 150 U.S. and 110 International medical schools.

/W-z-'im. IKis is ike kiakesttj-iefl source outtkere.


- Case Western reserve- University >cwf of HWtoirxe- SWent D\\ isa.area,tresource-1+ Ke|psre*9uou.4^use<sTa*v?S"bw(li-ii^flf^Wv**4A veiu e-Tne*\t Wait - pj TTbpkirvs iWersitj- VtMeftt to prepare-. ft
SUCCESSFUL STEP 1 SYKAPTOGENESIS!

pur vnarafa is oiwm'ml mJ( wveJl r*e iMfcensefu- ikrotuik tke "toua k tir*es of stuX-ir^ -for ike ewvy. - TVlLsfwes'tem An-z-ojvx University Swof of Osteopath 7he<$dr\e Vtu^ent I feft[ikeI WJla.sfyJLyarbwwitk DILI Akofu-teL\weJIittwJtai-so4W1 "took +J\ea>Urse-! - ^rtkeastDkio %JUi University SWent

job awp JUI a. supew ioW II kere is w? Way. I Wou|| kive scored as kiak witkout tke- DlTa>Urse!

- Uftiversit* of rUvaA>d\cofoi^e4iw^ >We*\t I kstaotrrvu-llSl^tv^u^lcy. resuftsWtkaivfl sr^e-)i1nwmw4-l>stwanWt W wtl wif pe-recowv\en??itt4 Dll toeveruoKeamwif^etwteLWSir\ih^Up -forike.ytep"^ course.TWkuou.a^airv
- wJMefpkia Cojtae of L\teopatki^7ne^idy\e > Went I (bvM tke structure of accw^w^ave- (ea.rni.vj. I ke <aui-z--7.es, ^laarajns an| supp(er*eniaru. rwateriaf rvvUe evenjtWa so simple <*W eastf if uwerstW. lr* DlTaXisors oreaWa stuX p|an -for r*e <WWere tkere -for i*e even*, step of ine wau.. ikank uou tc DM I - letkwtM Israef ms+itcte SWent

Visit DoctorslnTraining.com to see our additional medical education lecture series.

DOCTORS IN TRAINING

BETTER DOCTORS. BETTER WORLD.

You might also like